Download as pdf or txt
Download as pdf or txt
You are on page 1of 198

Any screen.

Any time.
Anywhere.
Activate the eBook version
of this title at no additional charge.

Student Consult eBooks give you the power to browse and find content,
view enhanced images, share notes and highlights—both online and offline.

Unlock your eBook today.


1 Visit studentconsult.inkling.com/redeem

2 Scratch off your code


Scan this QR code to redeem your
3 Type code into “Enter Code” box
eBook through your mobile device:
4 Click “Redeem”
5 Log in or Sign up
6 Go to “My Library”
It’s that easy!

Place Peel Off


Sticker Here

For technical assistance:


email studentconsult.help@elsevier.com
call 1-800-401-9962 (inside the US)
call +1-314-447-8200 (outside the US)

Use of the current edition of the electronic version of this book (eBook) is subject to the terms of the nontransferable, limited license
granted on studentconsult.inkling.com. Access to the eBook is limited to the first individual who redeems the PIN, located on the inside
cover of this book, at studentconsult.inkling.com and may not be transferred to another party by resale, lending, or other means.
2015v1.0
Self-Assessment in

Paediatrics
This page intentionally left blank
Self-Assessment in

Paediatrics
Edited by
Tom Lissauer MB BChir FRCPCH
Honorary Consultant Paediatrician, Imperial College Healthcare Trust, London, UK
Centre for International Child Health, Imperial College London UK

Will Carroll BM BCh MD FRCPCH


Consultant in Paediatric Respiratory Medicine,
University Hospital of the North Midlands, Stoke-on-Trent, UK
© 2018, Elsevier Limited All rights reserved.

The right of Tom Lissauer and Will Carroll to be identified as author of this work has been asserted by
them in accordance with the Copyright, Designs, and Patents Act 1988.
No part of this publication may be reproduced or transmitted in any form or by any means,
electronic or mechanical, including photocopying, recording, or any information storage and retrieval
system, without permission in writing from the publisher. Details on how to seek permission, further
information about the Publisher’s permissions policies and our arrangements with organizations such
as the Copyright Clearance Centre and the Copyright Licensing Agency, can be found at our website:
www.elsevier.com/permissions.
This book and the individual contributions contained in it are protected under copyright by the
Publisher (other than as may be noted herein).

Notices
Knowledge and best practice in this field are constantly changing. As new research and experience
broaden our understanding, changes in research methods, professional practices, or medical
treatment may become necessary.
Practitioners and researchers must always rely on their own experience and knowledge in
evaluating and using any information, methods, compounds, or experiments described herein. In
using such information or methods they should be mindful of their own safety and the safety of
others, including parties for whom they have a professional responsibility.
With respect to any drug or pharmaceutical products identified, readers are advised to check the
most current information provided (i) on procedures featured or (ii) by the manufacturer of each
product to be administered, to verify the recommended dose or formula, the method and duration of
administration, and contraindications. It is the responsibility of practitioners, relying on their own
experience and knowledge of their patients, to make diagnoses, to determine dosages and the best
treatment for each individual patient, and to take all appropriate safety precautions.
To the fullest extent of the law, neither the Publisher nor the authors, contributors, or editors,
assume any liability for any injury and/or damage to persons or property as a matter of products
liability, negligence or otherwise, or from any use or operation of any methods, products, instructions,
or ideas contained in the material herein.

ISBN: 978-0-7020-7292-5
978-0-7020-7293-2

Printed in Europe
Last digit is the print number: 9 8 7 6 5 4 3 2 1

Content Strategist: Pauline Graham


Content Development Specialist: Fiona Conn
Project Manager: Anne Collett
Design: Miles Hitchen
Illustration Manager: Amy Faith Heyden
Illustrator: Vicky Heim and TNQ
Marketing Manager: Deborah Watkins
Contents
Preface vii
Acknowledgements vii
1. The child in society 1
2. History and examination 3
3. Normal child development, hearing and vision 9
4. Developmental problems and the child with special needs 13
5. Care of the sick child and young person 21
6. Paediatric emergencies 26
7. Accidents and poisoning 33
8. Child protection 40
9. Genetics 45
10. Perinatal medicine 56
11. Neonatal medicine 63
12. Growth and puberty 70
13. Nutrition 74
14. Gastroenterology 78
15. Infection and immunity 85
16. Allergy 93
17. Respiratory disorders 95
18. Cardiac disorders 104
19. Kidney and urinary tract disorders 110
20. Genital disorders 117
21. Liver disorders 120
22. Malignant disease 124
23. Haematological disorders 131
24. Child and adolescent mental health 138
25. Dermatological disorders 142
26. Diabetes and endocrinology 150
27. Inborn errors of metabolism 157
28. Musculoskeletal disorders 159
29. Neurological disorders 168
30. Adolescent medicine 177
31. Global child health 180

Index 182
This page intentionally left blank
Preface
The aim of this book is to consolidate knowledge recognition of clinical conditions or interpretation
of paediatrics and aid revision for examinations. of investigations.
The questions accompany the Illustrated Textbook While some of the questions test knowledge,
of Paediatrics (5th edition), which contains all the we have also tried to assess understanding and
core information on which the questions are decision-making. We have concentrated on the
based. It is primarily designed for medical stu- most important topics in paediatrics and have
dents, however, it should also be a helpful revision avoided rare problems unless an important
aid for candidates preparing for postgraduate message is conveyed. Our experience and feed-
paediatric examinations, such as the Foundation back from many undergraduate medical schools
of Practice component of the Membership of the in the UK, Europe, the Middle East, Hong Kong,
Royal College of Paediatrics and Child Health Malaysia, Singapore, Australia and New Zealand,
examination or the Diploma of Child Health. and Africa is that the Illustrated Textbook of Paed­
Answers are provided for all questions, with iatrics and these self-assessment questions cover
additional comments or explanation to assist with the range and depth of topics of the paediatric
understanding and learning about the topic and curriculum.
not just checking if the answer is right or wrong. We very much hope that you will find the ques-
We have used two question formats, the Single tions helpful in your revision and welcome feed-
Best Answer and Extended Matching Question. back. And good luck in your examinations!
This reflects a change in examination assessment,
with less use of multiple true-false questions. We Tom Lissauer and Will Carroll
have also included some illustrations to assess

Acknowledgements
We have drawn extensively on questions and Body, Elizabeth Waddington and Claire Wensley
answers written for ‘Illustrated Self Assessment (https://studentconsult.inkling.com.2012) and
in Paediatrics’ by Tom Lissauer, Graham Roberts, wish to acknowledge and thank them for their
Caroline Foster and Michael Coren (Elsevier, contribution.
2001) and modified by Peter Cartledge, Caroline
This page intentionally left blank
1

The child in society

1.3
Questions: Single Best Answer In which age range is childhood mortality
greatest?
1.1
Poverty is associated with the greatest increased Select one answer only
risk of which one of the following conditions
A. <1 year
during childhood in the UK?
B. 1–5 years
Select one answer only C. 5–9 years
D. 10–14 years
A. Asthma E. 15–19 years
B. Cystic fibrosis
C. Developmental dysplasia of the hip
D. Febrile seizures
E. Pneumonia

1.2
Which of the following statements best describes
the negative effects of poverty on child
development?
Select one answer only
A. Especially harmful from the ages of birth to 5
years
B. More likely in the first child
C. Most pronounced in children from minority
ethnic groups
D. Rare in developed countries like the UK
E. Temporary and resolve if familial poverty is
addressed
By the age of 4 years, a development gap of
Answers: Single Best Answer more than 1.5 years can be seen between
the most disadvantaged and the most
1.1 advantaged children. Babies whose
1 A. Asthma
Correct. Asthma shows a marked relationship
development has fallen behind the norm during
the first year of life are much more likely to fall
with poverty as does low birthweight, injuries, even further behind in subsequent years rather
hospital admissions, behavioural problems, than catch up with those who have had a better
The child in society

special educational needs and child abuse. The start.


relationship between poverty and other health
problems is less clear cut. B. More likely in the first child
Birth order has little effect. Singletons may have
B. Cystic fibrosis some protection as they do not have to compete
A purely genetic condition; varies with ethnicity for parental attention.
but not poverty.
C. Most pronounced in children from minority
C. Developmental dysplasia of the hip ethnic groups
This is a congenital disorder of unknown Whilst poverty is more common in minority
aetiology. It is more common in girls, if there is a ethnic groups the effects are similar across all
family history, and breech presentation during groups.
pregnancy.
D. Rare in developed countries like the UK
D. Febrile seizures The UK has a higher proportion of children living
There is no socioeconomic influence. in relative poverty than many countries in the
European Union.
E. Pneumonia
Within the UK and the developed world, the E. Temporary and resolve if familial poverty is
association is lost. However, in developing addressed
countries the risk is higher in those with We remain a hostage to our early life
malnutrition. experiences. Paediatricians should act as
advocates for the alleviation of childhood
1.2 poverty.
A. Especially harmful from the ages of birth
to 5 years 1.3
Correct. Research indicates that being poor at A. <1 year
both 9 months and 3 years of age is associated Correct. Over half of deaths in childhood occur
with increased likelihood of poor behavioural, in infancy. Major causes are immaturity and
learning and health outcomes at age 5 years. congenital abnormalities.

2
2

History and examination

Questions: Single Best Answer


2.1
Ritha, aged 2 months, is admitted to hospital with
a 2-day history of mild coryza and tachypnoea
without significant intercostal recession. She has
been feeding poorly for the last 3 weeks.
Which clinical feature most supports her having
congenital heart disease rather than respiratory
disease? Right arm: Left arm:
Tone – normal Tone – increased
Select one answer only.
Reflexes – normal Reflexes – brisk
A. Sibling with congenital heart disease
B. Poor feeding
C. Generalized wheeze on auscultation
D. Hepatomegaly
E. Ejection systolic murmur, grade II/VI, at the
left sternal edge

2.2
Nazma, aged 4 years, presents with a 1-week
history of episodic central abdominal pain. She is
of Indian ethnicity, but the family live in Kenya
and are visiting relatives in the UK. She is
otherwise well. Her relative’s general practitioner
thinks she may be slightly pale and that her
spleen is enlarged, as it is 3 cm below the costal
margin. There are no other abnormalities on Right leg: Left leg:
examination. Tone – normal Tone – increased
Which of the following is the most likely cause Reflexes – normal Reflexes – brisk
for her enlarged spleen?
Select one answer only.
A. Acute lymphoblastic leukaemia
B. Malaria
C. Hookworm infestation
D. Wilms tumour
E. Sickle cell disease
Figure 2.1
2.3
Katie, an 18-month-old girl, is reviewed in the
paediatric clinic. She is unsteady on her feet but
has normal vision and gaze. She walks with a
limp and tends to fall to her left side. Her limb
tone and reflexes are as shown in Fig. 2.1.
Which is the site of her neurological lesion? 2.6
William, a 9-year-old boy, presents to the rapid
Select one answer only. access paediatric clinic with a fractured tibia
A. Upper motor neurone lesion following a fall from a wall. He is otherwise well
2 B.
C.
Lower motor neurone lesion
Cerebellar lesion
but has a history of shortness of breath and
wheeze when running. His mother denies that
D. Basal ganglia lesion he has ever needed treatment for his wheeze. He
E. Neuromuscular junction is not unwell currently.
History and examination

What clinical sign is best shown in Fig. 2.3?


2.4
Katie’s clinical problem and examination are Select one answer only.
described in Question 2.3.
A. Barrel chest
Which of the following best describes the B. Pectus excavatum
pattern of neurological signs? C. Pectus carinatum
D. Harrison sulcus
Select one answer only.
E. Sternal recession
A. Diplegia
B. Right hemiplegia
C. Left hemiplegia
D. Spastic quadriplegia
E. Choreoathetoid cerebral palsy

2.5
Jeremiah, a 6-year-old boy, is brought by his
mother to the ophthalmology outpatient clinic.
She is worried about her son’s ‘funny eyes’. You
examine him using the cover test, and you find
the signs shown in Fig. 2.2.

Right Left

C Figure 2.3

Figure 2.2 2.7


Ishmael, a 15-year-old boy from Pakistan, is seen
in the outpatient department. He has a long
history of chest infections needing recurrent
courses of antibiotics. He has a productive
What disorder does he have? cough. He opens his bowels once a day. On
examination, he has a normal temperature, his
Select one answer only.
skin and mucous membranes are pink and his
A. Right convergent squint heart sounds are normal. His hands look unusual
B. Right divergent squint (Fig. 2.4). On auscultation, there are some
C. Alternating convergent squint scattered crepitations at both bases. In view of
4 D. Left convergent squint his recurrent chest infections, you had ordered a
E. Left divergent squint sweat test, which is negative.
Subcostal and Respiratory
intercostal rate 70 breaths/min
recession

Scattered
wheezes throughout
both lung fields

History and examination


Fine end-expiratory
crackles
Heart rate 120 beats/
Hyperinflated min
chest Heart sounds normal
Figure 2.4 No murmurs
Figure 2.5

Which of the following is the most likely cause of 2.8.2


the appearance of Ishmael’s hands? Hatem, a 3-year-old boy, presents with fever and
difficulty breathing, getting worse for the last 3
Select one answer only. days. Examination shows the clinical signs shown
A. Cystic fibrosis in Fig. 2.6.
B. Infective endocarditis
C. Primary ciliary dyskinesia
D. Crohn’s disease
E. Tetralogy of Fallot
Respiratory
rate 40 breaths/min
Questions: Extended Matching
2.8
The following list (A–M) are causes of difficulty
breathing in children. For each of the following
clinical scenarios select the most likely diagnosis.
Each answer may be used once, more than once
or not at all.
A. Acute exacerbation of asthma Dull to percussion Heart rate 120 beats/min
B. Bronchiolitis Bronchial breathing Heart sounds normal
C. Chronic asthma Inspiratory crepitations No murmurs
D. Cystic fibrosis
E. Heart failure Figure 2.6
F. Inhaled foreign body (left side)
G. Inhaled foreign body (right side)
H. Pleural effusion (left sided)
I. Pleural effusion (right sided)
2.8.3
J. Pneumonia (left sided)
Darren, a 3-year-old boy, was eating peanuts 2
K. Pneumonia (right sided)
days ago when his younger brother pushed him
L. Pneumothorax (left sided)
over. He coughed up the peanuts and was all
M. Pneumothorax (right sided)
right. Today, he has been coughing and
becomes breathless as soon as he runs about.
2.8.1 He is afebrile. Examination findings are shown
Rob, a 9-month-old boy, presents with fever and in Fig. 2.7.
difficulty breathing for the last 3 days. His
breathing is now interfering with his feeding. He
has not had any previous illnesses and his
growth is normal.
Examination of his chest is shown in Fig. 2.5.
5
2.9
Respiratory The following list (A–M) are causes of difficulty
rate 36 breaths/min breathing in children. For the following clinical
scenario select the most likely diagnosis.
2 Reduced
air entry on
A.
B.
Asthma
Bronchiolitis
the left chest C. Cystic fibrosis
History and examination

D. Heart failure
Percussion
E. Inhaled foreign body (left side)
normal
F. Inhaled foreign body (right side)
G. Pleural effusion (left sided)
H. Pleural effusion (right sided)
Heart apex I. Pneumonia (left sided)
beat displaced J. Pneumonia (right sided)
to right K. Pneumothorax (left sided)
L. Pneumothorax (right sided)
Figure 2.7 M. Viral-induced wheezing

2.9.1
2.8.4 Jamalah, a 7-year-old girl, presents with difficulty
Tony, a 4-year-old boy, is admitted with breathing. She has had a cold for the last 2 days.
pneumonia. His chest X-ray shows consolidation This is the third time this has happened, each
at the right base. Despite antibiotic therapy, he time when she had a cold. She has not had any
remains febrile and unwell. Examination findings other medical problems but her mother has
are shown in Fig. 2.8. noticed she ‘coughs and whistles’ when she goes
out in the cold. On examination, she has an
upper respiratory infection and the signs shown
in Fig. 2.9.

Respiratory rate
50 breaths/min

Respiratory
Stony dull to rate 44 breaths/min
percussion Mild subcostal
and intercostal
recession
Reduced
breath Percussion –
sounds hyper-resonant
Scattered Scattered
inspiratory wheezes
crepitations

Figure 2.8 Figure 2.9

6
B. Lower motor neurone lesion
Answers: Single Best Answer Tone and reflexes would be decreased.

2.1 C. Cerebellar lesion


A. Sibling with congenital heart disease Cerebellar signs would be found including
A sibling with congenital heart disease slightly nystagmus, past-pointing, and ataxia.
increases the risk for Ritha, but it is still very low.
D. Basal ganglia lesion
B. Poor feeding There would be abnormal limb movements.

History and examination


Poor feeding may be a clinical feature of either
E. Neuromuscular junction
congenital heart or respiratory disease. The
This is a lower motor neurone lesion, and signs
3-week history may be related to her developing
would therefore include decreased tone and
heart failure.
reflexes.
C. Generalized wheeze on auscultation 2.4
Generalized wheeze on auscultation may be a A. Diplegia
clinical feature of either congenital heart or This affects all four limbs, the lower limbs more
respiratory disease. than the upper limbs.
D. Hepatomegaly B. Right hemiplegia
Correct. In infants, hepatomegaly is an Right hemiplegia describes the abnormal clinical
important sign of heart failure, usually secondary signs in the limbs, not the affected side of the
to congenital heart disease. Heart failure can brain. Katie’s left limbs are affected, so she does
result from inadequate cardiac output (forwards not have a right hemiplegia.
failure) or failure to pump enough blood away
from the system feeding it (backwards heart C. Left hemiplegia
failure). In adults, left ventricular failure Correct. Katie has a left hemiplegia due to the
predominates, but in children either ventricle increased tone and reflexes on her left arm
can fail. and leg.

E. Ejection systolic murmur, grade II/VI, at the D. Spastic quadriplegia


left sternal edge Spastic quadriplegia is when all four limbs are
Ejection systolic murmur, grade II/VI, at the left affected, often severely.
sternal edge in isolation is most likely to be an
E. Choreoathetoid cerebral palsy
innocent murmur.
There are abnormal movements of the limbs.
2.2 2.5
A. Acute lymphoblastic leukaemia A. Right convergent squint
Acute lymphoblastic leukaemia may cause Correct. It is a convergent squint because the eye
splenomegaly, but other clinical manifestations is facing inwards towards the midline. It affects
would also be present. the right eye when both eyes are uncovered. The
B. Malaria uncovered left eye usually focuses on objects. But
Correct. High prevalence in Kenya and may when the left eye is covered, the right eye takes
cause chronic anaemia and splenomegaly. up a normal position and focuses.

C. Hookworm infestation B. Right divergent squint


Hookworm infestation is associated with In a divergent squint, the eye deviates laterally
anaemia but not splenomegaly. away from the midline.

D. Wilms tumour C. Alternating convergent squint


Wilms tumour would cause enlarged kidney The left eye would remain convergent after
rather than spleen. uncovering.

E. Sickle cell disease D. Left convergent squint


Sickle cell disease may cause anaemia and The left eye is fixing until it is covered up, and
splenomegaly, but prevalence is very low in therefore is not squinting.
Indian ethnicity, whereas malaria is common in E. Left divergent squint
Kenya. The left eye is fixing until it is covered up, and
therefore is not squinting.
2.3
A. Upper motor neurone lesion 2.6
Correct. The increased tone and reflexes of her A. Barrel chest
left arm and leg are from an upper motor A barrel chest is hyperinflated with a large
neurone lesion, most likely in the right side of anterior–posterior measurement to look like a 7
her brain. round barrel.
B. Pectus excavatum failure is unlikely as cardiac examination is
This is when there is a hollow in the anterior normal and there is no hepatomegaly.
chest wall.
2.8.2
K. Pneumonia (right sided)
2 C. Pectus carinatum
This is a defect of the anterior chest wall caused
by bulging of the sternum (pigeon chest).
This child has ‘classic’ signs of a right-sided
pneumonia. Stony dullness (rather than just
dullness to percussion) would suggest fluid
D. Harrison sulcus
History and examination

accumulation – a pleural effusion. Percussion is


Correct. Harrison sulcus is caused by chronic possible in most children.
indrawing of the chest wall because of
diaphragmatic tug from chronic respiratory 2.8.3
disease. In this case he appears to have chronic F. Inhaled foreign body (left side)
under-treatment of asthma. In clinical practice, 50% of children with inhaled
foreign body do not volunteer a history of
E. Sternal recession inhalation. In this case, there is reduced air entry
Sternal recession is an acute sign of respiratory on the left side and heart apex beat displaced to
distress and is not consistent with the history the right, suggesting that the peanut has
given above. eventually ended up in the left bronchus with
2.7 reduced air entry on inspiration and a ball-valve
A. Cystic fibrosis effect causing hyper-expansion distally causing
This is effectively excluded by a negative sweat the apex beat to be displaced to the right. It is
test. most common for foreign bodies to initially
descend down the straighter, wider right main
B. Infective endocarditis bronchus and lodge here. However, inhaled nuts
Ishmael has no fever and examination does not can ‘move around’ in the larger airways as
support this diagnosis. coughing can dislodge them.
C. Primary ciliary dyskinesia 2.8.4
Correct. This child has marked clubbing of the I. Pleural effusion (right sided)
fingers due to bronchiectasis. Although cystic The signs are of a right pleural effusion.
fibrosis is the commonest cause of clubbing due
to respiratory disease, it may also be caused by 2.9.1
other respiratory conditions including primary A. Asthma
ciliary dyskinesia. Whilst not all that wheezes is asthma, this is a
common diagnosis and the most likely in this
D. Crohn’s disease scenario. In clinical practice, it is helpful to
Ishmael has no gastrointestinal symptoms. distinguish between viral-induced wheeze and
E. Tetralogy of Fallot asthma as the latter is steroid-responsive. Young
If Ishmael had tetralogy of Fallot, he would be children with asthma typically have interval
cyanosed and have an abnormal cardiac symptoms (wheezing and cough when well), as
examination. described by Jamalah’s mother, and most have
atopy (eczema). Lung function tests are
diagnostic and usually possible in children of 7
Answers: Extended Matching years.
2.8.1
B. Bronchiolitis
The child’s age, history, and clinical signs are
characteristic of bronchiolitis. The signs are
bilateral, which excludes unilateral lesions. Heart

8
3

Normal child development,


hearing and vision
At what age does Evie need to be referred for
Questions: Single Best Answer further review if the squint is still present?

3.1 Select one answer only.


Steven has just had his first birthday party. During
A. 2 weeks
his party he commando crawled with great
B. 6 weeks
speed, although he cannot walk. He managed to
C. 12 weeks
pick off all the Smarties (round chocolate sweets)
D. 8 months
from his birthday cake. He can say two words
E. 12 months
with meaning. After his birthday party, he
impressed his guests by waving goodbye.
3.4
Which area of Steven’s development is delayed? Sophie is a well 8-week-old baby who was born
at term. She has come for a routine
Select one answer only.
developmental check.
A. Fine motor and vision
Which of the following would you NOT expect
B. Gross motor
her to be able to do?
C. Social, emotional and behavioural
development Select one answer only.
D. Speech and hearing
E. None – his development is within normal A. Auditory brainstem response audiometry
limits B. Fix and follow a toy
C. Quieten to a loud noise
3.2 D. Raise her head when lying prone
Gerald is a 16-month-old boy who has not yet E. Reach out and grasp an object
said his first word and does not babble much.
His mother believes he does not hear well 3.5
because he does not startle when a door slams Joanna is an active toddler. She is just being
or show any response to his name. His potty trained and has had several days where
development is otherwise normal. she has remained dry. She enjoys pulling her
clothes off to use the potty but cannot dress
Which test would be best to assess Gerald’s
herself again. She enjoys playing by pretending
hearing?
to make her mother a cup of tea but does not
Select one answer only. play well with her older siblings, as she has not
yet learnt how to take turns. She is very bossy
A. Auditory brainstem response audiometry and demands things by saying ‘give me’ or ‘me
B. Distraction hearing test drink’. She can build a tower of six blocks and
C. Otoacoustic emission enjoys running and climbing on furniture.
D. Speech discrimination testing
E. Visual reinforcement audiometry What developmental age is Joanna?
Select one answer only.
3.3
Evie is a 10-day-old infant and was born in London. A. 12 months
Her health visitor reviews the family at home. She B. 18 months
is feeding well and has a normal examination C. 24 months
except that she has a squint. The health visitor tells D. 2.5 years
her parents that she will keep this under review. E. 3 years
3.6 3.7.6
Cordelia is a 4-month-old baby girl who is Rita’s father is thrilled that she has just said
assessed by her general practitioner because of her first word – says ‘dada’ to her father
constant crying and poor feeding. She is fed by only.
3 bottle on infant formula. Her mother tearfully
complains that she is finding it very difficult to 3.7.7
Rinah can follow her mother’s two-step
cope. She also has a 20-month-old son who has
recently been referred to the speech and commands, such as ‘Go to the cupboard and
Normal child development, hearing and vision

language therapist because of language delay. fetch your red shoes’.


Charlotte’s development, growth and physical 3.7.8
examination are normal. Blessing’s mother is so pleased; her baby has just
What is the likely cause of Cordelia’s problems? learnt to smile when she smiles at her!

Select one answer only. 3.7.9


Ivan can tie his shoe laces all by himself.
A. An inherited genetic condition
B. Cow’s milk protein allergy 3.7.10
C. Down syndrome Lizzie has just started crawling.
D. Gastro-oesophageal reflux
E. Maternal postnatal depression/stress 3.8
At which of these ages is the following action
Questions: Extended Matching (screening, examination, health promotion
activity) usually first taken in the child health
3.7 surveillance and promotion programme in the
At what age would one expect children to achieve United Kingdom? Each answer can be used once,
the milestones described in the scenarios (median more than once, or not at all.
age) described below? Each age can be used
once, more than once, or not at all. A. Newborn
B. 5–6 days
A. 6 weeks C. 12 days
B. 6 months D. 8 weeks
C. 8 months E. 3 months
D. 10 months F. 4 months
E. 12 months G. 8 months
F. 18 months H. 12 months
G. 2 years I. 2–3 years
H. 3 years J. 4–5 years (preschool)
I. 4 years K. 5 years (school entry)
J. 5 years
3.8.1
3.7.1 First measles/mumps/rubella (MMR)
Rosette can build a three-cube tower and can immunization.
point to her nose.
3.8.2
3.7.2 Biochemical screening test (Guthrie test).
Peace has just taken her first steps!
3.8.3
3.7.3 Advice on reducing the risk of sudden
Anthony has a friend at nursery, and they enjoy infant death syndrome by ‘back to sleep’,
playing with toy cars together. avoiding overheating and avoiding parental
smoking.
3.7.4
Herbert can transfer objects from one hand to 3.8.4
the other whilst sitting without support and with Hearing test using otoacoustic emission
a straight back. or auditory brainstem response
audiometry.
3.7.5
Grace enjoys drawing. She has just learnt to copy 3.8.5
drawing a square, and can build steps using An orthoptist assessment for visual
blocks after being shown. impairment.

10
E. Reach out and grasp an object
Answers: Single Best Answer Correct. An 8 week old infant will not be able to
voluntarily reach out to grasp an object; she will
3.1 only be able to grasp what is placed in her
A. Fine motor and vision hand.
Fine motor: he must be able to perform a pincer
grip to be able to pick small chocolates off his
3.5
birthday cake.
A. 12 months

Normal child development, hearing and vision


B. Gross motor The two-word sentences suggest that Joanna is
Gross motor: Steven commando crawls, so is more advanced than 1 year.
expected to walk later than the median age of 12
B. 18 months
months.
A six-block tower would be advanced for 18
C. Social, emotional and behavioural months.
development
Social: he can wave goodbye. C. 24 months
Correct. Joanna is dry by day, can undress,
D. Speech and hearing and has symbolic play. She is not yet playing
Speech: aged 1 year he is able to say two words interactively; she will learn this at about
with meaning. 3 years of age. She is constructing two word
sentences. She constructs a tower of six
E. None – his development is within normal
blocks and can run. In assessing development,
limits
find the most advanced skill that cannot be
Correct. He has achieved normal milestones for
performed.
a 12-month-old.
3.2 D. 2.5 years
E. Visual reinforcement audiometry We would expect more in language
Correct. This is the most reliable test for a child development by 30 months.
of Gerald’s age. The test requires an assistant to E. 3 years
play with the child and keep his attention. Joanna has imaginative play but is not yet
Behind a soundproof window, another assistant playing interactively, as would be expected at
will play sounds through a loudspeaker at this age.
particular frequencies. When the child turns
around to the noise, a glass-fronted box with a 3.6
previously dark toy inside lights up as visual A. An inherited genetic condition
reinforcement to reward the child for turning Although more than one child is affected,
around. besides genes they also share a home
3.3 environment.
C. 12 weeks B. Cow’s milk protein allergy
Correct. Newborns may appear to squint when A common and important cause of
looking at nearby objects because their eyes distress in bottle-fed infants which should be
over-converge. By 6 weeks of age, the eyes suspected if there is eczema, family history of
should move together when following an object, allergies and/or blood in the stool or faltering
and by 12 weeks of age there should be no growth.
squint present.
C. Down syndrome
3.4 The examination is normal.
A. Auditory brainstem response audiometry
Hearing in newborn children can be tested using D. Gastro-oesophageal reflux
this method. A common diagnosis but there are hints of other
issues. Why would this lead to speech delay in a
B. Fix and follow a toy
sibling?
A baby should be able to fix and follow either a
face or a brightly coloured object by 6 weeks. E. Maternal postnatal depression/stress
Correct. This may be detrimental to
C. Quieten to a loud noise
Cordelia’s development, as infants are totally
Usually a baby can do this by 1 month of age. It
dependent on their main caregiver. Her older
is a useful question to ask parents if hearing is a
brother may have speech and language delay
concern.
because he is not receiving enough stimulation
D. Raise her head when lying prone to develop his own language skills or there may
A 6 to 8 week old infant should be able to lift her be an underlying problem causing his speech
head by 45°. delay. 11
make sure it is a true ‘two-step’ command.
Answers: Extended Matching Children may fetch a toy (one-step command) if
they know where it is.
3.7.1
3.7.8
3 F. 18 months
By 18 months can usually build a small tower of A. 6 weeks
An important and remarkably constant
bricks and can point to several parts of the body.
milestone.
3.7.2
Normal child development, hearing and vision

E. 12 months 3.7.9
One year is the median age for children to walk J. 5 years
(a few steps) and say two to three words. These You might remember being ‘taught’ this skill!
are very important milestones. It is acquired at a variable age but requires good
fine motor skills and ‘memorization’ of the task.
3.7.3 Most children can do this by the time they finish
H. 3 years their first year of primary school education in
At 3 years, children develop interactive play and the UK.
turn-taking. It may emerge slightly sooner in
those who attend nursery or in those with 3.7.10
siblings. C. 8 months
3.8.1
3.7.4
H. 12 months
C. 8 months
It is usually given at 12 months in the UK.
Children transfer objects around 7 months of
age. A rounded back requires control of T1 to 3.8.2
T12, but a straight back requires T12 and L1 so B. 5–6 days
develops later. A child will often ‘sit’ at 6 months Screens for congenital hypothyroidism, cystic
but sit and be stable and with a straight back at fibrosis, haemoglobinopathies and a range of
8 months. inborn errors of metabolism. Some of the inborn
errors of metabolism cannot be detected until
3.7.5
metabolites have accumulated. Therefore, the
I. 4 years
age at which the test is done is a compromise, as
Asking a child to draw (copy) shapes is a good
some infants may present before day 5 of life.
way of ‘estimating’ their age. They scribble first
(age 2 years), then can copy a circle (by 3 years), 3.8.3
and a square (by 4 years). A triangle (by 5 years) A. Newborn
follows. This advice should also be given throughout
pregnancy.
3.7.6
D. 10 months 3.8.4
Many children will make double-syllable sounds A. Newborn
which are not specific to an object or person. In Performed in the neonatal period. Undertaking
this case, the word ‘dada’ is specific for her father this whilst still in hospital ensures that few
and so indicates 10 months of age. children miss their screening.
3.7.7 3.8.5
H. 3 years J. 4–5 years (preschool)
At about 2 12 years of age, children can follow a This should be performed prior to starting
two-step set of instructions. It is important to school.

12
4

Developmental problems and the child


with special needs

Select one answer only.


Questions: Single Best Answer
A. 0–12 months
4.1 B. 12–24 months
Jonathan is 4 years old and lives in a small village C. 2–4 years
in southern England. He attends a paediatric D. 4–8 years
outpatient clinic with his grandmother who is his E. Above 8 years
legal guardian. She is concerned that he only
seems to like to play with his toy train and insists 4.4
on watching the same DVD every night before Fortuna is an 8-month-old black African girl who
he goes to bed. He attends nursery where he was born at term. She is seen in the paediatric
plays with the toys but not with other children. outpatient department. She can roll over. She
His behaviour can be very difficult to manage at does not crawl. She can say ‘dada’, but says it to
times. He does not say any words, whereas the everyone not just her father. She reaches out and
grandmother’s children were speaking in grasps objects with her left hand but not with
sentences at his age. On examination you notice her right, and puts objects in her mouth. She
he does not make eye contact with you and smiles, but is not able to wave bye-bye.
pushes his toy train back and forth on the floor.
Which aspect of her development is of most
The rest of his examination is normal.
concern?
What is the most likely diagnosis?
Select one answer only.
Select one answer only.
A. Does not wave bye-bye
A. Asperger syndrome B. Inability to crawl
B. Attention deficit hyperactivity disorder C. Inability to use sounds discriminately to
C. Autism spectrum disorder parents
D. Developmental coordination disorder (also D. Left-hand preference
known as dyspraxia) E. None of the above
E. Expressive language disorder
4.5
4.2 Gloria is a 19-month-old girl who presents to you
At what age does autism spectrum disorder in primary care. Her health visitor is concerned
usually become evident? because she is still only babbling and says no
distinct words. She is able to walk, scribbles with
Select one answer only. crayons and feeds herself with a spoon.
A. 0–12 months What is the most appropriate first action?
B. 12–24 months
C. 2–4 years Select one answer only.
D. 4–8 years A. Hearing test
E. Above 8 years B. Assessment by a team specializing in autism
spectrum disorders
4.3 C. Reassure the health visitor
At what age would you expect the clinical D. Refer to an ear, nose and throat surgeon
features of spastic bilateral cerebral palsy to E. Refer to a paediatrician for a full
become evident? developmental assessment
Right ear Left ear
125 250 500 1000 2000 4000 8000 125 250 500 1000 2000 4000 8000
-20 -20 -20 -20
4 -10 -10 -10 -10

Hearing level (dB, Decibels)


0 0 0 0
10 10 10 10
20 20 20 20
Developmental problems and the child with special needs

30 30 30 30
40 40 40 40
50 50 50 50
60 60 60 60
70 70 70 70
80 80 80 80
90 90 90 90
100 100 100 100
128 256 512 1024 2048 4096 8192 128 256 512 1024 2048 4096 8192
Frequency (Hz) Frequency (Hz)
Air conduction: Right ear Left ear

Unmasked bone conduction:

Figure 4.1

4.6 4.7
Andrew is a 5-year-old boy. His father feels his Cruz is a 2-year-old boy. He has recently moved
behaviour has deteriorated and he is worried he to the UK from Mexico. He attends the audiology
is not hearing him all the time. He has poor department as he has marked problems with his
articulation of the few words that he can say. language development. His audiogram is shown
Andrew goes to the audiology department and in Fig. 4.2.
has his hearing tested. His audiogram is shown
in Fig. 4.1. What type of hearing loss does he have?

What type of hearing loss does he have? Select one answer only.

Select one answer only. A. Mild conductive hearing loss in both ears
B. Mild sensorineural hearing loss in
A. Mild conductive hearing loss in the right ear both ears
B. Mild sensorineural hearing loss in the C. Mixed hearing loss in both ears
right ear D. Severe conductive hearing loss in
C. Mixed hearing loss in the right ear both ears
D. Moderate sensorineural hearing loss in the E. Severe sensorineural hearing loss in
right ear both ears
E. Severe conductive hearing loss in the
right ear

14
Right ear Left ear
125 250 500 1000 2000 4000 8000 125 250 500 1000 2000 4000 8000
-20 -20 -20 -20
-10 -10 -10 -10

Hearing level (dB, Decibels)


0 0 0 0
10 10 10 10
20 20 20 20

Developmental problems and the child with special needs


30 30 30 30
40 40 40 40
50 50 50 50
60 60 60 60
70 70 70 70
80 80 80 80
90 90 90 90
100 100 100 100
128 256 512 1024 2048 4096 8192 128 256 512 1024 2048 4096 8192
Frequency (Hz) Frequency (Hz)
Air conduction: Right ear Left ear

Unmasked bone conduction:

Figure 4.2

4.8
Jenny is an 8-week-old girl who was born Questions: Extended Matching
preterm at 35 weeks’ gestation. She is seen by
her general practitioner for her surveillance 4.9
review. Her mother is concerned that she does Which of these investigations [A–J] would you
not smile. Her gross motor development appears choose to initially undertake to confirm the
to be normal and she startles to loud noises. diagnosis of developmental delay in the children
However, she will not follow a face or a colourful described in the following scenarios? Each
ball. The appearance of one of her eyes is investigation can be used once, more than once,
shown in Fig. 4.3. The other eye has a similar or not at all.
appearance. She has no other medical problems. A. Blood lactate
B. Chromosome karyotype
C. Congenital infection screen
D. Cranial ultrasound scan
E. Creatine kinase
F. CT or MRI scan of the brain
G. DNA fluorescent in situ hybridization (FISH)
analysis
H. EEG
I. Maternal amino acids for raised
phenylalanine
J. Thyroid function tests
4.9.1
Figure 4.3 Clarissa, a cheerful 20-month-old white British
girl, is referred to the child development clinic by
What is the likely underlying diagnosis? her health visitor because she is not yet walking.
Select one answer only. She was born at term with no complications. She
learnt to sit without support at 10 months, and is
A. Cataract able to crawl, although she drags her right leg
B. Conjunctivitis behind her. Her mother says that she has always
C. Corneal trauma been left handed. Examination of the right arm
D. Retinopathy of prematurity and leg reveals reduced power but increased 15
E. Vitamin A deficiency tone and reflexes.
4.9.2 4.10.2
Geoffrey is a 10-month-old black infant referred Gerald is a 4-year-old boy who was born in
to the child development clinic. His mother raised Tanzania, and had severe jaundice as a neonate
concerns because he is slower in his development that could not be treated because of lack of
4 than her four other children. He can sit but only if
he is propped up with cushions. He is not
medical services. He now has abnormal
movements of all his limbs where he adopts and
crawling or pulling to stand. On examination he is maintains unusual postures, and when he is
hypotonic, with some dysmorphic features, startled by a loud noise, the arm on one side
Developmental problems and the child with special needs

including upslanting palpebral fissures. There is a straightens and the opposite arm bends. When
skin fold of the upper eyelid covering the inner he is asleep he is hypotonic.
corner of the eye and a flat occiput. He has no
other medical problems except some vitiligo. He 4.10.3
was born at term by normal vaginal delivery. He Hassan is 3 years old and was born at 26 weeks’
has been slow to feed. gestation weighing 700 g. He sat at 10 months,
and has just started to walk. He can scribble and
4.9.3 build a tower of three blocks. When you examine
Batar is a 1-week-old baby born at term who is him, he is walking on tiptoes, and his legs
seen in the ophthalmology clinic because of ‘scissor’ when you lift him up. He is able to feed
cataracts. He has a head circumference of 32 cm himself.
(normal range 32.5–37 cm) and weight of
2.3 kg, mild jaundice, pallor and moderate 4.10.4
hepatosplenomegaly. He has been referred for Alan is a 3-year-old boy who developed a
further hearing assessment as he failed his preference for using his left hand at 7 months of
newborn hearing test. His mother had a mild age. He learnt to sit at 9 months and walked at
flu-like illness during pregnancy. 20 months. When he runs, he holds his right arm
flexed and limps with his right foot. On
4.9.4 examination, his right upper and lower limbs are
Dorcus, a 9-month-old infant, attends the clinic stiff, with increased reflexes.
because of unusual movements. She has
developed episodes of suddenly throwing her 4.10.5
head and arms forward. These occur in repetitive Ronaldo is a 6-year-old boy. He attends the
bursts. She was able to sit and babble but has outpatient department as his teacher has had
stopped doing so. some concerns. He has recently started school
and has been noted to be unsteady on his
4.9.5 feet. He has to walk and run with his legs quite
Darren is a 3-year-old boy who has difficulty wide apart to stop himself from falling over.
climbing stairs. He always needs to hold on to the His teacher also reports that he finds it difficult
railings or to have a supporting hand. He walked to grip a pen and to write because of
unsupported at 14 months. His development is unsteadiness.
otherwise normal. On examination the power in
his legs is reduced, he is somewhat hypotonic but 4.11
his reflexes are normal. Which of the following health professionals
(A–H) involved in the care of a disabled child
4.10 would be of MOST help to the children described
Considering these types of cerebral palsy, choose below?
the type of movement disorder to fit the following
scenarios. Each type of cerebral palsy [A–E] can be A. Dietician
used once, more than once, or not at all. B. Occupational therapist
C. Paediatrician
A. Dyskinetic D. Physiotherapist and occupational therapist
B. Ataxic E. Psychologist
C. Spastic diplegia F. Social worker
D. Spastic hemiplegia G. Specialist health visitor
E. Spastic quadriplegia H. Speech and language therapist
4.10.1 4.11.1
Moses is a 5-year-old boy who failed to attain his Adrianna is a 4-year-old girl who has recently
developmental milestones from shortly after moved from Poland to the UK. She has severe
birth. Currently, he cannot roll or talk, but he can learning difficulties and attends a special nursery.
smile. His mother complains it is difficult to dress She has epilepsy, which is difficult to control, and
him as both his arms and legs are stiff. On is on two different anti-epileptic drugs. She
examination, his left and right upper and lower continues to have seizures despite this
16 limbs are stiff and hyperreflexic. He has a medication. She does not have an underlying
primitive grasp reflex in both hands. diagnosis.
4.11.2 increasingly difficult to move around the
Frankie is a 6-month-old infant. classroom.
She has always struggled to gain weight. On
feeding she often had choking episodes 4.11.6
which led to two episodes of pneumonia. She Gloria is a 22-month-old girl whose health visitor
subsequently needed to have a nasogastric tube is concerned because she is still only babbling
and gastrostomy so she could be fed. Her and says no distinct words. She is able to walk,
mother wants to start trying to feed her some scribbles with crayons, and feeds herself with a

Developmental problems and the child with special needs


solid food. spoon.

4.11.3 4.11.7
Sian is 2 years old. She is being followed up for Cathy is 15 years old and was in a road traffic
growth faltering. All her investigations have accident. She spent a week in intensive care and
come back normal but she is still not gaining needed an operation on her spine. She is
adequate weight. She drinks a lot of dilute currently not able to walk and has been shown
squash but her mother complains she will not how to use a wheelchair. Her parents are
eat any of the food she gives her. desperate to get her back home. They live in a
town house where her bedroom is on the first
4.11.4 floor, and therefore her father would need to
Thomas is a 9-year-old boy who has suspected carry her up the stairs.
Asperger syndrome. He has problems interacting
with his siblings and classmates. His academic 4.11.8
performance at school is poor. He has a very Jake is a 20-month-old boy who burnt himself on
strict daily routine and becomes very upset if this a radiator whilst playing unsupervised. He
is broken. He sleeps poorly at night. attended the Accident and Emergency
department where analgesia was given and
4.11.5 dressings applied. He was seen by a paediatrician
Bilal is a 5-year-old boy who has Duchenne who performed a more detailed assessment for
muscular dystrophy. This presented with child abuse. There were no concerns and the
weakness and easy fatiguability when walking. child was discharged home to be seen later in
He is in mainstream school and is finding it the burns clinic.

17
D. 4–8 years
Answers: Single Best Answer Only very subtle problems will emerge in early
school years.
4.1

4 A. Asperger syndrome
Children with Asperger syndrome have similar
E. Above 8 years
Unusual. Cerebral palsy is the result of a fixed
insult usually in early life.
but less severe social impairments and near-
normal language development. 4.4
Developmental problems and the child with special needs

B. Attention deficit hyperactivity disorder A. Does not wave bye-bye


Attention deficit hyperactivity disorder also Usually this skill develops at about 1 year.
presents with difficult behaviour but there is a B. Inability to crawl
triad of attention deficit, hyperactivity and Children begin to crawl at around 8 months but
impulsivity. this would not be a worrying sign at this stage.
C. Autism spectrum disorder C. Inability to use sounds discriminately to
Correct. Autism is a triad of impaired social parents
interaction, speech and language disorder, and Functional expressive language begins at about
ritualistic and repetitive behaviour. 1 year.
D. Developmental co-ordination disorder (also D. Left-hand preference
known as dyspraxia) Correct. Fortuna has developed a preference for
There are no signs of developmental using her left hand at 8 months. Development of
co-ordination disorder reported. hand preference before 1 year of age is
E. Expressive language disorder abnormal.
Here language alone is affected. E. None of the above
Although examination technique suggests that
4.2 this is frequently the correct option, it is not in
A. 0–12 months this case.
Functional language has not developed yet.
4.5
B. 12–24 months A. Hearing test
Concerns may emerge at this stage but Correct. Speech delay can be due to hearing
problems are less obvious and overdiagnosis is a impairment and this should be assessed first
problem. prior to referring her to a specialist.
C. 2–4 years B. Assessment by a team specializing in autism
Correct. Autism spectrum disorder spectrum disorders
usually presents at this age because this Whilst autism is typified by expressive language
is when language and social skills rapidly difficulties this is only part of the problem.
develop.
C. Reassure the health visitor
D. 4–8 years The problem may resolve spontaneously but
It is unusual to get to school before problems further action is required if no clear words have
are noticed but this does occasionally emerged by 18 months.
happen.
D. Refer to an ear, nose and throat surgeon
E. Above 8 years Grommets for conductive hearing loss might be
Minor symptoms and conditions such as the solution but it would be important to check
Asperger syndrome may present later. hearing first.

4.3 E. Refer to a paediatrician for a full


A. 0–12 months developmental assessment
Correct. Cerebral palsy most often presents The service would be impossibly busy if all
during this time when acquisition of motor skills children with speech delay at this age were
occurs most rapidly. immediately referred.

B. 12–24 months 4.6


As the child gets older, the increased tone A. Mild conductive hearing loss in the right
becomes more evident. ear
Correct. The audiogram shows mild conductive
C. 2–4 years hearing loss in the right ear. It is mild at 25–
Most children with cerebral palsy have presented 39 dB hearing loss and conductive as high-
18 prior to this but very mild hemiplegia can be frequency hearing is relatively preserved and
missed. bone conduction (same for both ears) is normal.
B. Mild sensorineural hearing loss in the right
ear Answers: Extended Matching
The bone conduction is unaffected.
4.9.1
C. Mixed hearing loss in the right ear F. CT or MRI scan of the brain
The bone conduction is unaffected. Clarissa has spastic hemiplegic cerebral palsy.
She has delayed motor milestones, and a
D. Moderate sensorineural hearing loss in the
preference for using her left side from an earlier
right ear

Developmental problems and the child with special needs


age than expected. In most instances a MRI scan
Moderate hearing loss requires a loss of
would be performed rather than a CT scan, but
40–69 dB.
the family may have to wait a little longer for this
E. Severe conductive hearing loss in the right to be undertaken and it will require sedation.
ear
Severe hearing loss requires a loss of 70–94 dB. 4.9.2
Hearing loss is profound if greater than 95 dB. B. Chromosome karyotype
Geoffrey is likely to have Down syndrome,
4.7 trisomy 21. See Box 9.1 and Fig. 9.2. This can be
A. Mild conductive hearing loss in both ears confirmed with a karyotype or DNA FISH analysis.
Mild hearing loss would show a reduction in The latter has the advantage of being quicker
hearing of 25–39 dB and bone conduction would and results can be available in 1–2 days. The
be spared. main disadvantage of FISH analysis compared
with karyotyping is that it gives less information
B. Mild sensorineural hearing loss in both ears
about all of the chromosomes being studied. For
Mild hearing loss would show a reduction in
example, a typical prenatal FISH test will tell you
hearing of 25–39 dB.
how many of the number 13, 18, 21, X and Y
C. Mixed hearing loss in both ears chromosomes are present (i.e., whether there are
There would be a bigger air–bone gap. two copies or three) but will not give you any
information about any of the other
D. Severe conductive hearing loss in both ears chromosomes or any information about the
Bone conduction here is equally affected. actual structure of chromosomes. Therefore a
E. Severe sensorineural hearing loss in both karyotype will be required for counselling. By 10
ears months and with the clinical features listed, a
Correct. Severe sensorineural loss in both ears. diagnosis of Down syndrome is highly likely and
The hearing loss is severe (70–94 dB) to therefore it makes more sense to explain this to
profound (>95 dB) and sensorineural, as there is the family and arrange a chromosome karyotype.
no significant air–bone gap.
4.9.3
4.8 C. Congenital infection screen
A. Cataract This infant has features of a congenital infection,
Correct. This picture (Fig. 4.3) shows a white or including growth restriction, anaemia,
milky coloured object in the pupil. This is due to microcephaly and hepatosplenomegaly. If he is
opacification of the lens (congenital cataract). confirmed to have congenital cytomegalovirus
The red reflex would be absent. infection, then treatment with ganciclovir would
be indicated as this has been shown to improve
B. Conjunctivitis outcomes.
Conjunctivitis would present with normal
development and inflamed conjunctivae. 4.9.4
H. EEG
C. Corneal trauma Dorcus is described as having infantile spasms.
Corneal trauma would be difficult to see without These typically appear as rapid extensions or
fluorescein eye drops. flexions (salaam spasms). Her milestones have
D. Retinopathy of prematurity regressed. An EEG is required to identify
Retinopathy of prematurity is increased hypsarrhythmia. See Table 29.1 and Fig. 29.3 in
vascularization of the retina, which may be Illustrated Textbook of Paediatrics.
associated with excessive oxygen therapy in
4.9.5
premature infants. There may be loss of red
E. Creatine kinase
reflex on ophthalmology, but the lesion is at the
He may have Duchenne muscular dystrophy or
back and not the front of the eye.
one of the wide range of muscular dystrophies.
E. Vitamin A deficiency Checking his creatine kinase would be the initial
Vitamin A deficiency is an important cause of investigation as it is raised in muscular
blindness in developing countries. It presents at dystrophies. See Fig. 29.5 in Illustrated Textbook 19
an older age. of Paediatrics.
4.10.1 support, or work closely with the speech and
E. Spastic quadriplegia language therapist.
There is stiffness, hyperreflexia, and persistence
of primitive reflexes. He has quadriplegia as all 4.11.3
A. Dietician
4 four limbs are affected, with the arms severely
affected. Refer to Fig. 4.5 in Illustrated Textbook The most common cause of faltering growth is
inadequate calorie intake and dietician
of Paediatrics.
involvement is invaluable.
Developmental problems and the child with special needs

4.10.2
A. Dyskinetic 4.11.4
In this instance, caused by kernicterus because of E. Psychologist
his severe early jaundice. He has dystonic An educational psychologist will provide
movements and muscle spasms. cognitive testing and advice on education and
behaviour management. This can be helpful in
4.10.3 obtaining the best outcomes.
C. Spastic diplegia
Hassan’s lower limbs are much more affected 4.11.5
than his upper limbs; his feet are extended from D. Physiotherapist and occupational
markedly increased tone, causing him to walk on therapist
tiptoes, which also causes his legs to cross when Physiotherapist, usually in conjunction with
he is lifted up, i.e. ‘scissoring’. occupational therapist, assist with balance and
mobility problems, prevention of contractures
4.10.4 and scoliosis and advise on use of mobility aids
D. Spastic hemiplegia and orthoses.
Alan has a right hemiplegia.
4.11.6
4.10.5 H. Speech and language therapist
B: Ataxic A hearing test first is always required but if there
He is ataxic. Most cases are genetically is no hearing impairment, referral to speech and
determined and this is a relatively rare sub-type. language therapy team will be indicated.
4.11.1 4.11.7
C. Paediatrician B. Occupational therapist
She requires detailed assessment and An occupational therapist would assess home
investigation and management of her medical suitability and whether additional aids are
problems as well as coordination of input from required. A social worker may be needed if
therapists and other agencies. This requires financial assistance is required.
specialist expertise and the experience of a
paediatrician will be required. 4.11.8
G. Specialist health visitor
4.11.2 A healthcare professional should visit the family
H. Speech and language therapist home to ensure that it is safe and to offer the
A speech and language therapist will assist with family advice and support that may be required.
oro-motor coordination to establish feeding. In In this case a specialist health visitor would be
some centres a dietician would provide this the most appropriate professional.

20
5

Care of the sick child and young person

200 × 109/L and that there are blast cells. You


Questions: Single Best Answer ring Daniel’s parents at home and tell them they
need to come to the oncology ward at the
5.1 hospital as the results of his blood tests are
Molly is 18 months old and needs to be admitted abnormal. They ask you what the abnormalities
to the paediatric ward in the district hospital. are and you tell them you will explain more
There is one paediatric ward in the hospital. Her when they come in. You ring the consultant who
mother is concerned whether they will be says he will come to the hospital to speak to the
geared to caring for such a young child. parents. When the parents arrive you ask them
What is the most common age for children to be to wait 20 minutes until the consultant arrives
admitted to hospital? and he will explain more.

Select one answer only. What is least ideal about the situation?

A. Less than 1 year Select one answer only.


B. 1–3 years A. The family had to wait for the consultant,
C. 3–5 years so you should have told them the diagnosis
D. 5–10 years B. You asked the family to come to an
E. 10–16 years oncology ward rather than the paediatric
assessment unit
5.2 C. You did not answer the parent’s questions
Rhys is a 4-year-old boy from Wales who is on the phone. You should have told them
referred acutely to the paediatric team as he the diagnosis
has developed pneumonia. He has also had D. You have asked Daniel to come out of hours
increasing weakness in his legs. He has been rather than the following morning
admitted to the ward. Investigations reveal he E. You have told the parents over the phone
has Duchenne muscular dystrophy. the blood test was abnormal rather than in
Who is the most appropriate person to inform person
the parents about his diagnosis?
Select one answer only. Questions: Extended Matching
A. General practitioner
5.4
B. Junior doctor
For each of the scenarios below select the most
C. Nurse looking after patient
appropriate way to administer the medication
D. Senior doctor
required from the list (A–I) below. Each option
E. Sister on ward
may be used once, more than once, or not at all.
5.3 A. Inhaled via large-volume spacer device
Daniel is a 15-year-old boy who went to see his B. Intradermal
general practitioner as he has been tired and not C. Intramuscular
‘quite right’ for the last 2 months. The general D. Intravenous
practitioner obtained a full blood count to see if E. Liquid
he was anaemic. The haematology laboratory F. Nebulized
phone you, a newly qualified doctor, at 6 PM in G. Subcutaneous
the hospital saying they have received a full H. Tablets
blood count on Daniel and his white cells are I. Topical
5.4.1 5.5
Noel is a 7-day-old baby who is admitted to Each of the children listed below is in pain. Select
hospital with a fever. He is feeding 3–4-hourly the most appropriate next step in their pain
from the breast and has no obvious source for management from the list (A–K) below. Each
5 his fever. He was born by vaginal delivery
following a normal pregnancy.
option may be used once, more than once, or
not at all.
5.4.2 A. Distraction
Care of the sick child and young person

Luke is a 3-year-old boy who is seen in the B. Epidural analgesia


paediatric assessment unit. He is eating and C. Inhalation of nitrous oxide
drinking adequately. He has no other medical D. Intranasal diamorphine
history and is not on any medications. He has a E. Intravenous morphine via a nurse-controlled
fever, which is associated with tachypnoea and pump
crepitations at the right base. His oxygen F. Intravenous patient-controlled analgesia
saturation is 96% in air. with morphine
G. Regular oral long-acting morphine with
5.4.3 rapid action oramorph for breakthrough
Mark is a 15-year-old boy who is seen in the pain
emergency department with an exacerbation of H. Regular oral nonsteroidal anti-inflammatory
wheeze. He is able to speak in short sentences drug (NSAID)
but has an oxygen requirement and needs to be I. Regular oral paracetamol
given salbutamol. J. Regular oral weak opioid, e.g. codeine
5.4.4 K. Topical anaesthetic
A mother has just given birth to a baby girl in a
5.5.1
hospital in England. The midwife wants to give
Charlie is a 13-month-old boy who is attending
her vitamin K.
the community centre to receive his measles,
5.4.5 mumps, and rubella vaccination. He has no
Xu-Li is a 1-year-old with significant eczema medical problems, has no allergies, and is not
(Fig. 5.1). She has no other medical problems currently on any medication.
and is not on any medication. You decide to
prescribe medication. 5.5.2
Victoria is 6 months old. She attends the
outpatient department for a blood test.

5.5.3
Fiona is 3 years old. She has come from her
home in Northern Ireland to England to have a
liver transplant. She is day 1 postoperative.

5.5.4
Jake is 3 years old. He has had a hernia repair
and has just come back to the ward. You are
asked to write-up some pain relief by the nurses.

5.5.5
Noah is a 7-year-old boy. He attends the
Accident and Emergency department in severe
pain. He has been involved in a road traffic
accident and has a compound fracture of his
femur. He has had several episodes of vomiting.
He is extremely agitated. He has no intravenous
access.

5.5.6
Zac is a 10-year-old boy who was diagnosed with
Ewing sarcoma. He has severe pain from
metastatic disease, which is unresponsive to
therapy. You ask the palliative care team to help
with his management as the medications are
insufficient. You have tried regular, high-dose
paracetamol without success. He has acute
22 kidney injury secondary to his chemotherapy. He
Figure 5.1 has no allergies.
5.5.7 5.6.1
Achille is a 13-year-old black African boy who As Tolla’s doctor you feel she should be told she
has sickle cell disease. He presents with pain in is HIV positive, as you will be able to offer her
his left leg. He has already taken paracetamol more support and coping strategies if she knows
without effect. He has no other medical more about her diagnosis.
problems and has no allergies.
5.6.2
As Tolla’s doctor you feel she should not be
5.6

Care of the sick child and young person


told the diagnosis as this will potentially expose
Choose from the options which best describes
her and her family to more harm due to the
the reason for the doctor’s actions in managing
stigma attached to having HIV. The knowledge
Tolla, an 11-year-old girl who has recently arrived
that she is HIV positive may also affect her
in the UK with her mother. They are originally
self-esteem.
from Uganda. Tolla has been admitted to the
paediatric ward with a chest infection. Her 5.6.3
mother is HIV (Human immunodeficiency virus) As Tolla’s doctor you feel she has a right to be
positive but has not told any of her family involved in her own treatment and therefore
members. Her husband, Tolla’s father, died of an should be told she is HIV positive.
acquired immune deficiency syndrome-related
illness 6 months ago. Tolla has been tested for 5.6.4
HIV and her test result has come back positive. Her mother has asked you to tell her the HIV test
Her mother does not wish her to be told, as she is negative and as the doctor you do not feel this
is still very upset about the death of Tolla’s is ethically right.
father. 5.6.5
Each option [A–H] may be used once, more than The hospital manager has asked you not to start
once, or not at all. antiretroviral therapy as the family do not have
legal status to stay in the UK and cannot afford
A. Autonomy the medication. You start the therapy against
B. Beneficence this advice.
C. Duty
D. Justice
E. Non-maleficence
F. Rights
G. Truth-telling
H. Utility

23
C. You did not answer the parent’s questions on
Answers: Single Best Answer the phone. You should have told them the
diagnosis
5.1 You need to give some explanation of why
5 A. Less than 1 year
Correct. The most common age is infants less
Daniel should come back to hospital but it is
better to break the bad news in person rather
than 1 year. Most medical admissions are than over the phone.
emergencies in children under 5 years of age,
Care of the sick child and young person

whereas surgical admissions peak at 5 years of D. You have asked Daniel to come out of hours
age, one-third of which are elective. rather than the following morning
Daniel needs to come to hospital as soon as
5.2 possible because of the high white cell count, so
A. General practitioner the family needed to come out of hours rather
In some instances, the general practitioner will than waiting until morning.
know the family well and may be a great
support. However, they are unlikely to have E. You have told the parents over the phone
looked after a child with Duchenne muscular the blood test was abnormal rather than in
dystrophy before and may not be able to answer person
all the family’s questions. You need to give some explanation of why
Daniel should come back to hospital.
B. Junior doctor
Junior doctors often offer great support and
empathy to families, but families prefer serious Answers: Extended Matching
or complex information to be delivered by a
senior doctor. However, as junior doctors (and 5.4.1
nurses) are likely to be questioned by parents D. Intravenous
before being seen by a senior doctor, it can be In acutely ill neonates and infants, drugs are
difficult to avoid revealing information best given intravenously to ensure reliable and
reserved for an interview with the most adequate blood and tissue concentrations. Fever
appropriate professionals and members of the without a source in this age group is an
family being present. indication for a septic screen and starting
C. Nurse looking after patient intravenous antibiotics. With oral formulations,
The nurse looking after a patient will be able to intake cannot be guaranteed and absorption is
provide support to the family and can often help unpredictable as it is affected by gastric
the family with questions after the diagnosis has emptying and acidity, gut motility and the
been given to them. effects of milk in the stomach.

D. Senior doctor 5.4.2


Correct. Parents say that they would prefer a E. Liquid
senior doctor, in the presence of a nurse, to He has a community-acquired pneumonia but is
communicate serious or complex information to systemically well. For the treatment of
them. uncomplicated pneumonia, oral therapy is as
good as intravenous treatment. He could be
E. Sister on ward given antibiotics in liquid form; he is too young
May provide considerable support to the medical to take tablets. If he was systemically unwell and
and nursing teams and families but most parents had signs of a pleural effusion or empyema,
report that they would prefer serious or complex intravenous therapy would be indicated. (See the
information to be communicated by a senior PIVOT study for more details. Available at: http://
doctor. www.ncbi.nlm.nih.gov/pmc/articles/
PMC2094276/.)
5.3
A. The family had to wait for the consultant, so 5.4.3
you should have told them the diagnosis F. Nebulized
Breaking bad news is always difficult and should Nebulized treatment should be given when
be done by a senior doctor. A 20-minute wait is there is an oxygen requirement, as it is not
not a good reason to tell them the diagnosis. possible to administer supplemental oxygen
whilst treatment is given by inhaler and
B. You asked the family to come to an large-volume spacer.
oncology ward rather than the paediatric
assessment unit If he did not have an oxygen requirement, the
Correct. By telling them to come to an oncology best way to deliver the salbutamol is a metered-
ward you are telling them Daniel has cancer dose inhaler with a spacer, as this is as effective
24 without actually explaining his diagnosis without the extra cost of a nebulizer or the
properly. increased risk of paradoxical desaturation. He
should also be given oral steroids. (See: http:// 5.5.5
openaccess.sgul.ac.uk/2699/1/CD000052.pdf.) D. Intranasal diamorphine
The oral and intravenous routes are not available
5.4.4 in this situation. Inhalation of nitrous oxide is
C. Intramuscular also an effective analgesia when the oral or
Intramuscular (IM) vitamin K is recommended for intravenous route is not available. However, the
all infants to prevent haemorrhagic disease of child needs to cooperate to be able to use it
the newborn. Parents may request oral vitamin K effectively. Most 7-year-olds will not find it

Care of the sick child and young person


as an alternative; although care should be taken particularly helpful unless they already know
to identify why. Some poor evidence synthesis in how to use it. A child in severe pain who is upset
the 1990s raised questions about IM vitamin K will find it very difficult to learn a new skill.
and long-term risk of childhood leukaemia – this
was unsubstantiated and IM vitamin K is now 5.5.6
recommended. As oral absorption is variable, G. Regular oral long-acting morphine with
three doses are needed orally over the first 4 rapid action oramorph for breakthrough pain
weeks of life to achieve adequate liver storage, Pain from metastatic disease can be very
but protection cannot be guaranteed. [See: severe. NSAIDs should be avoided in renal
http://evidencebasedbirth.com/evidence-for-the failure.
-vitamin-k-shot-in-newborns/ for a very helpful
(if somewhat US centric) description of why IM 5.5.7
vitamin K is preferable to oral vitamin K and a H. NSAID
discussion of the evidence concerning childhood Achille needs a regular NSAID. Sickle cell crises
leukaemia.] can be very painful and there may be a need to
escalate to an oral opioid medication. However,
5.4.5 opioid dependence can become a problem in
I. Topical patients with sickle cell disease.
Most medication for eczema is topical. Emollients
are the first line of therapy (if it is dry, then wet 5.6.1
it). These should be administered topically and B. Beneficence
applied regularly. The skin needs to be kept The child’s interest is paramount. In the UK,
moist to prevent scratching. this is enshrined in the Children Act 1989 and
A good way of assessing how often it is applied the UN Convention on the Rights of the Child.
is to ask how quickly the pot of emollient was Beneficence is the positive obligation to do good
finished. (this principle has been part of medical ethics
since the Hippocratic Oath).
5.5.1
A. Distraction 5.6.2
Distraction is very effective for reducing pain and E. Non-maleficence
distress during minor procedures. Do no harm (psychological and/or physical). This
principle has been part of medical ethics since
5.5.2 the Hippocratic Oath.
K. Topical anaesthetic
Distraction would also be useful but applying 5.6.3
some topical anaesthetic cream is effective at A. Autonomy
preventing pain from a blood test. Respect for individuals’ rights to make informed
and thought-out decisions for themselves in
5.5.3 accordance with their capabilities.
E. Intravenous morphine via a nurse-
controlled pump 5.6.4
Liver transplantation is a major operation and G. Truth telling
the child would experience considerable pain. At This and confidentiality are important aspects of
3 years of age she is too young for a patient- autonomy that support trust, essential in the
controlled pump. doctor–patient relationship.
5.5.4 5.6.5
I. Regular oral paracetamol C. Duty
Paracetamol is a useful analgesic especially if Duty is the moral obligation to act irrespective of
given regularly. Nonsteroidal anti-inflammatory the consequences in accordance with moral
drugs (NSAIDs) should be given as second line; laws, which are universal, apply equally to all,
ibuprofen can be helpful if there is no contra- and which respect persons as autonomous
indication. Morphine would not normally be beings. There is also a degree of justice to this
required after a hernia repair. situation.
25
6

Paediatric emergencies

Where is the most appropriate position


Questions: Single Best Answer on the chest (Fig. 6.1) to do cardiac
compressions?
6.1
The paediatric team is resuscitating a 3-month- Select one answer only.
old boy who is in pulseless electrical activity. He
was discovered to be blue and lifeless when his A. A
parents went to wake him in the morning. The B. B
airway has been secured and despite bag valve C. C
mask ventilation the child remains blue. Cardiac D. None of the above
compressions are given. E. All of the above

A B C
Figure 6.1

6.2
You are in the Acute Assessment Unit and see
David, a 15-month-old boy, who has a fever of
38.5° C. He has had a runny nose, cough and a
fever for 3 days. Since this morning he has slept
and has been difficult to wake. His heart rate is
raised. He has a rash (Fig. 6.2) scattered over his
legs which does not disappear with pressure.
Which of the following is the most likely
diagnosis?
Select one answer only.
Figure 6.2
A. Acute lymphoblastic leukaemia
B. Henoch–Schönlein purpura
C. Immune thrombocytopenia
D. Non-accidental injury
E. Septicaemia
6.3 D. Remove wet clothing/towels and dry the
A 3-year-old boy who is unconscious arrives in baby vigorously
the Emergency Department. You manage his E. Stimulate the baby and shout for help
airway, breathing and circulation. His blood
glucose is normal. On examination you note his
pupils are as in Fig. 6.3. His temperature and 6.6
other vital signs are otherwise normal. Mohammed, aged 8 months, has been
vomiting and off his feeds for 2 days. Initially,

Paediatric emergencies
he had episodes of crying uncontrollably,
drawing his legs up into his abdomen as if in
pain, and appeared fractious. His mother gave
him some oral rehydration solution, but his
vomiting continued and he has become
lethargic. On admission to hospital he is in
shock.
Figure 6.3 What is the most likely diagnosis?
Select one answer only.
A. Gastroenteritis
What is the most likely cause?
B. Intussusception
Select one answer only. C. Malrotation and volvulus
D. Meckel diverticulum
A. Third nerve lesion E. Strangulated hernia
B. Severe hypoxia
C. Hypothermia
D. Tentorial herniation 6.7
E. Opiate poisoning Mohammed, aged 8 months, presented with the
clinical scenario described in question 6.7. He
6.4 weighs 8 kg. He needs a bolus of normal saline
You are called to see a 3-year-old boy with a 0.9% to treat his shock.
high fever. The nurse is worried that he is very
sleepy. As you walk into the resuscitation room What volume of fluid would you give initially?
he makes no spontaneous response. You try Select one answer only.
calling his name but he makes no response. On
stimulation, his eyes open, he cries and he raises A. 40 ml
his hand and pushes your hand away. B. 160 ml
C. 320 ml
What is this child’s Glasgow Coma Score (GCS)? D. 680 ml
Select one answer only. E. 800 ml

A. 8
B. 9 6.8
C. 10 Mohammed, aged 8 months, has presented with
D. 11 the clinical scenario described in Questions 6.6
E. 12 and 6.7. He has received the fluid bolus of
normal saline 0.9%, which has improved his
6.5 condition. From his presentation you suspect he
Ryan, aged 10 months, is rushed to the children’s is 10% dehydrated. You receive his laboratory
emergency department after being found results, which reveal a plasma sodium of
submerged in the bath. His mother runs 138 mmol/L (within the normal range). His
screaming into the department saying ‘Help my continuing fluid loss from vomiting is small and
baby, please’. can be ignored.
Which is the next most appropriate step? What is Mohammed’s total fluid requirement for
the initial 24 hours? He weighs 8 kg.
Select one answer only.
Select one answer only.
A. Commence chest compressions in a ratio of
15 : 2 A. 160 ml
B. If the child is not breathing, commence bag B. 320 ml
and mask ventilation C. 800 ml
C. Place the child onto the examination couch D. 880 ml 27
and put his head into the neutral position E. 1600 ml
6.9 A. Feet to foot of cot
You are called to the resuscitation room where B. Keeping baby in parent’s room until 6
there is a 6-year-old child who has arrived by months of age
ambulance. The child has been having a C. Keeping room cool to prevent
6 generalized seizure for 15 minutes. The
ambulance crew gave a dose of buccal
overheating
D. Parents not smoking in the same room as
midazolam 5 minutes ago. The emergency infant
doctor has maintained the airway and has E. Supine sleeping
Paediatric emergencies

applied oxygen. His capillary refill time is less


than 2 seconds and his heart rate 120 beats/min.
What is the next most appropriate management Questions: Extended Matching
step?
6.13
Select one answer only. For each of the following patients seen in the
A. Administer further anticonvulsant emergency department select the most
B. Check blood glucose level appropriate next step in the management
C. Gain intravenous access plan from the following list (A–L). Each
D. Request senior review option may be used once, more than once,
E. Start bag and valve mask ventilation or not at all.
A. Airway opening manoeuvres
6.10 B. Bag and mask ventilation
Seb, a 2-year-old boy, was at his cousin’s C. Check blood glucose
birthday party. His mother noticed that he has D. Check conscious level [alert, voice, pain,
suddenly developed a widespread urticarial rash unresponsive (AVPU)]
and has also become flushed in the face. His vital E. Check pupils
signs are normal and he has no respiratory F. Commence cardiac compressions using
compromise. both hands, one hand on top of
the other
Which medication would you give?
G. Commence cardiac compressions using the
Select one answer only. encircling method
H. Commence cardiac compressions using one
A. Intramuscular adrenaline hand on the sternum
B. Intramuscular antihistamine I. High-flow oxygen therapy
C. Intravenous hydrocortisone J. Intravenous access
D. Oral antihistamine K. Intravenous fluid
E. Oral corticosteroid L. Secondary survey

6.11 6.13.1
Jenny, a 3-year-old girl, was at a village fete. She Nathaniel, a 4-year-old boy, is brought to
suddenly developed swollen cheeks and lips and hospital with shortness of breath. He is able to
a widespread urticarial rash. She is rushed to the talk but has oxygen saturation of 90%. His
nearby general practice surgery, where it is capillary refill time is less than 2 seconds.
noted that her breathing is very noisy. She is
6.13.2
distressed and frightened. On auscultation she
Kelsey, a 2-year-old girl, is found unconscious
has widespread wheeze.
in the garden. When she is bought into the
Which medication would you give first? resuscitation room she is gasping and
moaning.
A. Intramuscular adrenaline
E. Intramuscular antihistamine 6.13.3
C. Intravenous hydrocortisone Ahmed, aged 2 months, is found by his
D. Oral antihistamine mother to be pale and floppy in his cot. The
B. Oral corticosteroid paramedics are giving bag and valve mask
ventilation when he arrives in the resuscitation
6.12 room and his chest is moving well. His heart rate
There has been a dramatic decline in the is 40 beats/min.
incidence of sudden infant death syndrome in
6.13.4
the UK.
Daniel, age 10 years, has diabetes mellitus
Which of the following is the single and has been playing football at his friend’s
28 most important factor responsible for this house. He has been brought to the
decline? emergency department as he has become
confused and is sweaty. He walks into the receiving high-flow oxygen, her breathing is
department. regular, and the cardiac monitor shows a heart
rate of 100 beats/min. She is unresponsive to
6.13.5 painful stimuli, as she does not flinch when her
Aisha, a 3-year-old girl, is bought to hospital by blood glucose is checked.
the paramedics as she has had a seizure. She is

Paediatric emergencies

29
pushing a hand away suggests localization.
Answers: Single Best Answer Remember, the lowest possible score in each
domain is 1 (rather than 0).
6.1
B. 9
6 B. B
Correct. In infants the heart is lower in relation Correct. The Glasgow Coma Scale is made up of
three parts. Best motor response (a score of 1–6 is
to the external landmarks than in older
children or adults. The area of compression possible); Best verbal response (a score of 1–5 is
Paediatric emergencies

over the sternum should be one finger possible) and Best eye opening (1–4 is possible).
breadth below an imaginary line between Here the child scores 5 for best motor response
the nipples. (localizes pain) and 2 each for best verbal and
best eye response.
6.2
A. Acute lymphoblastic leukaemia C. 10
This is a short history of the child being unwell. The verbal score here would have been higher if
In acute lymphoblastic leukaemia you would the child had responded with vocal sounds or
expect a longer history and other characteristic words and for eye opening if he had responded
clinical features. to sounds rather than pain.

B. Henoch–Schönlein purpura D. 11
The purpuric rash is localized to the legs and Remember that there is a separate scoring
buttocks, Henoch–Schönlein purpura is system for children under 4 years of age.
associated with abdominal pain and joint pain E. 12
but not with fever and being severely ill. This child is only responsive to pain. Using the
C. Immune thrombocytopenia AVPU (alert, voice, pain, unresponsive) scoring
With immune thrombocytopenia the children are system, a score of P usually corresponds to a GCS
usually well. of 8 or 9.

D. Non-accidental injury 6.5


Non-accidental injury is not suggested by this A. Commence chest compressions in a ratio of
acute febrile illness. 15 : 2
Chest compressions may be required but this is
E. Septicaemia
further down the resuscitation algorithm.
Correct. He has a purpuric rash, with lesions of
variable size. In a febrile child, meningococcal B. If the child is not breathing, commence bag
septicaemia is most likely. This is not invariably and mask ventilation
accompanied by meningitis. This is done after stimulating the child, shouting
for help and opening the airway.
6.3
A. Third nerve lesion C. Place the child onto the examination couch
Third nerve lesions and tentorial herniation and put his head into the neutral position
would cause a unilaterally dilated pupil. This will need to be done, but is not the first
step.
B. Severe hypoxia
Severe hypoxia would cause dilated pupils. D. Remove wet clothing/towels and dry the
baby vigorously
C. Hypothermia Drying is an essential first step in a newborn
With hypothermia the child’s temperature would infant but not the first thing to do here.
be low and it causes dilated pupils.
E. Stimulate the baby and shout for help
D. Tentorial herniation Correct. All resuscitation algorithms ensure that
Third nerve lesions and tentorial herniation the patient is assessed in a sequential manner,
would cause a unilaterally dilated pupil. adopting an Airway, Breathing and Circulation
E. Opiate poisoning approach. The paediatric life support algorithm
Correct. Bilateral, pinpoint pupils (as in Fig. 6.3) states that you: 1. Check safety (not strictly
with coma can be caused by a pontine lesion or necessary within hospital), stimulate and shout
opiate poisoning. Opiate poisoning may occur in for help. Calling for help early on in these
homes with substance abusers or adults on situations is paramount as you need many
methadone. people for resuscitation.

6.4 6.6
A. 8 A. Gastroenteritis
30 The Glasgow Coma Scale is shown in Table 6.1 Less likely, because episodes of crying
below. This child is scoring more than 8 as uncontrollably with drawing of legs up into his
Table 6.1 Glasgow Coma Scale, incorporating Children’s Coma Scale
Glasgow Coma Scale Children’s Coma Scale
(4–15 years) (<4 years)
Response Response Score

Eye opening Spontaneous Spontaneous 4

Paediatric emergencies
To sound To sound 3
To pressure To pain 2
None No response 1

Best motor response Obeys commands Obeys commands 6

Localizes pain Localizes pain 5


Normal flexion Flexion to pain 4
Abnormal flexion Abnormal flexion 3
(decorticate posture)
Extension Abnormal extension 2
(decerebrate posture)
No response No response 1

Best verbal response Oriented Talks normally, interacts 5


Confused Words 4
Words Vocal sounds 3
Sounds Cries 2
No response No response 1
A score of <8 out of 15 means that the child’s airway is at risk and will need to be maintained by a manoeuvre or
adjunct.

abdomen as if in pain are characteristic of Paediatrics for initial fluid resuscitation in shock.
intussusception. In trauma or diabetic ketoacidosis smaller
aliquots are given.
B. Intussusception
Correct. Intussusception is the most likely cause
6.8
of the pain and shock. Although this could be a
E. 1600 ml
strangulated hernia, this should be evident on
Correct. Mohammed’s fluid requirement is
clinical examination. Follow an Airway, Breathing,
calculated by adding
Circulation approach, get senior help and speak
to the radiologist. The diagnosis might be • Deficit: 10% of 8 kg = 800 ml
obvious on ultrasound. • Maintenance: 100 ml/kg per 24 hours =
800 ml
C. Malrotation and volvulus • Continuing losses: 0 ml
Less likely, because episodes of crying
Total = 1600 ml
uncontrollably with drawing of legs up into his
See Table 6.1 in Illustrated Textbook of
abdomen as if in pain are characteristic of
Paediatrics for maintenance fluid requirements
intussusception. Bile-stained vomiting is often
at different weights.
present in malrotation. However, the diagnosis
must be considered.
6.9
D. Meckel diverticulum A. Administer further anticonvulsant
Meckel diverticulum tends to present with After 10 minutes it is recommended to give a
bleeding per rectum as well as abdominal pain. further dose of an anticonvulsant if still having a
Blood loss is rarely so severe to result in shock. seizure.
E. Strangulated hernia
Although this could be a strangulated hernia, B. Check blood glucose level
this should be evident on clinical examination. Correct. This is the most appropriate next step
as, if the patient is hypoglycaemic, the only
6.7 treatment to stop the seizure would be to
B. 160 ml administer glucose. Ideally, intravenous glucose
Correct. This is 20 ml/kg initially, repeated as will be given but if access is not achieved, then 31
necessary. See Fig. 6.9 in Illustrated Textbook of glucose gel buccally.
C. Gain intravenous access C. Intravenous hydrocortisone
It is difficult to do this whilst the child is having a This should only be given after immediate
seizure. It can be very helpful but there are more treatment of the upper airway obstruction with
important treatment steps. This is often done at intramuscular adrenaline. Also, her upper airway
6 the same time by other team members.
D. Request senior review
obstruction may be further compromised by
the distress of establishing an intravenous
cannula. It takes about 6 hours to have optimal
It is important to make an ABC plus don’t ever effect.
Paediatric emergencies

forget glucose (ABCDEFG) assessment yourself


before seeking senior review. D. Oral antihistamine
Antihistamine alone will be slow to work and
E. Start bag and valve mask ventilation rapid treatment is required.
Although it can be difficult to assess breathing in
a child with seizures, it is usually sufficient to B. Oral corticosteroid
administer oxygen unless breathing stops This child has upper airway obstruction so
completely. This can occur after benzodiazepine is unlikely to be able to take oral medications
therapy but is uncommon after a single dose. and oral steroids would take too long
to work.
6.10
A. Intramuscular adrenaline 6.12
This is not needed as there is no respiratory E. Supine sleeping
compromise. Correct. All the answers have helped reduce the
risk of sudden infant death syndrome, but the
B. Intramuscular antihistamine single most important factor is putting babies to
This is not indicated as it is painful to administer sleep on their backs.
and the child is able to take oral medicines, as
there is no respiratory compromise.
Answers: Extended Matching
C. Intravenous hydrocortisone
This is not needed as this is a mild allergic 6.13.1
reaction and there is no respiratory compromise. I. High-flow oxygen therapy
He has a patent airway as he is able to talk. His
D. Oral antihistamine oxygen saturation is low, so oxygen needs to be
Correct. In children, the most common causes of given. This will improve co-operation and reduce
acute food allergy are ingestion or contact with anxiety.
nuts, egg, milk or seafood. Urticaria and facial
swelling are mild reactions. Immediate 6.13.2
management is with an oral antihistamine (e.g., A. Airway opening manoeuvres
chlorphenamine) and observation over 2 hours She is gasping and moaning, and thus needs her
for possible complications. airway repositioning before commencing bag
and valve mask ventilation.
E. Oral corticosteroid
This is not needed as this is a mild allergic 6.13.3
reaction and there is no respiratory compromise. G. Commence cardiac compressions using
The condition should respond to oral the hands encircling method
antihistamine. Ahmed has a heart rate of less than 40 beats/min
and his airway and breathing are being
6.11 managed. Therefore cardiac compressions need
A. Intramuscular adrenaline to be started. The most effective way in this age
Correct. This child has anaphylaxis, which is life group is the hand encircling technique. The main
threatening as she has both upper airway disadvantage of this technique is that it requires
obstruction (noisy breathing) and at least two rescuers to provide effective, timely
bronchoconstriction (wheeze). Either of these on basic life support.
their own or any signs of shock would be
enough to constitute a diagnosis of anaphylaxis. 6.13.4
Priority is to manage the airway and give oxygen C. Blood glucose
via a non-rebreathe mask. The first medication to The clinical features suggest hypoglycaemia.
give would be intramuscular adrenaline.
6.13.5
E. Intramuscular antihistamine E. Check pupils
This would make the situation worse, as it is Her pupils need checking as her airway,
painful and will not directly treat the upper breathing, circulation, and conscious level have
airway obstruction. already been checked.
32
7

Accidents and poisoning

Questions: Single Best Answer


7.1
What is the most common cause of death
in children aged 1 year to 14 years in
the UK?
Select one answer only.
A. Accidents
B. Congenital heart disease
C. Infectious disease
D. Malignant disease
E. Respiratory disease

7.2
Hamim, a boy aged 3 years, fell 3 metres from a
first-floor balcony on to a concrete path. He
presents to the Emergency Department with his
Figure 7.1
parents who are concerned that he has vomited
several times since the episode. After the fall he
immediately cried out in pain, but appeared to
be all right. His mother reports that he did not
lose consciousness. On examination he is found What does the X-ray show?
to be fully conscious but has a large bruise over
the left parietal region. There are neither focal Select one answer only.
neurological signs nor any other injuries. His A. Frontal bone fracture
heart rate is 110 beats/min, his respiratory rate is B. No abnormalities shown
25 breaths/min and his blood pressure is C. Occipital bone fracture
90/50 mmHg. D. Parietal bone fracture
Which of the following would be the most E. Temporal bone fracture
worrying additional clinical sign?
7.4
Select one answer only. Hamim (as outlined in Question 7.3) is admitted
to hospital for a period of observation.
A. A black eye (bruising around left eye)
B. A fractured nose with deviated septum Eight hours after admission, the nurses note a
C. A runny nose change in his level of consciousness. He is now
D. A temperature of 38.2° C responsive only to painful stimuli; his left pupil is
E. Further enlargement of the parietal bruising dilated although still responsive to light. His
airway, breathing and circulation are satisfactory.
7.3 A CT scan (Fig. 7.2) shows that there is a
Hamim (as outlined in Question 7.2) has a lateral haemorrhage and a skull fracture. He is stabilized
skull X-ray taken (Fig. 7.1). in the resuscitation room.
7.6
Louise (as outlined in Question 7.5) has
chest and abdominal X-rays which show
fractures of the 9th and 10th ribs on the
7 left-hand side.

What is the most important investigation


to perform to establish the cause of her
Accidents and poisoning

condition?
Select one answer only.
A. Abdominal ultrasound
B. Arrange CT head scan
C. Cervical spine X-ray
D. Full blood count
E. Serum creatinine, urea and electrolytes

Figure 7.2 7.7


Roberto is a 2.5-year-old boy. He pulled
a chip pan off the cooker and has been
extensively burnt. He is rushed to the
nearest Children’s Emergency Department.
His airway, breathing and circulation are
Which of the following is the most appropriate
satisfactory. His burns are distributed on
next step in his management?
his body as shown in Fig. 7.3. Most of the
Select one answer only. burnt area is now blistering and mottled in
colour, with a few white areas. Intravenous
A. Clotting studies analgesia is given.
B. EEG
C. Neurosurgical referral What is the estimated area of the burn?
D. Ophthalmology opinion
E. Skeletal survey Select one answer only.
A. 15%
7.5 B. 20%
Louise, aged 4 years, was hit by a car in C. 25%
the local supermarket car park. She is D. 30%
brought to the Emergency Department by E. 35%
ambulance.
An initial assessment shows 7.8
• Airway — talking to mother Roberto (as outlined in Question 7.7) is
• Breathing — receiving oxygen via a much more settled following intravenous
rebreathing circuit, oxygen saturation 99% analgesia.
• Breathing — air entry satisfactory and equal From the list of possible management options
bilaterally, respiratory rate 30/min below, which should be undertaken first?
• Circulation — pulse 160/min, blood
pressure 90/50 mmHg, capillary refill time Select one answer only.
3 seconds
• Disability — alert, but frightened and agitated, A. Commence intravenous 0.9% saline
moving all four limbs B. Cover the burns with sterile dressings
C. Intravenous antibiotics
She has abrasions to her left flank and pain in D. Intubation and artificial ventilation
her left shoulder. E. Place affected areas in cold water
What is the next intervention needed?
Select one answer only. 7.9
Jake is a 3-year-old boy. His brother spilt a
A. Analgesia pan of hot water over him and he has been
B. Blood glucose measurement extensively scalded. He is rushed to the nearest
C. Chest X-ray Emergency Department. His airway and
34 D. Intravenous access breathing are satisfactory. He is tachycardiac,
E. Intubation and ventilation his capillary refill time is 4 seconds and he has
A A

1 1

Accidents and poisoning


2 2 2 2
13 13
112 112 112 112

112 1 112 112 212 212 112

B B B B

C C C C

1 34 1 34 1 34 1 34

Surface area at

Area indicated 0 1 year 5 years 10 years 15 years

A 9.5 8.5 6.5 5.5 4.5


B 2.75 3.25 4.0 4.5 4.5
C 2.5 2.5 2.75 3.0 3.25

Figure 7.3

a low blood pressure. He has 20% burns 7.10


involving his chest, abdomen, and his right arm Solomon, aged 3 years, has been found eating
and hand. some of his pregnant mother’s iron tablets; up to
10 tablets are missing. Their general practitioner
What is the most likely underlying cause for his advised that he should be taken to hospital
shock? directly. On examination in the Emergency
Select one answer only. Department he is found to be talkative, with
no obvious abnormalities. He has no other
A. It is secondary to the pain from his burn medical problems and is not normally on any
B. Jake has shock due to loss of blood plasma, medications.
because of damage to his blood vessels
secondary to his scald What would be the first investigation you would
C. Jake has shock due to loss of red blood cells, perform?
because of damage to his blood vessels Select one answer only.
secondary to his scald
D. Jake has shock due to vasodilation of his A. Abdominal X-ray
blood vessels, secondary to his scald B. Full blood count
E. He has shock due to vasodilation of his C. Clotting studies
blood vessels, secondary to infection E. Liver function tests 35
developing in his scald D. Serum iron
7.11
Your investigation of Solomon (as outlined in Questions: Extended Matching
7.10) suggests a significant ingestion of iron.
7.13
Which of the following would you initiate?
7 Select one answer only.
For each of the following patients select the
most likely poison from the following list (A–J).
Each option may be used once, more than once,
A. Forced alkaline diuresis or not at all.
Accidents and poisoning

B. Intravenous desferrioxamine
A. Alcohol
C. Intravenous N-acetylcysteine
B. Button battery ingestion
D. Intravenous naloxone
C. Cannabis
E. Intubate and ventilate
D. Digoxin
E. Ecstasy (MDMA)
7.12
F. Iron
Rory is a 4-year-old boy. He was playing with his
G. Paracetamol
pocket money when he accidentally swallowed
H. Petroleum distillates
one of the coins. His father brings him to the
I. Salicylates (aspirin)
emergency department where an X-ray is
J. Tricyclic antidepressants
performed (Fig. 7.4).
7.13.1
Jacob is a 15-year-old boy who is brought to
hospital by ambulance. He was found in the local
park by paramedics after one of his friends
phoned them to say he was in trouble. When the
paramedics arrived there was no one else with
him. He is not coherent and unable to walk. His
blood glucose level is low.
7.13.2
Ruby is a 15-year-old girl who attends the
Emergency Department with her father as she
has ‘turned yellow’. When you ask her if she has
taken any medicines, she says no. She is being
bullied at school, which has been going on for
over a year now. On examination, she is
jaundiced and her liver function tests and
clotting are both deranged.
7.13.3
Callum, a 15-year-old boy, is bought into the
Emergency Department with his mother. She
found him in his bedroom with some pills by his
bed. He is disorientated and hyperventilating.
Figure 7.4 7.13.4
Syam, a 2-year-old boy, comes to the Emergency
Department with his father. His mother noticed
that his stools had become black. Syam lives at
home with his parents, grandparents, and two
Where, anatomically, is this swallowed foreign siblings. Syam’s mother is currently pregnant. On
body most likely to have lodged? examination, Syam is cardiovascularly stable.

Select one answer only. 7.13.5


Adrian is a 4-year-old who has been staying with
A. Cricopharyngeus muscle his grandmother over the weekend. He is bought
B. Lower oesophageal sphincter to the Accident and Emergency department by
C. Region where the aortic arch and carina his grandmother who is worried he has taken one
overlap of her heart pills. He complains of a ‘funny’ feeling
D. Thyroid cartilage in his chest. On examination he has an irregular
E. Upper trachea heart beat and ECG reveals an arrhythmia.

36
Answers: Single Best Answer
7.1
A. Accidents
This is now the second most common cause
of death in children aged 1 year to 14 years in
the UK.

Accidents and poisoning


B. Congenital heart disease
This is the most common congenital malformation
affecting 0.8% of liveborn children but not the
most common cause of death in this age group.
C. Infectious diseases
Most common cause worldwide but not in
the UK.
D. Malignant disease
Correct. Reduction in deaths from accidents, Figure 7.5
particularly in road traffic accidents, have
recently made malignancy the most common
cause of death in children aged 1 year to 14
years in the UK. B. EEG
The priority for this child is to prevent further
E. Respiratory disease secondary brain injury. An EEG is not required.
Asthma is the most common chronic disease of
childhood but fortunately deaths are rare. C. Neurosurgical referral
Correct. This child has sustained a potentially
7.2 serious head injury and now has reduced level of
A. A black eye (bruising around left eye) consciousness and focal neurological signs,
Black eyes (periorbital ecchymosis) are a which are indications to be assessed by a
significant sign if bilateral, as this can indicate a neurosurgical specialist. The priority for this
basal skull fracture. child is to prevent further secondary brain
injury.
B. A fractured nose with deviated septum
Parietal bruising, a fractured nose or facial D. Ophthalmology opinion
laceration are all distressing, but do not suggest His abnormal pupil is caused by the intracranial
significant brain or skull injury. lesion and not ocular pathology.

C. A runny nose E. Skeletal survey


Correct. A nasal discharge post head trauma A skeletal survey to identify other fractures is
is a significant sign. It may be leakage of important but the priority for this child is to
cerebrospinal fluid (CSF) that can indicate a basal prevent further secondary brain injury.
skull fracture. If it is CSF it will be positive for
7.5
glucose on testing.
A. Analgesia
D. A temperature of 38.2° C Analgesia and a blood sugar are important but
It is too soon for intracranial infection to be not until she has some intravenous fluid.
causing a fever.
B. Blood glucose measurement
E. Further enlargement of the parietal bruising Analgesia and a blood glucose measurement are
Parietal bruising, a fractured nose or facial important but not until she has some
laceration are all distressing, but do not suggest intravenous fluid.
significant brain or skull injury.
C. Chest X-ray
7.3 A chest X-ray is needed but this comes
D. Parietal bone fracture after ABC.
Correct. A parietal fracture (Fig. 7.5) is the most D. Intravenous access
common skull fracture sustained by children, Correct. Louise has cardiovascular compromise
whether accidental or non-accidental. and urgently needs fluid resuscitation. She
therefore needs intravenous access.
7.4
A. Clotting studies E. Intubation and ventilation
The priority for this child is to prevent further Her airway and breathing are stable therefore 37
secondary brain injury. the next appropriate treatment is C – circulation.
7.6 Correct. Jake has hypovolaemic shock secondary
A. Abdominal ultrasound to the loss of blood plasma. This is secondary to
Correct. The history and signs suggest loss of skin integrity.
hypovolaemic shock from splenic injury. She
7 needs an urgent abdominal ultrasound scan
(FAST scan, focused abdominal sonography in
C. Jake has shock due to loss of red blood cells,
because of damage to his blood vessels
secondary to his scald
trauma).
Jake has hypovolaemic shock secondary to the
Accidents and poisoning

B. Arrange CT head scan loss of blood plasma. This happens due to loss of
This would be indicated if there was focal skin integrity.
neurology on secondary survey.
D. Jake has shock due to vasodilation of his
C. Cervical spine X-ray blood vessels secondary to his scald
Important, but clinical assessment is needed for Jake has hypovolaemic shock secondary to the
‘clearing’ a cervical spine. An experienced loss of blood plasma. This happens due to loss of
Emergency Department doctor will be required skin integrity.
to guide investigation of this potential problem.
For now, keep the child immobilised. E. He has shock due to vasodilation of his blood
vessels, secondary to infection developing in his
D. Full blood count scald
Useful, but even if anaemic it will not tell you Though you can develop septic shock from
where the blood has been lost or the volume of infected burns this is likely to happen later rather
acute blood loss. than immediately.
E. Serum creatinine, urea and electrolytes
Commonly undertaken, but rarely informative in 7.10
an acute injury. A. Abdominal X-ray
Correct. An abdominal X-ray identifies if there is
7.7 a significant number of tablets in his stomach. A
B. 20% very useful consequence of iron showing up on
Correct. Burns are to chest and abdomen (about X-ray!
11%), right arm 2% and right leg 6%, which is B. Full blood count
19%, i.e. approximately 20%. The full blood count will be of no value at this
stage.
7.8
A. Commence intravenous 0.9% saline C. Clotting studies
Correct. There will be significant fluid loss The clotting studies will be of no value at this
through the burnt areas, which needs replacing. stage.
B. Cover the burns with sterile dressings D. Liver function tests
Covering the burns is helpful but not urgent. It It is too soon for LFTs to become deranged.
may help to reduce the pain experienced though.
E. Serum iron
C. Intravenous antibiotics The serum iron result will not be helpful at this
Intravenous antibiotics are not needed urgently. stage as he will not have absorbed the
medication.
D. Intubation and artificial ventilation
Intubation and ventilation are not required as his
airway and breathing are satisfactory. It should 7.11
be considered early for children with burns who A. Forced alkaline diuresis
have potential inhalation injury. For aspirin overdose.

E. Place affected areas in cold water B. Intravenous desferrioxamine


Placing scalded area in cold water is useful for Correct. Intravenous desferrioxamine binds
pain relief for small burns, but inappropriate for with iron in the blood excreting it in urine and
large burns. faeces.
C. Intravenous N-acetylcysteine
7.9 Intravenous acetylcysteine is the antidote for
A. It is secondary to the pain from his scald paracetamol poisoning.
His burn would be painful but this would be likely
to cause a tachycardia and increase in blood D. Intravenous naloxone
pressure rather than a drop in blood pressure. Intravenous naloxone is the antidote for opiate
poisoning.
B. Jake has shock due to loss of blood
38 plasma, because of damage to his blood E. Intubate and ventilate
vassels secondary to his scald Required if airway compromise is imminent.
7.12 Cannabis would usually result in red eyes with
A. Cricopharyngeus muscle dilated pupils. MDMA results in agitation but
Correct. About 70% of blunt objects that lodge neither of these would provoke hypoglycaemia.
in the oesophagus will do so at the
cricopharyngeus muscle. Once the object 7.13.2
reaches the stomach it is much less likely to lead G. Paracetamol
to complications, though the ileocecal valve is This is likely to be a paracetamol overdose with
another area that the object can lodge. The coin delayed presentation, as her liver function has

Accidents and poisoning


should be removed endoscopically. already been affected. This can be treated with
N-acetylcysteine. Teenagers can deny taking
B. Lower oesophageal sphincter anything, especially if asked with their parents
Objects tend not to get stuck here and it would present.
appear on the chest x-ray much closer to the
diaphragm if it was. 7.13.3
I. Salicylates (aspirin)
C. Region where the aortic arch and carina This is likely to be salicylate poisoning. The first
overlap stage of toxicity is characterized by
This would be in the mid-oesophagus. hyperventilation resulting from direct respiratory
centre stimulation, leading to a respiratory
D. Thyroid cartilage alkalosis.
It is too big to have passed through the larynx.
Think about the size of an endotracheal tube. 7.13.4
F. Iron.
E. Upper trachea Iron can turn your stools black. If the black stools
It is too big to have passed through the larynx. were melaena you would expect additional
Think about the size of an endotracheal tube. symptoms or signs. The presence of iron tablets
at home would be a crucial aspect to the history.

Answers: Extended Matching 7.13.5


D. Digoxin.
7.13.1 Treatment is with activated charcoal if the child
A. Alcohol presents <1 hour after ingestion. The child
Teenagers experimenting with alcohol can should have ECG monitoring and serum digoxin
become intoxicated quickly. Drinking patterns in concentration measured. To be sure that this
teenagers tend to be more extreme, with large wasn’t a tricyclic overdose (or indeed a red
amounts of alcohol consumed in very short herring altogether), a careful and quick check of
periods. Alcohol drops the blood glucose level Grandma’s actual medication, via her GP if
and can cause them to become comatose. necessary, can be very helpful.

39
8

Child protection

What is the most likely cause of this finding?


Questions: Single Best Answer
Select on answer only.
8.1
A. Accidental Injury
Alfie is a 2-year-old boy who presents to the
B. Non-accidental injury
Emergency Department with a painful right leg.
C. Osteogenesis imperfecta (brittle bone
His grandmother reports that he had fallen down
disease)
the stairs earlier that day. He has not been
D. Osteosarcoma
walking on his leg since then. She thought he
E. Vitamin D deficiency
had pulled a muscle. On returning home, his
parents brought him to the hospital. He has no
8.2
other injuries and his development is normal for
You are a junior doctor working on the
his age.
paediatric ward. You are asked to take some
The X-ray is shown in Fig. 8.1. bloods from Chloe, an 11-year-old girl. Her
parents do not wish to be present. When rolling
up her sleeve to look for a suitable place for
venepuncture you note numerous bruises from
strap marks to her upper arm. You ask her how
she got the bruising. She replies that her uncle
did it as she had been naughty. She does not
want you to tell her parents.
What should you do with the information?
Select on answer only.
A. Document what was said in the medical
notes including sketches and inform the
consultant on call
B. Document what was said in the notes
including sketches and where possible
photographs
C. Ignore it; she was being disciplined for
misbehaviour and she has expressed her
desire that no-one else is informed
D. Inform the health visitor and request a home
assessment
E. Inform her mother what she said and
suggest she asks the uncle about it

8.3
Pauline is a 6-year-old girl. Her teacher is
concerned, as she has been rubbing herself
‘down below’ in the classroom and touching
other girls. She later discloses to her teacher that
her stepfather has hurt her with his ‘willy’. She is
seen by the consultant paediatrician who notices
Figure 8.1 some vulval soreness and so takes a swab which
reveals gonococcus. She also notices that there is
some bruising to the thighs. She plots her Questions: Extended Matching
weight and finds it to be just above the 99th
centile. She has no other medical problems. 8.5
The following (A–I) is a list of possible diagnoses.
Which of the following findings is the most For each of the children described in the
suggestive of sexual abuse? following scenarios pick the most likely
diagnosis. Each answer may be used once, more
Select one answer only.

Child protection
than once, or not at all.
A. Bruising to the thighs
A. Accidental injury
B. Disclosure of event to teacher
B. Acute allergic reaction
C. Gonococcus on swab
C. Acute lymphoblastic leukaemia
D. Sexualised behaviour
D. Immune thrombocytopenic purpura (ITP)
E. Vulval soreness
E. Meningococcal septicaemia
F. Non-accidental injury
8.4
G. Poisoning
Chelsea is a 2-year-old girl who presented 6
H. Scald
months ago with a fractured femur which was
I. Staphylococcal scalded skin infection
felt to be accidental. She presents to the
Emergency Department having slipped in the 8.5.1
bath whilst briefly being left alone. On Ilsa is an 18-month-old child. Her mother has
examination there is swelling and bruising brought her to the Emergency Department
over Chelsea’s anterior right chest wall. She has with the marks on her left arm shown in
some older bruises on her right thigh. She has Fig. 8.3. She had been at her father’s house with
no other medical problems and is not on any her siblings all weekend and returned to her
medication. The chest X-ray (Fig. 8.2) reveals rib mother’s care on Sunday night. Her mother
fractures. asked her father what had happened and he had
said he thought that she knocked herself on the
coffee table.

Figure 8.2

What is the most appropriate next step in


management?
Select one answer only.
A. Check vitamin D status
B. Discharge home with follow up by GP the
next day
C. Ensure a child protection medical takes place
D. Genetic counselling 41
E. Health visitor home assessment Figure 8.3
8.5.2 8.5.4
Mona a 3-month-old child presents to the Denisa is a 14-year-old girl who presents to the
Emergency Department as her parents are Accident and Emergency department with fever
worried by the appearance of the right lower leg. and lethargy. On examination she is unwell and
8 On examination she has a temperature of 38.5° C
and her leg is as shown in Fig. 8.4. Her mother
has a rash over all her body which is shown in
Fig. 8.6. This rash does not disappear on applying
reports that she had an infection around her pressure.
mouth which she caught at nursery.
Child protection

Figure 8.4

8.5.3
Aisha, a 6-year-old girl, is brought by her mother Figure 8.6
to the paediatric emergency clinic because of
bruises. The family say that she had a viral upper 8.5.5
respiratory tract infection (URTI) the previous Lucy is a 22-month-old girl. She is brought to the
week for which she saw her general practitioner. Emergency Department by her mother. She was
The URTI had completely resolved. She was well playing in the kitchen when her mother says she
when she went to spend a long weekend with pulled a kettle of boiling water over her legs. The
relatives but returned with multiple bruises. On remainder of her examination is normal. She is in
examination she is well in herself and is afebrile. pain. Her legs are shown in Fig. 8.7. She has no
There is no hepatosplenomegaly. The appearance other medical problems and is not on any
of her legs is shown in Fig. 8.5, and there is a bruise medications.
around her eye and a few scattered petechiae.

Figure 8.5 Figure 8.7

42
E. Inform her mother what she said and suggest
Answers: Single Best Answer she asks the uncle about it
The parents will need to be informed and
8.1 consent should be sought for a full child
A. Accidental injury protection medical.
Correct. There is an oblique midshaft fracture of
the femur. The most common cause is an 8.3
accident, though non-accidental injury always A. Bruising to the thighs

Child protection
has to be borne in mind. Bruising over the thighs is common in active
B. Non-accidental injury children but it is more concerning if it is found in
The most common cause is an accident, though the inner thigh as this area is anatomically
non-accidental injury always has to be borne in ‘protected’ and not often bruised accidentally.
mind. B. Disclosure of event to teacher
C. Osteogenesis imperfecta (brittle bone The disclosure is very useful and a police
disease) investigation would need to be undertaken.
There is no evidence on the X-ray of generalised C. Gonococcus on swab
bone disease. Correct. All of these answers could suggest
E. Osteosarcoma sexual abuse but only the gonococcus on the
Although bone tumours may predispose to swab confirms it. It will not however inform you
fractures there is no evidence of a bone tumour of whom the perpetrator is.
on the X-ray. D. Sexualised behaviour
D. Vitamin D deficiency Sexualized behaviour may raise awareness of
There is no evidence on the X-ray of rickets. abuse as a potential problem but is not
confirmatory of it.
8.2 E. Vulval soreness
A. Document what was said in the medical Vulval soreness is common in young girls.
notes including sketches and inform the
Consultant on call. 8.4
Correct. This is a safeguarding issue, as the girl A. Check vitamin D status
has alleged physical abuse and this needs to be Bruising over the chest wall is uncommon and
taken seriously. Other agencies, e.g. social rib fractures without a reasonable explanation
services need to be contacted to identify any (e.g. involvement in significant road traffic
concerns, and the patient needs a full medical accident) are pathognomonic of abusive injury.
examination by a paediatrician trained in child Even if vitamin D is low, it would not explain
protection. Photographs will be helpful but these injuries.
consent must be obtained for these first. It is
vital that the consultant on call knows what is B. Discharge home with follow up by GP the
happening and is involved. next day
Inappropriate under these circumstances.
B. Document what was said in the notes
including sketches and where possible C. Ensure a child protection medical takes
photographs place
This is helpful but vital time may be lost in Correct. Even if you do not see the fractures on
ensuring that all injuries are identified. this X-ray, there are some features in this history
There may be more occult problems and it is which are very concerning. The child is left
better to get consultant involvement from unsupervised in the bath; she has had a previous
the outset. femur fracture and has bruises on her thigh. A
child protection medical review needs to be
C. Ignore it; she was being disciplined for undertaken now by a consultant and a place of
misbehaviour and she has expressed her desire safety found for this child. This is usually in
that no-one else is informed hospital but sometimes an alternative place of
You are not bound by the duty of confidentiality safety can be identified quickly.
in this instance. It is important to know the
exceptions to this. See: http://www.gmc-uk.org/ D. Genetic counselling
guidance/ethical_guidance/confidentiality.asp Genetic counselling is not required.
for more details and case discussions.
E. Health visitor home assessment
D. Inform the health visitor and request a home A health visitor home assessment would be
assessment useful in this situation to gain more information
The first action is to record the findings carefully but it would not be the most appropriate next 43
before a strategy meeting is arranged. step for this child.
8.5.3
Answers: Extended Matching D. ITP (Immune thrombocytopenic purpura)
This is likely to be ITP. It classically follows a viral
8.5.1 illness and here there is widespread purpura and
8 F. Non-accidental injury
This picture is consistent with a bite mark. There
bruising. A full blood count would reveal a low
platelet count. If it were acute lymphoblastic
appear to be two bite marks. There is a smaller leukaemia, she would be likely to be unwell and
mark over the hand which is from a child, and it have other abnormal symptoms and signs, e.g. a
Child protection

is not uncommon for toddlers to bite other febrile illness, splenomegaly.


children. However, the mark on the upper arm is
consistent with a much older child or adult and 8.5.4
as there is no adequate explanation, it is E. Meningococcal septicaemia
suggestive of a non-accidental injury. To be sure This is meningococcal sepsis and immediate
a forensic dentist may be helpful. treatment with intravenous antibiotics is
necessary. Her airway, breathing and circulation
8.5.2
all need to be supported.
I. Staphylococcal scalded skin syndrome
This is staphylococcal scalded skin syndrome, 8.5.5
caused by an exfoliative staphylococcal toxin F. Non-accidental injury
which causes separation of the epidermal skin This is a non-accidental injury. The image here
through the granular cell layers. This affects shows what is known as ‘glove and stocking’
infants and young children. They develop fever distribution of a burn on her feet. This type of
and malaise, may have a purulent, crusting, scald indicates that the feet have been
localized infection around the eyes, nose, and purposefully held in hot water. If a kettle had
mouth with subsequent widespread erythema caused this there would not be such uniform
and tenderness of the skin. Areas of epidermis demarcation and it is likely that there would be
separate on gentle pressure (Nikolsky’s sign) satellite lesions from splashes.
leaving denuded areas of skin, as shown here,
which subsequently dry and heal without
scarring.

44
9

Genetics

Questions: Single Best Answer


9.1
Michael is a 6-year-old boy who is prone to
bleeding. He has a problem with one of his
clotting factors and his parents have been told
that this is caused by a faulty gene. He is
otherwise well and has no other medical
problems. His parents have an appointment
with the geneticist who takes a family history
and draws a family tree, which is shown in
Fig. 9.1.
Figure 9.2
Select one answer only.
A. 1 in 1
B. 1 in 2
C. 1 in 3
D. 1 in 4
E. 1 in 5

9.3
Mr and Mrs Walsh attend the clinic with
their new baby, Ophelia, who has Down
syndrome. They are keen to have further
children and want to know more about their
Figure 9.1 future risk of having children with Down
syndrome. What chromosomal abnormality is
What is the most likely pattern of inheritance in likely to have caused Ophelia to have Down
this disorder? syndrome?
Select one answer only. Select one answer only.
A. Autosomal dominant A. Mosaicism
B. Autosomal recessive B. Nondisjunction
C. Imprinting from uniparental disomy C. Point mutation
D. Trinucleotide repeat expansion mutation D. Translocation
E. X-linked recessive E. Triplet repeat expansion

Hint: Why are only squares affected? 9.4


Mr and Mrs David are seen by the geneticists as
9.2 their baby, Sarah, has Down syndrome. Her
The female marked with an arrow in Fig. 9.2 is chromosomes are examined. Three copies of
planning on starting a family. If she were to give chromosome 21 are seen, one of which is
birth to a son, what would be the risk of him attached to chromosome 14. How would you
being affected by the disorder? describe this abnormality?
Select one answer only. C. Ischaemic heart disease
D. Leukaemia
A. Balanced Robertsonian translocation E. Pyloric stenosis
B. Mosaicism

9 C.
D.
E.
Nondisjunction
Triplet repeat expansion
Unbalanced Robertsonian translocation
9.8
A Pakistani couple are referred for genetic
counselling. They have lost two children who
both died in the first 2 years of life. They have
Genetics

9.5 one healthy girl, who is 3 years of age. The faulty


Olive is a 10-day-old baby with Down syndrome. gene has been identified and other members of
On examination, you hear a loud heart murmur. their family have been tested (Fig. 9.3).
What is the most likely cause?
Select one answer only.
A. Aortic stenosis
B. Atrioventricular septal defect
C. Coarctation of the aorta
D. Innocent murmur
E. Patent ductus arteriosus

9.6
Fiona is a well 4-year-old girl with Down
syndrome. She attends her yearly follow-up Figure 9.3
appointment with her mother. There are no real
problems other than constipation, for which her
general practitioner has started treatment. When What is the pattern of inheritance in this
you plot Fiona on the Down syndrome growth disorder?
chart, you notice that her height has gone from A. Autosomal dominant
the 75th centile to the 25th centile. Her weight, B. Autosomal recessive
however, has gone from the 50th centile to the C. Imprinting from uniparental disomy
75th centile. D. Trinucleotide repeat expansion mutation
Which of the following investigations would you E. X-linked recessive
perform?
9.9
Select one answer only. The same couple (Fig. 9.3) tell you that they
would like more children.
A. Coeliac screen
B. Abdominal ultrasound What is the risk of them having another affected
C. Full blood count baby?
D. Thyroid function tests
E. Vitamin D levels Select one answer only.
A. Around 1 in 200
9.7 B. 1 in 2
Mary is an infant with Down syndrome. Her C. 1 in 4
antenatal scans had all been normal and an D. 2 in 3
echocardiogram on the neonatal unit shortly E. 3 in 4
after birth was normal. She attends the
community clinic for the first time at 4 weeks of 9.10
age. She is now thriving and feeding well. Her A Bangladeshi couple are referred for genetic
parents have many questions about what is counselling. Both parents are carriers of a faulty
going to happen in the future. In particular, they gene PEX1; possession of two abnormal copies of
have been reading that she is still at an increased this gene leads to death in infancy. They have
risk of certain diseases because she has Down lost two children who both died in the first 2
syndrome. years of life. They have one healthy daughter,
Out of the following, which condition is Mary at who is 3 years of age. What is the risk that their
increased risk of developing compared with the daughter is a carrier?
general population? A. None
Select one answer only. B. 1 in 4
C. 1 in 2
46 A. Congenital heart disease D. 2 in 3
B. Duodenal atresia E. 100%
9.11 9.13
Gemma and Mark, who are both well, are George has Klinefelter syndrome. What is his
planning to start a family. Gemma’s older brother karyotype?
has cystic fibrosis. There is no history of cystic
Select one answer only.
fibrosis in Mark’s family. The family tree is shown
in Fig. 9.4. What is the chance that they will have A. 46, XO
a child with cystic fibrosis? The carrier rate is 1 in B. 46, XY
25 in their population. C. 45, XO

Genetics
D. 45, XY
E. 47, XXY
9.14
Louise (Fig. 9.6) is a 14-year-old girl. She is attending
the endocrinology clinic, as she is the shortest girl in
her class and wants to know if there is treatment to
make her taller. She is otherwise well and has no
other medical complaints. She has always been the
shortest girl in her class. Her clinical appearance is
Gemma Mark shown in the picture. Her height plots well below
the 0.4th centile but her weight is on the 9th
centile. Her chromosome karyotype is 46, XX.

Figure 9.4

Select one answer only.


A. 1 in 6
B. 1 in 37.5
C. 1 in 100
D. 1 in 150
E. 1 in 2500

9.12
A mother comes to see the geneticist. She has two
children, Robert and Elizabeth. They both suffer
from a genetic disorder. The geneticist takes a
history and draws the family tree (Fig. 9.5).

Figure 9.5

What is the likely pattern of inheritance in this Figure 9.6


disorder? What is the most likely cause of her short stature.
Select one answer only. Select one answer only.
A. Autosomal dominant A. Cushing syndrome
B. Autosomal recessive B. Down syndrome
C. Imprinting from uniparental disomy C. Noonan syndrome
D. X-linked dominant D. Turner syndrome 47
E. X-linked recessive E. Normal variant (constitutional short stature)
9.15.2
Questions: Extended Matching Amelia is a 14-year-old girl who has not started
puberty. Her friends at school have all started
9.15 their periods but she hasn’t even started any
9 The following (A–N) is a list of genetic disorders.
For each child described select the most likely
breast development. She is an otherwise well girl
with no other medical complaints. Her mother
genetic diagnosis from the list. Each answer may mentions she had puffy hands and feet as a
be used, more than once, or not at all. baby. On examination you note that she is short
Genetics

A. Achondroplasia (<0.4th centile for height) and has a systolic heart


B. Angelman syndrome murmur and diminished femoral pulse pressure
C. Cystic fibrosis compared with brachial pulses.
D. Down syndrome 9.15.3
E. Duchenne muscular dystrophy Cyril is a 2-day-old infant born to a mother who
F. Edward syndrome is a refugee and had no antenatal care. He was
G. Fragile X syndrome born at term by a normal vaginal delivery.
H. Klinefelter syndrome Examination at birth revealed a cleft lip and
I. Neurofibromatosis palate, a small area on his scalp where there is
J. Noonan syndrome no skin and six toes on both feet. He is fed by a
K. Patau syndrome nasogastric tube as he has no suck reflex. He has
L. Prader–Willi syndrome a loud systolic heart murmur.
M. Turner syndrome
N. Williams syndrome 9.15.4
Rafael is a 6-year-old boy. He is referred to the
9.15.1 paediatric outpatient department by the school
Rachael is a 3-month-old girl who attends the nurse. His class teacher reports that he finds
Emergency Department because of rapid most learning activities very difficult. His
breathing. She is the 5th infant born to a behaviour is difficult to manage in class. On
39-year-old mother. When you take her examination you note that he has a large head
developmental history, you find she smiled at 7 which plots on the 99th centile. His height and
weeks but is unable to hold her head weight are on the 25th centile. He has a long
unsupported. On examination, she is face and large ears. The remainder of
tachypnoeic, sweaty and hypotonic. Her clinical examination is normal.
appearance can be seen in Fig. 9.7. You plot her
weight and she is below the 0.4th centile. 9.15.5
Roger is a 12-year-old boy. He is referred to the
paediatric outpatient department because he
has developed breasts. His class teacher reports
that his behaviour is difficult to manage in class.
On examination you note that he is very tall and
slim, with his height on the 99th centile and his
weight on the 40th centile. He has no signs of
puberty and on assessing his scrotum you note
he has small, firm testicles.

9.16
For each of the case summaries below, select the
most likely mode of genetic inheritance from the
list (A–G) below. Each answer can be used once,
more than once, or not at all.
A. Autosomal dominant disorder
B. Autosomal recessive disorder
C. Imprinting from uniparental disomy
D. Microdeletion
E. Polygenic
F. X-linked dominant
G. X-linked recessive
9.16.1
Jack is a 3-year-old boy whose father has a
genetic disorder associated with short stature.
48 Jack is also very short, has bowed legs and a very
Figure 9.7 prominent forehead. His head is
disproportionately large compared with his He is also doing well academically at school. On
body size. examination you note he is very tall. He has a
high arched palate, stretch marks on his skin and
9.16.2 lax joints. His height is well above the 99th
Mercy is a 7-month-old, Black-African girl who centile and he has long arms, legs, and fingers.
presents to the Emergency Department with His mother is also tall (178 cm) but reports that
fever and severe pain in her digits. She has a past she is healthy.
medical history of being chronically tired. On

Genetics
examination you note that she has pale 9.16.4
conjunctiva and mild hepatomegaly. She is A child is suspected of having Prader-Willi
febrile and appears to have an upper respiratory syndrome. This is confirmed on genetic analysis
tract infection. Her fingers are swollen. but there is no deletion of genetic material.
9.16.3 9.16.5
Bruce is a 12-year-old boy who is referred by the Gregor, aged 6 weeks, is vomiting his feeds. His
ophthalmology team. He presented with blurred vomit is ‘like a fountain’, and immediately
vision. Ophthalmological examination revealed a afterwards he is hungry again. When examined
dislocated lens. He is very disappointed, as he after a vomit, a mass could be felt in the right
had been doing very well in his basketball team. upper quadrant of his abdomen.

49
B. Nondisjunction
Answers: Single Best Answer Correct. Meiotic nondisjunction accounts for
94% of cases of Down syndrome. Most cases are
9.1 a result of an error at meiosis.
9 A. Autosomal dominant
It could only be autosomal dominant if there was C. Point mutation
No, Down syndrome is a trisomy. There is an
incomplete and variable penetrance of this
condition. extra copy of an entire chromosome (21).
Genetics

B. Autosomal recessive D. Translocation


Whilst it is theoretically possible to see a pattern This accounts for 5% of cases. It must be
like this in autosomal recessive conditions it unbalanced to result in the clinical syndrome.
would be highly unlikely. Both great
E. Triplet repeat expansion
grandparents would have to be carriers and two
No, Down syndrome is a trisomy. There is an
‘new’ partners would also have to be carriers.
extra copy of an entire chromosome (21).
Unlikely unless there is significant consanguinity.
9.4
C. Imprinting from uniparental disomy A. Balanced Robertsonian translocation
Following meiosis, imprinting proceeds in a Balanced translocations do not result in clinical
‘parent of origin’ specific manner. In general this features but increase the risk of having a child
‘resets’ the genetic information correctly and with the condition.
therefore uniparental disomy is not heritable.
B. Mosaicism
D. Trinucleotide repeat expansion mutation In mosaicism some of the cells are normal and
This would result in genetic anticipation where some have trisomy 21.
the disease would be more severe or be evident
sooner in subsequent generations. C. Nondisjunction
This would be more common and simply result
E. X-linked recessive in three copies of chromosome 21.
Correct. Only males are affected and females
can be carriers. Very rarely female carriers can D. Triplet repeat expansion
show mild signs and symptoms. An example of This would result in an alteration in the DNA
such a genetic disorder is haemophilia A. sequence, often in a noncoding region (intron).

9.2 E. Unbalanced Robertsonian


A. 1 in 1 translocation
This would be close to the risk if she had a Correct. This is important because there is an
mitochondrial disorder. increased risk of recurrence as one of the parents
is likely to have a balanced translocation
B. 1 in 2 whereby one of their copies of chromosome 21
This would be the risk if she was a known carrier. is attached to chromosome 14.
C. 1 in 3 9.5
Risks can deviate from expected ratios if diseases A. Aortic stenosis
are fatal in utero. But this is not the case. It does occur in children with Down syndrome
but it is much less common than atrioventricular
D. 1 in 4
septal defect and ventricular septal defect.
Correct. There are two questions to answer:
(1) What is the risk that this person has the faulty B. Atrioventricular septal defect
gene and then (2) what is the risk of passing Correct. This is the most common congenital
it on? cardiac anomaly in children with Down
This is an X-linked recessive disorder. As her syndrome. 40% of children with Down syndrome
brother is affected, her mother is a carrier. The have a congenital heart defect and therefore, all
female in question therefore has a 50% chance children with Down syndrome should have an
of also being a carrier. If she were to have a son echocardiogram in the neonatal period.
there would be a further 50% chance of passing
this condition on. Therefore there is a total 25% C. Coarctation of the aorta
risk (1 in 4) of her son being affected. It does occur in children with Down syndrome
but it is much less common than atrioventricular
E. 1 in 5 septal defect and ventricular septal defect.
It is very hard to get a situation where the risk of
recurrence is 1 in 5. D. Innocent murmur
It would be unusual for an innocent murmur to
9.3 be loud. They are usually ‘soft, short, systolic,
50 A. Mosaicism localized in site, and there are no signs or
This accounts for about 1% of cases symptoms’.
E. Patent ductus arteriosus feature would be one of ‘genetic anticipation’: it
Common in preterm babies but not the typical would be clinically more severe in each
abnormality seen in a child with Down syndrome. generation.

9.6 E. X-linked recessive


A. Coeliac antibodies There are male carriers evident in the family tree.
Whilst children with Down syndrome are at
increased risk of coeliac disease this is usually 9.9
A. Around 1 in 200

Genetics
associated with faltering growth.
This is the recurrence risk of Down syndrome for
B. Abdominal ultrasound a mother under 35 years of age if the underlying
Unhelpful in this situation. cause was nondisjunction.
C. Full blood count B. 1 in 2
Children with Down syndrome are at increased This is the recurrence risk of autosomal dominant
risk of leukaemia – but this is not the correct conditions when one parent is affected or X-linked
clinical history. recessive conditions where mother is a carrier.
D. Thyroid function tests C. 1 in 4
Correct. Children with Down syndrome are at an Correct. This is an autosomal recessive
increased risk of hypothyroidism. Growth failure condition. If both parents are carriers for an
and constipation are symptoms of autosomal recessive condition then there is a 1
hypothyroidism. in 4 chance of a child being affected.
E. Vitamin D levels D. 2 in 3
Severe vitamin D deficiency might cause a This is the recurrence risk of an autosomal
reduction in height but not an increase in weight dominant condition when homozygosity is
and is not more common in children with Down associated with fetal death and both parents
syndrome. carry the gene.

9.7 E. 3 in 4
A. Congenital heart disease This is the recurrence risk of an autosomal
Her echocardiogram is normal though. Therefore dominant condition when homozygosity results
her risk is not increased. in liveborn children and both parents have the
condition.
B. Duodenal atresia
This would have presented soon after birth. 9.10
A. None
C. Ischaemic heart disease Whilst it is clear that she does not possess two
There is no increased risk of this ischaemic heart copies of the faulty gene (she is healthy), she
disease in children with Down syndrome. may be a carrier.
D. Leukaemia B. 1 in 4
Correct. Children with Down syndrome are at There is a 1 in 4 chance of a future pregnancy
increased risk of leukaemia. resulting in an affected child, but this is not the
E. Pyloric stenosis clinical scenario.
No increased risk. C. 1 in 2
9.8 There is a 1 in 2 chance that a pregnancy will
A. Autosomal dominant result in a child being a carrier. However, this is a
There are carriers shown. subtly different question, as one possibility does
not exist as this child is definitely not affected.
B. Autosomal recessive
Correct. Autosomal recessive conditions are D. 2 in 3
more commonly seen in families with Correct. As she does not have the condition,
consanguinity. there are three possible outcomes, one (1 in 3) is
that she is not a carrier, the other two (2 in 3) are
C. Imprinting from uniparental disomy that she is a carrier.
Following meiosis, imprinting proceeds in a
‘parent of origin’ specific manner. In general this E. 100%
‘resets’ the genetic information correctly and This is incorrect as there is a possibility that
therefore uniparental disomy is not heritable. parents passed on their ‘good’ copies of gene
PEX1. (This is the gene responsible for one form
D. Trinucleotide repeat expansion mutation of Zellweger syndrome, a rare but fatal condition
The inheritance of trinucleotide repeat expansion resulting in defective peroxisomes). 51
disorders is usually autosomal dominant. The key
9.11 E. X-linked recessive
A. 1 in 6 Males and females are equally affected in this
This would be the risk if Mark was known to be a family’s pedigree therefore, it cannot be X-linked
carrier but Gemma’s status was unknown. A recessive.
9 more complicated pedigree where Mark already
has an affected child by another partner could 9.13
A. 46, XO
give this risk.
An interesting karyotype. This could only occur if
Genetics

B. 1 in 37.5 you had an extra autosome and a missing sex


This is roughly the risk that both parents are chromosome.
carriers. As Gemma does not have the condition
there are three possible outcomes, one (33%) is B. 46, XY
that she is not a carrier, the other two (66%) are Normal male karyotype.
that she is a carrier, i.e. 2/3 × 1/25 = 1 in 37.5. C. 45, XO
C. 1 in 100 Turner syndrome.
This is a tempting option but incorrect. If Gemma D. 45, XY
was known to be a carrier and she found a There is a missing autosome. This would not be
partner from the general population, this would survivable.
be the chance that a pregnancy would result in
an affected child, i.e. 1/25 × 1/4 = 1/100. E. 47, XXY
Correct. This is Klinefelter syndrome.
D. 1 in 150
Correct. As Gemma does not have the condition 9.14
there are three possible outcomes, one (1 in 3) is A. Cushing syndrome
that she is not a carrier, the other two (2 in 3) are In Cushing syndrome there is growth failure but
that she is a carrier. The risk of Mark being an you would also expect other clinical features as
unaffected carrier is the population frequency of listed in Box 9.1.
heterozygosity. The risk of two carriers having an
affected child is 1 in 4 per pregnancy. Therefore
the risk is roughly 2/3 × 1/25 × 1/4. This could be
refined slightly if Gemma’s status is known, but Box 9.1 Clinical features of Cushing
this might not influence the couples decision syndrome
about screening. Checking whether Mark is a
carrier has a bigger influence on the risk. • Growth failure/short stature
• Face and trunk obesity
E. 1 in 2500 • Red cheeks
This is incorrect but is the risk for Caucasian
• Hirsutism
couples having a child with cystic fibrosis in
the UK. • Striae
• Hypertension
9.12 • Bruising
A. Autosomal dominant • Carbohydrate intolerance
Correct. This is the most common mode of
• Muscle wasting and weakness
Mendelian inheritance. An affected individual
carries the abnormal gene in the heterozygous • Osteopenia
state. Offspring have a 50% chance of inheriting • Psychological problems.
the abnormal gene.
B. Autosomal recessive
Providing that new partners came from outside
this family, the chance that this pattern is B. Down syndrome
autosomal recessive is extremely low. Children with Down syndrome are small but
have different clinical features. See Box 9.2.
C. Imprinting from uniparental disomy
Following meiosis, imprinting proceeds in a C. Noonan syndrome
‘parent of origin’ specific manner. In general this Correct. She has a round face with short,
‘resets’ the genetic information correctly and webbed neck, and short stature since infancy,
therefore uniparental disomy is not heritable. characteristic of Noonan syndrome. There is
some overlap with the phenotype of Turner
D. X-linked dominant syndrome, but her karyotype is normal.
In X-linked dominant inheritance all the
daughters of an affected male have the D. Turner syndrome
52 condition as he must pass on a copy of the gene The normal karyotype excludes Turner
to any daughter. syndrome.
Box 9.2 Characteristic clinical manifestations of Down syndrome
Typical craniofacial appearance Later medical problems
• Round face and flat nasal bridge • Delayed motor milestones
• Upslanted palpebral fissures • Learning difficulties – severity is variable,
• Epicanthic folds (a fold of skin running across usually mild to moderate but may be severe
the inner edge of the palpebral fissure) • Short stature
• Brushfield spots in iris (pigmented spots) • Increased susceptibility to infections

Genetics
• Small mouth and protruding tongue • Hearing impairment from secretory otitis
• Small ears media (75%)
• Flat occiput and third fontanelle • Visual impairment from cataracts (15%),
squints, myopia (50%)
Other anomalies • Increased risk of leukaemia and solid tumours
• Short neck (<1%)
• Single palmar creases, incurved and short fifth • Acquired hip dislocation and atlanto-axial
finger and wide ‘sandal’ gap between first instability
and second toes • Obstructive sleep apnoea (50–75%)
• Hypotonia • Increased risk of hypothyroidism (15%) and
• Congenital heart defects (in 40%) coeliac disease
• Duodenal atresia • Epilepsy
• Hirschsprung disease (<1%) • Early onset Alzheimer disease.

E. Normal variant (constitutional short The mandible is often prominent, with a broad
stature) forehead. They have macro-orchidism post-
Not with the clinical features shown. puberty. See Box 9.3 and Fig. 9.8.

Answers: Extended Matching Box 9.3 Clinical findings in males with


fragile X syndrome
9.15.1
D. Down syndrome • Moderate–severe learning difficulty (IQ
The picture shows the facial features of Down 20–80, mean 50)
syndrome – round face with a flat nose, • Macrocephaly
upslanted palpebral fissures and epicanthic folds • Macro-orchidism – postpubertal
(a fold of skin running across the inner edge of • Characteristic facies – long face, large everted
the palpebral fissure). Other features are her ears, prominent mandible, and broad
hypotonia. Her mother’s older age increases her forehead, most evident in affected adults
risk of having a child with Down syndrome. The
• Other features – mitral valve prolapse, joint
tachypnoea, sweatiness and poor weight gain
laxity, scoliosis, autism, hyperactivity.
could reflect heart failure from congenital heart
disease. Plotting her weight on a Down
syndrome chart would be more appropriate.
9.15.2
M. Turner syndrome
Features include delayed puberty, primary
amenorrhoea, short stature, coarctation of the
aorta and lymphoedema as a neonate. Noonan
syndrome shares many of these features.
9.15.3
K. Patau syndrome (trisomy 13)
Cyril has a cleft lip and palate, a scalp defect,
polydactyly, and a heart defect. These are all
features of Patau syndrome.
9.15.4
G. Fragile X syndrome
Fragile X is one of the most common causes of
learning difficulties in boys. Characteristic 53
features are a long face with large everted ears. Figure 9.8
9.15.5 high incidence of this condition in populations
H. Klinefelter syndrome who originate from endemic areas as
heterozygote status confers a survival benefit.
9.16.1
A. Autosomal dominant disorder
9 Jack and his father have the same condition. Jack
has achondroplasia.
9.16.3
A. Autosomal dominant disorder
Bruce has Marfan syndrome. The causes of tall
9.16.2 stature are described in Table 9.1. Whilst the
Genetics

B. Autosomal recessive disorder description alone could suggest an X-linked


Mercy has sickle cell disease. Carrier status is dominant inheritance, conditions that are
protective against malaria and has resulted in a inherited in this manner are rare.

Table 9.1 Causes of excessive growth or tall stature

Familial Most common cause

Obesity Puberty is advanced, so final height centile is less than in childhood

Secondary Hyperthyroidism
Excess sex steroids – precocious puberty from whatever cause
Excess adrenal androgen steroids – congenital adrenal hyperplasia
True gigantism (excess growth hormone secretion)

Syndromes Long-legged tall stature


Marfan syndrome
Homocystinuria
Klinefelter syndrome (47, XXY karyotype)
Sotos syndrome – associated with large head, characteristic facial features and learning
difficulties

9.16.4 two maternal copies of chromosome 15 will


C. Imprinting from uniparental disomy have Prader-Willi syndrome from imprinting.
Uniparental disomy accounts for 10% of See Fig. 9.9.
Prader–Willi syndrome. A child who inherits

Imprinting from a uniparental disomy

Chromosome 15

Prader-Willi Angelman
syndrome syndrome

54
Figure 9.9
Answer 9.16.5 female is more likely to pass the condition on to
E. Polygenic her children than an affected male. This is
Gregor has pyloric stenosis. This is a classic because there is a sex specific threshold for
polygenic disorder. It is more common in boys expression of pyloric stenosis that is lower in
than girls and this unequal sex incidence leads to males. Affected females are more likely to pass it
the Carter phenomenon, where an affected on.

Genetics

55
10

Perinatal medicine
C. Breech presentation
Questions: Single Best Answer D. Fetal breathing movements seen
E. Reverse end-diastolic flow in the umbilical
10.1 artery
A mother has just found out she is pregnant and
asks for advice about how to look after her 10.4
health and nutrition during pregnancy. She A mother has just found out she is pregnant with
smokes 15 cigarettes a day. You recommend she twins. Her antenatal scan reveals dichorionic,
gives up smoking. diamniotic twins.
If she continues to smoke despite your advice Which condition carries the biggest increased
the baby is at increased risk of which of the risk in her twin pregnancy?
following health problems?
Select one answer only.
Select one answer only.
A. Congenital abnormalities
A. Growth restriction B. Gestational diabetes
B. Dysmorphic syndromes C. Macrosomia
C. Neural tube defects D. Post-term gestation
D. Shoulder dystocia E. Twin-to-twin transfusion
E. Vitamin D deficiency
10.5
10.2 You perform a routine newborn examination on
A mother has her routine 20-week antenatal scan. a baby who is 20 hours old.
The sonographer finds the fetal abdominal and
head circumference measurements are normal, Which one of the following features requires
but is concerned that there is an abnormally further immediate assessment?
small amount of amniotic fluid (oligohydramnios). Select one answer only.
What is the most likely cause for this. A. Acrocyanosis (cyanosis of the hands
Select one answer only. and feet)
B. A heart murmur
A. Duodenal atresia C. An undescended testis
B. Gastroschisis D. Breast enlargement
C. Maternal diabetes E. Subconjunctival haemorrhages
D. Poorly functioning fetal kidneys
E. Severe intrauterine growth restriction 10.6
A black mother is found to have glycosuria at her
10.3 midwife appointment at 32 weeks’ gestation. Her
A midwife is concerned that a mother who is at glucose tolerance test and fasting glucose is
32 weeks’ gestation has a symphysis–fundal abnormal. She is given dietary advice to control
height smaller than expected. An ultrasound her blood glucose.
confirms intrauterine growth restriction.
What problem is her newborn baby at most
Which feature would be of most concern to the increased risk of?
sonographer?
Select one answer only.
Select one answer only.
A. Anaemia
A. Accelerations of fetal heart rate B. Hyperglycaemia
B. Active fetal movements C. Respiratory distress syndrome
D. Neonatal bacterial infection Which of the following is not tested for in the UK?
E. Neonatal type 1 diabetes mellitus
Select one answer only.
10.7 A. Cystic fibrosis
Jonathan, a newborn baby, is noted to have B. Duchenne muscular dystrophy
hepatosplenomegaly and a petechial rash. His C. Hypothyroidism
red eye reflex is normal and there is no heart D. Phenylketonuria
murmur. He fails his newborn screening hearing E. Sickle cell disease

Perinatal medicine
test. His mother is from the UK and her antenatal
screening bloods were all normal. 10.11
What is the most likely congenital infection that Sam, a two-day-old infant, weighs 3.6 kg at
has caused these symptoms? birth. He was born by vaginal delivery with
Apgar scores of 7 at 1 minute and 10 at 5
Select one answer only. minutes. On day 2 he is reported to be jittery,
A. Cytomegalovirus crying inconsolably and feeding poorly. He
B. Rubella sneezes and yawns, and is thought to have some
C. Syphilis abnormal movements, possibly seizures. No
D. Toxoplasmosis dysmorphic features are present and he is not
E. Varicella zoster jaundiced.

10.8 What is the most likely explanation for his


A male infant is born at term. At 1 minute of age problems?
he is breathing regularly and has a heart rate of Select one answer only.
140 beats/min. He is grimacing but has not yet
cried. He is pink centrally but still blue around his A. Congenital rubella syndrome
extremities and his tone is reduced although he B. Hypoxic-ischaemic encephalopathy
has good limb flexion but is not actively moving C. Fetal alcohol syndrome
his limbs. D. Kernicterus
E. Maternal opiate use
What is his Apgar score at one minute?
Select one answer only. 10.12
Mohammed is 24 hours old. His mother
A. 10 develops chicken pox (varicella) 1 day after his
B. 9 delivery.
C. 8
D. 7 From the following list of options what is the
E. 6 best advice you could give?
Select one answer only.
Hint: The measurement of Apgar score is
shown in Chapter 10. Perinatal medicine in A. Breastfeeding is contraindicated
Illustrated Textbook of Paediatrics. B. Neonatal infection is unlikely due to
transplacentally acquired antibodies
10.9 C. Reassure and discharge home asking mother
A mother is known to have pre-eclampsia and to return if the baby develops symptoms.
her fetus has shown signs of intrauterine growth D. The infant’s varicella antibody status should
restriction on antenatal scans. He is delivered at be checked
37 weeks and weighs 2.2 kg. He is admitted to E. There is a significant risk of serious neonatal
the Special Care Baby Unit because of his size. infection
He appears well and has had a breast feed.
What is he most at risk of? Questions: Extended Matching
Select one answer only.
10.13
A. Anaemia From the following list (A–K) pick the diagnosis
B. Congenital cardiac abnormality that fits best with the attached clinical
C. Group B streptococcus infection description and picture. Each option may be
D. Hypoglycaemia used once, more than once, or not at all.
E. Hypercalcaemia

10.10
In the UK all newborn babies have a heel-prick
blood sample taken at 5–7 days of age for 57
biochemical screening.
A. Bruising The doctor notices some small white spots on
B. Capillary haemangioma (stork bites) Oliver’s nose and cheeks (Fig. 10.3). Oliver was
C. Erythema toxicum (neonatal urticarial) born by normal vaginal delivery and is currently
D. Group B streptococcal infection feeding well.
10 E.
F.
Milia
Mongolian blue spots
G. Neonatal varicella zoster
H. Peripheral cyanosis
Perinatal medicine

I. Port wine stain (naevus flammeus)


J. Strawberry naevus (cavernous
haemangioma)
K. Traumatic cyanosis

10.13.1
George, a 2-day-old baby boy, is reviewed by the
paediatric doctor at the request of the midwife.
He was born by forceps delivery and has been
Figure 10.2
feeding well. There were no risk factors for sepsis
and mother was well during her pregnancy.
When examined, he appears very well but has
the rash shown in Fig. 10.1. His mother reports
the rash keeps moving around his body.
Hint: The baby looks well.

Figure 10.3

10.13.4
Jessica, a 2-month-old infant, presents to her
family doctor with a mass on her forehead as
shown in Fig. 10.4. It was not present at birth,
and has been gradually increasing in size since it
was first noticed when Jessica was about 3
weeks old. Jessica is well. The only past medical
history of note is an uncomplicated premature
delivery at 33 weeks gestation.

Figure 10.1

10.13.2
Anna, a month-old infant, is reviewed by the
general practitioner. She was born by normal
vaginal delivery, and was well during and after
delivery. Her mother is worried about the mark
on her face (Fig. 10.2). It has not changed in Figure 10.4
appearance since birth.
10.13.3 Hint: It is raised and her mother is not too
58 Oliver, a 4-day-old infant, is having a routine worried as her older brother had a similar lesion
newborn check by the junior paediatric doctor. that resolved between 18 and 24 months.
10.13.5
Adam, a 3-day-old black infant, has his routine
newborn check. The paediatric doctor notices
blue and black macules on his back and buttocks
(Fig. 10.5). Adam was born by normal vaginal
delivery and has been feeding well since birth.

Perinatal medicine
Figure 10.5

59
C. Breech presentation
Answers: Single Best Answer It is common for the baby to not have fully
engaged at this stage of pregnancy.
10.1

10 A. Being growth restricted


Correct. Smoking reduces birthweight and is
D. Fetal breathing movements seen
Fetal breathing movements are a normal sign.
associated with increased risk of stillbirths and
E. Reverse end-diastolic flow in the umbilical
miscarriages. It also is known to adversely affect
artery
Perinatal medicine

fetal lung growth.


Correct. Absent or reversed flow velocity during
B. Dysmorphic syndromes diastole carries an increased risk of morbidity
Alcohol ingestion in pregnancy is associated with from hypoxic damage to the gut or brain, and of
increased risk of fetal alcohol syndrome. intrauterine death.
C. Neural tube defects 10.4
Folic acid should be taken by mothers prior to A. Congenital abnormalities
conception and in the first trimester to reduce Correct. There is a higher rate of congenital
the risk of neural tube defects. abnormalities in twin pregnancies. These occur
D. Shoulder dystocia twice as frequently as in a singleton (but the risk
Shoulder dystocia is associated with infants who is increased four-fold in monochorionic twins).
are large for gestational age and is more B. Gestational diabetes
common in mothers with gestational diabetes or Multiple births do not significantly increase the
poorly controlled diabetes. mother’s risk of gestational diabetes.
E. Vitamin D deficiency
C. Macrosomia
Smoking does not increase the risk of vitamin D
Multiple births are at risk of intrauterine growth
deficiency.
restriction rather than macrosomia.
10.2 D. Post-term gestation
A. Duodenal atresia Multiple births are at risk of preterm rather than
Duodenal atresia may be associated with post-term births. The median gestation for twins
polyhydramnios (excess amniotic fluid). is 37 weeks, 34 weeks for triplets and 32 weeks
B. Gastroschisis for quadruplets.
Gastroschisis should not affect the volume of E. Twin-to-twin transfusion
amniotic fluid. Twin–twin transfusion syndrome is found in
C. Maternal diabetes monochorionic twins (with a shared placenta).
Maternal diabetes may be associated with She has dichorionic, diamniotic twins, and
polyhydramnios (excess amniotic fluid). therefore they are not at risk of this as they have
two separate placentas.
D. Poorly functioning fetal kidneys
Correct. Oligohydramnios, when there is 10.5
reduced or lack of amniotic fluid, may be A. Acrocyanosis (cyanosis of the hands
associated with decreased fetal urine output and feet)
because of abnormal kidneys. Early scans will This is common on the first day and resolves
miss this problem as the fetal skin is not spontaneously.
keratinised and amniotic fluid volume is
proportional to fetal size. By 20 weeks it is B. A heart murmur
dependent upon fetal urine production and that Correct. Although many heart murmurs heard at
the membranes are intact. the first day of life are innocent and will disappear,
some are from congenital heart disease. Further
E. Severe intrauterine growth restriction clinical assessment is required to determine if the
Oligohydramnios may be associated with severe murmur is innocent or significant.
intrauterine growth restriction, but the
sonographer has identified a normal head and C. An undescended testis
abdominal circumference. Urgent surgical opinion for an undescended
testis is not required as many descend
10.3 spontaneously in the first few weeks of life, and
A. Accelerations of fetal heart rate surgical repair is therefore delayed. The testis
Accelerations of the fetal heart rate are a normal should be rechecked at a few weeks of age.
finding.
D. Breast enlargement
B. Active fetal movements Breast enlargement can occur in either sex and a
60 Active fetal movements are a normal sign. small amount of milk may be discharged. This
Reduced fetal movements would suggest fetal resolves spontaneously and does not require
compromise. investigation.
E. Subconjunctival haemorrhages severe scarring of the skin, and possibly
These occur commonly during delivery and ocular and neurological damage. However, this is
reassurance should be given. rare.
10.8
10.6
A. Anaemia D. 7
These newborn infants are at increased risk of Correct. He scores 2 each for a heart rate >100
polycythaemia (venous haematocrit >0.65) rather beats/min and regular respirations, but only 1

Perinatal medicine
than anaemia. each for some hypotonia and flexion of his limbs,
B. Hyperglycaemia a grimace but no cry, and although his body is
Transient hypoglycaemia is common during the pink, his extremities are blue.
first day of life in these infants due to fetal 10.9
hyperinsulinism, which is in response to the high A. Anaemia
levels of maternal glucose crossing the placenta. This baby has intrauterine growth restriction.
C. Respiratory distress syndrome These babies are at risk of polycythaemia rather
Correct. This mother has gestational diabetes than anaemia.
which places the infant at increased risk of B. Congenital cardiac abnormality
respiratory distress syndrome as lung maturation Congenital cardiac anomalies are not caused by
is delayed. intrauterine growth restriction.
D. Neonatal bacterial infection C. Group B streptococcus infection
Infants of diabetic mothers are not at particular This baby has intrauterine growth restriction.
risk of neonatal infection. However, this does not increase the baby’s risk
E. Neonatal type 1 diabetes mellitus of Group B streptococcus infection compared
There is no increased risk of neonatal type 1 with babies of normal weight.
diabetes mellitus in babies born to mothers with D. Hypoglycaemia
gestational diabetes. Correct. The baby has intrauterine growth
restriction. These babies are liable to
10.7 hypoglycaemia from poor fat and glycogen
A. Cytomegalovirus stores.
Correct. This is the most common congenital
infection in the UK. Most babies born to mothers E. Hypercalcaemia
with cytomegalovirus infection during This baby has intrauterine growth restriction.
pregnancy are normal at birth, but 5% have These babies are at risk of hypocalcaemia rather
clinical features such as hepatosplenomegaly than hypercalcaemia.
and petechiae and sensorineural hearing loss. 10.10
Most of these babies will go on to have A. Cystic fibrosis
neurodevelopmental disabilities. 5% who appear This is now screened for in the UK.
normal at birth develop sensorineural hearing
loss later in life. B. Duchenne muscular dystrophy
Correct. This is not routinely screened for.
B. Rubella
Screening with creatine kinase measurement is
Congenital rubella can cause similar signs but is
unreliable at this age, with many false-positive
often associated with cataracts and congenital
and false-negative results, and there is
heart disease as well as deafness; congenital
insufficient evidence of long-term benefit of
rubella is extremely rare in the UK since the MMR
diagnosis in the newborn period.
vaccine was introduced, and all mothers are
screened for rubella antibodies. C. Hypothyroidism
Thyroid function is screened. As this is with TSH
C. Syphilis
in the UK, it may miss the rare cases of central
This is unlikely because mothers in the UK
hypothyroidism.
are screened for syphilis infection, and we
know that this mother’s screening bloods were D. Phenylketonuria
normal. This was the initial disorder screened for and was
called the Guthrie test.
D. Toxoplasmosis
Babies with toxoplasmosis can have some of the E. Sickle cell disease
features described, with the addition of This forms part of the newborn screen in
intracranial calcification, hydrocephalus, and the UK.
retinopathy, but this infection is rare in the UK.
10.11
E. Varicella zoster A. Congenital rubella syndrome 61
Babies whose mothers develop chickenpox in Characteristic clinical features are not present in
the first 20 weeks of pregnancy are at risk of this baby.
B. Hypoxic-ischaemic encephalopathy will be insufficient time for protective antibodies
This could cause some of these neurological to develop and be transferred to the infant. A
abnormalities, but sneezing and yawning and quarter of such infants become infected, with a
satisfactory condition at birth (good Apgar significant mortality.
10 scores) are against this diagnosis.
C. Fetal alcohol syndrome Answers: Extended Matching
These infants have severe growth restriction and
10.13.1
Perinatal medicine

have characteristic facies.


C. Erythema toxicum
D. Kernicterus Erythema toxicum or neonatal urticaria is
Kernicterus is unlikely as severe jaundice is an extremely common rash and appears
absent. at day 2–3 of age. It is usually concentrated on
E. Maternal opiate use the trunk and comes and goes at different sites.
Correct. The clinical features are consistent with The fluid within the papules contains
neonatal abstinence syndrome. eosinophils.

10.12 10.13.2
A. Breastfeeding is contra-indicated I. Port wine stain (naevus flammeus)
Antibodies acquired via breast milk are important This is a port wine stain. It is due to a vascular
for an infant’s immunity, although they will not malformation of the capillaries in the dermis. If
protect against this episode of varicella. along the distribution of the trigeminal nerve, it
may be associated with intracranial vascular
B. Neonatal infection is unlikely due to anomalies.
transplacentally acquired antibodies
The mother is unlikely to have antibodies to 10.13.3
varicella, because she herself has developed E. Milia
chickenpox. Milia are white pimples on the nose and
cheeks. They are due to the retention of
C. Reassure and discharge home asking mother keratin and sebaceous material in the
to return if the baby develops symptoms pilaceous follicles.
The infant is at risk of morbidity and mortality if
discharged home. This advice would be 10.13.4
inappropriate. J. Strawberry naevus (cavernous
haemangioma)
D. The infant’s varicella antibody status should They are not usually present at birth, but appear
be checked in the first month of life. They are more common
It is unlikely that the mother will have any in preterm infants. They increase in size until 3–9
antibodies to varicella, as she herself has months of age and then gradually regress.
developed chickenpox. Therefore, transplacental
transfer of antibodies cannot have occurred, and 10.13.5
the infant will not be immune to varicella. F. Mongolian blue spots
These are blue/black discolorations at the base
E. There is a significant risk of serious of the spine and on the buttocks, most
neonatal infection commonly found in Asian and African infants.
Correct. If the mother develops chickenpox from They fade slowly over the first few years of life,
5 days before until 5 days after delivery, there and are of no clinical significance.

62
11

Neonatal medicine
11.2
Questions: Single Best Answer Robert is a full term male infant, born 10 hours
ago. His mother is blood group O rhesus positive
11.1 and her membranes ruptured 2 days before
Natasha, a female infant, is delivered by delivery. He is breastfeeding well but the
caesarean section at 32 weeks’ gestation because midwife noticed he looks jaundiced. On
of maternal pre-eclampsia. Her birth weight is examination the baby is clinically well but
1.9 kg. No resuscitation is required. At 2 hours of markedly jaundiced.
age she develops respiratory distress, with a
respiratory rate of 70 breaths/min, grunting What investigation should be performed first?
respirations, and indrawing of her rib cage. Select one answer only.
Respiratory support with CPAP (continuous
positive airway pressure) and 45% oxygen is A. Bilirubin level
required. A chest X-ray is taken at 4 hours of age, B. Blood culture
and is shown (Fig. 11.1). C. Blood group
D. Congenital infection screen
E. Direct antibody test

11.3
You are asked to review a newborn baby
who is only 24 hours old and has developed
very swollen eyelids with a purulent
discharge.
What management is required?
Select one answer only.
A. Clean with cool boiled water
B. Intravenous antibiotics
C. Oral antibiotics
D. Reassure that it will resolve spontaneously
E. Topical antibiotic therapy

11.4
Isabelle was born at term weighing 4 kg. At 6
Figure 11.1
hours of age she was noted to be breathing fast
What is the most likely reason that this baby and have a low temperature. She was born by
needs oxygen therapy and respiratory support? normal vaginal delivery and the membranes had
Select one answer only. ruptured 24 hours previously. Isabelle has not
breastfed since birth and has had one vomit. On
A. Aspiration of meconium has resulted in lung
examination, she is lethargic and her core
collapse
temperature is 35.5° C. She has a respiratory rate
B. Blood is still flowing from the pulmonary
of 90 breaths/min, her central capillary refill time
artery to the aorta as in the fetal circulation
is 4 seconds, pulse 180/min and oxygen
C. The alveoli still contain fluid
saturation is 89% in air.
D. The chest wall and ribs are too compliant
E. There is ventilation-perfusion mismatch from Her chest X-ray shows consolidation at the right
surfactant deficiency base.
What is the most likely causative organism for
her infection? Questions: Extended Matching
Select one answer only. 11.7

11 A.
B.
Escherichia coli (E. coli)
Group B streptococcus
The following is a list of possible diagnoses(A–L).
For each clinical case described below select the
most likely diagnosis. Each answer may be used
C. Herpes simplex virus (HSV) infection
once, more than once, or not at all.
D. Listeria monocytogenes
Neonatal medicine

E. Staphylococcus aureus A. Anaemia


B. Bronchopulmonary dysplasia (BPD)
11.5 C. Coarctation of the aorta
James was born at 39 weeks’ gestation by D. Diaphragmatic hernia
elective caesarean section because of E. Heart failure
pre-eclampsia. His birth weight was 3.7 kg. His F. Meconium aspiration
mother breastfed him as soon as she had G. Persistent pulmonary hypertension of the
recovered from the general anaesthetic. He fed newborn (PPHN)
well but is vomiting after every feed. He is now H. Pneumonia
18 hours old and after the last 2 feeds he I. Pneumothorax
‘vomited everything up’ and it was greenish in J. Respiratory distress syndrome
colour. On examination his temperature is 36.5° C K. Tracheo-oesphageal fistula
and he is alert and hungry. L. Transient tachypnoea of the newborn

His abdomen is not distended. He has not yet 11.7.1


passed meconium. Mohammed was born at term, weighing 3 kg. He
is 6 hours old and on the postnatal ward with his
What is the most likely diagnosis? mother, who has asked the midwife to review
Select one answer only. him as he is breathing very quickly. Mohammed
did not breathe well immediately after birth and
A. Duodenal atresia needed mask ventilation to establish breathing.
B. Hirschprung disease By 5 minutes of age he was crying and was
C. Meconium ileus wrapped and handed to his mother. On
D. Neonatal sepsis examination he is breathing at 64 breaths/min,
E. Pyloric stenosis and there is mild chest recession. Breath sounds
on the left side are reduced compared with the
11.6 right. His oxygen saturation is 95%. The chest
Aaron was born at term, birth weight 3.2 kg. He X-ray is shown (Fig. 11.3).
is now 2 weeks old, and his mother is concerned
that his umbilicus looks abnormal. A photo is
shown in Fig. 11.2.

Figure 11.2
Select the most likely diagnosis.
Select one answer only.
A. Exomphalos
B. Gastroschisis
C. Umbilical granuloma
64 D. Umbilical hernia
E. Umbilical infection (omphalitis) Figure 11.3
11.7.2 11.8
Sabrina was born at 37 weeks’ gestation, birth The following is a list of diagnoses that are
weight 2.8 kg. She was noted by the midwife to associated with jaundice in the newborn
be breathing very fast at 2 hours of age. On period. For each of the following scenarios pick
examination she is breathing at 72 breaths/min the most likely cause of the jaundice. Each
and has moderate chest recession. The heart answer may be used once, more than once, or
sounds are difficult to hear on the left and her not at all.
apex beat is palpable on the right side of the

Neonatal medicine
chest. A chest X-ray is taken (Fig. 11.4). A. ABO incompatibility
B. Biliary atresia
C. Breast milk jaundice
D. Congenital hypothyroidism
E. Congenital infection with cytomegalovirus
F. Crigler–Najjar syndrome
G. G6PD deficiency
H. Physiological jaundice
I. Rhesus haemolytic disease
J. Sepsis
K. Urinary tract infection

11.8.1
Stewart is a full-term baby boy, born 16 hours
ago. His mother is blood group O rhesus
positive. The baby is breastfeeding well but the
midwife has noticed he looks jaundiced. On
examination the baby is clinically well. His
bilirubin was 150 µmol/L at 10 hours and he was
started on intensive phototherapy. Six hours
later his bilirubin is 250 µmol/L. Stewart’s blood
group was identified as group A rhesus positive.

11.8.2
Alfie is a 3-week-old male infant whose mother is
concerned that his stools are pale. He is
breastfed. On examination he is jaundiced, has
mild hepatomegaly and has only just regained
his birthweight.
Figure 11.4
11.8.3
Poppy is 2 weeks old and is breastfed. She is
11.7.3 thriving but is jaundiced. The midwife does a
Thomas, a baby boy with a birth weight of 875 g bilirubin which is moderately raised at
at 28 weeks’ gestation required artificial 170 µmol/L, and is nearly all unconjugated. A
ventilation for 2 weeks. At 10 weeks of age he urine dipstick is negative.
still needs additional oxygen via nasal cannulae.
This is his chest X-ray (Fig. 11.5) 11.8.4
Dimitri is 20 hours old and is noted to be
markedly jaundiced, needing intensive
phototherapy. Maternal blood is group A rhesus
positive. His blood group is group A rhesus
positive. He has been breastfeeding well. On
examination he is markedly jaundiced, but is
alert and active.

65
Figure 11.5
11.9 11.9.1
The following is a list (A–L) of diagnoses that are Rebecca was born 48 hours ago at term. She
associated with respiratory distress in the weighed 3.2 kg. You are asked to review her on
newborn period. For each of the following the postnatal ward as she is breathing very
11 scenarios pick the most likely cause of the
jaundice. Each answer may be used once, more
quickly. She is not feeding. On examination she
is breathing at 68 breaths/min and has mild
than once or not at all. chest recession. She looks unwell. You cannot
confidently feel her femoral pulses. Her oxygen
Neonatal medicine

For each of the scenarios, choose the most likely saturation is 85% in air.
diagnosis from the list.
11.9.2
A. Anaemia Zak, a full-term male infant, with a birth weight
B. Bronchopulmonary dysplasia (BPD) of 3.7 kg, is born by elective caesarean section.
C. Coarctation of the aorta His mother was well during pregnancy and had a
D. Diaphragmatic hernia normal blood glucose screen. Zak becomes
E. Heart failure tachypnoeic with indrawing between his ribs at
F. Meconium aspiration 2 hours of age. Examination is otherwise normal.
G. Persistent pulmonary hypertension of the A chest X-ray looks normal.
newborn
H. Pneumonia
I. Pneumothorax
J. Respiratory distress syndrome
K. Tracheo-oesphageal fistula
L. Transient tachypnoea of the newborn

66
11.3
Answers: Single Best Answer A. Clean with cool boiled water
The eyes should be cleaned and carefully
11.1 examined but also treated pending the results of
A. Aspiration of meconium has resulted in lung any swabs.
collapse
Meconium aspiration is associated with term or B. Intravenous antibiotics
post-term infants. There are patchy changes on Correct. Purulent discharge and eyelid swelling

Neonatal medicine
chest X-ray (CXR). in the first 48 hours of life must be taken
seriously. This should be treated promptly, e.g.
B. Blood is still flowing from the pulmonary with a third-generation cephalosporin
artery to the aorta as in the fetal circulation intravenously, as permanent loss of vision can
Some blood may ‘shunt’ across a PDA (persistent occur. The most common cause of severe
ductus arteriosus) but this would usually flow neonatal purulent conjunctivitis is Chlamydia
from the high pressure aorta into the pulmonary trachomatis, but gonococcus may also be the
artery as pulmonary vascular resistance should cause.
drop after birth and inflation of the lungs.
C. Oral antibiotics
C. The alveoli still contain fluid Oral antibiotic absorption in the newborn is
Retention of a small amount of fluid is common highly variable and therefore cannot be relied
and results in transient tachypnoea of the upon to treat this (or other) infections.
newborn. D. Reassure that it will resolve spontaneously
D. The chest wall and ribs are too compliant Whilst incomplete canalization of the
The ribs are compliant but this is not the main nasolacrimal duct commonly leads to sticky eyes,
reason for respiratory distress. the discharge is rarely purulent and the eyes are
not swollen.
E. There is ventilation-perfusion mismatch E. Topical antibiotic therapy
from surfactant deficiency Topical antibiotic therapy is often prescribed for
Correct. Surfactant deficiency is common in a sticky eye in older infants and children but is
preterm infants. This results in alveolar collapse insufficient in this situation.
which in turn results in areas of lung being
perfused but not ventilated. The CXR appearance 11.4
here is typical with a diffuse granular or ‘ground A. Escherichia coli (E. coli)
glass’ appearance. An important cause of neonatal sepsis but not as
common as Group B streptococcus. Antibiotic
regimens to treat newborn infants must cover
11.2
this organism though.
A. Bilirubin level
Correct. Jaundice starting at less than 24 hours B. Group B streptococcus
of age is most likely due to haemolysis and Correct. Isabelle is most likely to have early-
may rapidly rise to dangerously high levels. It onset sepsis and this can be caused by infection
needs urgent assessment and close monitoring. in the chest, urine or cerebrospinal fluid
The most urgent investigation is to measure the (meningitis). The most common organism
bilirubin level, as this will determine the causing early-onset sepsis in the UK is Group B
management required. streptococcus.
Listeria monocytogenes, E. coli, and
B. Blood culture Staphylococcus aureus may cause early-onset
Blood cultures are taken to exclude infection but sepsis, but are less common causes in the UK.
do not influence immediate management.
C. Herpes simplex virus (HSV) infection
C. Blood group HSV infection in the neonate is rare but very
Blood group will inform if ABO or Rhesus serious when it occurs. It can present with
incompatibility is a likely cause but will not localized lesions, encephalitis, or disseminated
influence immediate management. disease. Skin lesions may not be present.
Treatment is with intravenous aciclovir. Primary
D. Congenital infection screen maternal HSV infection may not be diagnosed at
Congenital infection can cause early jaundice but delivery.
there are usually clinical features and the
jaundice is mild. D. Listeria monocytogenes
An important cause of neonatal sepsis and
E. Direct antibody test meningitis but rare in the UK. Antibiotic
Direct antibody test is positive with rhesus regimens to treat newborn infants must cover 67
disease and ABO incompatibility. this organism though.
E. Staphylococcus aureus
A common cause of skin infection but can cause Answers: Extended Matching
sepsis or staphylococcal scalded skin syndrome
in the newborn. Blistering rashes or inflammation 11.7.1

11 around the umbilicus increase the likelihood of


this infection.
I. Pneumothorax
This may have been spontaneous, but the risk is
increased by the positive pressure ventilation
11.5 during resuscitation. Nonetheless, a pneumothorax
Neonatal medicine

A. Duodenal atresia may occur spontaneously in 1–2% of newborn


Correct. This is the most likely cause of infants. The reduced breath sounds are because of
persistent bilious vomiting on the 1st day of life. the pneumothorax. The diagnosis is confirmed on
Malrotation and volvulus can also cause this chest X-ray, with hypertranslucency on the left, the
presentation, but the vomit may contain blood left lung edge is visible, and the mediastinum is
and the abdomen may be tender. A plain displaced to the right.
abdominal X-ray would be helpful as it may
show the classic ‘double bubble’. 11.7.2
D. Diaphragmatic hernia
B. Hirschprung disease The chest X-ray shows loops of bowel in the left
Affects the rectum and sometimes the colon, chest and there is displacement of the
therefore causes distal bowel obstruction mediastinum
causing marked abdominal distension.
11.7.3
C. Meconium ileus B. Bronchopulmonary dysplasia (BPD)
Affects the lower ileum, so there would be This is the name used for infants born preterm
abdominal obstruction and therefore distension. who still have an oxygen requirement at a
postmenstrual age of 36 weeks. The X-ray
D. Neonatal sepsis
characteristically shows widespread areas of
Neonatal sepsis can cause vomiting, which is
opacification and cystic changes.
sometimes slightly bile-stained, but there are no
risk factors from labour and delivery, and no 11.8.1
other features on clinical examination. A. ABO incompatibility
This is now more common than rhesus
E. Pyloric stenosis haemolytic disease. Most ABO antibodies are IgM
In pyloric stenosis the vomit is not bile-stained as and do not cross the placenta, but some group O
the obstruction is above the ampulla of Vater women have an IgG anti-A-haemolysin in the
and it presents at about 6 weeks of age. blood which can cross the placenta and
11.6 haemolyses the red cells of a group A infant.
A. Exomphalos Occasionally, group B infants are affected by
In exomphalos the abdominal contents protrude anti-B haemolysins.
through the umbilical ring, covered by the 11.8.2
amniotic membrane and peritoneum. B. Biliary atresia
B. Gastroschisis Conjugated hyperbilirubinaemia is suggested by
In gastroschisis the bowel protrudes through a the pale stools accompanying his jaundice. His
defect in the anterior abdominal wall adjacent to urine will be dark but this may not be
the umbilicus. recognized. There are other causes of conjugated
hyperbilirubinaemia in infants of this age but it is
C. Umbilical granuloma important to diagnose biliary atresia as early as
Correct. In umbilical granuloma there is a pink, possible, as early surgery improves prognosis.
pedunculated lesion of granulation tissue as
shown here. 11.8.3
C. Breast milk jaundice
D. Umbilical hernia In most infants with prolonged neonatal
In umbilical hernia there is protrusion of the jaundice, the hyperbilirubinaemia is
umbilicus. These are common and resolve unconjugated, but this needs to be confirmed on
spontaneously. laboratory testing. In prolonged unconjugated
hyperbilirubinaemia, ‘breast milk jaundice’ is the
E. Umbilical infection (omphalitis) most common cause, affecting up to 15% of
In umbilical infection (omphalitis) there is healthy breastfed infants; the jaundice gradually
redness of the skin surrounding the umbilicus, fades and disappears by 4–5 weeks of age.
with spread of the redness onto the anterior
abdominal wall (umbilical flare). Sometimes there 11.8.4
is purulent discharge, although an umbilical G. G6PD deficiency
68 granuloma can also be ‘sticky’ as it produces Dimitri has severe jaundice at less than 24 hours
mucus. of age, therefore it is likely to be haemolytic. His
mother’s and his blood group, A and rhesus arteriosus closes as this is a classic ‘duct-
positive, exclude rhesus disease and ABO dependent’ lesion.
incompatibility. His parents’ Mediterranean
origin is compatible with G6PD deficiency, a 11.9.2
common and important cause of haemolytic L. Transient tachypnoea of the newborn
jaundice worldwide. The most common cause of respiratory distress
in a term infant. It is due to a delay in the
11.9.1 resorption of lung liquid – an increased risk

Neonatal medicine
C. Coarctation of the aorta following caesarean section, as the babies have
The absence of femoral pulses should always not had to undergo the same physical and
suggest that there is coarctation of the aorta. Her physiological stressors as those who pass
left brachial pulse will also be difficult to palpate. through the birth canal.
Heart failure develops when the ductus

69
12

Growth and puberty

(see Fig. 12.5, Illustrated Textbook of Paediatrics).


Questions: Single Best Answer She has not started her periods, but has had a
growth spurt recently. An ultrasound of her
12.1 pelvis shows multicystic ovaries and enlarging
Janine, a 9-month-old female infant, is seen by uterus.
her family doctor because of concern that she is
not growing fast enough. She is only on the 5th What is the most likely cause of her early
centile for height and 2nd centile for weight. pubertal development?

What is the greatest influence on her growth rate Select one answer only.
at her age?
A. Brain tumour
Select one answer only. B. Congenital adrenal hyperplasia
C. Idiopathic precocious puberty
A. Genes D. Ovarian tumour
B. Growth hormone E. Turner syndrome
C. Nutrition
D. Oestrogen 12.4
E. Testosterone Sophia, an 18-month-old girl, is brought to
outpatients by her mother, who is very worried
12.2 as she has developed breasts. She is otherwise
John, a 3-year-old boy, is referred to paediatric well and has been growing normally. On
outpatients because of concern about the examination she has breast development stage 3
development of pubic and axillary hair. His testes (BIII) but no pubic or axillary hair. Her bone age is
are 1.5 ml in size (prepubertal). His blood 20 months.
pressure is 100/75 mmHg. Gonadotrophin levels
are normal for a prepubertal boy (undetectable) What is the most likely cause of her early
but his bone age is 5 years. pubertal development?

What is the most likely cause of his early pubertal Select one answer only.
development? A. Brain tumour
Select one answer only. B. Congenital adrenal hyperplasia
C. Idiopathic precocious puberty
A. Adrenal tumour D. Premature thelarche
B. Brain tumour E. Premature pubarche
C. Idiopathic precocious puberty
D. Prader–Willi syndrome 12.5
E. Testicular tumour Tom, a 7-year-old boy, is referred by his general
practitioner with concerns about his growth. He
12.3 is an adopted child and no details
Chelsea, a 7-year-old girl, is brought by her are available about his biological father although
mother to the paediatric clinic. Her mother is his biological mother was ‘of average height’.
concerned about Chelsea’s early development Physical examination reveals a happy and
of puberty. Chelsea has started to develop playful boy with no dysmorphic features. His
breasts and, more recently, some pubic hair. height is 110 cm which is just below the 0.4th
On examination she has breast development centile. His weight is on the 0.4th centile.
stage 3 and pubic hair development stage 2 He has a normal physical examination.
What is the most likely cause for his short stature? 12.6.1
May, a 14-year-old girl, is referred by her
Select one answer only. school nurse because she is found to be
A. Achondroplasia below the 0.4th centile for height. He mother
B. Constitutional delay of growth and puberty is 167 cm tall and father is 175 cm tall. She was
C. Familial short stature born at term, birth weight 3.2 kg. She says
D. Growth hormone deficiency she has always been the shortest in the class.
E. Vitamin D deficiency Physical examination reveals a girl in the early

Growth and puberty


stages of puberty. Her previous history is
unremarkable. Her blood pressure is
Questions: Extended Matching
135/65 mmHg.
12.6
The following (A–M) is a list of investigations for 12.6.2
children referred with short stature. From the Eleanor, a 7-year-old girl, is referred because of
following clinical scenarios pick the investigation short stature. She is on the 5th centile for height
which is most likely lead to a correct diagnosis. and 25th centile for weight. Her growth chart
Each answer may be used once, more than once, shows that 9 months previously her height and
or not at all. weight were on the 15th centile. Her mother
says that she has become lethargic, her school
A. Bone age (wrist X-ray) work has deteriorated and that she now refuses
B. Coeliac screen to do sport.
C. C-reactive protein (CRP) and erythrocyte
sedimentation rate (ESR) 12.6.3
D. Creatinine and electrolytes Jake, a 5-year-old boy with Down syndrome
E. Full blood count has a check-up with his general practitioner.
F. Growth hormone provocation tests He is short but until 6 months ago was following
G. Insulin-like growth factor-1 (IGF-1) his growth centiles. You note that his height
H. Karyotype has dropped one centile line and his weight
I. MRI scan of the brain two centile lines. His mother reports that he has
J. Skeletal survey become very irritable and difficult to manage.
K. Thyroid stimulating hormone (TSH) On direct questioning, his appetite has
L. Ultrasound scan of the uterus deteriorated, with meal times becoming
M. Vitamin D levels problematic.

71
B. Congenital adrenal hyperplasia
Answers: Single Best Answer In congenital adrenal hyperplasia, the sequence
of pubertal changes is abnormal, with isolated
12.1. pubic hair and virilisation of genitalia.
12 A. Genes
Genes are important but without adequate C. Idiopathic precocious puberty
Correct. Precocious puberty in females is usually
nutrition a child’s growth potential will not be
achieved. Genetic influences begin to affect due to premature onset of normal puberty. The
Growth and puberty

growth mostly after the 1st year of life. sequence of puberty in this child is normal and
there is also been an associated growth spurt,
B. Growth hormone which makes an idiopathic cause for the
Growth hormone is particularly important in precocious puberty more likely.
determining growth after infancy and continues
to exert an effect until growth ceases. D. Ovarian tumour
The pelvic ultrasound findings are consistent
C. Nutrition with premature onset of normal puberty.
Correct. Along with good health, happiness and
thyroid hormones, infant growth (from birth to E. Turner syndrome
12 months of age) is most dependent on good Turner syndrome is associated with delayed
nutrition. Growth in the 1st year of life rather than precocious puberty.
contributes about 15% to final adult height. 12.4.
D. Oestrogen A. Brain tumour
The sex hormones cause the back to A brain tumour is unlikely as she has isolated
lengthen and boost growth hormone breast development.
secretion. This occurs during the pubertal B. Congenital adrenal hyperplasia
growth spurt. Congenital adrenal hyperplasia would cause
E. Testosterone premature pubarche rather than isolated breast
The sex hormones cause the back to development.
lengthen and boost growth hormone C. Idiopathic precocious puberty
secretion. This occurs during the pubertal Sophia does not have precocious puberty
growth spurt. because she does not have axillary or pubic hair,
12.2. nor has she had a growth spurt.
A. Adrenal tumour D. Premature thelarche
Correct. Premature sexual development in boys Correct. Sophia has breast development and no
is uncommon and usually has an organic (rather other signs of puberty. She has premature
than constitutional or familial) cause. With his thelarche.
prepubertal testes, hypertension and normal
gonadotrophin levels, the abnormality is likely to E. Premature pubarche
be in his adrenal glands. She has not developed pubic or axillary hair.

B. Brain tumour 12.5.


John has prepubertal testes; therefore he does A. Achondroplasia
not have true precocious puberty and the cause He has no dysmporhic features. There would be
is not likely to be central. obvious disproportion with shorter limbs if this
was the diagnosis.
C. Idiopathic precocious puberty
He has prepubertal testes so does not have B. Constitutional delay of growth and puberty
precocious puberty. Tom is too young to present with this. His
pubertal growth spurt would not be expected
D. Prader–Willi syndrome until the second decade of life.
Boys with Prader–Willi syndrome have delayed
puberty. C. Familial short stature
Correct. Most short children have short parents
E. Testicular tumour and fall within the centile target range allowing
John has prepubertal testes; therefore, he does for midparental height. Care needs to be taken,
not have precocious puberty and the cause is though, that both the child and a parent do not
not likely to be coming from his testes. have an inherited growth disorder, such as a
skeletal dysplasia.
12.3.
A. Brain tumour D. Growth hormone deficiency
You would be more worried about the likelihood It is possible, but not the most likely cause of
72 of a brain tumour if this was a boy showing signs short stature. Careful re-evaluation and
of early puberty. assessment of the growth velocity will be helpful.
E. Vitamin D deficiency 12.6.2
It is unlikely to be the cause of growth failure K. Thyroid stimulating hormone (TSH)
without other signs of rickets (swollen wrists, Her height has stopped increasing and she has
bent legs). put on weight. This suggests an endocrine
problem. Her clinical symptoms suggest
Answers: Extended Matching hypothyroidism.

12.6.1 12.6.3

Growth and puberty


H. Karyotype B. Coeliac screen
Turner syndrome needs to be excluded as she Children with Down syndrome are at increased
has always been short. The elevated blood risk. He is also at increased risk of
pressure may be because of an undetected hypothyroidism, but his decreased appetite,
coarctation of the aorta which is associated with irritability and weight loss are characteristic of
the syndrome. coeliac disease.

73
13

Nutrition
13.2
Questions: Single Best Answer Sarah, a 9-year-old girl, is referred by the school
nurse to the paediatric clinic because of her
13.1 weight. She weighs 43 kg (98th centile) and is
Sunit, a 13-month-old boy, presents with 141 cm tall (91st centile). She has followed her
faltering growth. He is still entirely breastfed. height centiles for the last 9 months but her
On examination, he is miserable and his wrist is weight centile has increased. Her body mass
shown in Fig. 13.1a. An X-ray is taken of his wrist index is on the 97th centile. Her mother reports
is shown in Fig. 13.1b. that she hardly eats at all and when she does she
has a very healthy diet.
Which of the following statements is most likely
to be correct?
Select one answer only.
A. A calorie-restricted diet is the treatment of
choice
B. Sarah’s adrenocortical axis should be
checked to exclude Cushing syndrome
C. Sarah has a higher risk of an abnormal lipid
profile and raised blood pressure in adult life
D. Sarah is obese
(a) E. Sarah’s main problem is that she has a low
metabolic rate

13.3
Which of the following term newborns has the
lowest risk of cardiovascular disease in later life?
Select one answer only.
A. 1.8 kg
B. 2.1 kg
C. 2.4 kg
D. 3.9 kg
E. 4.6 kg

(b) 13.4
A mother asks you whether there are any
Figure 13.1 a) Courtesy of Nick Shaw. disadvantages to breastfeeding. Although you
would prefer to inform her about the many
What is the most likely diagnosis? advantages of breastfeeding, you wish to answer
Select one answer only. her question honestly. Which of the following is
most likely to be a true potential disadvantage?
A. Vitamin A deficiency
B. Vitamin B1 deficiency
C. Vitamin D deficiency
D. Vitamin E deficiency
E. Vitamin K deficiency
Select one answer only. diagnosis from the list (A–J) below. Each option
may be used once, more than once, or not at all.
A. Breastfeeding will reduce her chance of
having more children A. Cow’s milk protein allergy
B. The absence of cow’s milk protein in breast B. Cystic fibrosis
milk increases the risk that the child will C. Kwashiorkor
develop milk allergy at weaning D. Marasmus
C. The higher interferon level in breast milk E. Normal

Nutrition
increases the risk of severe bronchiolitis in F. Obesity
children who develop respiratory syncytial G. Vitamin A deficiency
virus infection H. Vitamin C deficiency
D. The lower vitamin K concentration in breast I. Vitamin D deficiency
milk can result in life-threatening bleeding J. Vitamin K deficiency
E. The strong bond developed during
breastfeeding will prevent paternal bonding 13.7.1
Ahmed is an 18-month-old Pakistani boy who
was born in the UK with a weight of 3.2 kg. He is
13.5
on a mixed diet. His height is on the 10th centile
Anil is a 2½-year-old boy who lives in India and
and his weight is on the 0.4th centile. He is
attends the local health clinic near their village
noted to be miserable. On examination, his
for a routine check. Both his parents are
wrists also feel wider than normal.
subsistence farmers. He is asymptomatic. On
examination, he is very thin but his hair and skin 13.7.2
appear normal and there is no oedema or other Harry is a 3-week-old infant who has been
clinical abnormalities. His height is on the 5th exclusively breastfed by his mother. He was born
centile but his weight is well below the 0.4th at home as his mother wanted ‘everything to be
centile (z-score between −2 and −3 below the natural’ and declined all interventions. His
median). birthweight was 3.4 kg. He presents to the
hospital with severe rectal bleeding and shock.
What is the most likely diagnosis?
13.7.3
Select one answer only.
Jonas is an 18-month-old black African boy in
A. Kwashiorkor KwaZulu Natal, South Africa. He was born
B. Marasmus weighing 3.2 kg. He was breastfed until 9
C. Normal child months of age when his sibling was born. He
D. Rickets now mainly eats the traditional maize-based
E. Severe gastro-oesophageal reflux porridge, which is grown on the family farm. His
weight is just below the 0.4th centile. He looks
13.6 thin but has a distended abdomen. There is
Harry is a 13-year-old boy who attends the oedema around his eyes and the top of his feet.
paediatric clinic because of obesity. His height is His hair has a red tinge.
on the 98th centile and his weight is above the
13.7.4
99.6th centile.
Jamie is a 5-month-old male infant who was
Which of the following is least likely to be born with a weight of 3.5 kg (25th centile). He
associated with obesity? was initially breastfed and was growing well. His
mother developed mastitis and so he was
Select one answer only. changed to formula milk feeds. He now weighs
A. Asthma 5.0 kg (<0.4th centile). He has frequent loose
B. Hypertension stools and eczema.
C. Low self-esteem 13.7.5
D. Slipped upper femoral epiphysis Tanya, an 11-month-old Caucasian girl, is being
E. Type 1 diabetes monitored by her health visitor. Her birthweight
was 2.4 kg (0.4th centile) and she has remained
on the 2nd centile, now weighing 7.0 kg. Her
Questions: Extended Matching mother is on the 5th centile and her father is on
the 40th centile for height. She is well, has a
13.7 good appetite and has never needed to visit her
For each of the following patients with doctor. She has no abnormal signs on
nutritional problems select the most likely examination and her development is normal.

75
D. Sarah is obese
Answers: Single Best Answer For clinical use, obese children are those with a
BMI above the 98th centile of the UK 1990
13.1 reference chart for age and sex. Sarah is
13 A. Vitamin A deficiency
Clinical manifestations of vitamin A deficiency
overweight (BMI > 91st centile).
E. Sarah’s main problem is that she has a low
include eye damage from corneal scarring and
impairment of mucosal function and immunity. metabolic rate
Nutrition

These complications are seen in low-resource In most cases, obesity results from an increased
countries. intake of energy-dense foods and reduced
exercise.
B. Vitamin B1 deficiency
Thiamine (vitamin B1) is a cofactor for many 13.3
enzymes. Clinical features include cardiac A. Birthweight 1.8 kg
features (e.g. heart failure) and neurological Evidence suggests that undernutrition in utero
features (e.g. polyneuropathy). resulting in growth restriction is associated with
an increased incidence of coronary heart disease,
C. Vitamin D deficiency
stroke, type 2 diabetes, and hypertension in
Correct. He has vitamin D deficiency resulting in
later life.
rickets. This classically causes bowing of the legs
but now more often presents with poor growth. B. Birthweight 2.1 kg
The ends of the radius and ulna, as shown on the The risk is significantly higher than babies born
x-ray of the wrists, (and the tibia and fibula at the weighing between 3.9 kg and 4.3 kg.
ankles) are expanded and rarefied and cup
shaped. At 13 months of age, breast milk alone C. Birthweight 2.4 kg
does not provide adequate intake of vitamin D. The risk is still significantly higher than
babies born weighing between 3.9 kg and
D. Vitamin E deficiency 4.3 kg.
Vitamin E deficiency is rare. It is a fat-soluble
vitamin and can therefore result from severe fat D. Birthweight 3.9 kg
malabsorption, e.g. abetalipoproteinemia or in Correct. The lowest risk of cardiovascular
cystic fibrosis. Individuals with CF are routinely disease, almost half that of babies born weighing
prescribed vitamin supplements to counter these less than 2.5 kg. This is the Barker hypothesis –
problems. Vitamin E deficiency causes a see Figure 13.3 in Illustrated Textbook of
neuropathy and ataxia. Paediatrics.

E. Vitamin K deficiency E. Birthweight 4.6 kg


Vitamin K deficiency affects the clotting factors II, The risk appears to increase again for very heavy
VII, IX, and X. It is a fat-soluble vitamin, and can babies and this may be related to factors such as
be due to fat malabsorption or present as maternal diabetes.
haemorrhagic disease of the newborn.
13.4
13.2 A. Breastfeeding will reduce her chance of
A. A calorie-restricted diet is the treatment of having more children
choice Breastfeeding leads to an increased period
The current recommendation is for a well- between children on a population basis.
balanced healthy diet with increased physical However, it is not an effective form of
activity rather than calorie restriction alone. Many contraception, and does not decrease
surveys of food intake of overweight children fertility.
show that the quantity of food they consume is
no greater than normal weight children. B. The absence of cow’s milk protein in breast
milk increases the risk that the child will develop
B. Sarah’s adrenocortical axis should be checked milk allergy at weaning
to exclude Cushing syndrome Cow’s milk allergy is more commonly seen in
Cushing syndrome is rare and associated with a bottle-fed infants. However, a small amount of
failure of linear growth. milk protein does pass into the breast milk and
cow’s milk allergy occurs in up to 1 in 200
C. Sarah has an increased risk of an abnormal breastfed babies. Exposure to small amounts of
lipid profile and raised blood pressure in cow’s milk protein probably lowers rather than
adult life increases the risk.
Correct. Sarah is overweight [body mass index
(BMI) > 91st centile] rather than obese (BMI > C. The higher interferon level in breast milk
98th centile). It places her at increased risk of an increases the risk of severe bronchiolitis in
76 abnormal lipid profile and raised blood pressure children who develop respiratory syncytial virus
in adult life unless action is taken. infection
Interferon is present in breast milk and has an due to increased mechanical stress placed across
antiviral property. Breastfeeding does not lead to the space between the upper femoral epiphysis
a more severe bronchiolitis. and the femur.
D. The lower vitamin K concentration in E. Type 1 diabetes
breast milk can result in life-threatening Correct. Obese children are more likely to have
bleeding non-insulin dependent diabetes mellitus (type 2
Correct. The vitamin K concentration in breast diabetes). Type 1 diabetes is an autoimmune

Nutrition
milk is lower. There is insufficient vitamin K in disorder and is not related to a child’s weight.
breast milk to reliably prevent haemorrhagic
disease of the newborn. This risk is minimized by Answers: Extended Matching
giving prophylactic vitamin K.
13.7.1
E. The strong bond developed during
I. Vitamin D deficiency
breast-feeding will prevent paternal bonding
This is more common in children with dark skin.
Breastfeeding enhances mother–child
The clinical features of rickets in this child may
relationship but it does not decrease paternal–
include bowing of his legs and widening of the
child bonding.
wrists. There is increased prevalence of rickets in
13.5 young children from Southeast Asia living in the
A. Kwashiorkor UK, from dietary deficiency and inadequate
Kwashiorkor is another manifestation of severe exposure to sunlight.
protein malnutrition, but oedema is present.
13.7.2
B. Marasmus J. Vitamin K deficiency, causing
Correct. In marasmus there is severe protein– haemorrhagic disease of the newborn
energy malnutrition. Oedema is absent. There is insufficient vitamin K in breast milk to
reliably prevent the disorder. Most babies are
C. Normal child given prophylactic vitamin K at birth, usually
Anil is severely underweight. intramuscularly or else orally, but this requires
D. Rickets parental consent. Formula feeds are
Children with rickets can have faltering growth, supplemented with vitamin K.
but they may also have clinical features of
13.7.3
rickets: bowing of legs, rachitic rosary, and
C. Kwashiorkor
swelling of ankles and wrists.
He has severe protein malnutrition accompanied
E. Severe gastro-oesophageal reflux by oedema. Because of the oedema, the weight
There is no history of vomiting. may not be as severely reduced as in marasmus.
He has other features of kwashiorkor, the
13.6 depigmented hair and distended abdomen.
A. Asthma
For reasons that are unclear, asthma is more 13.7.4
common in obese children and adults. A. Cow’s milk protein allergy
He developed faltering growth, loose stools and
B. Hypertension
eczema when changed from breast milk to
Children who are obese are more likely to
formula milk, which is based on cow’s milk
develop hypertension.
protein.
C. Low self-esteem
Children who are obese are more likely to have 13.7.5
low self-esteem. E. Normal infant
She is growing normally along the 2nd centile
D. Slipped upper femoral epiphysis for weight, and has no symptoms to suggest an
Children who are obese are more likely to underlying illness. She has short parents and is
develop this complication. This is thought to be constitutionally small.

77
14

Gastroenterology

colour and usually contain undigested food. The


Questions: Single Best Answer rest of the family are well. He has never been
abroad. Examination is normal and his personal
14.1 child health record shows that he is growing
Benjamin is a 6-year-old boy who is seen in the along the 50th centile.
paediatric emergency department. He has been
vomiting and has had diarrhoea for 3 days. What is the most likely diagnosis?
His stool is watery and foul smelling but has no
A. Chronic non-specific diarrhoea
blood in it. He has not been out of the UK since
B. Coeliac disease
he was born. Examination reveals mild
C. Cow’s milk protein allergy
dehydration but is otherwise normal.
D. Inflammatory bowel disease
What is the most likely organism that has caused E. Lactose intolerance
his symptoms?
14.4
Select one answer only.
Ellie is a 4-year-old girl who has been
A. Campylobacter complaining of pain in her tummy for a
B. Escherichia coli month. It is worse when she goes to the toilet;
C. Giardia lamblia her stools are firm and she opens her bowels only
D. Rotavirus every 2–3 days. She has not had any vomiting.
E. Shigella For the last 2 weeks her stools have become
loose. On examination she has a mass in the left
14.2 iliac fossa.
Emma is an 18-month-old girl who is seen in the
What is the most likely diagnosis?
outpatient department. She presents with loose
stools two to three times each day, with no Select one answer only.
blood in them. She is generally difficult and it
has become a battle to get her to feed. A. Appendix mass
Examination of her abdomen, although difficult B. Constipation
due to distress at being examined, is C. Gastroenteritis
unremarkable apart from being distended. She is D. Inguinal hernia
not on any medication. Her weight was on the E. Wilms tumour
9th centile and is now below the 0.4th centile.
14.5
What is the most likely cause for the weight loss?
Aiysha is a 2-month-old baby who is seen in the
Select one answer only. paediatric outpatient department. She was born
at term, weighing 3.5 kg and is breastfed. Her
A. Chronic non-specific diarrhoea mother is concerned as she has vomited some of
B. Coeliac disease the milk after most feeds since birth. She cries
C. Hirschsprung disease when she vomits. She is continuing to grow
D. Lactose intolerance along the 50th centile.
E. Ulcerative colitis
What is the most likely diagnosis?
14.3 Select one answer only.
Rodney, a boisterous 2-year-old, has had
diarrhoea for the last 3 months. He produces up A. Gastro-oesophageal reflux
to four stools a day, which are loose, brown in B. Helicobacter pylori infection
C. Infant colic What is the most likely diagnosis?
D. Overfeeding
E. Pyloric stenosis Select one answer only.
A. Congenital hypothyroidism
14.6 B. Cystic fibrosis
Claire, a 7-year-old girl, has had abdominal pain C. Duodenal atresia
for the last 6 months. On several occasions it has D. Hirschsprung disease
been sufficiently severe for her to be sent home E. Rectal atresia

Gastroenterology
early from school. The pain happens once or
twice a week in the afternoon or early evening. It Questions: Extended Matching
is periumbilical in nature. It does not wake her at
night. She has not had vomiting or diarrhoea. 14.9
She is growing well. Her examination is normal. The following (A–O) is a list of possible diagnoses
Her urine is clear on dipstick testing. which result in vomiting. For each of the
What is the most likely cause for her pain? following scenarios pick the most likely cause for
the vomiting. Each answer may be used once,
Select one answer only. more than once, or not at all.
A. Functional abdominal pain A. Appendicitis
B. Gastritis B. Coeliac disease
C. Hepatitis A C. Cyclical vomiting syndrome
D. Irritable bowel syndrome D. Diabetic ketoacidosis
E. Meckel diverticulum E. Gastroenteritis
F. Gastro-oesophageal reflux
14.7 G. Intussusception
Ben, aged 9 months, has had a 3 day history of H. Malrotation
diarrhoea and vomiting. On examination he is I. Meningitis
found to be quiet but alert, is tachypnoeic, has a J. Migraine
tachycardia but normal pulses, dry mouth, no K. Pyloric stenosis
mottling of the skin but reduced skin turgor and L. Raised intracranial pressure
a sunken fontanelle. Capillary refill time is 2 M. Sepsis
seconds. His blood pressure is normal for his age. N. Strangulated inguinal hernia
He continues to vomit even with oral rehydration O. Urinary tract infection
solution given via a nasogastric tube. Ben’s 14.9.1
plasma sodium is found to be 156 mmol/L James, an 8-month-old infant, is bought to the
(normal range, 135–145 mmol/L). He needs fluid Emergency Department by his parents. He is
as he has clinical dehydration. How would this having episodes of abdominal pain and is just
fluid best be replaced? recovering from an upper respiratory tract
Select one answer only. infection. He seems well in-between, but then
suddenly seems to be in pain and looks pale. He
A. Immediate bolus of 20 ml/kg of 0.9% sodium has vomited several times. On questioning he
chloride followed by reassessment and has had no blood in his stool but has not opened
replacement of remaining deficit over 24 his bowels for 24 hours.
hours with 0.9% sodium chloride solution
B. Rehydration over 6 hours followed by repeat 14.9.2
urea and electrolyte measurement and Bridgitta, an 8-year-old girl, presents to her
maintenance fluid only for a further 18 hours family doctor with vomiting and abdominal
C. Rehydration over 24 hours with 0.18% pain. Her vomiting only started today and she
sodium chloride/5% glucose solution has no diarrhoea or fever. She looks unwell
D. Rehydration over 24 hours with 0.9% sodium and has clinical dehydration on examination
chloride/5% glucose solution and has deep rapid breathing. She is thirsty
E. Rehydration over 48 hours with 0.9% or and pale. She has lost weight over the last
0.45% saline few weeks.
14.9.3
14.8 Noah is 5 weeks old and has been breastfeeding
Matthew is a 3-day-old term infant who has not well and putting on weight. However, over
passed meconium since birth. On examination the last 36 hours he has been vomiting after
his abdomen is distended but the remainder of almost every feed. The vomit goes everywhere
the examination is normal. An x-ray of the and he then wants to feed again. All the vomits
abdomen shows distended loops throughout the are milky. He was born at term (birth weight 79
bowel, including the rectum. 3.8 kg).
14.9.4 yogurt. He has not opened his bowels. His
Amir was born by elective caesarean section for temperature is 38.2° C. His throat is red and he
maternal pre-eclampsia. His birthweight was has tender cervical lymph nodes. He is not
3.3 kg. He is 36 hours old. He has started to dehydrated. He has mild generalized tenderness
14 establish breastfeeding but has been vomiting
after every feed. The vomit is noted by the
of the abdomen, with no guarding.
14.10.2
midwife to be green. On examination his
temperature is 37.2° C. His abdomen is slightly Pete, aged 4 years, is brought by his mother to
Gastroenterology

distended. The rest of his examination the Emergency Department as he is crying and
is normal. saying his tummy hurts. He has had a 2-day
history of fever, coryza and cough. He is sitting
14.9.5 quietly on his mother’s lap, and is reluctant to
Jennifer, a 14-year-old girl, presents to the play. He has a temperature of 38.2° C and a
emergency department with a severe headache respiratory rate of 50 beats/min. On examination,
for the last 6 hours, mainly affecting the left side his throat is red and he has tender cervical
of her head. She just wants to lie still in the dark lymph nodes. He complains of tenderness on
and dislikes being disturbed, but her mother is palpation of the right upper quadrant of the
concerned as she has never had such an episode abdomen.
before and is normally a very lively girl who is
doing well at school. She has been vomiting for 14.10.3
the last 2 hours and cannot keep anything down Molly, aged 10 years, is brought to the Paediatric
and is also complaining of tummy pain. On Assessment Unit as she has been vomiting and
examination she is distressed by her headache had central abdominal pain for 2 days. She has
and dislikes having the examination light shone also had some diarrhoea. She has only had apple
on her. Her temperature is 37.2° C. She does not juice and no food for the last day. Her pain is
have neck stiffness or papilloedema. The rest of getting worse. On examination, she has a
her examination is normal. temperature of 38.2° C and a heart rate of
110 beats/min. She has mild dehydration. There
is tenderness in the lower right abdomen, but no
14.10 guarding. When asked to walk, she is unable to
The following (A–M) is a list of diagnoses stand up straight because of pain.
associated with abdominal pain in children. For
each of the following scenarios select the most 14.10.4
likely diagnosis from the list. Each answer may Francis is 12 years old and is reviewed in the
be used once, more than once, or not at all. paediatric outpatient department. He has been
referred as he has had abdominal pain for the
A. Appendicitis last 3 months. The pain is cramp-like, all over his
B. Coeliac disease tummy. He also has developed diarrhoea and is
C. Constipation getting up twice in the night to open his bowels.
D. Diabetic ketoacidosis There has not been any blood. He no longer
E. Functional abdominal pain wants to play football in the team, and is
F. Gastroenteritis increasingly refusing to do his homework. He has
G. Hepatitis lost 1 kg in weight. There are no abnormalities
H. Inflammatory bowel disease on examination. A blood test shows his
I. Inguinal hernia haemoglobin level to be 101 g/L, and a raised
J. Intussusception C-reactive protein of 85 mg/dL (normal <5).
K. Mesenteric adenitis
L. Pneumonia 14.10.5
M. Urinary tract infection Ted, aged 2 years, has had a 2-day history of
low-grade fever and coryza. His mother has
14.10.1 brought him to the Emergency Department as
Max, aged 9 years, has been brought to the he is crying inconsolably. She thinks his tummy is
Emergency Department as he is crying and hurting him. He has not opened his bowels for 2
saying his tummy hurts. He has had a 2-day days. He appears reasonably well, has minimal
history of fever and coryza. He has been drinking abdominal tenderness but has an indentable
orange juice but has only eaten some jelly and mass on the left side of the abdomen.

80
14.3
Answers: Single Best Answer A. Chronic non-specific diarrhoea
Correct. In chronic non-specific diarrhoea there
14.1 are loose stools with undigested food present.
A. Campylobacter The children grow well and have plenty of energy.
The most common cause of bloody diarrhoea
in the UK. All bacterial forms are notifiable B. Coeliac disease
diseases. Although coeliac disease is now relatively

Gastroenterology
common, you would expect more characteristic
B. Escherichia coli clinical features. Early weaning (before 3 months)
This occurs in outbreaks. Verotoxin-producing is a risk factor for early onset. Faltering growth is
strains can result in haemolytic uraemic common.
syndrome.
C. Cow’s milk protein allergy
C. Giardia lamblia Although this is common, affecting at least 1 in
Giardia lamblia is a parasitic infection and is 50 children, stools containing undigested food is
usually acquired whilst travelling abroad, not characteristic and it is usually accompanied
although there are rare cases in the UK. by some atopic markers such as eczema. Parents
Persistent diarrhoea would warrant testing for often report flecks of blood in the stool.
this organism.
D. Inflammatory bowel disease
D. Rotavirus Diarrhoea with blood and colicky abdominal
Correct. Rotavirus is a common cause of pain are characteristic of ulcerative colitis.
foul-smelling, watery diarrhoea. Escherichia coli, Crohn’s disease usually presents with diarrhoea,
Shigella, and Campylobacter also cause explosive abdominal pain and weight loss and general ill
watery diarrhoea but can be associated with health; oral lesions and perianal skin tags are
blood in the stools and in the UK are much less other features.
common than rotavirus. The incidence of
rotavirus in the UK should decline with the E. Lactose intolerance
introduction of rotavirus vaccine into the Primary lactose intolerance is common and the
standard childhood immunization schedule. norm for many non-Caucasian, non-Arabic
populations after infancy. Bloating, discomfort
E. Shigella and acidic stool causing perianal soreness are
Consider this if there is blood in the stool. more suggestive. Either a breath test or trial of
lactose free diet can be helpful in diagnostic
14.2 uncertainty. The presence of undigested food in
A. Chronic non-specific diarrhoea the stool and normal growth are against this
This is not the characteristic history for this diagnosis.
condition.
14.4
B. Coeliac disease A. Appendix mass
Correct. Coeliac disease (gluten-sensitive An appendix mass would be in the right iliac
enteropathy) usually presents at age 8 fossa following appendicitis.
to 24 months with abnormal stools, faltering
growth, abdominal distension, wasting of the B. Constipation
muscles of the buttocks (a difficult clinical Correct. The loose stool is overflow from her
sign), and irritability. Can also present with constipation. Children may present with loose
short stature or anaemia. It is increasingly stools when they actually have constipation.
detected on screening high-risk groups and at
older ages. C. Gastroenteritis
Gastroenteritis is unlikely as there is no vomiting
C. Hirschsprung disease and the problem has been going on for a month.
Hirschsprung disease presents with constipation
and a distended abdomen. D. Inguinal hernia
An inguinal hernia is not associated with loose
D. Lactose intolerance stools. It may be associated with pain in the
Lactose intolerance develops after a bout of groin or abdomen if it is strangulated, which is
gastroenteritis with the child continuing to have not the case in this child. It is also much less
diarrhoea. It is usually transient. common in girls than boys.
E. Ulcerative colitis E. Wilms tumour
Ulcerative colitis usually presents with bloody Wilms tumour usually presents as an abdominal
diarrhoea in older children. Nocturnal waking to mass but as it is a renal mass it would not be
defecate associated with abdominal pain should confined to the left iliac fossa. Blood in the urine 81
alert you to inflammatory bowel disease. may be present.
14.5 it does it can be a real diagnostic conundrum
A. Gastro-oesophageal reflux but it usually presents with either melaena or
Correct. Gastro-oesophageal reflux is caused by fresh blood in the stool. It can also be mistaken
the involuntary passage of gastric contents into for acute appendicitis.
14 the lower oesophagus. These infants can vomit
several times per day but still continue to gain 14.7
weight appropriately. If complications are A. Immediate bolus of 20 ml/kg of 0.9% sodium
present, it is called gastro-oesophageal reflux chloride followed by reassessment and
Gastroenterology

disease and needs treatment. replacement of remaining deficit over 24 hours


B. Helicobacter pylori infection with 0.9% sodium chloride solution
Helicobacter pylori infection presents with A bolus of intravenous fluid is not warranted as
abdominal pain, typically in an older child. he is not in shock. Although the pulse rate is not
provided, a normal capillary refill time and
C. Infant colic normal blood pressure confirm this.
Whilst this is common, it is typified by pain (and
crying) in the early evening lasting for a few B. Rehydration over 6 hours followed by
hours. Only a minority of cases are thought to be repeat urea and electrolyte measurement
due to gastro-oesophageal reflux. and maintenance fluid only for a further
18 hours
D. Overfeeding As he has hypernatraemic dehydration, rapid
Whilst this is a common cause of vomiting, it is rehydration should not be given to avoid
uncommon in breastfed babies. The clue in an potential brain damage from cerebral oedema.
examination will be that the feed volumes are This is therefore not a safe plan.
significantly higher than you would expect. For a
bottlefed infant, the feed volume will often be in C. Rehydration over 24 hours with 0.18%
excess of 200 ml/kg per day. sodium chloride/5% glucose solution
Rapid rehydration should be avoided to avoid
E. Pyloric stenosis potential brain damage from cerebral oedema. In
Although this child has the correct age, there are this case the use of hypotonic sodium chloride
no ‘red flags’ to suggest pyloric stenosis. By 8 solution is dangerous as a rapid fall in serum
weeks most affected infants would have either sodium increases this child’s risks. The use of
lost weight or at least had a significant 0.18% sodium chloride solutions is now
plateauing in their weight gain. It is also much restricted in the UK.
less common than reflux. It usually starts to
present at a few weeks rather than from birth. D. Rehydration over 24 hours with 0.9% sodium
chloride/5% glucose solution
In most circumstances rehydration over 24 hours
14.6
is appropriate; however, in hypernatremic
A. Functional abdominal pain
dehydration it is important to make the
Correct. Functional abdominal pain is classically
correction more slowly.
periumbilical and is not associated with any
other symptoms. The pain has been going on for E. Rehydration over 48 hours with 0.9% or
6 months and Claire is well in herself, which 0.45% saline
suggests that this is not pathological. The pain is Correct. Ben has hypernatremic dehydration, so
very real to her and needs to be explored, as in rehydration must be slow, i.e. over 48 hours to
some children it is a manifestation of stress. avoid cerebral oedema. In hypernatremic
dehydration the brain cells are contracted as
B. Gastritis
water has moved to the extracellular
Uncommon in this age group and usually felt in
compartment, as this has a higher osmolality
the epigastric area when it is the cause. If this
(see Fig. 14.1). Changes in extracellular
was the case, consider testing for Helicobacter
osmolality need to be slow to avoid rapid
pylori and if it persists then refer for endoscopy.
expansion of cells in the brain.
C. Hepatitis A Hypernatremic dehydration is clinically more
A common infection worldwide but would difficult to detect than other forms of
usually presents with jaundice and dark urine. dehydration on clinical grounds alone (see
Fig. 14.1 for reasoning).
D. Irritable bowel syndrome
A diagnosis of exclusion to be made with caution
in prepubertal children. 14.8
A. Congenital hypothyroidism
E. Meckel diverticulum Whilst babies with untreated congenital
2% of the population have a Meckel diverticulum hypothyroidism may develop constipation, it
82 but in the majority it causes no symptoms. When does not cause bowel obstruction.
Hypernatraemic dehydration Hyponatraemic dehydration

ECC
ECC O
O H2

Gastroenterology
H2
ICC

ICC

Na Na

In hypernatraemic dehydration, In hyponatraemic dehydration,


the sodium concentration in the the sodium concentration in the
extracellular compartment (ECC) ECC is reduced.
is increased. Therefore, water moves by osmosis
Therefore, water moves by osmosis from the extracellular to the
from the intracellular compartment intracellular compartment.
(ICC) to the ECC. So, for a given degree of
So, for a given degree dehydration, the ECC
of dehydration, the ECC loses more volume and
loses less water than the dehydration is readily
in other types of dehydration. apparent clinically.
This is why this form of dehydration
is readily underestimated clinically.

Figure 14.1

B. Cystic fibrosis the child cries with pain and goes pale. The
Meconium ileus, which is usually a manifestation passage of redcurrant jelly per rectum from
of cystic fibrosis, causes abdominal distension blood-stained mucus is a late sign.
from bowel obstruction and in some cases a
mass may be present. Distension of the colon 14.9.2
and rectum would not be a feature. D. Diabetic ketoacidosis
Diabetic ketoacidosis classically presents
C. Duodenal atresia
with a history of weight loss, polydipsia
Duodenal atresia causes bile-stained vomiting.
and polyuria. The fast breathing is due to
D. Hirschsprung disease the metabolic acidosis. This can sometimes
Correct. Hirschsprung disease is caused by the be mistaken as being a respiratory infection
absence of ganglion cells from the myenteric but she is afebrile. A urine dipstick and blood
and submucosal plexuses of part of the large glucose and blood gas will confirm the
bowel, which results in a narrow, contracted diagnosis.
segment. The abnormal bowel extends from the
rectum for a variable distance proximally, ending 14.9.3
in a normally innervated, dilated colon. K. Pyloric stenosis
This is a typical history. Pyloric stenosis occurs at
E. Rectal atresia 2–7 weeks of age; the vomiting is milky and
Rectal atresia causes large bowel obstruction but projectile and occurs after each feed. A mass can
examination of Matthew’s perineum is normal. be felt in the abdomen during or after a test
feed. A blood gas will usually show a
Answers: Extended Matching hypochloremic metabolic alkalosis because
chloride and acid are lost from the vomitus. An
14.9.1 ultrasound scan of the pylorus is usually
G. Intussusception diagnostic and is used in many centres to
In intussusception there is invagination of one confirm the diagnosis made on clinical
part of the bowel into another. When this occurs, examination. 83
14.9.4 cause of abdominal pain. Laparotomy is not a
H. Malrotation good treatment for pneumonia!
Bile-stained vomiting (green vomit) is a red-flag
symptom and in a neonate is most likely to be 14.10.3
A. Appendicitis
14 from intestinal obstruction. Malrotation is often
not evident until the baby experiences a twisting Appendicitis starts with central abdominal pain
that then localizes to the right iliac fossa.
of the intestine known as a volvulus; it usually
presents in the first 3 days of life, and requires Anorexia and loose stools are also associated
Gastroenterology

immediate surgery. Bilious vomiting may also clinical features. The inability to stand up straight
result from other causes of bowel obstruction, because of pain is known as the ‘appendix
such as duodenal atresia, or sepsis. Amir appears shuffle’ and is adopted to minimize the painful
clinically well, making sepsis less likely, but it still movement of the inflamed adjacent peritoneal
needs to be excluded. surfaces.

14.9.5 14.10.4
J. Migraine H. Inflammatory bowel disease
Migraine classically is associated with headache Getting up during the night to open his bowels
with photophobia and vomiting. It may be is a ‘red-flag’ sign and should always alert one to
accompanied by abdominal pain. Meningitis is a the possibility of inflammatory bowel disease.
differential diagnosis that should be considered Crohn’s disease usually presents with malaise
but she is afebrile and does not have any neck (fever, lethargy, weight loss), which may be
stiffness. Cyclical vomiting, by definition, occurs accompanied by abdominal pain, diarrhoea, and
recurrently. faltering growth. His anaemia and raised
inflammatory markers are consistent with a
14.10.1 diagnosis of Crohn’s disease.
K. Mesenteric adenitis
The mesenteric nodes in the abdomen become 14.10.5
inflamed following an upper respiratory tract C. Constipation
infection. Can be difficult to differentiate from Constipation can cause severe abdominal pain
appendicitis and requires frequent clinical and a mass may be palpable on examination.
re-assessment to check it resolves. The condition The mass is indentable, which is a sign that this
is frequently only diagnosed at laparotomy when is a faecal mass. The abdomen may be distended
a non-inflamed appendix is identified (and but is not tender to touch.
removed).
14.10.2
L. Pneumonia
He has tachypnoea, fever and cough. It is
important to consider pneumonia as a possible

84
15

Infection and immunity

15.3
Questions: Single Best Answer Henry is 4 years old and has a 3 day history of
fever. He presents to the Accident and
15.1 Emergency department with a headache. A
Joseph, aged 3 years, has been unwell for 24 lumbar puncture is performed. You receive the
hours with irritability, vomiting and fever. He was following result from the laboratory:
seen by his general practitioner earlier in the day
• Cerebrospinal fluid (CSF) microscopy:
and started on amoxicillin. On examination his
• 250 red blood cells/mm3
temperature is 38.5° C, pulse 140 beats/min and
• 1200 neutrophils/mm3
blood pressure 90/60 mmHg. He has a rash on his
• 250 lymphocytes/mm3
upper limbs and abdomen as shown in Fig. 15.1.
• CSF protein: 0.6 g/L
• CSF glucose: 2.1 mmol/L
• blood glucose: 7.2 mmol/L
What is the most likely diagnosis?
Select one answer only.
A. Bacterial meningitis
B. Blood-stained tap
C. Normal lumbar puncture result
D. Tuberculosis meningitis
E. Viral meningitis

15.4
Figure 15.1 Graham is 5 years old and has had an
intermittent fever for 4 weeks. He presents to the
What is the most likely diagnosis? Emergency Department with a headache and
neck stiffness. A CT scan is performed, which is
Select one answer only. normal. A lumbar puncture is performed. You
receive the following result from the laboratory:
A. Henoch–Schönlein purpura
B. Immune thrombocytopenic purpura • cerebrospinal fluid (CSF) microscopy: 95
C. Measles lymphocytes, 10 neutrophils and 0 red blood
D. Meningococcal sepsis cells/mm3
E. Pertussis • CSF protein: 2.2 g/L
• CSF glucose: 1.3 mmol/L
15.2 • blood glucose: 6.3 mmol/L
What investigation is most likely to give a What is the most likely diagnosis?
definitive diagnosis in Joseph’s case?
Select one answer only.
Select one answer only.
A. Ascending polyneuritis (Guillain–Barré
A. Blood culture syndrome)
B. Lumbar puncture B. Bacterial meningitis
C. Polymerase chain reaction (PCR) C. Blood-stained tap
D. Pernasal swab D. Tuberculosis meningitis
E. Throat swab E. Viral meningitis
15.5 weeks and has not put on any weight since then.
John, a 2-year-old boy, is brought to the general On examination he appears pale and has marked
practitioner by his mother. He has the skin rash intercostal recession. His oxygen saturation in air
shown in Fig. 15.2 on both his feet. He also has a is 82%. His chest X-ray is shown in Fig. 15.3 below.
15 few lesions on both hands. They are tender to
touch. He is well in himself.
Infection and immunity

Figure 15.3

Figure 15.2 What is the most likely cause of his respiratory


failure?
What is the most likely cause from the list below? Select one answer only.
Select one answer only. A. Influenza virus
A. Coxsackie A B. Pneumocystis jiroveci (carinii) pneumonia
B. Flea bites C. Respiratory syncytial virus
C. Herpes zoster D. Rhinovirus
D. Scabies E. Staphylococcal pneumonia
E. Varicella zoster
15.8
15.6 Imran is a 3-year-old boy who moved to the UK
Ahmed, a 1-year-old boy, is brought to the from Bangladesh 4 months ago. He has not
Paediatric Assessment Unit. He has had a high gained any weight for the last couple of months.
temperature for the last week. His mother has He has a cough. The general practitioner has
taken him to the general practitioner on two requested a chest X-ray (Fig. 15.4).
occasions and he has completed a course of
amoxicillin. His eyes are injected and his throat is
red. He has marked cervical lymphadenopathy.
The skin on his fingers has started to peel.
Everyone else in the family is well.
What is the most likely diagnosis?
Select one answer only.
A. Infectious mononucleosis (glandular fever)
B. Kawasaki disease
C. Staphylococcal scalded skin syndrome
D. Scarlet fever
E. Tuberculosis

15.7
Prince, a 4-month-old black African infant, who
has recently moved to the UK from Swaziland
with his mother is seen in the Paediatric
86 Assessment Unit. He is feeding poorly and is
breathless. He has had loose stools for the last 4 Figure 15.4
What is the most likely diagnosis? Q. Rubella virus
R. Staphylococcus aureus
Select one answer only. S. Streptococcus pneumoniae
A. Asthma T. Tuberculosis (Mycobacterium tuberculosis)
B. Neuroblastoma 15.10.1
C. Pertussis infection Harry, aged 21 months, presents with a 3 day
D. Pneumonia history of cough and fever and is very miserable
E. Tuberculosis

Infection and immunity


and feeding poorly. Examination reveals
conjunctivitis and the widespread rash shown in
15.9 Fig. 15.5 below.
You are in the baby clinic at a local general
practice. One of the mothers is questioning
whether or not she should get her baby
immunized as she wonders if it is really
necessary. He does not go to nursery and has no
siblings so is not at risk of infection.
What advice would you give?
Select one answer only.
A. As most other children are immunized it is
not crucial for her child to be immunized as
he is unlikely to be exposed to any of the
infections in the immunization schedule
B. Immunization is important as a high
proportion of children need to be
immunized to remove the infections from
the community
C. Immunization is important to provide
immunity for her child from serious
infections and to remove some of these
infections from the community
D. It is her choice and you do not feel you
should give an opinion
E. Vaccines are associated with side-effects, so
you can understand her reasoning

Questions: Extended Matching


Figure 15.5
15.10
The following (A–T) is a list of infectious agents 15.10.2
seen in children. Select the organism that is most Maryam, a 10-month-old infant is brought to her
likely to be the causative agent in the clinical general practitioner. She has had an intermittent
scenarios outlined below. Each option may be fever for 1 day and has a runny nose. Her mother
used once, more than once, or not at all. is concerned as her appetite is reduced, but she
A. Aspergillus fumigatus is still drinking. She is generally miserable when
B. Chickenpox virus (varicella zoster virus) febrile. On examination she has a temperature of
C. Cytomegalovirus infection 37.9° C, a runny nose and a fine macular rash,
D. Epstein–Barr virus mainly on her trunk.
E. Herpes simplex virus 15.10.3
F. Human immunodeficiency virus infection Jennifer, aged 12 years, developed a severe sore
G. Influenza virus throat and lethargy. Amoxicillin was prescribed.
H. Lyme disease (Borrelia burgdorferi) The next day she developed the florid
I. Malaria parasites (Plasmodium sp.) maculopapular rash shown (Fig. 15.6).
J. Measles virus
K. Mumps virus 15.10.4
L. Parvovirus infection (fifth disease) Sebastian, aged 10 years, has recently returned
M. Pseudomonas aeruginosa from a safari holiday in the Kruger National Park
N. Respiratory syncytial virus in South Africa. He has had a fever for 4 days
O. Rhinovirus associated with rigors. Sebastian’s blood film is 87
P. Roseola infantum (sixth disease) shown in Fig. 15.7 below.
15.10.6
Fiona, who is 2 years old, was burnt by a falling
mug of tea but it did not seem a bad burn and
so her mother did not seek medical attention.
15 The burn developed yellow crusting at the edges
and appears to be infected. She now develops
profound diarrhoea and collapses. She is
admitted to the paediatric intensive care unit.
Infection and immunity

It is noted that she has red lips. She is


hypotensive and requires respiratory and
circulatory support.
Hint: Although more than one organism
on the list can cause this, the ‘yellow crusting’
suggests a particular bacterium.
15.10.7
Katie is 6 years old. She missed a few days of
school as she had a fever and was generally
Figure 15.6 lethargic. Now she has a temperature of 38.3° C
and has a marked erythematous rash on her
cheeks.
Hint: Also known as ‘slapped cheek’.

15.11
For each of the following patients with an
infectious disease, select the NEXT step in
management from the list (A–O) below. Each
option may be used once, more than once, or
not at all.
A. Antipyretic/analgesia
B. Combined anti-tuberculosis medication
Figure 15.7 (Courtesy of Dr Saad Abdalla). C. Highly active antiretroviral therapy
D. Intravenous aciclovir
15.10.5 E. Intravenous antibiotics
Philip is 4 years old and has recently started F. Intravenous bolus of 20 ml/kg of normal
school. He presents with a widespread rash saline
(Fig. 15.8) that is intensely itchy. G. Intravenous immunoglobulin
H. Intravenous quinine
I. Nasogastric rehydration therapy
J. Oral antibiotic
K. Oral rehydration solution
L. Oral ACT (artemisinin-based combination
therapy)
M. Oxygen
N. Topical antibiotic
O. Topical emollient
15.11.1
Mohammed is 2 years old and presents with a
fever and cough for 2 days. He is drinking sips of
water but is off his food. He has a temperature of
39.5° C, respiratory rate of 40 breaths/min and his
oxygen saturation is normal. On examination he
has mild indrawing between his ribs. On
auscultation there are some crackles at the left
base. There is no wheeze. He is not clinically
dehydrated.
15.11.2
88 Imran is 2 years old and presents with fever and
Figure 15.8 a cough for 2 days. He has a fever of 38° C, a
respiratory rate of 25 breaths/min and oxygen saturation is 92% in air. Her temperature is 39° C
saturation of 98%. He is coryzal and has an and she has a respiratory rate of 50 breaths/
inflamed pharynx. minute. She has marked indrawing between her
ribs and is using her accessory muscles of
15.11.3 respiration. On percussion there is ‘stony
Mustafa, an 18-month-old boy, presents with 3 dullness’ at her left base and on auscultation
days of vomiting and diarrhoea. The vomiting decreased air entry at the left base.
has now settled but he continues to have loose

Infection and immunity


stools (he had about eight loose stools in the last
24 hours). His mucous membranes are dry, he 15.11.5
has sunken eyes and appears lethargic. His skin Michael is a 2-year-old boy whose parents were
turgor is reduced. His extremities are warm and born in Taiwan. He has been unwell now for 6
capillary refill time is normal. days with a fever, sore eyes and throat. He is
noted to have red eyes, an injected throat and
15.11.4 cervical lymphadenopathy. He also has a
Josie is 3 years old. She has had a cough and generalized maculopapular rash and the skin is
fever for 6 days. On examination her oxygen peeling from his hands and feet.

89
B. Blood-stained tap
Answers: Single Best Answer The red cell count would be much higher if this
was to account for the number of white cells
15.1 present.
15 A. Henoch–Schönlein purpura
Henoch–Schönlein purpura presents with C. Normal lumbar puncture result
The CSF results are abnormal. The normal CSF
‘palpable’ purpura confined to the buttocks and
extensor surfaces. It is a clinical diagnosis and values in a child are:
Infection and immunity

often presents with colicky abdominal pain, • white blood cells: 0–5/mm3
which may be followed by joint pains. • red blood cells: 0/mm3
B. Immune thrombocytopenic purpura • protein: 0.15–0.4 g/L
Immune thrombocytopenic purpura often • glucose: ≥50% blood glucose.
follows a minor viral illness but the child is well; D. Tuberculosis meningitis
however, the child develops bruising over bony This is rare in the UK and difficult to diagnose.
prominences as well as petechiae. Characteristic findings are lymphocytes rather
C. Measles than neutrophils in the CSF, the CSF protein is
The rash with measles is small reddish brown, markedly raised, and the glucose very low.
flat or slightly raised macules that may coalesce
E. Viral meningitis
into larger blotchy patches. Usually they first
The white blood cells are predominantly
appear on the head or neck, before spreading to
lymphocytes rather than neutrophils in the CSF.
the rest of the body.
The neutrophil count may be raised initially in
D. Meningococcal sepsis viral meningitis and may predominate in
Correct. Meningococcal sepsis is most likely as the enterovirus infections, but is nearly always less
child has fever, malaise, and has the characteristic than 1000/mm3.
purpuric rash spreading across the abdomen.
15.4
E. Pertussis
A. Ascending polyneuritis (Guillain–Barré
Whooping cough may cause facial and
syndrome)
conjunctival petechiae, but there is no history of
In ascending polyneuritis (Guillain–Barré
cough with this child. Coughing or severe vomiting
syndrome), the protein level would also be high
result in petechiae only in the distribution of the
but the white cells would not be raised and the
superior vena cava (above the nipples).
glucose would not be so low. It is important to
15.2 ensure that a paired blood sample is taken for
A. Blood culture serum glucose.
Blood culture may be negative as oral
B. Bacterial meningitis
bactericidal antibiotics have been given.
The white cell count shows predominantly
B. Lumbar puncture lymphocytes, and the neutrophil count is only
This would reveal if there was an associated marginally raised.
meningitis but a negative result would not rule
out septicaemia. The priority in this child is to C. Blood-stained tap
achieve cardiovascular stability and lumbar No red blood cells are reported in this sample.
puncture should be deferred. D. Tuberculosis meningitis
C. Polymerase chain reaction Correct. The white cell count shows
Correct. Meningococcal polymerase chain predominantly lymphocytes. This could also be
reaction is most likely to give definitive diagnosis. seen in viral meningitis, but the very markedly
Treatment should not be delayed whilst awaiting raised protein and very low glucose in the CSF
results, which may take 2–3 days. together with the clinical history are suggestive
D. Pernasal swab of tuberculosis.
Pernasal swab is used to diagnose pertussis E. Viral meningitis
infection and not meningococcal disease. The white cell count shows predominantly
E. Throat swab lymphocytes. This could also be seen in viral
Throat swab may be negative as oral antibiotics meningitis, but the very markedly raised
have been given and throat carriage is not protein and very low glucose in the CSF together
indicative of systemic infection. with the clinical history are suggestive of
tuberculosis.
15.3
A. Bacterial meningitis 15.5
Correct. There is a markedly raised white cell A. Coxsackie A
90 count, mainly neutrophils. The CSF protein is Correct. This is hand, foot and mouth disease
raised, with a reduced ratio of CSF to blood caused by Coxsackie A. Other infections would
glucose concentration. not cause painful vesicles in this distribution.
B. Flea bites C. Respiratory syncytial virus (RSV)
These would be itchy. Often other family This is the most common cause of bronchiolitis.
members would be affected. The soles of the However, he does not have crepitations and
feet would be an unusual site for flea bite marks. wheeze and his oxygen saturation is much lower
than you might expect.
C. Herpes zoster
This would be vesicular and crusting. D. Rhinovirus
Some strains of rhinovirus can cause a very
D. Scabies

Infection and immunity


severe pneumonitis but they are very rare.
This can be a difficult diagnosis to exclude. Very
careful close examination may reveal burrows. E. Staphylococcal pneumonia
Other family members would be likely to have an There is no focal consolidation on the chest X-ray
itchy rash as it is highly contagious. as in staphylococcal pneumonia.
E. Varicella zoster 15.8
This would cause vesicles. They would not be A. Asthma
confined to the palms and soles. Asthma can present with a persistent cough, but
15.6 the X-ray would be normal or the lungs
A. Infectious mononucleosis (glandular fever) hyperinflated.
Often there are limited signs in this age group B. Neuroblastoma
although enlarged tonsils with characteristic Malignancy is a possibility, but the chest X-ray
‘slimy’ appearance might prompt testing. and history are suggestive of tuberculosis.
B. Kawasaki disease C. Pertussis infection
Correct. This is an important diagnosis peculiar Pertussis causes a persistent cough, but not
to children. He fulfils the major diagnostic criteria these chest X-ray changes.
of fever for more than 5 days, with cervical
lymphadenopathy, injected pharynx, D. Pneumonia
conjunctivitis and peeling fingers. Treatment Bronchopneumonia would make Imran acutely
with intravenous immunoglobulin reduces the unwell, but there would be consolidation on the
risk of coronary artery aneurysm formation. chest X-ray.

C. Staphylococcal scalded skin syndrome E. Tuberculosis


There are areas of denuded skin as well as fever Correct. Tuberculosis is the most likely diagnosis
and malaise. as he has marked left hilar lymphadenopathy on
the chest X-ray, and it is endemic in Bangladesh.
D. Scarlet fever
There may be inflammation of the throat but 15.9
usually a generalized maculopapular rash. This A. As most other children are immunized, it is
may be followed by peeling of the skin of fingers not crucial for her child to be immunized as he is
and toes. unlikely to be exposed to any of the infections
This is not true. Herd immunity cannot be
E. Tuberculosis guaranteed, as shown by measles outbreaks
Failure to respond to antibiotics might make you when immunization rates in the UK dropped
consider tuberculosis but his clinical features, following adverse publicity (which was false).
especially peeling of the skin, do not fit this
diagnosis. B. Immunization is important, as a high
proportion of children need to be immunized to
15.7
remove the infections from the community
A. Influenza virus This is true but does not stress the potential
An important cause of fever in the under 1-year-old benefits to her child and therefore is less likely to
patient. However, it rarely causes respiratory failure. be convincing.
There would be an adult respiratory distress
syndrome pattern on chest X-ray. C. Immunization is important to provide
immunity for her child from serious
B. Pneumocystis jiroveci (carinii) infections, and to remove some of these
pneumonia infections from the community
Correct. This patient is from Swaziland, which Correct. Immunization would provide benefits
markedly increases his risk of HIV infection. All to both her child and the wider community.
the other infections listed can cause respiratory
failure but are less likely in this particular patient. D. It is her choice, and you do not feel you
Marked hypoxia in the presence of a relatively should give an opinion
normal looking chest X-ray should always make Whilst it is her choice to determine whether her
you consider this diagnosis. Prophylactic child should be immunized, it is important to 91
treatment with cotrimoxazole could have explore these issues with families to ensure that
prevented this infection. they are making an informed decision.
E. Vaccines are associated with side-effects, so 15.10.7
you can understand her reasoning L. Parvovirus infection (fifth disease)
However, the risks are much smaller than the Parvovirus B19 causes erythema infectiosum or
potential benefits in every study ever conducted. fifth disease, which is also known as ‘slapped-
15 Answers: Extended Matching
cheek syndrome’ because of its characteristic
facial rash. This infection will temporarily reduce
red cell production, which can result in serious
15.10.1 anaemia in children with more rapid red cell
Infection and immunity

J. Measles virus turnover (hereditary spherocytosis or sickle cell


Harry has measles. It is uncommon in countries disease) or the fetus.
with immunization programmes, though
15.11.1
outbreaks still occur in children who have not
J. Oral antibiotic
been immunized because of parental and public
Mohammed has pneumonia. He has cough, fever
anxiety about the safety of MMR (measles,
and focal chest signs. As he does not have an
mumps and rubella) vaccination. It also
oxygen requirement or signs of a pleural
continues to be a cause of morbidity and death
effusion, but has only mild respiratory distress,
worldwide. Measles virus is a type of
he can be managed at home with oral
paramyxovirus.
antibiotics. The family should be given advice to
15.10.2 see a doctor promptly if the breathing becomes
O. Rhinovirus more laboured or he becomes more unwell.
Upper respiratory tract infection is one of the
15.11.2
most common problems seen in general
A. Antipyretic/analgesia
practice. The most common causative agent is
This child has an upper respiratory tract infection
rhinovirus. This is also the most common trigger
and so can be given an antipyretic/analgesic, i.e.
for exacerbations of asthma, accounting for more
paracetamol. This also has the advantage of
than 80% of exacerbations in children.
making the child more comfortable by easing
15.10.3 pain. NICE (National Institute for Health and Care
D. Epstein–Barr virus Excellence) guidelines are that fever does not
Ampicillin or amoxicillin may cause a florid need treating with an antipyretic unless the child
maculopapular rash in children infected with is distressed.
Epstein–Barr virus (glandular fever) and therefore
15.11.3
should be avoided in children who may have
glandular fever. If examination reveals ‘slimy’ K. Oral rehydration solution
enlarged tonsils, this should be suspected. This child has gastroenteritis and has clinical
dehydration, but is not shocked. This is an
15.10.4 important differentiation. As he is not shocked,
I. Malaria parasites (Plasmodium sp.) start with oral rehydration solution, but keep him
The infection is diagnosed by examination of a under observation for about 4 hours to ensure
thick film. The species (P. falciparum, P. vivax, P. that the oral fluid is tolerated. If not, a
ovale, or P. malariae) is confirmed on a thin film. nasogastric tube can be sited and fluids given via
Repeated blood films may be necessary. This this route. Intravenous fluids should be avoided
blood film shows falciparum malaria. where possible.
15.10.5 15.11.4
B. Chickenpox (varicella zoster virus) E. Intravenous antibiotics
Clinical features of chickenpox are fever and Josie has pneumonia with an effusion. She has
itchy, vesicular rash, which crops for up to 7 days. borderline oxygen saturation and marked
respiratory distress and may need oxygen during
15.10.6
sleep if her oxygen saturation drops further.
R. Staphylococcus aureus
Intravenous antibiotics are initially required to
Fiona has toxic shock syndrome. This is usually
treat this infection. When she has been afebrile
caused by a toxin-producing Staphylococcus
for 48 hours, this can be changed to oral
aureus, although group A streptococci can also
antibiotics.
cause this syndrome. It is characterized by:
• fever ≥39° C 15.11.5
• hypotension G. Intravenous immunoglobulin
• diffuse erythematous, macular rash Michael has Kawasaki disease and is at risk of
• can also cause mucositis involving developing coronary artery aneurysm, so he
conjunctivae, oral and genital mucosa and requires treatment with intravenous
multiorgan dysfunction. immunoglobulin.
92
16

Allergy

Questions: Single Best Answer


16.1
Jonathan, aged 6 years, is brought to the
Emergency Department after becoming unwell at
a family party. He is unable to say more than a
single word and he indicates that he is finding it
hard to breathe. He is very anxious. He has a raised
itchy rash that is spreading from his face down to
his chest. He has never had an episode like this
before, although his mother explains that he has
asthma and he has been prescribed a salbutamol
inhaler previously for wheezy episodes. Figure 16.1
What is the most likely diagnosis?
Select one answer only. What term provides the best description of
this rash?
A. Acute asthma
B. Allergic reaction A. Annular
C. Anaphylaxis B. Maculopapular
D. Idiopathic urticaria C. Purpuric
E. Inhaled foreign body D. Urticarial
E. Vesicular
16.2
What will be your first step in Johnathan’s 16.4
management? Eddy is an 8-month-old infant who presents to
his general practitioner following a bout of
Select one answer only.
diarrhoea and vomiting, associated with fever for
A. Administer a budesonide nebulizer 3 days. This was managed at home with oral
B. Assess his airway and give him high-flow rehydration solution. The vomiting had settled
oxygen but the diarrhoea has continued for three weeks.
C. Give intramuscular benzylpenicillin The stool microscopy and culture were negative.
D. Insert an intravenous cannula He has no other medical problems and is not on
E. Lie him flat any other medication. On examination he is not
dehydrated. He has a soft, mildly distended
16.3 abdomen.
Cordelia, a 5-month-old infant, was exclusively What is the most likely cause of his prolonged
breastfed up until yesterday when her mother diarrhoea?
started her on formula as she is planning to go
back to work. She noticed that she developed a A. Coeliac disease
rash very soon after the formula feed. She has no B. Giardiasis
other medical problems and is not routinely on C. IgE mediated cow’s milk protein allergy
any medications. On examination she has the D. Non-allergic food hypersensitivity
rash shown in Fig. 16.1 all over her body. E. Non-IgE mediated cow’s milk protein allergy
D. Insert an intravenous cannula
Answers: Single Best Answer The treatment he needs is oxygen and if
necessary intramuscular adrenaline.
16.1

16 A. Acute asthma
The most common reason for admission to
E. Lie him flat
Lying him flat is contra-indicated as it makes
upper airways obstruction worse. If there is no
hospital in the UK during childhood but the rash
points to an allergic cause. difficulty breathing but signs of shock, then this
Allergy

can be helpful but this is not appropriate in this


B. Allergic reaction instance.
This is a severe allergic reaction, or anaphylaxis,
as it is resulting in difficulty breathing. 16.3
C. Anaphylaxis D. Urticarial
Correct. His stridor in association with Correct
angioedema and urticaria is characteristic of
anaphylaxis. The likely cause is an allergic 16.4
reaction to a peanut or tree nut encountered at A. Coeliac disease
the party. Although this is a possible cause of prolonged
diarrhoea, it is less likely than secondary lactose
D. Idiopathic urticaria intolerance. If a lactose-free diet does not result
This is the most common cause for an urticarial in an improvement in symptoms, then this
rash but would not account for the sudden onset should be considered.
in difficulty breathing.
B. Giardiasis
E. Inhaled foreign body The stool microscopy is negative.
This is a common cause of sudden-onset difficulty
breathing but would not explain the rash. C. IgE mediated cow’s milk protein allergy
IgE mediated reactions are rapid and typified by
16.2 an urticarial reaction.
A. Administer a budesonide nebulizer
Nebulized budesonide has been used as a D. Non-allergic food hypersensitivity
treatment for croup (although it is probably no Correct. This is likely to be temporary lactose
more effective than oral dexamethasone) but is intolerance (a non-allergic food hypersensitivity)
not the treatment for anaphylaxis. following gastroenteritis. The stool is likely to
have reducing substances in it. This should
B. Assess his airway and give him high-flow resolve over several weeks.
oxygen
Correct. Jonathan requires oxygen as he has E. Non-IgE mediated cow’s milk protein allergy
upper airways obstruction and is having an Children with non-IgE mediated cow’s milk
anaphylactic reaction. Oxygen will improve allergy usually present with faltering growth
co-operation and allow time for adrenaline to be in conjunction with at least one or more
drawn up. gastrointestinal symptoms including
gastroesophageal reflux, loose or frequent stools,
C. Give intramuscular benzylpenicillin abdominal pain, infantile colic, food refusal,
This is the treatment for meningococcal constipation and perianal erythema. They are
septicaemia. variably affected with atopic eczema.

94
17

Respiratory disorders

Questions: Single Best Answer


17.1
Liam, a 7-year-old boy, complains to his family
doctor of a sore throat and has a mild fever. The
appearance of his throat is as shown in Fig. 17.1.

Figure 17.2 (Courtesy of Mr Neil Tolley)

What is the most likely diagnosis?


Select one answer only.
A. Bacterial tracheitis
B. Croup
C. Epiglottitis
Figure 17.1 (Courtesy of Mr Neil Tolley) D. Foreign body
E. Laryngomalacia

Hint. When the question stem includes a


What is the most likely diagnosis? child who is a refugee the question may be
Select one answer only. asking indirectly about a disease which is
preventable by immunization.
A. Diphtheria
B. Glandular fever (Ebstein Barr virus) 17.3
C. Group A Streptococcal tonsillitis Amber, a 9-month-old girl, presents with a 4-day
D. Measles history of coughing spasms which are followed
E. Herpes simplex stomatitis by vomiting. Whooping cough (Bordetella
pertussis infection) is suspected.
17.2
Mohammed, a 5-year-old refugee from Somalia, Which of the following tests would be most
presents acutely unwell to the Emergency useful in confirming the diagnosis?
Department. He has a 1-day history of sore Select one answer only.
throat and a high temperature (40.1° C). Over the
last 8 hours he has been having increasing A. Blood culture
difficulty breathing with quiet stridor. He has B. Chest X-ray
never been immunized. He is noted to be unable C. Full blood count and film
to swallow his saliva. Fig. 17.2 was taken when D. Nasopharyngeal aspirate
he was intubated. E. Pernasal swab
17.4 17.7
Tak, a 3-year-old Asian boy, presents to his family Zak, a 3-year-old boy, is seen by his general
doctor. He has a ‘hacking’ cough that started practitioner because of recurrent wheezing
several weeks ago and has failed to respond to associated with upper respiratory tract
17 two courses of antibiotics. He is otherwise well
and has had no previous chest problems. On
infections.
Which of the following features most supports
examination there is decreased air entry in the
right lower zone with normal percussion note. the diagnosis of asthma?
Respiratory disorders

His growth is normal. Select one answer only.


Which is the most appropriate next step? A. Daytime cough
Select one answer only. B. Finger clubbing
C. Peak-flow variability diary
A. Admit for intravenous antibiotic therapy D. Persistent moist cough
B. Assess bronchodilator response E. The presence of symptoms between coughs
C. Organize for ultrasound-guided drainage of and colds
his pleural effusion
D. Request a chest X-ray 17.8
E. Request a sweat test and evaluation Boris, a 5-month-old infant from Poland, is
of immunoglobulins and functional admitted to hospital with breathing problems
antibodies and poor feeding. On examination he has
a respiratory rate of 50 breaths/min. On
17.5 auscultation of the chest he has widespread
Amir, a 4-year-old boy, presents to his family crackles. He has moderate intercostal recession,
doctor with a history of eczema, rhinitis, chronic and oxygen saturation of 92% in air. He was born
nocturnal cough and intermittent wheeze. at term with a birthweight of 3.6 kg (50th
Asthma is suspected and a bronchodilator is centile). His weight is now 5.2 kg (<0.4th centile).
prescribed. This is his first admission to hospital but he ‘is
always chesty’.
How should his bronchodilator be delivered?
You suspect Boris has cystic fibrosis. When he is
Select one answer only.
stabilized which would be the most appropriate
A. Dry powder inhaler investigation to perform?
B. Metered dose inhaler (MDI)
Select one answer only.
C. Metered dose inhaler with large-volume
spacer A. Genetic screening for cystic fibrosis
D. Nebulizer B. Heel prick for immunoreactive trypsin
E. Syrup C. Measurement of faecal elastase
D. Measurement of serum bilirubin
17.6 E. Sweat test
Sarah, a 10-year-old girl, has frequent attacks
of asthma. She attends the Emergency 17.9
Department with increasing difficulty in Norah, an 18-month-old girl, presents to her
breathing over the last 12 hours. Initial family doctor with coryza, cough and a mild
observation shows that she is anxious, sitting fever for 3 days. She feeds poorly and is
upright, has a marked tracheal tug and is unable unsettled at night. Her respiratory rate is normal
to complete a sentence. and there is no chest recession.
Which of the following statements is most likely What is the most likely diagnosis?
to be correct?
A. Bronchiolitis
Select one answer only. B. Frontal sinusitis
A. Sarah’s asthma attack is of moderate C. Pneumonia (lower respiratory tract infection)
severity D. Tonsillitis
B. Sarah’s condition is likely to improve if she is E. Upper respiratory tract infection
encouraged to lie flat
C. Sarah’s oxygen saturation should be 17.10
measured Fiona, a 10-month-old infant, has been unsettled
D. Sarah should be taken promptly to the X-ray and febrile with a runny nose for 2 days. Her
department for a chest X-ray family doctor examines her ear canal and the
96 E. The lack of wheeze should make you tympanic membrane appears as in Fig. 17.3
consider a panic attack below.
Select the ONE most likely diagnosis from the list
below.
A. Acute epiglottitis
B. Anaphylaxis
C. Bronchiolitis
D. Laryngeal foreign body
E. Laryngotracheobronchitis (croup)

Respiratory disorders
Questions: Extended Matching
17.12
Below is a list of conditions affecting the
respiratory tract (A–O). For each of the clinical
scenarios described pick the most likely
diagnosis from the list. Each answer may be used
once, more than once, or not at all.
Figure 17.3 A. Acute exacerbation of asthma
B. Bronchiolitis obliterans
What is the most likely diagnosis? C. Bronchiolitis
D. Bronchopulmonary dysplasia (BPD)
Select one answer only. E. Chronic asthma
F. Cystic fibrosis
A. Acute otitis externa
G. Inhaled foreign body
B. Cholesteatoma
H. Laryngotracheobronchitis (croup)
C. Chronic otitis externa
I. Obstructive sleep apnoea
D. Foreign body in the external ear canal
J. Pertussis
E. Otitis media with effusion
K. Pneumonia
L. Pneumothorax
17.11 M. Retropharyngeal abscess
Jake is a 10-month-old boy from the UK who N. Tracheitis
presents to the Emergency Department with a O. Tuberculosis
2-day history of fever and runny nose. He has
been otherwise well. During the night he 17.12.1
gradually developed a barking cough in A 10-year-old Caucasian boy has had recurrent
association with a loud noise on inspiration. On chest infections requiring admission to hospital
examination he has a temperature of 38° C and for intravenous antibiotics. He is smaller than his
noisy inspiration accompanied by marked sternal classmates: his weight is on the 2nd centile and
recession (Fig. 17.4). His capillary refill time is height on the 25th centile. His chest X-ray is
normal. shown (Fig. 17.5).

97
Figure 17.4 Figure 17.5
17.12.2
A 10-year-old African boy woke up four nights
ago with a sudden onset of coughing and
choking. Since then he has been noted to be
17 intermittently wheezy. He has wheeze on
auscultation of his right chest only. His chest
X-ray is shown in Fig. 17.6 below.
Respiratory disorders

Figure 17.8

17.13
Below is a list of diagnoses (A–N) that result in
tachypnoea in children. For each of the following
patients with respiratory symptoms, select the
Figure 17.6 most likely diagnosis. Each option may be used
once, more than once, or not at all.
17.12.3
A 3-year-old Asian girl has been coughing for 10 A. Acute asthma
days, with fever and lethargy for 2 days. On B. Bronchitis
examination, she has a respiratory rate of 45 C. Bronchiolitis
breaths/min and crepitations with decreased air D. Bronchopulmonary dysplasia (BPD)
entry on her left lung base. Her chest X-ray is E. Chronic asthma
shown (Fig. 17.7). F. Cystic fibrosis
G. Heart failure
H. Inhaled foreign body
I. Laryngotracheobronchitis (croup)
J. Obstructive sleep apnoea
K. Pertussis (whooping cough)
L. Pneumonia
M. Retropharyngeal abscess
N. Tuberculosis

17.13.1
Jamal, a 10-month-old Asian boy, is brought at 1
am to the Emergency Department because he has
woken up with noisy breathing. He has had coryzal
symptoms for 2 days and now has a barking cough.
On examination he has a fever of 37.8° C. He is alert
and watches you but clings to his mother. On
crying, he has marked inspiratory stridor.

17.13.2
Jack, a 4-month-old infant, has rapid, laboured
breathing that has been getting worse over the
last 2 days. His mother is concerned as she is
Figure 17.7 struggling to get him to feed. He was born at 27
weeks’ gestation, birth weight 979 g and was
17.12.4 discharged home at 3 months of age. On
Annoushka, a 3-month-old infant, was born at 25 examination he has a temperature of 37.4° C and
weeks’ gestation (birthweight 695 g). She a respiratory rate of 60 breaths/min. He is
required prolonged artificial ventilation and coughing. His chest is hyperinflated with marked
98 continues to require oxygen delivered via nasal intercostal recession. On auscultation there are
cannulae. Her chest X-ray is shown (Fig. 17.8). generalized fine crackles and wheezes.
17.13.3 C. Inhaled salbutamol via metered dose
Connor, a 5-month-old infant, from a travelling inhaler (MDI)
family visiting from Ireland, is admitted to D. Inhaled salbutamol via MDI and spacer
hospital with difficulty breathing and poor E. Inhaled steroid via MDI
feeding. He was born at term with a birthweight F. Inhaled steroid via MDI and spacer
of 3.6 kg (50th centile). His weight is now 5.2 kg G. Intravenous antibiotics
(<0.4th centile). He has never fed well, and has H. Nebulized adrenaline
always tended to regurgitate his milk. This is his I. Nebulized salbutamol

Respiratory disorders
first admission to hospital but he ‘is always J. Nebulized steroid (budesonide)
chesty’. On examination he has temperature K. Oxygen therapy
of 37.9° C, a respiratory rate of 50 breaths/min L. Oral antipyretic/analgesic
with widespread crackles on auscultation of M. Oral antibiotics
the chest. N. Oral corticosteroids

17.13.4 17.14.1
Fred is a normally well 4-year-old boy. He has Sara, an 8-month-old Asian girl, presents with a
had a runny nose and fever for 3 days. He has 3-day history of being unsettled. She has been
now developed a cough and difficulty breathing. coryzal and has had a mild fever. On examining
On examination his temperature is 39° C. He the right ear canal you note a bulging red
watches you but sits quietly on his mother’s tympanic membrane. Her respiratory rate is 25
lap. He has a respiratory rate of 55 breaths/min. breaths/min and she does not have any chest
His breaths are rapid but shallow with some recession.
mild substernal recession. There is no wheeze 17.14.2
on auscultation but some coarse crackles at Chardonnay is a 6-year-old Caucasian girl who
the right base. His oxygen saturation is 91% has asthma. Her mother smokes cigarettes and
in air. there is poor compliance with her preventative
17.13.5 steroid therapy. She presents with a 2-day
Hannah is a 3-month-old infant who has had a history of cough, runny nose, mild fever and
cough for over 2 weeks. She has now developed ‘breathlessness’. Her mother cannot remember
prolonged bouts of coughing. She has started to her previous peak-flow result. On examination
vomit at the end of the bout of coughing. Her she has a respiratory rate of 30 breaths/min, mild
temperature is 38° C. Her respiratory rate is 25 intercostal recession and oxygen saturation of
breaths/min. On auscultation of her chest there 96% in air.
are some scattered crackles. 17.14.3
Jake, a 7-month-old infant, presents with a
17.14 2-day history of fever and runny nose. During
Below is a list of management options (A–N). For the night he has developed a harsh cough
each of the following patients with respiratory in association with noisy inspiration. On
disease, select the next step in management. examination, you note he has moderate stridor
Each option may be used once, more than once, mainly on inspiration and mild intercostal and
or not at all. subcostal recession. His respiratory rate is 30
breaths/min. He has a temperature of 37.8° C.
A. Continuous positive airway pressure (CPAP) His capillary refill time is normal. His oxygen
B. Endotracheal intubation and ventilation saturation is 96% in air.

99
17.3
Answers: Single Best Answer A. Blood culture
It is incredibly rare to isolate Bordetella pertussis
17.1 from blood cultures.
17 A. Diphtheria
Diptheria is now extremely rare as children are B. Chest X-ray
Chest X-ray changes can occur in whooping
immunized in the UK. Also the appearance is not
‘typical’. In diphtheria a thick grey pharyngeal cough but are not diagnostic.
Respiratory disorders

membrane is characteristic.
C. Full blood count and film
B. Glandular fever (Ebstein Barr virus) A marked lymphocytosis is characteristic
Infection with Ebstein Barr virus can lead to of pertussis, but not diagnostic. The
tonsillitis although typically the tonsils are lymphocytosis is secondary to pertussis toxin.
coated with a grey membrane. Palatal petechiae Bordetella parapertussis does not produce
(pinpoint spots on the soft palate) may also pertussis toxin.
be seen.
D. Nasopharyngeal aspirate
C. Group A Streptococcal tonsillitis Immunofluorescence of a nasopharyngeal
Correct. There is intense inflammation of the aspirate is used to identify respiratory syncytial
tonsils with purulent exudates. In this age group virus, which causes bronchiolitis, but is not
Group A beta-haemolytic Streptococcus is the helpful in the diagnosis of pertussis.
most likely causative pathogen.
E. Pernasal swab
D. Measles Correct. Culturing a pernasal swab allows
In measles there may be white spots (Koplik the pathogen (Bordetella pertussis) to be
spots) visible on the buccal mucosa. identified (though PCR (polymerase chain
reaction) is more sensitive). This can also be
E. Herpes simplex stomatitis helpful in isolating the related Bordetella
There are lesions on the lips, gums and parapertussis.
tongue.
17.4
17.2 A. Admit for intravenous antibiotic therapy
A. Bacterial tracheitis Tak has already failed to respond to two courses
The child would have loud, harsh stridor and of antibiotics, so further investigation is
would not have an enlarged epiglottis as shown warranted.
in the figure.
B. Assess bronchodilator response
B. Croup There is no wheeze on examination and so
The child would be less seriously unwell or ‘toxic’ inhaled bronchodilator is unlikely to be
and would have a mild fever and loud stridor effective.
and would not have the abnormal appearance
shown in the figure. C. Organize for ultrasound-guided drainage of
his pleural effusion
C. Epiglottitis His signs are not consistent with a pleural
Correct. The photograph shows the effusion. The percussion note would be stony
characteristic grossly enlarged ‘cherry red’ dull if this was a pleural effusion.
epiglottis of acute epiglottitis. It is caused by
Haemophilus influenzae type b. In the UK and D. Request a chest X-ray
many other countries, the introduction of The most likely diagnosis is the inhalation of a
universal H. influenzae type b immunization in foreign body, e.g. a peanut. Tak also has focal
infancy has led to a >99% reduction in the chest signs and so needs a chest X-ray to be
incidence of epiglottitis and other invasive performed.
H. influenzae type b infections. E. Request a sweat test and evaluation of
D. Foreign body immunoglobulins and functional antibodies
The history would be of a sudden onset of cough A sweat test is used to diagnose cystic fibrosis
or respiratory distress, and there would not be a and is not indicated here because this is his first
high fever. episode of chest problems and he is growing
normally. Functional antibodies and
E. Laryngomalacia immunoglobulins are useful screening tests for
The child would not be unwell and would have possible immunodeficiency and this ‘triad’ of
had recurrent or continuous stridor since infancy. investigations is often used in children with
Laryngomalacia generally resolves within the 1st faltering growth and/or recurrent respiratory
100 year of life. infections.
17.5 B. Finger clubbing
A. Dry powder inhaler Clubbing suggests suppurative lung disease or
Dry powder inhaler is appropriate only if MDI and congenital heart disease.
spacer have failed in this age group. Most 4 year
olds will perform better with MDI and spacer. C. Peak-flow variability diary
Zak is too young to perform peak flow reliably.
B. Metered dose inhaler (MDI)
Children should be prescribed a MDI with spacer D. Persistent moist cough
This suggests persistent bacterial bronchitis.

Respiratory disorders
as they cannot co-ordinate an MDI alone. It is
always preferable to use a spacer, even in adults E. The presence of symptoms between
as delivery is more reliable. coughs and colds
C. Metered dose inhaler with large-volume Correct. The presence of interval symptoms and
spacer atopic conditions (eczema/hay fever) helps to
Correct. The best mode of delivery is direct to distinguish asthma from viral-induced wheeze.
the lungs. Children under 5 years of age should 17.8
be prescribed a MDI with spacer as they cannot A. Genetic screening for cystic fibrosis
co-ordinate an MDI alone. ΔF508 is the most common mutation in cystic
D. Nebulizer fibrosis, but is not diagnostic as some children
Nebulizers provide very effective delivery of with cystic fibrosis will have a different mutation.
bronchodilators. However, they produce more Even screening for the most common mutations
hypoxia than bronchodilators delivered by will miss some affected children. If the sweat test
pressurized MDI and spacer, are more expensive, is positive or borderline, then genetic screening
and are not as safe. In hospital the nebulizers are is helpful in determining prognosis. Some
driven by oxygen to offset the risk. mutations are associated with milder disease or
may be amenable to treatment.
E. Syrup
Syrup should not be used as it is results in high B. Heel prick for immunoreactive trypsin
blood levels and unacceptable side-effects. Immunoreactive trypsin remains high (elevated)
only for a few weeks before returning to
17.6 normal levels. This test is used for newborn
A. Sarah’s asthma attack is of moderate severity screening.
Sarah’s attack is severe as she is unable to C. Measurement of faecal elastase
complete a sentence. Children with untreated cystic fibrosis will have
B. Sarah’s condition is likely to improve if she is low faecal elastase and faltering weight. This
encouraged to lie flat confirms pancreatic insufficiency. However, cystic
Sitting upright assists lung mechanics and fibrosis is not the only cause for this and a sweat
enables her to use her accessory muscles. test will still be required.

C. Sarah’s oxygen saturation should be D. Measurement of serum bilirubin


measured Cystic fibrosis can cause prolonged jaundice (>14
Correct. Oxygen saturation should be measured days of age), but this is not specific.
to further assess the severity of the asthma E. Sweat test
attack and to guide treatment. Correct. Abnormal function of the sweat glands
D. Sarah should be taken promptly to the X-ray results in excessive concentrations of sodium
department for a chest X-ray and chloride in the sweat, and this is the basis of
The priority is to treat Sarah’s asthma. If a chest the essential diagnostic test for cystic fibrosis.
X-ray is indicated, then a portable X-ray machine
17.9
should be brought to any patient who is
A. Bronchiolitis
significantly unwell.
Norah’s chest has no abnormal signs on
E. The lack of wheeze should make you auscultation. With bronchiolitis, you would
consider a panic attack expect to hear crepitations and wheeze, and to
The absence of wheeze implies that little air is see signs of respiratory distress.
moving in and out of the chest and would
B. Frontal sinusitis
indicate severe bronchoconstriction.
This is unusual in children of this age because
frontal sinuses do not develop until late
17.7 childhood.
A. Daytime cough
Nocturnal cough is classically experienced with C. Pneumonia (lower respiratory tract infection)
asthma. A cough that always resolves during Young children with pneumonia can sometimes 101
sleep is likely to be habitual (psychogenic). present without chest signs on auscultation, but
there would be respiratory distress including a
raised respiratory rate. Answers: Extended Matching
D. Tonsillitis Answer 17.12.1

17 Examination of the oropharynx would show


intense inflammation of the tonsils, often with a
purulent exudate.
F. Cystic fibrosis
The history of recurrent severe chest infections
with poor weight gain is suggestive of cystic
fibrosis. The chest X-ray shows extensive changes
E. Upper respiratory tract infection
Respiratory disorders

with large-volume lungs with extensive


Correct. This child has an upper respiratory tract
reticulonodular shadowing, and peribronchial
infection, most likely a common cold.
thickening with hilar lymphadenopathy,
consistent with cystic fibrosis. With screening,
17.10 late presentation like this will become
A. Acute otitis externa uncommon.
Acute otitis externa is an infection of the outer
ear canal, not of the tympanic membrane. Answer 17.12.2
G. Inhaled foreign body
B. Cholesteatoma The acute history of coughing and focal wheeze
A rare problem in children. The two most is suggestive of an inhaled foreign body. The
common symptoms are persistent, often smelly, tooth is visible in the right main bronchus.
discharge from the affected ear and gradual loss
Answer 17.12.3
of hearing in the affected ear.
K. Pneumonia
C. Chronic otitis externa The history of fever, cough and being unwell
Chronic otitis externa is an infection of the outer (lethargy), with signs of tachypnoea and focal
ear canal, not of the tympanic membrane. crepitations are suggestive of pneumonia. The
Fiona’s history is only 2 days long and therefore chest X-ray shows that there is loss of contour of
not chronic. the left heart border and of the left
hemidiaphragm.
D. Foreign body in the external ear canal
Fiona has symptoms of an upper respiratory tract Answer 17.12.4
infection and fever, and no foreign body is seen D. Bronchopulmonary dysplasia (BPD)
on examination. Preterm infants who still have an oxygen
requirement at a postmenstrual age of 36 weeks
E. Otitis media with effusion are described as having bronchopulmonary
Correct. Here the ear drum is dull and bulging. dysplasia (BPD). The lung damage may be
caused by pressure and volume trauma from
17.11 artificial ventilation, oxygen toxicity and
A. Acute epiglottitis infection, but delay in lung maturation is now
Epiglottitis would present with a shorter history, the most common predisposing factor. The chest
and the child would be extremely unwell X-ray, as shown here, characteristically shows
(toxic). It is also very rare in the UK and widespread areas of opacification, sometimes
countries with Haemophilus influenzae type b with cystic changes.
immunization. Answer 17.13.1
B. Anaphylaxis I. Laryngotracheobronchitis (croup)
Anaphylaxis may cause stridor but also causes Jamal has laryngotracheobronchitis (croup). This
other symptoms, often including an urticarial is the correct diagnosis because he has stridor
rash. It would not cause a fever. accompanied by a low-grade fever, a viral upper
respiratory tract infection, and is alert.
C. Bronchiolitis
Bronchiolitis causes wheeze and crepitations Answer 17.13.2
rather than stridor as in this child. C. Bronchiolitis
Jack has signs and symptoms of bronchiolitis,
D. Laryngeal foreign body including poor feeding, increased work of
The child has a fever and a gradual onset of breathing, mild fever, cough and crackles and
symptoms. wheeze on auscultation. Jack is at increased risk
of respiratory failure from bronchiolitis because
E. Laryngotracheobronchitis (croup)
he was preterm.
Correct. Laryngotracheobronchitis (croup) is
mucosal inflammation and increased secretions Answer 17.13.3
affecting the airway. Croup occurs from 6 F. Cystic fibrosis
months to 6 years of age. Fever suggests The clinical features suggesting cystic fibrosis are
102 infection. Inspiratory noises suggest upper respiratory infection together with being ‘always
airway obstruction. chesty’ and poor weight gain since birth and his
Irish descent (the cystic fibrosis gene frequency Around 80% of cases of acute otitis media
is particularly high in Ireland). Routine screening resolve spontaneously. Antibiotics marginally
is performed on the newborn biochemical blood shorten the duration of pain but have not been
test at about 5 days of age. However, he may shown to reduce the risk of hearing loss, and
have missed the screening because the test is there is an increased risk of minor side-effects.
done at home following discharge from hospital They would be indicated if the child is still unwell
and he is from a travelling community. after 48 hours.

Respiratory disorders
Answer 17.13.4 Answer 17.14.2
L. Pneumonia D. Inhaled salbutamol via metered dose
Fred has signs and symptoms of pneumonia, inhaler and spacer
including tachypnoea, fever, and localized Chardonnay has an acute exacerbation of
crepitations, as well as a preceding viral upper asthma of moderate severity. She should be
respiratory tract infection. given a high dose of bronchodilator via a
Answer 17.13.5 metered dose inhaler and spacer. Using a
K. Pertussis (whooping cough) nebulizer is no more effective (and might
This infant has paroxysms of coughing, which are encourage her mother to attend hospital again
so severe that they cause her to vomit. This is as she does not have one at home). She does
suggestive of pertussis. The characteristic not need oxygen as her oxygen saturation is
inspiratory whoop may be absent in infants, but normal in air.
apnoea can be a feature at this age. Vaccination
reduces the risk of developing pertussis and Answer 17.14.3
reduces the severity of disease in affected infants N. Oral corticosteroid
but does not guarantee protection. Maternal Jake has croup of moderate severity. Nebulized
immunization against pertussis should reduce steroid (budesonide) or oral corticosteroids have
the risk of their infants developing the disease. been shown to reduce the severity of croup and
need for hospitalization. However, nebulizers are
Answer 17.14.1 more costly and have no advantage over oral
L. Oral antipyretic/analgesic corticosteroids. In the UK oral dexamethasone
This child has acute otitis media. Pain should be rather than prednisolone is usually used as it has
treated with an analgesic such as paracetamol. a longer half-life.

103
18

Cardiac disorders

18.4
Questions: Single Best Answer Tariq, who is 6 weeks old, is admitted directly
from the cardiology clinic with heart failure. He
18.1 has a large ventricular septal defect. The
Alan, a 4-month-old boy, sees his general cardiologist has recommended treatment with
practitioner for an ear infection. On listening to furosemide and spironolactone. His mother
his chest a heart murmur is heard. wants to know why he has only now started to
Which one of the following features most have problems. Which of the following
suggests that it requires further investigation? statements provides the best explanation?

Select one answer only. Select one answer only.

A. A thrill A. At birth and for the first few weeks the


B. Disappearance of murmur on lying flat ductus arteriosus remained patent and this
C. Murmur maximal at the left sternal edge balanced the flow across the septal defect
D. Sinus arrhythmia B. Pulmonary vascular resistance is increasing
E. Systolic murmur and blood is now flowing from right to left
C. The left ventricle is now failing due to its
progressive dilatation
18.2 D. The pulmonary vascular resistance falls after
Which of the following is the most common type birth and now flow from left to right across
of congenital heart disease in the UK? the septal defect is much greater
Select one answer only. E. Volume overload results in decreased return
to the left ventricle and a reduction in
A. Atrial septal defect cardiac output related to a reduced
B. Persistent arterial duct end-diastolic filling pressure
C. Pulmonary stenosis
D. Tetralogy of Fallot 18.5
E. Ventricular septal defect John, who is 6 years old, presents to the
Emergency Department feeling sick and dizzy.
18.3 He was brought to hospital by a paramedic crew
Sunil, a 3-month-old infant, presents with who were called after he became unwell at
breathlessness and sweating on feeding. He has school. His heart rate was noted to be very quick,
had several chest infections. You suspect heart at 260 beats/min and supraventricular
failure. tachycardia is diagnosed. He says he can feel his
heart beating quickly and looks pale. He is
Which of the following is most likely to be crying, saying he wants his mother.
correct regarding his heart failure?
Which of the following should be undertaken by
Select one answer only. the attending team?
A. Hepatomegaly is not a common feature at Select one answer only.
this age
B. It is caused by Eisenmenger syndrome A. Adenosine via a large bore intravenous line
C. It is due to left heart obstruction B. Bilateral carotid sinus massage
D. It is due to a left-to-right shunt C. Direct current cardioversion
E. It is due to an increase in right-to-left D. Reassure that it will resolve spontaneously
shunt E. Vagal stimulation manoeuvre
D. Dextrocardia
Questions: Extended Matching E. Mitral regurgitation
F. Mitral stenosis
18.6 G. Normal
The following is a list (A–J) of congenital heart H. Persistent ductus arteriosus
problems encountered in children. From the list I. Ventricular septal defect
select the most likely diagnosis given the J. Pulmonary stenosis
associated findings on clinical examination.

Cardiac disorders
A. Aortic stenosis The site and heart sounds and murmurs
B. Atrial septal defect are depicted below in Fig. 18.1. For each
C. Coarctation of the aorta presentation, select the most likely diagnosis.

18.6.1
18.6.5 A2 P2
EC A2 P2

4/6

18.6.2
A2 P 2
EC

3/6 soft or
absent

18.6.3
18.6.4 A2 P2
A2 P2

2/6 Fixed

Figure 18.1. EC = ejection click. A2 = aortic component of second heart sound. P2 = pulmonary
component of second heart sound.

18.7 J. Pulmonary stenosis


Below is a list (A–M) of possible findings on K. Tetralogy of Fallot
echocardiography. For each of the following L. Transposition of the great arteries
clinical scenarios described select the most likely M. Ventricular septal defect
findings on echocardiography. Each option may
be used once, more than once, or not at all. 18.7.1
Jack is 24 hours old and his mother notices when
A. Aortic stenosis he is about to breastfeed that he is blue around
B. Atrial septal defect the mouth. On examination, his tongue looks
C. Coarctation of the aorta blue and there is peripheral cyanosis. His
D. Dextrocardia with situs inversus respiratory rate is 65 breaths/min. On
E. Dextrocardia with situs solitus auscultation of the chest there is no murmur.
F. Mitral regurgitation Pulses in all four limbs can be palpated and
G. Mitral stenosis are equal in volume. He is watching you and
H. Normal moving his arms and legs vigorously whilst you 105
I. Persistent ductus arteriosus examine him.
18.7.2 E. Dextrocardia with situs solitus
Sarah was born at term by spontaneous vaginal F. Mitral regurgitation
delivery and went home at 8 hours of age G. Mitral stenosis
following a normal neonatal discharge H. Normal
18 examination. At 48 hours of age her mother
found her looking pale and was unable to
I.
J.
Persistent ductus arteriosus
Pulmonary stenosis
wake her. She was rushed to the Emergency K. Tetralogy of Fallot
Department. Her breathing was noted to be very L. Transposition of the great arteries
Cardiac disorders

shallow, her skin was cool and mottled and she M. Ventricular septal defect (VSD)
was unresponsive to pain. She is resuscitated,
and given intravenous fluids and broad- 18.8.1
spectrum antibiotics. On examination, the only Anoushka, a 1-year-old girl, presents to the
palpable pulse is the right brachial pulse. Emergency Department with a respiratory tract
infection. She is pink and well-perfused. There is
18.7.3 a thrill and pansystolic murmur at the lower left
Azam, a 5-year-old boy, presents with frequent sternal edge.
chest infections. On examination of the chest
there are bilateral crackles at the bases. His heart 18.8.2
sounds can be heard throughout the Debbie, a 3-month-old female infant is being
praecordium but are louder on the right. His reviewed in the paediatric outpatient clinic. She
apex beat is palpable on the right. was referred as on her 6-week check the general
practitioner heard a continuous murmur
18.7.4 throughout the praecordium. She is well and
Jane, a previously fit and well 18-month-old girl, thriving. All peripheral pulses are present and
presents with frequent respiratory tract easily palpable. Oxygen saturation is 96%
infections and wheeze. On examination there is a post-ductal.
fixed and widely split second heart sound, an
ejection systolic murmur best heard at the upper 18.8.3
left sternal edge. Nada, a 5 month old female infant has a fever
and runny nose for 2 days. On examination she
has a fever of 38.3° C and a runny nose. Her
18.8 tongue is pink. Her breathing is normal. Pulse is
Below is a list (A–M) of possible findings on 160 beats/min. Her heart sounds are normal but
echocardiography. For each of the following she has a soft systolic murmur at the left sternal
children who present with a heart murmur select edge. Pulses are normal.
the most likely findings on echocardiography.
Each option may be used once, more than once, 18.8.4
or not at all. Robert, a 3-year-old boy, has had a runny nose
and wheeze for 3 days. On examination his pulse
A. Aortic stenosis is 100 beats/min. Pulses are normal. There is an
B. Atrial septal defect ejection systolic murmur heard loudest at the
C. Coarctation of the aorta upper right sternal edge, which can also be heard
D. Dextrocardia with situs inversus over the carotid arteries but not at the back.

106
E. It is due to an increase in right-to-left shunt
Answers: Single Best Answer It is due to increasing left-to-right shunt.

18.1 18.4
A. A thrill A. At birth and for the first few weeks the
Correct. A thrill is a palpable murmur, i.e. a loud ductus arteriosus remained patent and this
murmur. It always requires further investigation. balanced the flow across the septal defect
A ventricular septal defect is not a ‘duct-
B. Disappearance of murmur on lying flat

Cardiac disorders
dependent’ lesion. A widely open ductus will not
The disappearance of the murmur on lying flat is prevent heart failure here.
characteristic of a venous hum, which is innocent.
B. Pulmonary vascular resistance is increasing
C. Murmur maximal at the left sternal edge and blood is now flowing from right to left
Murmurs at the left sternal edge may be This is Eisenmenger syndrome and would not
innocent or pathological. Further investigation is occur in a 6-week-old baby. It is a late
not indicated if the features of an innocent complication of untreated ventricular septal
murmur are fulfilled. defect/atrioventricular septal defect.
D. Sinus arrhythmia C. The left ventricle is now failing due to
Sinus arrhythmia is a variation in heart rate with progressive dilation
respiration and is a normal finding in children. Dilated cardiomyopathy is a very late change. In
E. Systolic children with ventricular septal defect the heart
Systolic murmurs may be innocent or pathological. remains healthy but heart failure occurs from
Hallmarks of an innocent ejection murmur (all volume overload.
have an ‘S’, therefore innoSent) are: D. The pulmonary vascular resistance falls
• aSymptomatic patient after birth and now flow from left to right
• soft blowing murmur across the septal defect is much greater
• systolic murmur only, not diastolic Correct. The pulmonary vascular resistance falls
• left sternal edge. over the first few weeks of life. This increases the
Additional requirements are: flow across the septal defect and leads to
• normal heart sounds with no added sounds progressively worsening heart failure.
• no parasternal thrill
• no radiation. E. Volume overload results in decreased return
Further investigation is not indicated under to the left ventricle and a reduction in cardiac
these circumstances. output related to a reduced end-diastolic filling
pressure
18.2 In hypovolemic shock there is decreased cardiac
E. Ventricular septal defect filling leading to heart failure. However, this is
Correct. This is the most common single group the result of hypovolemia rather than volume
of structural congenital heart disease (30%). overload and is not the case here.
18.3 18.5
A. Hepatomegaly is not a common feature at A. Adenosine via a large bore intravenous line
this age This is likely to be effective but should be tried
Hepatomegaly is an important clinical feature of only after vagal manoeuvres have been
heart failure in children at all ages. Percussion of undertaken.
the upper border of the liver will help
discriminate between hepatomegaly and B. Bilateral carotid sinus massage
downward displacement of a liver from Whilst this might work, it should not be
hyperexpansion of the chest. undertaken bilaterally as this might be
dangerous.
B. It is caused by Eisenmenger syndrome
Eisenmenger syndrome causes cyanosis from C. Direct current cardioversion
pulmonary hypertension, and usually occurs in This is painful and even synchronized shock
the second decade of life. requires a general anaesthetic first.
C. It is due to left heart obstruction D. Reassure that it will resolve spontaneously
Only in the 1st week of life is heart failure usually He is symptomatic and it should be treated.
from left heart obstruction, e.g. coarctation of
the aorta. E. Vagal stimulation manoeuvre
Correct. Instruct John how to perform a Valsalva
D. It is due to a left-to-right shunt manoeuvre – ask him to put his thumb in his
Correct. After the 1st week of life, progressive mouth and try to blow on it like a trumpet; this
heart failure is most likely due to a left-to-right often works. Cold ice pack to the face is an 107
shunt, most often from a ventricular septal defect. alternative.
help confirm the diagnosis, with the blood
Answers: Extended Matching pressure in the legs being lower than in the
right arm.
Answer 18.6.1
Answer 18.7.3
18 H. Persistent ductus arteriosus
There is flow during systole and diastole D. Dextrocardia with situs inversus
The right-sided apex indicates dextrocardia.
suggesting that there is a pressure gradient
in both. This makes one think of a shunt This may be associated with primary ciliary
Cardiac disorders

between arteries – in this case the aorta to the dyskinesia (Kartagener syndrome), which is
pulmonary artery across the persistent ductus likely to be responsible for his frequent
arteriosus. respiratory infections. Cilia are required to
decide the polarity of an embryo. Without
Answer 18.6.2 adequate ciliary function, the lateralization
J. Pulmonary stenosis of organs occurs randomly and therefore
An ejection click tells you this is a valvular approximately half of children will have
problem. The quiet P2 suggests the pulmonary dextrocardia with situs inversus (stomach
valve is the source. A difficult set of signs to on the right and liver on the left). Isolated
detect clinically. dextrocardia with situs solitus (with the
stomach and liver in their normal positions)
Answer 18.6.3 is not a ciliary problem.
B. Atrial septal defect.
The variation in timing of closure of aortic Answer 18.7.4
and pulmonary valves is lost when there is B. Atrial septal defect
an atrial septal defect. The fixed and widely An atrial septal defect. This classically presents
split second heart sound (often difficult to with an ejection systolic murmur best heard at
hear) is due to the right ventricular stroke the upper left sternal edge – due to increased
volume being equal in both inspiration and flow across the pulmonary valve because of
expiration. the left-to-right shunt. The fixed and widely
split second heart sound (often difficult to
Answer 18.6.4 hear) is due to the right ventricular stroke
I. Ventricular septal defect volume being equal in both inspiration and
The lack of an opening click and the presence of expiration.
a pansystolic murmur are highly suggestive of
ventricular septal defect. In general, all Answer 18.8.1
pansystolic murmurs are appreciated most easily M. Ventricular septal defect
below the level of the nipples. Children with VSDs may have recurrent chest
infections. VSDs classically present with a
Answer 18.6.5 pansystolic murmur loudest at the left
A. Aortic stenosis sternal edge.
An ejection click tells one this is a valvular
problem. The quiet A2 suggests that the aortic Answer 18.8.2
valve is the source. A difficult set of signs to I. Persistent ductus arteriosus
detect clinically. In clinical practice radiation to Most children present with a continuous
the neck (or a suprasternal thrill) is a useful sign murmur beneath the left clavicle. The murmur
of left outflow tract obstruction. continues into diastole because the pressure
in the pulmonary artery is lower than that
Answer 18.7.1 in the aorta throughout the cardiac cycle.
L. Transposition of the great arteries Some sources describe the murmur as sounding
He has cyanotic congenital heart disease. This is like ‘machinery’. Whilst it varies in intensity
transposition of the great arteries as all his pulses during the cardiac cycle (it is usually loudest in
are present on examination. This is also systole and quieter but still present in diastole),
classically the age when this presents. the key feature is its presence throughout systole
and diastole. The pulse pressure is increased,
Answer 18.7.2 causing collapsing or bounding pulses, making
C. Coarctation of the aorta them easy to feel.
Collapse of a newborn can be caused by:
septicaemia/meningitis, congenital heart disease Answer 18.8.3
or an inborn error of metabolism. In this case the H. Normal
likely diagnosis is outflow obstruction in a sick Innocent murmurs can often be heard in
neonate – severe coarctation of the aorta or children. It is obviously important to be able to
interrupted aortic arch. When the ductus distinguish an innocent murmur from a
arteriosus closes, perfusion of the left arm and pathological one. During a febrile illness or
108 lower body is compromised. Performing a anaemia, innocent or flow murmurs are often
four-limb blood pressure measurement might heard because of increased cardiac output.
Answer 18.8.4 indicate valvular involvement but they are
A. Aortic stenosis difficult to hear even with experience. The
It can be difficult to discriminate between left presence of radiation to the neck and/or a
and right outflow tract obstruction as both result suprasternal thrill is a very useful sign and is a
in ejection systolic murmurs. Opening clicks feature of left-sided obstruction.

Cardiac disorders

109
19

Kidney and urinary tract disorders

afebrile and his abdomen is soft with no


Questions: Single Best Answer guarding. He is tender in his left loin and there
are no palpable masses. He has 2+ of haematuria
19.1 on dipstick of his urine but no protein or
Emily is a 2-year-old girl. She presents to the leucocytes. He is not on any medication.
Emergency Department with a 2-day history of
fever, vomiting and ‘smelly’ urine. She has no What is the most likely cause of his pain?
significant medical history and is not on any Select one answer only.
medication. On examination she has a
temperature of 39° C and has a heart rate of 126 A. Constipation
beats/min. She has generalized tenderness over B. Glomerulonephritis
her abdomen. C. Renal stone
D. Trauma
What is the best way to collect a urine sample
E. Wilms tumour
from her?
Select one answer only. 19.4
In Johnathan’s particular case (Question 19.3),
A. Bag sample
what is the most likely organism to have been
B. Catheter sample
the cause of his previous urinary tract infections?
C. Clean catch
D. Pad/cotton wool balls in nappy Select one answer only.
E. Suprapubic aspirate
A. Escherichia coli
19.2 B. Klebsiella sp.
Becky is a 7-year-old girl. She is seen by her C. Proteus sp.
general practitioner with a 2-day history of fever D. Pseudomonas aeruginosa
and abdominal pain. Her mother has noted that E. Streptococcus faecalis
she is going to the toilet to pass urine more
often than usual. On examination she is now 19.5
afebrile and has no abdominal tenderness. Eesa, a 1-month-old Pakistani infant, is taken to
his general practitioner by his mother. He is
What is the most likely diagnosis?
vomiting and not taking his feeds as well as
Select one answer only. normal. He is irritable and has a temperature of
39° C. His heart rate is 170 beats/min and his
A. Appendicitis respiratory rate 45 breaths/min. The remainder
B. Cystitis of his examination is unremarkable. A clean
C. Diabetic ketoacidosis catch urine sample is obtained and is positive for
D. Glomerulonephritis nitrites, leucocytes and protein on dipstick.
E. Pyelonephritis
Which is the most appropriate next course of
19.3 action?
Jonathan is an 8-year-old boy. He has a medical Select one answer only.
history of urinary tract infections. On this
occasion he presents to hospital with acute A. Intravenous antibiotics
spasmodic pain on the left side of his abdomen. B. Intravenous fluids
He says it is the worst pain he has ever C. Oral antibiotics
experienced. He played football yesterday but D. Oral paracetamol
had not hurt himself. On examination he is E. Oral rehydration solution
19.6 19.8
Freddie is a 3-year-old boy. He is referred to the Finlay, a 5-year-old boy, recently had a birthday
hospital as his mother has noticed that his face is party at his local farm. Since then he has had
swollen (Fig. 19.1). His mother was concerned 3 days of diarrhoea. The loose stool had some
that he had an allergic reaction to some peanuts blood in it. His mother is concerned as he is
he ate at a party. Examination reveals abdominal still not himself. He appears to be very pale
distension, bilateral scrotal swelling and pitting and has not passed urine for 12 hours. He has
oedema of his lower limbs. Urine dipstick had 4+ no other medical problems and is not normally

Kidney and urinary tract disorders


protein, 2+ blood and nitrite and leucocyte on any medication. On examination you note
negative. His blood pressure is normal for his that he has pale conjunctivae. His blood
age. You perform renal function tests, which pressure is 120/70 mmHg. You decide to
reveal normal levels of sodium, potassium, urea, take some blood tests and get the following
and creatinine. His complement levels (C3 and results:
C4) are normal.
• haemoglobin: 76 g/L; white blood cells:
14.2 × 109/L; platelet count: 50 × 109/L
• creatinine: 200 µmol/L (normal:
20–80 µmol/L)
• prothrombin time: 13 seconds (control:
12–15 s)
• activated partial thromboplastin time: 34
seconds (control: 25–35 s)
What is the most likely diagnosis?
Select one answer only.
A. Acute lymphatic leukaemia
B. Haemolytic-uraemic syndrome
C. Henoch–Schönlein purpura
D. Immune thrombocytopenic purpura
E. Post-streptococcal glomerulonephritis

19.9
Jane is a 5-year-old girl who presents to her
general practice with bed-wetting. She has been
wetting for 4 months and wets on average three
nights per week. Her mother is upset as she had
been dry during the day and night for almost a
year. She recently started school and has had
two episodes of wetting at school. She has no
other medical problems and is not on any
Figure 19.1 medication. She has always liked to drink water
from a bottle she carries.
What is the most likely diagnosis?
Select one answer only. Which of the following would you do first?
A. Acute allergic reaction to peanuts Select one answer only.
B. Acute glomerulonephritis
C. Haemolytic-uraemic syndrome A. Blood glucose
D. Nephrotic syndrome B. Ultrasound of the abdomen
E. Urinary tract infection C. Urinary dipstick
D. Urinary microscopy and culture
Hint: This child is losing lots of protein in E. Water deprivation test
his urine.
19.10
19.7 Hamza, a 14-year-old Indian boy, has chronic
What is the initial treatment of Freddie’s condition? kidney disease secondary to renal dysplasia. His
Select one answer only. mother wants him to take more responsibility for
his medication and would like you to talk to him
A. Diuretics about the different medicines he takes.
B. Fluid restriction
C. Intravenous human albumin solution 20% Which of the following dietary changes or
D. Intravenous hydrocortisone medications is Hamza likely to be taking to 111
E. Oral prednisolone prevent renal osteodystrophy?
Select one answer only. A urine sample was sent from the Emergency
Department and you receive the following
A. Bicarbonate supplements results: white blood cells <50/mm3; red blood
B. Calcium restriction cells, none seen; organisms, none seen; red cell
19 C.
D.
E.
Phosphate supplements
Sodium supplements
Vitamin D supplements
casts, none seen; culture, mixed coliforms.
19.11.4
George is a 7-year-old boy who presents to his
Kidney and urinary tract disorders

family doctor with cloudy urine. He has no other


Questions: Extended Matching symptoms and his examination is normal. You
obtain the following result from the microscopy
19.11 and culture: white blood cells 50–100/mm3; red
Below is a list of possible diagnoses (A–H) for blood cells, many seen; organisms, none seen;
which urinalysis may be undertaken. For each red cell casts seen; culture, negative at 48 hours.
of the following patients who have urine
19.11.5
microscopy and culture sent from the Emergency
Gregor is a 6-week-old baby who has recently
Department, select the most likely diagnosis.
moved to the UK from Estonia. He presents with
Each option may be used once, more than once,
fever and irritability. On examination he is
or not at all.
clinically shocked. Because of the urgency to
A. Balanitis obtain a sample you take a catheter urine sample
B. Glomerulonephritis and start him immediately on intravenous
C. Nephrotic syndrome antibiotics after a bolus of saline. You obtain the
D. Normal result following result from the microscopy and
E. Perineal contamination culture: white blood cells >200/mm3; red blood
F. Renal stone (calculi) cells, none seen; organisms seen on microscopy;
G. Urinary tract infection red cell casts not seen; culture >105 coliforms.
H. Vulvovaginitis
19.12
19.11.1 Below is a list (A–J) of renal investigations. For
Ji is a 12-month-old Japanese girl who presents each of the following cases which investigation
to the Emergency Department. She has a 2-day would you perform next? Each option may be
history of fever and vomiting. Her mother is used once, more than once, or not at all.
concerned that she is dehydrated. On
examination she is well hydrated and has A. Antistreptolysin O titre
marked coryza. She has an inflamed pharynx. A B. CT scan of the abdomen
urine sample from a bag is sent to the laboratory. C. DMSA scan
You obtain the following result from the D. MAG3 renogram
microscopy and culture: white blood cells E. Micturating cystourethrogram (MCUG)
100/mm3; red blood cells negative; organisms, F. Plain abdominal X-ray
none seen; red cell casts, none seen; culture, not G. Plasma creatinine and electrolytes
available. H. Ultrasound of the kidneys and urinary tract
I. Urinary electrolytes
19.11.2 J. Urine microscopy and culture
Harriet, a 12-year-old girl, presents to the
Emergency Department. She has abdominal 19.12.1
pain and pain on micturition. A mid-stream Max is a 12-year-old boy with cerebral palsy and
urine sample is obtained and is positive for epilepsy. He presents to the Emergency
leucocytes but negative for nitrites. She was Department with severe right-sided colicky
started on oral antibiotics and discharged; abdominal pain. He has had one episode of this
48 hours later you receive the following result previously, which ended as quickly as it started.
from the microscopy and culture: white blood Today the pain is so severe that he has needed
cells >200/mm3; red blood cells, many seen; morphine in the department. On examination
organisms, none seen; red cell casts, none seen; you find a generally tender abdomen. He has a
culture, >105 coliforms. temperature of 37° C, heart rate of 160 beats/
min, and his blood pressure is 120/80 mmHg.
19.11.3 Urine dipstick has been performed and reveals
Gary is a 6-month-old boy. He has been febrile haematuria.
for 1 day, is feeding poorly and has difficulty
breathing. On examination he has a respiratory 19.12.2
rate of 60 breaths/min, marked chest recession Rosa, a 2-year-old girl, presents to her general
and on auscultation has widespread wheeze and practitioner. She has a 24-hour history of
112 fine crepitations. He needs admission for oxygen vomiting and fever. She has a temperature of
therapy. 38° C. She has generalized tenderness of the
abdomen. A dipstick reveals leucocytes and Usmaan has not passed urine. He has a normal
nitrites. Her blood pressure is 90/50 mmHg. blood pressure for a neonate.
19.12.3 19.12.4
Usmaan is an 18-hour-old newborn baby boy John is a 9-year-old boy. He has a history of
who you are seeing for his routine discharge numerous urinary tract infections. He has had an
baby check. You read in his notes that he was ultrasound scan, which reveals that his kidneys
found to have bilateral hydronephrosis and a are dysplastic. You are seeing him in the routine

Kidney and urinary tract disorders


distended bladder on his antenatal ultrasound paediatric follow-up clinic. His mother does not
scan. He mother is breastfeeding and does not feel he is growing as well as his siblings. His
feel she has started producing milk yet as blood pressure is 130/90 mmHg.

113
B. Glomerulonephritis
Answers: Single Best Answer Usually little or no pain and many red cells
and often protein and leucocytes on urine
19.1 analysis.
19 A. Bag sample
A bag sample involves placing an adhesive C. Renal stone
Correct. He is afebrile and the pain is spasmodic
plastic bag to the perineum after washing.
This may also result in contamination from and severe. His previous urinary tract infections
Kidney and urinary tract disorders

skin flora. will have put him at higher risk of stone


formation.
B. Catheter sample
A catheter sample can be obtained if there is D. Trauma
urgency in obtaining a sample and no urine has There is no history of trauma.
recently been passed. However, it is invasive and
E. Wilms tumour
can introduce infection.
Wilms tumour can present with flank pain and
C. Clean catch haematuria but a mass is usually palpable.
Correct. This involves waiting with a sterile bowl
to catch the urine. It is the least invasive and has 19.4
less risk of contamination. The child is well A. Escherichia coli
enough to wait before starting treatment and This is the most common organism to cause
you want to be sure that you are actually urinary tract infection but would not result in
treating a urinary tract infection. The smell of stone formation.
urine is an unreliable sign of genuine infection.
B. Klebsiella sp.
D. Pad/cotton wool balls in nappy The second most common cause of urinary
A pad or cotton wool balls can be used in a tract infection but does not result in stone
nappy but risks contamination, resulting in a formation.
false-positive result.
C. Proteus sp.
E. Suprapubic aspirate Correct. Proteus infection predisposes to the
A suprapubic aspirate is sometimes used in formation of phosphate stones by splitting urea
severely ill infants requiring urgent diagnosis and to ammonia and thus alkalinizing the urine. It is
treatment but it is invasive. It is increasingly more commonly diagnosed in boys than in girls,
replaced by urethral catheter sampling. possibly because of its presence under the
prepuce.
19.2
A. Appendicitis D. Pseudomonas aeruginosa
The absence of abdominal tenderness makes Common only in children with indwelling
appendicitis less likely. catheters, when colonization can become
chronic.
B. Cystitis
Correct. She most likely has cystitis as she has E. Streptococcus faecalis
urine frequency and abdominal pain. A cause but rarer than the others listed.

C. Diabetic ketoacidosis 19.5


This must be considered. The urine must also be A. Intravenous antibiotics
checked for glucose but it is a much less Correct. All infants less than 3 months old
common diagnosis than lower urinary tract who are febrile and systemically unwell should
infection. be referred immediately to a paediatric
D. Glomerulonephritis department as they need treatment with
There would be dark brown urine or frank intravenous antibiotics. He probably is
haematuria. septicaemic from a urinary tract infection as
his urine is positive for nitrites, leucocytes and
E. Pyelonephritis protein on dipstick. However, dipsticks should
As she does not have features of upper urinary not be relied upon to diagnose urinary tract
tract infection, namely fever, feeling systemically infection in children aged under 3 months; at
unwell, localised abdominal or flank pain or this age, urine microscopy and culture should
tenderness, pyelonephritis is unlikely. be used.
19.3 B. Intravenous fluids
A. Constipation Whilst he may require intravenous fluids,
He does not have a history of infrequent this will not effectively treat his urinary tract
114 defaecation and the pain is unlikely to be so infection. A bolus of 0.9% saline is required
acute or severe. if in shock.
C. Oral antibiotics E. Oral prednisolone
Oral antibiotics are suitable first-line treatment Correct. In 85% to 90% of children with
for children with uncomplicated urinary tract nephrotic syndrome, the proteinuria resolves
infection without evidence of serious bacterial with corticosteroid therapy (steroid-sensitive
infection. nephrotic syndrome).
D. Oral paracetamol 19.8
Children in pain should be given paracetamol or A. Acute lymphatic leukaemia

Kidney and urinary tract disorders


another analgesic agent. However, this should Acute lymphatic leukaemia can present with
not delay treatment with antibiotics in this case. anaemia and thrombocytopenia; however, the
E. Oral rehydration solution other features of the history make this less likely.
Oral rehydration solution is an essential B. Haemolytic-uraemic syndrome
treatment for children with gastroenteritis who Correct. This is a characteristic presentation with
are dehydrated. This will not treat his urinary the triad of acute renal failure, microangiopathic
tract infection. haemolytic anaemia and thrombocytopenia.
Typical haemolytic-uraemic syndrome is
19.6
secondary to gastrointestinal infection with
A. Acute allergic reaction to peanuts
verocytotoxin-producing Escherichia coli O157:H7
In acute allergic reaction to peanuts there may
acquired through contact with farm animals or
be periorbital oedema but there is no
eating uncooked beef, or, less often, Shigella.
generalized oedema.
C. Henoch–Schönlein purpura
B. Acute glomerulonephritis
Henoch–Schönlein purpura usually presents with
In acute glomerulonephritis there may be
a rash.
periorbital oedema but there is no generalized
oedema. Complement level would be low. D. Immune thrombocytopenic purpura
Immune thrombocytopenic purpura presents
C. Haemolytic-uraemic syndrome
with a purpuric rash.
Expect marked pallor and possibly bloody stools
if this is the diagnosis. E. Post-streptococcal glomerulonephritis
Post-streptococcal glomerulonephritis would
D. Nephrotic syndrome
present with acute nephritis after a streptococcal
Correct. This is the characteristic presentation.
infection such as tonsillitis but would not be
He will have very low serum albumin which
accompanied by anaemia and
leads to widespread capillary leak and fluid
thrombocytopaenia.
accumulation. He will have gained a significant
amount of weight (from fluid). 19.9
E. Urinary tract infection A. Blood glucose
Not all dipstick-positive urine tests are the result This would identify hyperglycaemia but is
of infection. The absence of nitrites and unnecessarily invasive with this history if the
leucocytes makes infection unlikely. It would not urine has no glucose in it.
result in generalized oedema. B. Ultrasound of the abdomen
19.7 A useful non-invasive test but not the first line
A. Diuretics investigation.
Diuretics are not indicated at presentation in this C. Urinary dipstick
child. They may be used with caution if fluid Correct. She has secondary enuresis, which is
accumulation is causing significant symptoms the loss of previously achieved urinary
(e.g. painful scrotal oedema). continence. A urine dipstick would be useful
B. Fluid restriction immediately as it would be abnormal if she had
Fluid restriction is not indicated in this child. a urinary tract infection or diabetes. The urine
In some instances, children with nephrotic dipstick helps guide one to further
syndrome become hypovolemic. Abdominal pain investigations. The likely cause is emotional
is often a feature and should prompt a thorough upset from starting school and advice can be
review. given once medical conditions have been
ruled out.
C. Intravenous human albumin solution 20%
Human albumin solution is not indicated in this D. Urinary microscopy and culture
child. It is sometimes given for hypovolaemia or A helpful test, but it would be prudent to
in steroid-resistant disease. dipstick the urine first as this would identify
abnormalities suggestive of a urinary tract
D. Intravenous hydrocortisone infection as well as glycosuria from diabetes and 115
Intravenous corticosteroids are not indicated. is easier to perform.
E. Water deprivation test yet a urine sample was sent. Though he has a
Diabetes insipidus is a rare diagnosis. If the positive culture, it is a mixed growth and there is
symptoms persist and the urinalysis is normal, no pyuria (white cells in urine). This is a likely
this may need to be excluded. contaminant.
19 19.10
A. Bicarbonate supplements
Answer 19.11.4
B. Glomerulonephritis
Bicarbonate supplementation is required to George has glomerulonephritis. The presence
Kidney and urinary tract disorders

prevent acidosis. This does not have a big effect of red cell casts in the urine is always
on bone health though. pathological. It is strongly indicative of
glomerular damage.
B. Calcium restriction
On the contrary, a high calcium diet is indicated Answer 19.11.5
as children with renal failure typically have low G. Urinary tract infection
levels. This also helps to bind dietary phosphate, Gregor was systemically unwell on arrival and
which otherwise would be absorbed and result needed resuscitation. Sometimes a urine sample
in elevated phosphate levels. cannot be gained before starting antibiotics but
in this case a catheter was employed.
C. Phosphate supplements
Phosphate intake should be restricted in children Answer 19.12.1
with chronic kidney disease. H. Renal ultrasound
He is likely to have renal stones. Some anti-
D. Sodium supplements epileptic medications can increase the risk. The
Sodium supplements are sometimes needed in first-line investigation would be an ultrasound
children with renal disease due to the loss of scan as this would usually demonstrate the
sodium into their urine but these do not prevent site of obstruction and the presence of most
renal osteodystrophy. stones (even radiolucent ones). Max’s blood
E. Vitamin D supplements pressure and heart rate are mildly elevated.
Correct. Reduced activation of vitamin D results These should be re-checked once his pain is
in secondary hyperparathyroidism, which causes ameliorated.
phosphate retention and hypocalcaemia. Answer 19.12.2
Activated vitamin D supplements help prevent J. Urine microscopy and culture
renal osteodystrophy. Urinary tract infection is the most likely
diagnosis. In children under the age of 3 years,
Answers: Extended Matching urine microscopy and culture should be used to
diagnose urinary tract infection rather than urine
19.11.1 dipstick alone.
E. Perineal contamination
The number of white blood cells is mildly raised. Answer 19.12.3
This is most likely a false-positive result, from H. Ultrsound of kidneys and urinary tract
contamination of the sample. This occurs more This child needs an urgent ultrasound. Bilateral
often if the sample is not a clean catch. White hydronephrosis on antenatal scanning can
cells from the skin, including the vagina in girls indicate urinary obstruction in the lower renal
or foreskin in boys, are ‘washed’ into the sample tract. Usmaan may have posterior urethral valves,
as the urine waits in the bag. This is why a which may be preventing him from voiding
clean-catch sample is better. urine. If the ultrasound confirms bilateral
hydronephrosis and/or a distended bladder, a
Answer 19.11.2 MCUG (micturating cystourethrogram) would
G. Urinary tract infection need to be performed prior to surgery. This
This girl had a ‘false negative’ on her dipstick management should be planned antenatally.
test. The clinical story is very important for
diagnosing urinary tract infection in this age Answer 19.12.4
group. The culture 48 hours later confirms the G. Plasma creatinine and electrolytes
presence of coliforms in the urine. John may be in renal failure and it is important
to measure his renal function. The serum
Answer 19.11.3 creatinine will be the most useful to identify
E. Perineal contamination chronic kidney disease. It is also important
This case highlights that our tests should be to know what his current serum potassium
clinically guided. He presented with bronchiolitis level is.

116
20

Genital disorders

20.2 Testicular swelling


Questions: Single Best Answer Jean-Paul, a 13-year-old boy, has developed an
acutely painful red scrotum. On examination, his
20.1 right testis is found to be swollen and lying
Gary, a 2-month-old infant, is referred urgently to higher in the scrotum than the left. It is tender
outpatients because of an intermittent swelling on palpation.
in both groins as shown in Fig. 20.1.
What is the most important diagnosis to
exclude?
Select one answer only.
A. Epididymitis
B. Hydrocele
C. Idiopathic scrotal oedema
D. Torsion of the appendage of the testes
(hydatid of Morgagni)
E. Torsion of the testis

Hint: Which will lead to testicular loss if


not diagnosed promptly?

20.3
David, a 1-year-old infant, is brought to the
Emergency Department by his mother because
he has a swollen and red foreskin. It has become
worse over the last few days and there is a
discharge from the end of his penis. He has not
suffered from anything like this before.
Which of the following represents the best
treatment plan?

Figure 20.1 Select one answer only.


A. Broad-spectrum antibiotic and warm baths
B. Broad-spectrum antibiotics followed by
How would you best manage this child? circumcision
C. Circumcision on an elective list
Select one answer only.
D. Ice-cold baths
A. Needs immediate emergency surgical repair E. Immediate circumcision on the
B. Needs surgical repair promptly on next emergency list
routine surgical list
C. Place on the waiting list for routine surgical 20.4
repair Harry is a newborn infant. His parents have
D. Reassess at 1 year of age to determine if noticed that his genitalia look abnormal.
surgery is still required Examination findings are shown in Fig. 20.2
E. Reassure that this will resolve spontaneously below. Both testes are palpable.
What management would you initiate?
Select one answer only.
A. Advise that surgery will be needed when
20 older and will correct the problem
B. Advise that he is at an increased risk of other
congenital abnormalities
C. Chromosomal analysis to check his gender
Genital disorders

D. Reassure his parents that it is a variant of


normal
E. Ultrasound of the bladder and kidneys

Figure 20.2

118
20.3
Answers: Single Best Answer A. Broad-spectrum antibiotic and warm
baths
20.1 Correct. This boy is suffering from
A. Needs immediate emergency surgical balanoposthitis. Treatment is with warm baths
repair and a broad-spectrum antibiotic.
This boy has bilateral inguinal hernias, which are
present intermittently and therefore are B. Broad-spectrum antibiotics followed by

Genital disorders
reducible and do not pose an immediate threat circumcision
to the contents of the hernia. The antibiotics will treat the condition, and
circumcision will not be required.
B. Needs surgical repair promptly on next
routine surgical list C. Circumcision on an elective list
Correct. Repair should be done on the next Circumcision is only indicated for recurrent
routine operating list because of the risk of balanoposthitis.
strangulation.
D. Ice-cold baths
C. Place on the waiting list for routine surgical There is active inflammation and infection, and
repair this will not be treated by an ice-cold bath.
The risk of strangulation is high in infants.
E. Immediate circumcision on the emergency
Therefore surgery should be done promptly.
list
D. Reassess at 1 year of age to determine if This is balanoposthitis (inflammation of the glans
surgery is still required and foreskin); recurrent attacks of
The hernia is due to a patent processus balanoposthitis are uncommon, and only then is
vaginalis, which will not close spontaneously. circumcision indicated.
Surgery should be done promptly to avoid 20.4
strangulation.
A. Advise that surgery will be needed when
E. Reassure that this will resolve spontaneously older and will correct the problem
The hernia is due to a patent processus vaginalis, Correct. This is a glanular hypospadias. Surgery
which will not close spontaneously. Therefore is performed to ensure that the boy can pass
surgery is necessary. urine in a single stream in a straight line, has a
straight penile shaft and straight erection and
20.2 has a normal looking penis.
A. Epididymitis
This diagnosis is not the most common or B. Advise that he is at an increased risk of other
dangerous cause of an acute red scrotum in this congenital abnormalities
age group. Hypospadias affecting the glans is not associated
with other congenital abnormalities.
B. Hydrocele
A hydrocele should not be red or present C. Chromosomal analysis to check his gender
acutely. As both testes are descended and the phallus is
of normal size, chromosomal analysis is not
C. Idiopathic scrotal oedema necessary.
This is not usually painful.
D. Reassure his parents that it is a variant of
D. Torsion of the appendage of the testes normal
(hydatid of Morgagni) Hypospadias is a congenital structural
More common than torsion but not as dramatic abnormality of the urethra and meatus and is
clinical features. Usually before puberty. frequently associated with ventral curvature of
the shaft of the penis and a hooded foreskin. It is
E. Torsion of the testis not a variant of normal.
Correct. Surgical exploration is mandatory
unless torsion can be excluded, because it must E. Ultrasound of the bladder and kidneys
be relieved within 6–12 hours of the onset of Hypospadias is not associated with renal tract
symptoms for there to be a good chance of abnormalities unless very severe, so ultrasound
testicular viability. of the kidneys and bladder is not indicated.

119
21

Liver disorders

stool and urine are a normal colour. He is


Questions: Single Best Answer breastfed, although he has not been feeding
well. His mother is concerned that this could be
21.1 due to his constipation. When you examine the
Manuel, a 3-day-old infant is born to healthy infant (Fig. 21.1) you note that he has dry skin
parents. He presents with oozing from the and an umbilical hernia.
umbilical stump and sleepiness. On examination
he is pale and grunting. He responds only to
painful stimuli. He has marked hepatomegaly.
Oxygen is delivered and senior help summoned
as he is very unwell. The nurse practitioner
inserts an intravenous line and asks what blood
tests you would like first.
From the following list of blood tests pick the
one you would undertake first.
Select one answer only.
A. Ammonia
B. Blood culture
C. Blood gas
D. Blood glucose
E. Coagulation studies

21.2. Figure 21.1


Reece, a 4-week-old male infant living in the UK,
is taken to his family doctor because he is
jaundiced. He was born at term and is breastfed.
His mother reports that he has always looked Which of the following is the most likely
yellow and has started to develop bruises. His diagnosis?
stools are now pale in colour. On examination he Select one answer only.
has hepatomegaly.
A. Biliary atresia
Which of the following investigations would you B. Congenital infection
undertake first? C. Galactosaemia
Select one answer only. D. Rhesus haemolytic disease of the newborn
E. Hypothyroidism
A. Faecal elastase
B. Serum conjugated and unconjugated 21.4.
bilirubin Luna, a 32-year-old Cantonese woman has just
C. Sweat test given birth to her third child. She arrived in the
D. Ultrasound scan of the liver UK 3 months ago to live with her extended
E. Urinalysis family following the death of her husband.
Antenatal screening shows that she is hepatitis B
21.3. surface antigen (HBsAg) positive and hepatitis B
Javid is a 5-month-old Asian baby born at term e antigen (HBeAg) negative. The newborn infant
in rural Pakistan. He presents with jaundice. His looks well and has fed. The postnatal team are
mother’s blood group is AB rhesus positive. His keen to send the mother and baby home.
Which of the following is the best advice to give
concerning immunization of the family? Questions: Extended Matching
Select one answer only. 21.6.
The following is a list of diagnoses (A–L). For
A. Hepatitis B vaccination for the baby
each of the following patients with liver disease,
B. Hepatitis B vaccination for the baby and all
select the most likely diagnosis from the list.
other children
Each option may be used once, more than once,
C. Hepatitis B vaccination for the baby and

Liver disorders
or not at all.
mother
D. Hepatitis B vaccination for the baby with A. α1-antitrypsin deficiency
hepatitis B immunoglobulin B. Autoimmune hepatitis
E. No treatment required C. Bacterial infection
D. Biliary atresia
21.5. E. Cystic fibrosis
Summer, a 12-year-old girl, is seen in the F. Hepatitis A
Emergency Department. Her parents report that G. Hepatitis B
her school performance has been deteriorating H. Hepatitis C
and recently she has become confused and I. Inflammatory bowel disease
unsteady on her feet. J. Galactosaemia
K. Primary sclerosing cholangitis
Examination findings of her eyes are shown in
L. Wilson disease
Fig. 21.2.
21.6.1.
Jason, a 3-week-old boy, is still jaundiced. His
mother is reassured that this is likely to be
‘breast milk’ jaundice as she is fully breastfeeding
him. He presents 3 weeks later with poor
feeding, vomiting and bruising on his forehead
and limbs. He has pale stools. On examination
the liver is palpable 4 cm below the costal
margin.
21.6.2.
Raj, a previously well 14-year-old Asian boy, is
noted to be jaundiced. He has recently returned
to the UK from India where he was visiting
relatives in a rural village. He had a 10-day
diarrhoea and vomiting illness whilst in India.

Figure 21.2 (Courtesy of Prof Deidre Kelly). 21.6.3.


A 5-week-old southern Asian male infant born in
the UK presents to the Paediatric Assessment
Unit with vomiting. He has not gained weight
since birth. On examination you find an infant
Select the most likely diagnosis. who is jaundiced, lethargic and hypotonic.
Cataracts are present.
Select one answer only.
21.6.4.
A. Glaucoma Lee is a 12-year-old Chinese boy who moved
B. Hyperthyroidism with his parents from China 2 years ago. He
C. Illicit drug use presents with episodes of vomiting which is
D. Intracranial tumour blood-stained. On examination he is jaundiced,
E. Wilson disease malnourished and has splenomegaly.

121
21.3
Answers: Single Best Answer A. Biliary atresia
Biliary atresia is very unlikely here due to the
21.1 normal stool colour. However, a blood test to
21 A. Ammonia
Manuel may have liver failure and high
assess the proportion of conjugated bilirubin
should be performed. If this is raised, then biliary
ammonia. However, knowing this will not help atresia would be considered.
your initial management.
Liver disorders

B. Congenital infection
B. Blood culture The TORCH congenital infections (toxoplasmosis,
There may be infection here and blood cultures syphilis, rubella, cytomegalovirus, or herpes) are
are indicated. However, failure to do so will not acquired by the mother during pregnancy and
change initial management. passed on to the developing fetus. Affected
C. Blood gas babies may be jaundiced, but they
This is almost certain to demonstrate a marked characteristically have other problems of the
acidosis. In practice it may give a blood glucose heart, skin, eye, and central nervous system.
measurement too but a blood gas is not the C. Galactosaemia
priority. Galactosaemia is a rare disorder of carbohydrate
D. Blood glucose metabolism that presents with poor feeding,
Correct. This must be done. Hypoglycaemia is vomiting, jaundice and hepatomegaly when
an important consequence of serious illness fed milk.
including liver failure, e.g. from galactosaemia in D. Rhesus haemolytic disease of the newborn
view of his hepatomegaly and bleeding and Not the cause as the mother is AB rhesus
sepsis. The priorities for management are: positive. Rhesus disease could be possible if the
• maintain blood glucose mother was rhesus negative. ABO incompatibility
• treat sepsis with broad-spectrum antibiotics is usually only seen in mothers with an O blood
• prevent haemorrhage with intravenous group.
vitamin K and fresh frozen plasma.
E. Hypothyroidism
E. Coagulation studies Correct. Hypothyroidism is a cause of prolonged
Important but likely to be deranged as there is jaundice in infants. Clinical features include dry
bleeding. skin, constipation, coarse facial features including
21.2 a large tongue as in the figure, umbilical hernia
A. Faecal elastase and a hoarse cry. In the UK it is usually identified
A useful test to determine exocrine pancreatic on newborn biochemical screening (Guthrie test).
insufficiency, but not the priority here.
21.4
B. Serum conjugated and unconjugated A. Hepatitis B vaccination for the baby
bilirubin This is required but it is important to also
Correct. This will enable you to identify whether, vaccinate other family members (in this case the
as the history suggests, this child has a conjugated older siblings).
hyperbilirubinaemia. If this is the case then urgent
further investigation is necessary as the child may B. Hepatitis B vaccination for the baby and
have biliary atresia. Delay in diagnosis and all other children
definitive treatment adversely affects outcome. Correct. Prevention of hepatitis B virus infection
is important. All pregnant women should have
C. Sweat test antenatal screening for HBsAg. Babies of all
Reece should have had a screening test for cystic HBsAg-positive mothers should receive a course
fibrosis at birth. It may cause prolonged of hepatitis B vaccination (given routinely in
conjugated jaundice but is not a first-line many countries).
investigation.
C. Hepatitis B vaccination for the baby and
D. Ultrasound scan of the liver mother
This can help identify the likely underlying cause. The other children in the family should be
A fasting abdominal ultrasound scan may vaccinated.
demonstrate an absent or shrunken gallbladder
in biliary atresia but one first needs to confirm D. Hepatitis B vaccination for the baby with
conjugated jaundice. hepatitis B immunoglobulin
Hepatitis B immunoglobulin is also given if the
E. Urinalysis mother is also HBeAg-positive. Antibody
The urine is likely to be dark if the child has response to the vaccination course should be
122 neonatal liver disease. However, laboratory checked in high-risk infants at 12 months as 5%
testing is not usually necessary. require further vaccination.
E. No treatment required
Incorrect. Approximately 30% to 50% of carrier Answers: Extended Matching
children will develop chronic hepatitis B virus
liver disease. Answer 21.6.1
D. Biliary atresia
21.5 All infants with jaundice after 2 weeks of age
A. Glaucoma should be investigated to exclude this. The
Blurred vision and a sore eye or unilateral parents should never be reassured that this is

Liver disorders
headache should make you consider glaucoma. breast milk related until a conjugated bilirubin
It is a rare diagnosis in children but important has been shown to be normal. The pale stool
not to miss. occurs due to failure of the liver to excrete bile,
and indicates that the jaundice is conjugated, and
B. Hyperthyroidism
therefore not due to breast milk jaundice.
This may cause reduced concentration at school
and proptosis. However, the upper margin of the Answer 21.6.2
iris is not exposed here. Assessment for F. Hepatitis A
tachycardia/sweating will be required. Hepatitis A virus. Vaccination is recommended
C. Illicit drug use for those travelling to endemic areas but it
A cause of deterioration in school performance. incurs a charge and many families do not
Teenagers may also become secretive and undertake it.
withdrawn. Cannabis use may result in red eyes
Answer 21.6.3
with dilated pupils. However, this is not what is
J. Galactosaemia
shown.
In this very rare disorder infants develop poor
D. Intracranial tumour feeding, vomiting, jaundice and hepatomegaly
Eye signs are a common but rather late sign of when fed milk. Liver failure, cataracts and
brain tumours. The most obvious is a ‘false- developmental delay are inevitable if untreated.
localizing sign’ of VI nerve palsy. This results in A rapidly fatal course with shock and
double-vision and a failure to fully abduct the disseminated intravascular coagulation, often
affected eye. Early morning headache and head due to Gram-negative sepsis, may occur. For this
tilt are both ‘red-flag’ signs. reason, galactosaemia is being screened for in
some countries (but not the UK).
E. Wilson disease A bacterial infection (such as a urinary tract
Correct. You are being shown Kayser-Fleischer infection) would need to be excluded.
rings from copper in the cornea. This is a very
rare but treatable cause of liver failure/ Answer 21.6.4
neurological deterioration. Neuropsychiatric G. Hepatitis B
features are more common in those presenting Hepatitis B virus has a high prevalence and
from the second decade onwards and include carrier rate in the Far East and sub-Saharan
deterioration in school performance, mood and Africa. This child probably acquired the infection
behaviour change and extrapyramidal signs such vertically from his mother or horizontally from
as incoordination, tremor and dysarthria. A very another family member. His vomiting blood
rare diagnosis – a typical hospital in the UK will most likely represents oesophageal varices from
see one case every 30 years (it has an incidence portal hypertension. The main clue to hepatitis B
of 1 in 200 000). is his country of origin.

123
22

Malignant disease

Which of the following investigations would be


Questions: Single Best Answer the most useful in making the diagnosis?

22.1 Select one answer only.


Concerning the epidemiology of childhood
A. Plasma ammonia
cancer, which of the following types is most
B. Plasma LDG (lactate dehydrogenase)
common in the UK?
C. Serum α-fetoprotein
Select one answer only. D. Serum beta-HCG (human chorionic
gonadotrophin)
A. Bone tumour E. Urinary catecholamines
B. Brain tumour
C. Leukaemia
D. Neuroblastoma 22.4
E. Wilms tumour (nephroblastoma) Josh is a 2-year-old boy who presents to the
Emergency Department with weight loss and
22.2 lethargy. On examination he has splenomegaly.
Polly is a 2-year-old girl who is receiving You are worried that he has leukaemia. You
chemotherapy for her acute lymphoblastic perform an urgent full blood count.
leukaemia. She is known to be neutropenic and What are the most likely haematology results if
has developed a fever of over 38.5° C. he has leukaemia?
What is the most appropriate course of action? Select one answer only.
Select one answer only. A. Low haemoglobin and low platelets
A. Be admitted to hospital for observation B. High haemoglobin and low platelets
B. Be started immediately on oral antibiotics C. Normal haemoglobin and normal platelets
C. Have her blood count and inflammatory D. High haemoglobin and raised platelets
markers (e.g. C-reactive protein) measured E. Low haemoglobin and raised platelets
D. Have blood cultures taken and be started on
intravenous antibiotics 22.5
E. See her general practitioner for further Amy is a 2-year-old girl who presents to the
assessment and decision regarding outpatient department with a history of
antibiotics developmental regression. She was able to walk
but has ‘gone off her legs’ in the last few weeks.
22.3 Her parents are very worried about her. On
Mohammed is a 3-year-old boy who is reviewed further questioning she had been well with no
in the Paediatric Assessment Unit. He has a preceding coryzal illness. On examination, she is
history of weight loss and lethargy. His mother very unsteady on her feet and her power is
is also concerned as he keeps crying and reduced in her lower limbs. She has no obvious
complains of pain in his tummy. On examination visual problems and seems able to follow the
of his abdomen he has an extensive mass. The picture book her mother has brought to clinic.
doctor is worried he may have a childhood A CT scan is obtained and is shown below in
malignancy. Fig. 22.1.
Mark’s parents are worried and phone the
oncology ward for advice.
What information should they be given?
Select one answer only.
A. Advise them to see their general practitioner
to check that he is all right

Malignant disease
B. Mark needs urgent treatment to prevent him
from becoming unwell
C. Monitor Mark and if he becomes unwell with
a fever, bring him to the ward
D. Monitor Mark and if he has any signs of the
rash, bring him to the ward
E. Reassurance – this is a common illness that
most children get

Figure 22.1 22.7


Essa is a 2-year-old boy whose parents have
noticed his two eyes look different. He has no
other medical history and is not currently on any
What is the most likely underlying cause? medication. On examination, the movement of
Select one answer only. the eyes is normal and the pupils are equal and
reactive to light. On checking his pupillary reflex
A. Cerebral abscess you observe his left pupil looks red but the right
B. Cortical astrocytoma looks white. His systemic examination is normal.
C. Craniopharyngioma
D. Medulloblastoma What is the likely cause of this?
E. Viral encephalitis Select one answer only.

22.6 A. Congenital cataract


Mark, a 3-year-old boy, is currently receiving B. Glaucoma
chemotherapy. His sister has developed the rash C. Retinoblastoma
shown in Fig. 22.3 below. Mark is well and does D. Allergic conjunctivitis
not have a fever. E. VI nerve palsy

22.8
Brittney, aged 5 years, goes to her optician. She
has needed glasses for 4 months but her mother
thinks her prescription needs changing as she
has been getting progressively worsening
headaches. When the optician examines her eyes
she finds the examination shown in Fig. 22.4.
Both her eyes have a red reflex.

Looking left

Looking right

125
Figure 22.3 Figure 22.4
What is the most likely diagnosis? have increased in intensity. These have woken
him from sleep. His mother reports
Select one answer only. that he has recently begun vomiting in the
A. Craniopharyngioma morning. His teachers have also commented
22 B.
C.
Concomitant squint
Optic glioma
his school work is getting worse. His mother
thinks this may be because his sight has
D. Posterior fossa tumour deteriorated as he keeps complaining of
E. Retinoblastoma double vision.
Malignant disease

22.10
Questions: Extended Matching Below is a list (A–K) of investigations. For each of
the following patients with malignant disease
22.9 select the investigation most likely to confirm
Below is a list (A–K) of possible diagnoses. For the diagnosis. Each option may be used once,
each of the clinical scenarios, select the most more than once, or not at all.
likely diagnosis. Each option may be used once,
more than once, or not at all. A. Bone marrow aspirate
B. Blood film
A. Acute lymphoblastic leukaemia C. Chest X-ray
B. Bone tumour D. Clotting screen
C. Brain tumour E. CT scan
D. Burkitt lymphoma F. Excision biopsy
E. Henoch–Schönlein purpura G. Full blood count
F. Hodgkin disease H. Magnetic resonance imaging (MRI) scan
G. Immune thrombocytopenia purpura I. Positron emission tomography (PET) scan
H. Langerhans cell histiocytosis J. Ultrasound of abdomen
I. Neuroblastoma K. Urine catecholamines
J. Retinoblastoma
K. Wilms tumour (nephroblastoma) 22.10.1
Connor, a 4-year-old boy, presents to the
22.9.1 Paediatric Assessment Unit with his parents.
Natalia, a 4-year-old girl, presents to her general They are worried as he seems to be very tired
practitioner. This is her second bad episode of and complains of his legs hurting. He also
tonsillitis in the same month. She is tired and seems to have a fine rash that has developed
looks pale and has a number of bruises on her on his arms. On examination he is pale, has a
lower legs. On examination she has pallor, petechial rash on his arms and legs and has
scattered purpuric skin lesions and hepatosplenomegaly. A full blood count shows
hepatosplenomegaly. low haemoglobin and platelet count.
22.9.2 22.10.2
Angel, a 2-year-old boy, presents to his general Niamh, a 4-year-old girl, is taken to her general
practitioner with an abdominal mass noticed by practitioner as her mother has noticed she has
his mother on dressing him. He has no other red urine. On further questioning she has been
medical problems and is not on any medication. more tired than normal and has been
His stool pattern is regular. He is otherwise complaining of abdominal pain. On examination
relatively well in himself. You examine his she is pale with a left-sided abdominal mass. Her
abdomen and feel a mass in his left abdomen, urine is red and a dipstick confirms that this is
which does not cross the midline. There is no blood.
hepatosplenomegaly.
22.10.3
22.9.3 Oscar, a 3-year-old boy, attends the Emergency
Kay, a 3-year-old girl, presents to the Paediatric Department as his mother is worried he has lost
Assessment Unit as her mother is worried she is weight and looks pale. On examination he has a
pale, tired and ‘not quite right’. She has also lost large irregular mass extending across his
2 kg of weight in the last month. On examination abdomen. His blood pressure is high.
the child looks unwell, has pallor and a large
firm, irregular abdominal mass in the centre of 22.10.4
her abdomen. Francis, a 2-year-old boy, is taken to his general
practitioner by his father who is worried as he
22.9.4 has become ‘cross-eyed’. Otherwise he is very
Douglas, a 7-year-old boy, visits his general well in himself and has no history of vomiting.
practitioner with his mother. He has been On examination he appears to be well but has an
126 getting headaches over the last 3–5 weeks which absent red reflex in his left eye.
22.10.5 reports that the ‘glands’ in his neck have been
Carla is a 4-year-old girl who is seen in the enlarged for several months now. He has no
Paediatric Assessment Unit complaining of other medical problems. He has not been having
headaches. Her mother has noticed that her eye any episodes of fever or night sweats. On
movements are not normal. The whole family examination he has several large, irregular, hard
has recently had a sickness bug but Carla seems lymph nodes in his neck. They are all greater
to have continued vomiting. than 2 cm in size. You order a full blood count
and blood film, which show normal results.

Malignant disease
22.10.6
Solomon is a 12-year-old boy. He has recently
lost weight and his ‘glands are up’. His mother

127
Merely observing or just giving oral antibiotics
Answers: Single Best Answer would not be aggressive enough for what could
be a life-threatening infection.
22.1

22 A. Bone tumour
Bone tumours represent about 4% of the total.
E. See her general practitioner for further
assessment and decision regarding antibiotics
This would add delay to the process of initiating
They are uncommon before puberty. In
adolescents they are an important cancer type appropriate treatment.
Malignant disease

and outcomes are not as good as those for


leukaemia. 22.3
A. Plasma ammonia
B. Brain tumour Useful for the diagnosis of urea cycle disorders
Brain and spinal tumours make up about but not elevated in neuroblastoma.
one-quarter of all childhood malignancies (24%).
They often present late and treatment is difficult. B. Plasma LDG (lactate dehydrogenase)
Elevated (often significantly) in lymphoma and
C. Leukaemia other solid tumours. Rather non-specific though,
Correct. Leukaemia is the most common and will be increased in any process where there
malignancy in childhood (about a third of all is increased cell breakdown, e.g. haemolysis.
cases; see Fig. 22.1 in Illustrated Textbook of
Paediatrics). Acute lymphoblastic leukaemia is C. Serum α-fetoprotein
most likely. It accounts for 80% of leukaemia in Along with β-human chorionic gonadotropin,
children. this is elevated in germ-cell tumours. However,
these are rare.
D. Neuroblastoma
This is a relatively rare tumour after the first 5 D. Serum β-HCG (human chorionic
years of life but is a fairly common type in gonadotrophin)
children under the age of 5 years. It makes up Along with α-fetoprotein, β-HCG is elevated in
about 7% of the total. germ-cell tumours.

E. Wilms tumour (nephroblastoma) E. Urinary catecholamines


This is a rare tumour but affects a relatively Correct. An extensive abdominal mass in an
narrow age range, being more common in unwell child of his age suggests neuroblastoma.
children under 5 years of age and rarely seen Urinary catecholamine levels would be elevated.
after 10 years of age. Most children look
remarkably well and may present with either an 22.4
abdominal mass or haematuria. A. Low haemoglobin and low platelets
Correct. A low haemoglobin and low platelet
count are often present at diagnosis due to bone
22.2
marrow failure. Circulating leukaemic blast cells
A. Be admitted to hospital for observation
may be present. There is often either a very high
Children with febrile neutropenia become very
or a low white cell count.
unwell very quickly. This is a case where it is
important to treat early and before B. High haemoglobin and low platelets
overwhelming sepsis takes hold. Thrombocytopenia is common as is a high white
cell count but there is normally an associated
B. Be started immediately on oral antibiotics
anaemia.
Oral antibiotics are insufficient protection against
the wide range of potential pathogens. Broad- C. Normal haemoglobin and normal platelets
spectrum intravenous antibiotics are required. This can occur in the early stages. However, by
the time there is splenomegaly this is unlikely.
C. Have her blood count and inflammatory
markers (e.g. C-reactive protein) measured D. High haemoglobin and raised platelets
This would be done, but is not likely to change This does not occur.
treatment. The main purpose of blood tests in
this instance is to determine whether the child is E. Low haemoglobin and raised platelets
neutropenic. A thrombocytosis is sometimes seen in chronic
blood loss states or following viral infection.
D. Have blood cultures taken and be started However, it is an unusual presenting feature of
on intravenous antibiotics leukaemia.
Correct. Children with fever and neutropenia are
at high risk of serious infection and must be 22.5
admitted promptly to hospital for cultures and A. Cerebral abscess
treatment with broad-spectrum antibiotics. The The clinical history usually gives a fever and
128 absence of neutrophils may lead to less obvious there would be a ‘ring enhancing lesion’ on CT
clinical signs than otherwise expected. (or MRI) scan.
B. Cortical astrocytoma D. Monitor Mark and if he has any signs of the
Cortical astrocytoma is not likely as there is no rash, bring him to the ward
evidence of raised intracranial pressure or focal This is not appropriate as his sibling has
neurological signs or seizures typical of cortical chickenpox and the aim is to prevent him from
tumours and the mass is in the posterior fossa. becoming seriously ill from catching it.
C. Craniopharyngioma E. Reassurance – this is a common illness that
Craniopharyngioma is unlikely as there is no most children get

Malignant disease
evidence of pituitary deficiency or visual field It is very dangerous in children who are receiving
loss and the mass is in the posterior fossa. chemotherapy.
D. Medulloblastoma 22.7
Correct. Medulloblastoma (~20% of brain A. Cataract
tumours) arises in the midline of the posterior Congenital cataract can cause leukocoria but
fossa (Fig. 22.2). These children present with would have been detected at an earlier age.
truncal ataxia and incoordination.
B. Glaucoma
Whilst this is a rare disease in childhood, it would
be most likely to lead to a swollen red eye. There
is often excess tear production and light
sensitivity in children. It is more common in
children with neurofibromatosis or Sturge–
Weber syndrome.
C. Retinoblastoma
Correct. White papillary reflex (also known as
leukocoria, see Fig. 22.17 in Illustrated Textbook
of Paediatrics) requires urgent ophthalmological
assessment as it can be caused by:
• retinoblastoma
• corneal opacity
• congenital cataract (usually presents shortly
after birth)
• vitreous opacity
• retinal disease, e.g. retinal detachment.
Figure 22.2 There is no history of trauma or a foreign body
making this unlikely.
E. Viral encephalitis D. Allergic conjunctivitis
Unlikely as there is no fever and she has ataxia Would cause bilateral conjunctivitis rather than a
and reduced power only of the lower limbs. The unilateral white reflex.
imaging also shows a mass.
E. VI nerve palsy
22.6 A VI nerve palsy would not cause a white
A. Advise them to see their general practitioner papillary reflex.
to check that he is all right 22.8
This merely adds delay to the process. Nearly all A. Craniopharyngioma
children undergoing chemotherapy have open Craniopharyngioma can cause loss of visual
access to the ward. fields, classically bitemporal hemianopia, rather
B. Mark needs urgent treatment to prevent than a VI nerve palsy.
him from becoming unwell B. Concomitant squint
Correct. His sister has varicella zoster (chicken This is a paralytic squint and not a concomitant
pox) infection, and this can be life-threatening in (non-paralytic) squint.
immunocompromised patients. He needs
treatment with varicella zoster immunoglobulin C. Optic glioma
unless he is immune. The incubation period An optic glioma would affect vision but not the
for varicella is 14–21 days, so one cannot be eye movements.
reassured by the fact that he is currently well.
D. Posterior fossa tumour
C. Monitor Mark and if he becomes unwell with Correct. Brittney has bilateral VI nerve palsy –
a fever, bring him to the ward she is unable to look to her left or right because
This is not appropriate as his sibling has of lateral abducens (VI) nerve palsy. She is likely
chickenpox and the aim is to prevent him from to have a posterior fossa tumour. It is a false 129
becoming seriously ill from catching it. localizing sign from raised intracranial pressure.
E. Retinoblastoma blood film would be very useful but to confirm
Retinoblastomas cause a white reflex but do not and classify the diagnosis a bone marrow
affect the eye movements. examination is essential.
Answer 22.10.2
22 Answers: Extended Matching J. Ultrasound of abdomen
This child is likely to have a Wilms tumour
Answer 22.9.1 (nephroblastoma). An ultrasound is the easiest
Malignant disease

A. Acute lymphoblastic leukaemia test to perform and so would be performed prior


Clinical symptoms and signs result from to either a CT or MRI scan.
disseminated disease and systemic ill health
from infiltration of the bone marrow or other Answer 22.10.3
organs with leukaemic blast cells. In most K. Urine catecholamines
children, leukaemia presents insidiously over This child is likely to have a neuroblastoma.
several weeks but in some it presents and Urinary catecholamine levels would be raised.
progresses very rapidly. A confirmatory biopsy is usually obtained
and evidence of metastatic disease is detected
Answer 22.9.2 with bone marrow sampling and MIBG
K. Wilms tumour (nephroblastoma) (metaiodobenzylguanidine) scan with or without
Most children present in this way and before 5 a bone scan.
years of age.
Answer 22.10.4
Answer 22.9.3 H. MRI scan
I. Neuroblastoma This child is likely to have a retinoblastoma
Most common before 5 years of age and most and this is best confirmed using a MRI scan
children present with an abdominal mass, which of the head and an eye examination under
is large and complex, crossing the midline. Over anaesthesia.
the age of 2 years, clinical symptoms are mostly
from metastatic disease, particularly bone pain, Answer 22.10.5
bone marrow suppression causing weight loss H. MRI scan
and malaise. This child is likely to have a brain tumour.
Brain tumours are best characterized on MRI
Answer 22.9.4 scan. Often a CT scan is performed first if MRI
C. Brain tumour is not readily available, but is not as good at
The HeadSmart campaign (www.headsmart identifying tumour type. Magnetic resonance
.org.uk) highlights presentation of brain tumours spectroscopy can be used to examine the
in children of his age: biological activity of a tumour. Lumbar puncture
• persistent/recurrent headaches must not be performed without neurosurgical
• persistent/recurrent vomiting advice if there is any suspicion of raised
• abnormal balance/walking/coordination intracranial pressure.
• abnormal eye movements, blurred or
double vision Answer 22.10.6
• behaviour change F. Excision biopsy
• seizures It is common for children to have cervical
• abnormal head position, e.g. head tilt, wry lymphadenopathy, but here the warning
neck, stiff neck. signs are weight loss and large and irregular
lymph nodes. The concern is that this boy has
Answer 22.10.1 a lymphoma. A normal full blood count and
A. Bone marrow aspirate blood film cannot rule out the diagnosis and
This child has most likely got acute a lymph node biopsy would have to be
lymphoblastic leukaemia. A full blood count and performed.

130
23

Haematological disorders

Select one answer only.


Questions: Single Best Answer
A. Acute lymphatic leukaemia
23.1 B. β-Thalassaemia trait
Tia is a 4-year-old Caucasian girl. She is referred C. G6PD (Glucose-6-phosphate dehydrogenase)
to the paediatric ward by her general deficiency
practitioner as her mother noted that she had a D. Pyelonephritis
yellow tinge to her eyes since developing an E. Sickle cell disease
upper respiratory tract infection. She is well in
herself and has no history of weight loss. There is
no family history of any blood disorders. On 23.3
examination she was pale and her spleen was Tom is a 5-year-old boy who presents to hospital
enlarged 3 cm below the costal margin. with a recent history of bruising easily. Two
weeks ago he had an upper respiratory tract
You perform a full blood count, which reveals: infection which resolved spontaneously. On
• Hb (haemoglobin): 60 g/L examination today he is afebrile but has many,
• WBC (white blood cell count): 8 × 109/L widespread bruises with some scattered
• platelet count: 255 × 109/L petechiae.
• blood film: small red cells
• MCV (mean cell volume): 60 fL (normal: Investigations reveal:
75–87 fL) • Hb (haemoglobin): 116 g/L
• WBC (white blood cell count): 10.2 × 109/L
What is the most likely diagnosis? • platelet count: 32 × 109/L
• prothrombin time: 15 seconds (control:
Select one answer only.
12–15 s)
A. Acute lymphoblastic leukaemia • activated partial thromboplastin time: 30
B. Glucose-6-phosphate dehydrogenase (G6PD) seconds (control: 25–35 s)
deficiency • fibrinogen: 2.5 g/L (normal: 2–4 g/L)
C. Hereditary spherocytosis
What is the most likely diagnosis?
D. Sickle cell disease
E. Thalassaemia Select one answer only.

23.2 A. Acute lymphoblastic leukaemia


Ahmed, whose parents come from Egypt, is a B. Haemophilia A
10-year-old boy who presents to his general C. Immune thrombocytopenic purpura
practitioner. This evening he is more lethargic D. Non-accidental injury
than usual and his urine has become dark in E. Vitamin D deficiency
colour. There is no history of excessive exercise
or beetroot consumption; in fact, they had a
festive meal of chicken, fish, broad beans and 23.4
rice for lunch. His examination is normal and he Amir is a 2-year-old Bangladeshi boy who
is afebrile. He has no medical history of note. He has eczema. His general practitioner is
has not had a recent upper respiratory tract undertaking a routine review of his care and
infection. notices that Amir is pale. The remainder of his
examination is normal. His mother reports that
What is the most likely underlying cause of his he has been eating bits of carpet from his room
new symptoms? recently.
Investigations reveal: 23.6
• Hb (haemoglobin): 66 g/L Peter, aged 9 months, presents to the Emergency
• WBC (white blood cell count): 10.2 × 109/L Department. His family moved to the UK when
• platelet count: 350 × 109/L Peter was 6 weeks old. He has a 6 hour history of
23 • MCV (mean cell volume): 60 fL (normal:
75–87 fL)
pain in his fingers. He has had an upper
respiratory tract infection for the past 24 hours.
He has no other medical history and is not on
What is the most appropriate treatment? any medication.
Haematological disorders

Select one answer only. The appearance of his left hand is shown in
A. Dietary advice Fig. 23.2. What is the most likely diagnosis?
B. Folic acid
C. Iron supplements
D. Multivitamin tablets
E. Vitamin B12 injections

23.5
Joseph, a 2-year-old black Caribbean boy from
London, is admitted to hospital for an elective
repair of an inguinal hernia. He has no other
medical problems. His pre-operative assessment
reveals the following results and his blood film is
shown below (Fig. 23.1):

Figure 23.2

Select one answer only.


A. Acute lymphoblastic leukaemia
B. β-Thalassaemia major
C. Glucose-6-phosphate dehydrogenase
deficiency
D. Haemophilia A
E. Sickle cell disease
Figure 23.1 (Courtesy of Prof Paula
Bolton-Maggs). Questions: Extended Matching
23.7
Below is a list of possible diagnoses (A–R).
• Hb (haemoglobin): 86 g/L For each of the following patients with
• MCV (mean cell volume): 68 fL (normal: haematological problems select the most likely
75–87 fL) diagnosis. Each option may be used once, more
• MCHC (mean corpuscular haemoglobin than once, or not at all.
concentration): 22 g/dl (normal: 32–35 g/dl)
• WBC (white blood cell count): 11.2 × 109/L A. Acute lymphoblastic leukaemia
• platelets: 262 × 109/L B. Acute myeloid leukaemia
• HB electrophoresis: haemoglobin A (HbA) C. α-Thalassaemia major
98%; haemoglobin A2 (HbA2) 2% D. α-Thalassaemia trait
E. β-Thalassaemia major
What is the most likely diagnosis? F. β-Thalassaemia trait
Select one answer only. G. Glucose-6-phosphate dehydrogenase
deficiency
A. β-Thalassaemia trait H. Haemophilia A
B. Glucose-6-phosphate dehydrogenase I. Immune thrombocytopenic purpura
deficiency J. Infectious mononucleosis
C. Iron-deficiency anaemia K. Iron deficiency anaemia
132 D. Normal variation for age and ethnicity L. Liver disease
E. Sickle cell trait M. Meningococcal disease
N. Normal variant Further testing reveals 5% haemoglobin A2
O. Sickle cell disease (HbA2) and 3% fetal haemoglobin (HbF).
P. Vitamin D deficiency
Q. Vitamin K deficiency 23.8
R. von Willebrand disease Below is a list (A–K) of treatment options.
23.7.1 For each of the following patients with
Shlomo is a 9-day-old Jewish boy who was born haematological problems select the appropriate
treatment. Each option may be used once, more

Haematological disorders
in the UK. He had a religious circumcision
yesterday, but the wound will not stop bleeding. than once, or not at all.
On examination he is pale and has tachycardia. A. Blood transfusion
There is oozing of blood around the circumcision B. Chemotherapy
wound. A cannula is inserted and a blood C. Desmopressin
cross-match sent. There is now oozing around D. Folic acid supplementation
the cannula site. E. Intravenous antibiotics
Investigation reveals: F. Iron supplementation
• Hb (haemoglobin): 84 g/L G. No action required at present
• WBC (white blood cell count): 12 × 109/L H. Oral antibiotics
• platelet count: 322 × 109/L I. Recombinant factor VIII
• prothrombin time: 16 seconds (control: J. Vitamin D
12–15 s) K. Vitamin K
• activated partial thromboplastin time: >120 23.8.1
seconds (control: 25–35 s) Ahmed is a 4-week-old infant of Somali refugees
who have just fled to the UK. He was circumcised
23.7.2 yesterday, but the wound will not stop bleeding.
Charlie, aged 5 years, is troubled by recurrent On examination there is oozing of blood around
nose bleeds, the last of which took 1.5 hours to the circumcision wound.
stop. He has no other medical problems. His
examination is normal except for pale Investigation reveals:
conjunctivae. • Hb (haemoglobin): 122 g/L
• WBC (white blood cell count): 11 × 109/L
Investigation reveals: • platelet count: 312 × 109/L
• Hb (haemoglobin): 86 g/L • prothrombin time: 36 seconds (control:
• WBC (white blood cells): 10.2 × 109/L 12–15 s)
• platelet count: 350 × 109/L • activated partial thromboplastin time: 25
• prothrombin time: 16 seconds (control: seconds (control: 25–35 s)
12–15 s)
• activated partial thromboplastin time: 46 23.8.2
seconds (control: 25–35 s) Lola is an 8-month-old girl from Cyprus. She is
• fibrinogen: 2.5 g/L (normal: 2–4 g/L) referred to the paediatric department because
• factor VIII: just below the normal range she is clinically anaemic and has faltering
growth. On examination you find that she has a
23.7.3 large liver and spleen. Electrophoresis reveals an
Melissa is a 3-year-old girl. She presents to her absence of haemoglobin A (HbA).
general practitioner with a 3–4 week history of
lethargy and weight loss. On examination she is 23.8.3
pale and has widespread bruising. She has no George is a 3-month-old boy. He presents to the
other medical history and is not currently on any paediatric ward with a swollen leg. He had an
medications. immunization yesterday and there is now a large
swelling at the injection site. Haematological
The general practitioner orders a full blood count investigation reveals:
which reveals: • Hb (haemoglobin): 102 g/L
• Hb (haemoglobin): 66 g/L • WBC (white blood cell count): 9.0 × 109/L
• WBC (white blood cell): 43.2 × 109/L • platelet count: 312 × 109/L
• platelet count: 50 × 109/L • prothrombin time: 13 seconds (control:
12–15 s)
23.7.4 • activated partial thromboplastin time: 100
Xevera is a 7-year-old Greek boy who is seen by seconds (control: 25–35 s)
a paediatrician for constipation and was noted to
look pale. Haematological testing reveals that he There is currently no bleeding and he is
is anaemic with Hb of 100 g/L (both MCV and haemodynamically stable. His two older brothers
MCHC are low). He is given a course of iron both suffer from a bleeding disorder but both his 133
therapy but his anaemia does not improve. parents and his older sister do not.
23.8.4 reveals a normochromic normocytic anaemia
Lizzie is a 9-year-old girl who presents to the with no blast cells.
paediatric clinic. She is known to have hereditary
spherocytosis. Her mother is concerned that she 23.8.5

23 is very pale. Three weeks ago Lizzie had an upper


respiratory tract infection. Her mother reports
Angie is 6 weeks old. She was jaundiced at 24
hours of age, when her haemoglobin (Hb) was
that she had a fever and was very flushed with checked and was 150 g/L. At 2 months of age
bright red cheeks. She is otherwise well and has she presents with an upper respiratory tract
Haematological disorders

fully recovered from her infection. Her mother infection. The full blood count is repeated and
informs you that she has a very good diet. A full she has Hb 102 g/L.
blood count reveals a Hb 88 g/L. A blood film

134
B. Haemophilia A
Answers: Single Best Answer In haemophilia A, thrombocytopenia is not a
feature and the clotting profile would not be
23.1 normal.
A. Acute lymphoblastic leukaemia
Acute lymphoblastic leukaemia is unlikely as she C. Immune thrombocytopenic purpura
is well, has not had weight loss, and her Correct. Affected children develop petechiae,
splenomegaly is not accompanied by purpura, and/or superficial bruising. The platelet

Haematological disorders
hepatomegaly or lymphadenopathy. Her blood count can be very low (single digits) but
count only shows anaemia, with normal white treatment is not usually required and most cases
cell and platelet counts. resolve spontaneously.

B. Glucose-6-phosphate dehydrogenase (G6PD) D. Non-accidental injury


deficiency Non-accidental injury can present with unusual
G6PD deficiency may be associated with acute bruising but this boy has a low platelet count,
haemolysis but there would not usually be which explains the bruising.
splenomegaly. In girls, clinical abnormalities are
E. Vitamin D deficiency
uncommon.
In vitamin D deficiency, thrombocytopenia is not
C. Hereditary spherocytosis a feature.
Correct. In hereditary spherocytosis the anaemia
is usually mild (Hb 90–110 g/L), but the 23.4
haemoglobin level may transiently fall during A. Dietary advice
infections. Mild to moderate splenomegaly is Dietary advice is required but iron therapy is a
common. must as his anaemia is severe.

D. Sickle cell disease B. Folic acid


Sickle cell disease would not be likely at all in a An important supplement for women prior to
Caucasian child (see Fig. 23.12 in Illustrated conception as it reduces the risk of neural tube
Textbook of Paediatrics). defects in the fetus.

E. Thalassaemia C. Iron supplements


Thalassaemia is usually encountered in children Correct. Iron-deficiency anaemia is the most
from the Indian subcontinent, Mediterranean, likely diagnosis and iron therapy is indicated. The
and Middle East (see Fig. 23.12). presence of ‘pica’ strongly suggests iron
deficiency. He may well be a picky eater and
23.2 survives on cow’s milk and little else.
A. Acute lymphatic leukaemia In some children anaemia can be due to a
Acute lymphatic leukaemia is unlikely as he does β-thalassaemia trait. The blood parameters
not have abnormal clinical signs. should be repeated after a course of treatment
B. β-Thalassaemia trait and if there has not been a response he should
Thalassaemia trait patients rarely have symptoms be tested for β-thalassaemia trait.
but may be found to have anaemia. D. Multivitamin tablets
C. Glucose-6-phosphate dehydrogenase Vitamins are recommended for all preschool
deficiency children but with such a significant anaemia and
Correct. In G6PD deficiency, acute haemolysis iron deficiency, a treatment course of iron is
may be precipitated by infection, the most required first.
common precipitating factor. Certain drugs E. Vitamin B12 injections
including nitrofurantoin, fava beans (broad A macrocytosis is usually seen in vitamin B12
beans) and naphthalene in mothballs can also deficiency, which is rare in children.
precipitate haemolysis. The urine is dark as it
contains haemoglobin as well as urobilinogen. 23.5
A. β-Thalassaemia trait
D. Pyelonephritis
See Table 23.2 and Fig. 23.7 in Illustrated
Pyelonephritis is unlikely as the child is afebrile
Textbook of Paediatrics. In β-thalassaemia
and has no pain.
trait, the haemoglobin A (HbA) would be
E. Sickle cell disease approximately 90%. There would be extra
Sickle cell disease is mostly seen in black children fetal haemoglobin (HbF) and haemoglobin
and adults. A2 (HbA2).
23.3 B. Glucose-6-phosphate dehydrogenase
A. Acute lymphoblastic leukaemia deficiency
In acute lymphoblastic leukaemia you would Usually presents as either neonatal jaundice or 135
expect a range of abnormal clinical features. acute haemolysis. Here the mean corpuscular
volume and the mean corpuscular haemoglobin start to crawl or walk. However, it can present in
are low suggesting a more chronic problem. the neonatal period with intracranial
haemorrhage, bleeding after circumcision or
C. Iron-deficiency anaemia prolonged oozing after heel sticks or
Correct. A common diagnosis in children that
23 results in these findings. The most common
cause is dietary insufficiency (see Case History
venepuncture. The results of investigations in
children with Haemophilia and von Willebrand
disease are summarized in Table 23.3 in
23.1 in Illustrated Textbook of Paediatrics). Illustrated Textbook of Paediatrics.
Haematological disorders

D. Normal variation for age and ethnicity


Answer 23.7.2
At 6 weeks of age children are in the ‘physiological
R. von Willebrand disease
trough’ for haemoglobin concentration and the
On investigation, haemophilia A and von
lower limit of normal is 100 g/L. However, this is
Willebrand disease (vWD) will both have a
short lived and by 6 months, children maintain
deranged activated partial thromboplastin time.
haemoglobin concentrations close to adult
However, in haemophilia A it is much more
female levels. After puberty, boys have a higher
pronounced. The clinical features are also less
normal range. See Fig. 23.2 in Illustrated Textbook
pronounced in vWD, as in this child.
of Paediatrics for more details.
Spontaneous soft-tissue bleeding, such as large
E. Sickle cell trait haematomas, does not occur in vWD. A family
In sickle cell trait, there is inheritance of sickle history is common.
haemoglobin (HbS) from one parent, so
Answer 23.7.3
approximately 40% of the haemoglobin is HbS.
A. Acute lymphoblastic leukaemia
Whilst carriers are asymptomatic, this is easily
Lethargy, weight loss, bruising and anaemia,
detectable on haemoglobin electrophoresis.
thrombocytopenia and raised white cell count
23.6 suggest acute lymphoblastic leukaemia.
A. Acute lymphoblastic leukaemia
Although a small number of children present Answer 23.7.4
with bony pain, it would be unusual to have pain F. β-Thalassaemia trait
in the fingers. This boy most likely has a β-thalassaemia trait
(see Table 23.2 in Illustrated Textbook of
B. β-Thalassaemia major Paediatrics). A serum ferritin should have been
Would present with anaemia and if severe, this performed prior to starting the iron therapy. This
could lead to heart failure with peripheral would most likely have been normal and iron
oedema but pain would be unusual and there therapy could have been avoided.
would be other signs. See Fig. 23.13 in Illustrated
Textbook of Paediatrics for more details. Answer 23.8.1
K. Vitamin K
C. Glucose-6-phosphate dehydrogenase This child probably has haemorrhagic disease of
deficiency the newborn, due to vitamin K deficiency. In
After the neonatal period, this would present high-income countries prophylactic vitamin K is
with acute haemolysis precipitated by infection, offered after birth. Vitamin K is essential for the
drugs or fava beans. Haemolysis is production of active forms of factors II, VII, IX,
predominantly intravascular and is associated and X and for the production of naturally
with fever, malaise and dark urine. occurring anticoagulants such as proteins C and
S. Vitamin K deficiency therefore causes reduced
D. Haemophilia A
levels of all of these factors. A clotting profile
Haemophilia typically presents with bleeding
reveals a prolonged prothrombin time. He needs
into the larger joints.
vitamin K treatment.
E. Sickle cell disease
Answer 23.8.2
Correct. He is experiencing a vaso-occlusive crisis.
A. Blood transfusion
A common clinical feature in late infancy is the
This girl has thalassaemia major. She will need
hand–foot syndrome, in which there is dactylitis,
life-long blood transfusions due to her profound
with swelling and pain of the fingers and/or feet.
anaemia. She will need iron chelation therapy to
His condition would be identified on the routine
prevent the toxic effects of iron overload from
newborn screening blood test in the UK.
excessive transfusions.

Answers: Extended Matching Answer 23.8.3


I. Recombinant factor VIII
Answer 23.7.1 In haemophilia A and von Willebrand disease
H. Haemophilia A there is an abnormal activated partial
136 Haemophilia usually presents with spontaneous thromboplastin time and normal prothrombin
bleeding into joints or muscles when infants time. However, in haemophilia A the abnormality
is far more pronounced. The family history also transfusion is required. There is no absolute level
points towards haemophilia A, which is an below which transfusion is required but most
X-linked recessive disorder. His factor VIII is low children require blood when the haemoglobin is
and treatment is needed. less than 50 g/L.
Answer 23.8.4 Answer 23.8.5
G. No action required at present G. No action required at present
She probably had a parvovirus infection This is a normal variant. At birth, the

Haematological disorders
associated with bone marrow suppression. This haemoglobin in term infants is high,
level of anaemia does not need transfusion. If 140–215 g/L, to compensate for the low oxygen
she has a good diet, she will not need iron concentration in the fetus. The haemoglobin falls
supplementation. However, she will need a over the first few weeks, mainly due to reduced
further full blood count in a couple of weeks to red cell production, reaching a nadir of around
ensure that her haemoglobin has not dropped 100 g/L at 2 months of age (see Fig. 23.2 in
further and reached the level where a blood Illustrated Textbook of Paediatrics).

137
24

Child and adolescent mental health

reports that Florence is hyperactive and that


Questions: Single Best Answer she is ‘at the end of her tether’. Florence has
always been a very active girl since learning to
24.1 walk at 11 months of age. Although being
James is a 3½-year-old boy who wets the bed described as an ‘angel’ at school, she is very
about four of every seven nights. He does not difficult to manage at home. She is always on
wake after wetting. He has been dry during the the go. She has tantrums when not getting her
day for 1 year. There is no history of constipation. own way. She will often move from task to task
He is otherwise well and has a normal at home without completing one before going
examination. What would be the most on to the next. She has never been in any
appropriate initial management? accidents and is not impulsive. Florence is
Select one answer only. currently staying with her grandparents two
nights a week to give her mother a break. She
A. Desmopressin therapy has no other medical problems and is not on any
B. Enuresis alarm medication. There is no family history of similar
C. ‘Lifting’ at midnight problems.
D. Reassurance
E. Star chart Which of the following would be the first step in
management of this girl?
24.2
Zoe is a 14-year-old girl who has always been Select one answer only.
quiet and hard working. She is very thin and is A. Parenting classes
losing weight despite being very interested in B. Psychostimulant, e.g. methylphenidate
food; she enjoys baking cakes for her classmates C. Referral to child psychiatrist
and is always keen to go to the supermarket D. Referral to community paediatrician for
with her mother. Zoe herself denies that there specialist paediatric opinion
is a problem with her weight or that she has E. Sedative medication, e.g. sedating
been vomiting. Her parents are both extremely antihistamine
worried about her weight loss and poor appetite;
they have not noticed any vomiting. There are 24.4
no other symptoms and her examination is Georgio is a 6-year-old, normally healthy boy,
normal except that her height is on the 30th who attends the Paediatric Assessment Unit.
centile but her weight is below the 0.4th centile. Three weeks ago he had a diarrhoeal illness
What is the most likely diagnosis? lasting 3 days, which was not associated with
vomiting and resolved without any treatment.
Select one answer only. Now he is having episodes of cramping
A. Anorexia nervosa abdominal pains. He has not opened his
B. Bulimia bowels for a week. He reports that his stools
C. Chronic fatigue syndrome were normal prior to the diarrhoeal illness.
D. Coeliac disease He is his normal self between the episodes of
E. Depression abdominal pain. On examination he is a well
child, not currently in pain, with an indentable
24.3 mass in his left iliac fossa. His weight and height
Florence is a 7-year-old girl whose mother brings are plotted and are found to lie on the 25th
her to her general practitioner. Her mother centile.
What is the most likely diagnosis? C. Home tuition
D. Intensive exercise
Select one answer only. E. Neurostimulant medication
A. Appendicitis
B. Constipation
C. Intussusception
Questions: Extended Matching
D. Malrotation with volvulus 24.8
E. Recurrent abdominal pain of childhood

Child and adolescent mental health


Below is a list (A–H) of possible interventions.
For the scenarios described below, choose the
24.5 most appropriate treatment interventions for
Jeremy, aged 4 years, is brought to his general emotional and behavioural problems in children
practitioner because he wakes some nights at and adolescents. Each option may be used once,
about 10 pm with a shout. When his parents go more than once, or not at all.
to him he looks awake but confused. He goes
back to sleep soon afterwards and next morning A. Admit to paediatric ward
cannot recall any of these night-time events. He B. Cognitive behavioural therapy
is otherwise well and has no medical problems. C. Drug therapy
D. Explanation, advice and reassurance
What is the most likely explanation? E. Family therapy
Select one answer only. F. Individual or group dynamic psychotherapy
G. Parenting group
A. Complex partial seizures H. Referral to a child psychiatrist
B. Motor tics
C. Nightmares 24.8.1
D. Night terrors Steven is a 15-year-old boy who is brought to
E. Temper tantrums the Emergency Department. His mother is
distressed because Steven is complaining that
the television is putting thoughts into his head,
24.6
and he is becoming increasingly suspicious of
For the last 3 months, Fatima, aged 13 years and
their next-door neighbour, who he believes is
born in the UK to parents of Pakistani origin, has
spying on him. His mood is normal and he is fully
complained of difficulty in doing school work,
orientated. There is no suspicion of intoxication
increased tiredness on the slightest exercise,
or medical cause for his symptoms. He has no
pain in her joints and headaches with tenderness
other medical problems and is not on any
over the top of her head. This began shortly after
medications.
a febrile illness, associated with a cough and sore
throat, which lasted 3 days. She has not attended 24.8.2
school for the last 8 weeks. Examination is Olivia is a 2-year-old girl who comes to the
normal. You plot her height and weight, which general paediatric clinic with her exasperated
are on the 30th and 9th centile, respectively. mother. Olivia throws tantrums whenever her
There are no other medical problems and she is mother asks her to do something that Olivia is
not on any medications. reluctant to do. When you ask for an example,
Olivia’s mother describes breakfast time that
On the basis of this history alone which of the
morning, when Olivia screamed and threw her
following is the most likely diagnosis?
toast across the room when she was asked to stop
Select one answer only. flicking her yoghurt at her baby brother. Olivia
was born at term and has no medical history.
A. Chronic fatigue syndrome
B. Depression 24.8.3
C. Hypothyroidism Cynthia is a 15-year-old girl whose mother brings
D. Juvenile dermatomyositis her to her general practitioner because of
E. Tuberculosis headaches and abdominal pain. These episodes
of pain have been present for 6–9 months. There
24.7 are no symptoms or signs to suggest an organic
Jane is 14 years old and has a diagnosis of cause, so you ask some further questions. These
chronic fatigue syndrome. reveal that Cynthia is bored most of the time,
stays in her room and does not want to go out
What would be the most appropriate with her friends as she did previously. She used
management plan? to be a star student, but now is doing less well in
Select one answer only. school. She often misses school because of
feeling tired. She has no suicidal ideation and
A. Complete bed rest has never deliberately self-harmed. She is not on 139
B. Gradual rehabilitation programme any medication.
incorrectly labelled by family members or
Answers: Single Best Answer teachers as having attention deficit hyperactivity
disorder. In this case her difficult behaviour is not
24.1 pervasive, as it is not present at school.
24 A. Desmopressin therapy
This is likely to give short-term relief as B. Psychostimulant, e.g. methylphenidate
Although methylphenidate is a good treatment
desmopressin will stop the child making so
much urine. Whilst this temporarily ‘solves’ the choice for moderately severe attention deficit
Child and adolescent mental health

problem, it will not help him wake when his hyperactivity disorder (ADHD), medication is
bladder becomes full. It is, however, very helpful reserved for those in whom behavioural
for holidays or providing exhausted parents with interventions have been tried and failed. In
some short-term relief. He is though too young the true hyperkinetic disorder or ADHD, the
to start this therapy. child is undoubtedly overactive in most
situations (i.e. it is pervasive) and has impaired
B. Enuresis alarm concentration with a short attention span or
If an older child does not respond to a star chart, distractibility.
it may be supplemented with an enuresis alarm.
C. Referral to child psychiatrist
C. ‘Lifting’ at midnight The symptoms described are common.
This might prevent the child waking with a wet Secondary care services would be overwhelmed
bed but will not lead to any improvement in if the first step is to refer all these children.
wakening.
D. Referral to community paediatrician for
D. Reassurance specialist paediatric opinion
Correct. Nocturnal enuresis is common at his The symptoms described are common.
age. About 6% of children aged 5 years and 3% Secondary care services would be overwhelmed
of children aged 10 years have nocturnal if the first step is to refer all these children.
enuresis.
E. Sedative medication, e.g. sedating
E. Star chart antihistamine
Probably the most effective treatment for most This would be unhelpful and would be likely to
behavioural problems and most paediatricians make things worse rather than better.
would start using this around 5 years of age.
24.4
24.2 A. Appendicitis
A. Anorexia nervosa There would be pain on abdominal examination.
Correct. Zoe has several characteristic features – Classically, this would be in the right iliac fossa
female, adolescent, weight below the 0.4th although a pelvic appendix can be more difficult
centile, but has a distorted body image in that to detect.
she denies there is a problem with her weight.
She has perfectionist personality traits and is B. Constipation
overly interested in food without eating it herself. Correct. The mass in his left iliac fossa is most
likely to be impacted stool.
B. Bulimia
Her parents do not think that she has been C. Intussusception
vomiting and so bulimia is less likely. Intussusception causes intermittent abdominal
pain, but has a shorter history, is usually found in
C. Chronic fatigue syndrome younger children, and there would not be an
There is no report of fatigue. Moreover, the indentable mass in the abdomen.
weight loss and distorted body image are not
usually symptoms of chronic fatigue syndrome. D. Malrotation with volvulus
Malrotation with volvulus may cause bilious
D. Coeliac disease vomiting and would not be associated with an
Zoe denies that there is a problem with her indentable mass.
weight. Although coeliac disease may (rarely)
coexist, it does not fit well with the clinical history. E. Recurrent abdominal pain of childhood
This is a common and important diagnosis but
E. Depression the presence of an indentable mass coupled
Features of depression are not present: she is not with failure to open bowels for a week makes
socially withdrawn and has enjoyment in life (e.g. constipation more likely.
baking cakes).
24.5
24.3 A. Complex partial seizures
A. Parenting classes Complex partial seizures do not present in this
140 Correct. Parenting classes are often effective in way. If the child wakes many times per night
managing such problems. Many children are with episodes though, it is worth considering
rarer forms of seizure (nocturnal frontal lobe findings are an elevated creatinine kinase and a
seizures). heliotrope rash across the eyelids.
B. Motor tics E. Tuberculosis
Motor tics tend to be worse towards the end of No fevers and no family history of tuberculosis. It
the day (when the child is tired) or when would be highly unusual for this to present in
watching/playing violent video games. this way.
C. Nightmares 24.7

Child and adolescent mental health


Nightmares usually occur later in the night or A. Complete bed rest
early morning as they occur most commonly Complete bed rest can exacerbate the problem,
during REM stages of sleep. They can usually be leading to muscle atrophy and further fatigue.
recalled.
B. Gradual rehabilitation programme
D. Night terrors Correct. Usually the most successful approach in
Correct. Night terrors classically occur about chronic fatigue syndrome. It consists of a gradual
1–2 hours after falling asleep during the increase in exercise, school work and activities of
non-REM stage of sleep. The child is often daily living. Successful therapies include graded
confused, distressed and can appear terrified exercise tolerance programmes, pacing and
despite reassurances offered from parents. cognitive behavioural therapy.
Unlike nightmares, there is no recollection of
events the next day. C. Home tuition
Home tuition can be useful but will not hasten
E. Temper tantrums recovery.
These do not wake a child from sleep. A child
might have a tantrum upon being told to do D. Intensive exercise
something they do not want to. Intensive exercise can lead to relapse.

24.6 E. Neurostimulant medication


A. Chronic fatigue syndrome Neurostimulants have no place in management.
Correct. By definition, this is chronic fatigue
syndrome. There are persisting high levels of
subjective fatigue. Myalgia, migratory arthralgia Answers: Extended Matching
and headaches are almost universal.
Answer 24.8.1
B. Depression H. Referral to a child psychiatrist
She does not report low mood or a lack of Steven has clinical features suggestive of
motivation. schizophrenia, and needs a psychiatrist to assess
him and manage his treatment.
C. Hypothyroidism
Some of the features described fit well with Answer 24.8.2
hypothyroidism but it is not the most likely D. Explanation, advice, and reassurance
cause. However, thyroid function tests form part This is a normal developmental stage, and needs
of the investigations done in management of advice about behaviour management.
chronic fatigue syndrome.
Answer 24.8.3
D. Juvenile dermatomyositis B. Cognitive behavioural therapy
A rare diagnosis but one which can present with This may help her link feelings, thoughts and
tiredness and muscle aches. The characteristic behaviour.

141
25

Dermatological disorders

Questions: Single Best Answer


25.1
William is a previously well 8-year-old boy who
presents to the Emergency Department with a
fever and arthralgia for 2 days, which is getting
worse. On examination, he is noticed to have a
painful, raised rash on his shins (Fig. 25.1).

Figure 25.2 (Courtesy of Dr Rob Primhak)

What is the most likely diagnosis?


Select one answer only.
A. Kawasaki disease
B. Langerhans cell histiocytosis
Figure 25.1 (Courtesy of Prof Julian Verbov) C. Primary herpes simplex virus infection
D. Stevens–Johnson syndrome
E. Allergic keratoconjunctivitis
Which of the following is the most likely cause?
Select one answer only. 25.3
A 1-month-old male infant is brought to his
A. Football injuries family doctor. He has a rash confined to the
B. Henoch–Schönlein purpura perineum (Fig. 25.3). He has had diarrhoea for
C. Herpes simplex infection the last week.
D. Mycoplasma pneumoniae infection
E. Streptococcal infection What is the most likely cause for his rash?
Select one answer only.
25.2
This 8-year-old boy presents to the Emergency A. Atopic eczema
Department. He is on co-trimoxazole (Septrin) as B. Candida napkin rash
treatment for a urinary tract infection. He went C. Chicken pox
on to develop severe ulceration of his mouth D. Infantile seborrhoeic dermatitis
and corneal ulceration (Fig. 25.2). E. Irritant napkin rash
Which of the following is most likely to have
caused his eczema to flare-up?
Select one answer only.
A. Bacterial infection
B. Exposure to allergen
C. Increase in heat due to summer
D. Reduction in his emollients

Dermatological disorders
E. Viral infection

25.7
Robert is a 16-year-old boy with severe acne.
He has had it for the last 3 years, but it is
Figure 25.3 (Courtesy of Prof Julian Verbov) getting worse. He has tried some topical
benzoyl benzoate sporadically but the skin
lesions are now attracting adverse comments
from his school friends, and he would like
25.4 treatment. The lesions over his shoulder are
Many rashes in childhood are itchy and can shown (Fig. 25.4).
cause a lot of discomfort for the child.
Which of the following rashes is least likely to be
itchy?
Select one answer only.
A. Atopic eczema
B. Chicken pox
C. Infantile seborrhoeic dermatitis
D. Pityriasis rosea
E. Scabies

25.5
You see William, a 10-month-old baby boy who
has been diagnosed with atopic eczema. His
parents are struggling to control it with herbal
remedies. He is scratching himself all day and at Figure 25.4 (Courtesy of Prof Julian Verbov)
night, and he appears in discomfort.
Which of the following is the most useful advice?
Select one answer only. What management would you recommend?
A. Apply emollients once a day Select one answer only.
B. Bandages can only be used in children over
1 year of age A. Continue with current regimen
C. Regularly wash the baby with soap to avoid B. Oral retinoid isotretinoin
infection C. Topical antibiotics
D. Using nylon instead of cotton clothes D. Topical benzoyl peroxide alone
E. Use ointments instead of creams when the E. Topical benzoyl peroxide and oral
skin is dry tetracycline

25.6 25.8
Mikey is a 2-year-old boy who has eczema. He Joseph is a 4-year-old boy who is brought to the
comes to his family doctor with his mother, as general paediatric clinic with this lesion in his
his eczema has become troublesome over the hair (see Fig. 25.5 below).
last few weeks. His mother had been using What is the most likely diagnosis?
emollients twice a day, but for the last few
months during the summer she stopped as she Select one answer only.
did not think his eczema had been sufficiently A. Alopecia areata
bad. Today he has erythematous, weeping areas B. Cutis aplasia
of skin in the flexor surfaces of his knees and C. Kerion
ankles. There are also some areas of yellow D. Langerhans cell histiocytosis 143
crusting and he is pyrexial. E. Psoriasis
25
Dermatological disorders

Figure 25.5 (Courtesy of Prof Julian Verbov)


Figure 25.7 (Courtesy of Prof Julian Verbov)

25.9
Adam, a 2-day-old baby presents to the paediatric Which of the following is most likely to be the
department as his mother is concerned about the cause for the above rash?
appearance of his axilla, as shown in Fig. 25.6. A. Drug reaction
B. Inflammatory bowel disease
C. Mycoplasma pneumoniae infection
D. Tuberculosis
E. Varicella zoster

Questions: Extended Matching


25.11
For each of the following children select the
most likely diagnosis (A–M). Each option may be
used once, more than once, or not at all.
A. Acne vulgaris
B. Atopic eczema
C. Erythema multiforme
D. Guttate psoriasis
Figure 25.6 E. Henoch–Schönlein purpura
F. Immune thrombocytopenic purpura
G. Measles
H. Meningococcal disease
I. Molluscum contagiosum
What is the cause of this? J. Pityriasis rosea
Select one answer only. K. Ringworm
L. Scabies
A. Birth trauma
M. Systemic-onset juvenile idiopathic arthritis
B. Burn
C. Eczema 25.11.1
D. Erythema toxicum Ashleigh is a 5-year-old girl. She presents to the
E. Infection Emergency Department with a non-blanching
rash over her buttocks and legs (see Fig. 25.8
25.10 below). She is well in herself and has a heart rate
Stephan, a 5-year-old boy, presents with this rash of 100 beats/min, capillary refill of less than 2
(Fig. 25.7), which started 2 days ago. He had a seconds and is afebrile.
mild fever, headache and cough for the last 5
days but is not particularly unwell. He was given 25.11.2
144 amoxicillin by his general practitioner this Edward, a 5-year-old boy presents to his general
morning and has taken one dose. practitioner with a rash consisting of small
25.11.3
A mother brings Noah, her 3-year-old son, to the
general practitioner as she is concerned that her
son’s rash (Fig. 25.10) has been present for over
4 months.

Dermatological disorders
Figure 25.8

lesions which are erythematous and scaly on his


trunks and limbs (Fig. 25.9). He has recently been
diagnosed with a throat infection, but did not
receive antibiotics.
Figure 25.10 (Courtesy of Prof Julian Verbov)

25.11.4
Julie is a 10-month-old girl who has an intensely
itchy rash on her hands, feet and wrists
(Fig. 25.11).

Figure 25.11

145
Figure 25.9 (Courtesy of Prof Julian Verbov)
25.11.5
Damian is an 8-year-old boy. He has a persistent,
itchy rash (see Fig. 25.12).

25
Dermatological disorders

Figure 25.12 (Courtesy of Prof Julian Verbov)

146
25.3
Answers: Single Best Answer A. Atopic eczema
Atopic eczema is rarely present at this age, and
25.1 appears predominantly on the face and trunk of
A. Football injuries infants.
Would be bruised. Why would he have
a fever? B. Candida napkin rash
Correct. In this condition the flexures are
B. Henoch–Schönlein purpura

Dermatological disorders
involved and there are satellite lesions (isolated
Whilst joints are painful, the rash itself is not erythematous lesions away from the main
painful and is purpuric and has a characteristic rash area).
distribution.
C. Chicken pox
C. Herpes simplex infection Chicken pox is a vesicular rash.
Herpes simplex can cause erythema
multiforme. D. Infantile seborrhoeic dermatitis
Infantile seborrhoeic dermatitis is more discrete
D. Mycoplasma pneumoniae infection than this rash, which is widespread.
Mycoplasma pneumoniae can cause erythema
multiforme. E. Irritant napkin rash
In irritant napkin rash the flexures are spared.
E. Streptococcal infection
Correct. This boy has erythema nodosum and the 25.4
causes include streptococcal infection, primary A. Atopic eczema
tuberculosis, inflammatory bowel disease, drug A key feature of eczema is itch. This
reaction or idiopathic. causes the child to scratch, which exacerbates
the condition and predisposes to local
25.2 infection.
A. Kawasaki disease B. Chicken pox
Kawasaki disease does cause a sore mouth and This is very itchy, particularly in the healing phase.
bilateral non-purulent conjunctivitis. However, it Scratching can lead to permanent scarring and is
would not usually lead to ulceration. Fever often treated with calamine lotion.
would be present for more than 5 days.
C. Infantile seborrhoeic dermatitis
B. Langerhans cell histiocytosis Correct. The causes of itchy rashes are: atopic
Langerhans cell histiocytosis is a rare diagnosis. It eczema, chicken pox, urticaria, allergic reactions,
sometimes is mistaken for eczema (see Fig. 22.19 contact dermatitis, insect bites, scabies, fungal
in Illustrated Textbook of Paediatrics). infections and pityriasis rosea.
C. Primary herpes simplex virus infection All the conditions listed in the question are itchy
The rash shown does not have the vesicular other than seborrhoeic dermatitis.
pattern typically seen with herpes simplex virus D. Pityriasis rosea
infection (HSV), nor the characteristic clinical A typical feature is itch.
features.
E. Scabies
D. Stevens–Johnson syndrome If the itch is shared, then always consider
Correct. This boy has Stevens–Johnson scabies. Indeed, it is a truism in dermatology
syndrome, which is a severe bullous form of that any persistent itchy rash should be
erythema multiforme. Its relative frequency in considered as scabies until this has been
children treated with co-trimoxazole has led excluded.
to this antibiotic rarely being used. It is still
helpful for the treatment or prevention of 25.5
Pneumocystis jirovecii (carinii) in children with A. Apply emollients once a day
immunosuppression. The eye involvement may There are many different management
include conjunctivitis, corneal ulceration and options for eczema. Simple advice includes
uveitis, and ophthalmological assessment is using emollients frequently – at least twice
required. It may also be caused by sensitivity to a day, but do not use sparingly. A typical
other drugs or infection, with morbidity and infant will use 250–500 ml of emollient every
sometimes even mortality from infection, fortnight.
toxaemia or renal damage.
B. Bandages can only be used in children over 1
E. Allergic keratoconjunctivitis year of age
Severe allergy can be very unpleasant but would Occlusive bandages are extremely helpful at
not cause this severity of conjunctivitis or lip/ any age, especially when excoriation and 147
mouth involvement. lichenification are a problem.
C. Regularly wash the baby with soap to avoid E. Topical benzoyl peroxide and oral
infection tetracycline
There are many different management Correct. Robert has severe acne with marked
options for eczema. Simple advice includes cystic and nodular lesions. Benzoyl peroxide will
25 avoiding soap.
D. Using nylon instead of cotton clothes
need to be combined with oral antibiotics.

25.8
Avoiding nylon and woollen clothes is useful. A. Alopecia areata
Dermatological disorders

E. Use ointments instead of creams when the Usually flat, colourless, and painless. Not swollen
skin is dry and red.
Correct. Ointments are preferable to creams B. Cutis aplasia
when the skin is dry. A scalp defect that presents at birth.
C. Kerion
25.6 Correct. Kerions are caused by infection with
A. Bacterial infection tinea capitis (scalp ringworm). Topical treatment
Correct. Causes of flare-up of eczema are: is usually tried first but oral antifungals may be
• bacterial infection, e.g. Staphylococcus, required for very severe cases.
Streptococcus spp.
• viral infection, e.g. herpes simplex virus D. Langerhans cell histiocytosis
• ingestion of an allergen, e.g. egg Causes a seborrhoeic rash in infants. It is rare.
• contact with an irritant or allergen E. Psoriasis
• environment: heat, humidity The rash shown is localized and not scaly as in
• change or reduction in medication psoriasis.
• psychological stress.
The yellow crusting and pyrexia are suggestive 25.9
of infection with Staphylococcus, therefore a A. Birth trauma
bacterial cause for the exacerbation. There would be bruising and this site would be
B. Exposure to allergen highly unusual with birth trauma.
But why would he be pyrexial? B. Burn
C. Increase in heat due to summer A burn in that distribution and no history is very
Would this cause the crusting? unlikely.

D. Reduction in his emollients C. Eczema


A cause but why would there be weeping? A Eczema does not cause blistering and the infant
weeping skin should prompt one to start is too young.
antibiotics. D. Erythema toxicum
E. Viral infection Erythema toxicum is a benign pustular rash that
If it was vesicular, then always consider herpes migrates and requires no treatment.
simplex. If he is ill enough to warrant hospital E. Infection
admission, then antivirals are often given in Correct. Staphylococcal infection causing
conjunction with antibiotics whilst awaiting blistering. There are several variations of this
improvement. from bullous impetigo to staphylococcal scalded
skin syndrome, as shown here.
25.7
A. Continue with current regimen 25.10
It is not working. A. Drug reaction
This is a possible cause, but here the rash
B. Oral retinoid isotretinoin preceded the first dose of antibiotic.
Oral retinoid isotretinoin is reserved for
severe acne that is unresponsive to other B. Inflammatory bowel disease
treatments. Inflammatory bowel disease is associated with
erythema nodosum.
C. Topical antibiotics
Topical antibiotics and sunshine can be helpful C. Mycoplasma pneumonia infection
but probably only in milder disease and this is Correct. The rash is erythema multiforme.
severe. Although amoxicillin can cause this rash, it was
given after the rash appeared. Other causes of
D. Topical benzoyl peroxide alone erythema multiforme include herpes simplex
148 Benzoyl peroxide on its own will probably not be virus or idiopathic. His symptoms are consistent
adequate. with Mycoplasma pneumoniae infection.
D. Tuberculosis Answer 25.11.3
A cause of erythema multiforme but unlikely I. Molluscum contagiosum
with this history. This is caused by a poxvirus. The lesions are
small, skin-coloured, pearly papules with central
E. Varicella zoster umbilication. They may be single but are usually
Varicella zoster virus causes a vesicular rash. multiple. Lesions are often widespread but
tend to disappear spontaneously within
a year.
Answers: Extended Matching

Dermatological disorders
Answer 25.11.4
Answer 25.11.1 L. Scabies
E. Henoch–Schönlein purpura Always consider scabies if a rash is very itchy.
The rash here is typical of Henoch–Schönlein It is a highly contagious (and unpleasant)
purpura. A classic triad of colicky abdominal pain, condition.
arthralgia and purpuric rash often develop, but in
the early stages the rash usually predominates. Answer 25.11.5
B. Atopic eczema
Answer 25.11.2 Atopic eczema affects 20% of children.
D. Guttate psoriasis Lichenification and erythema are shown. The
The presentation here is typical and often a flare skin looks sore and the excoriations suggest itch.
of guttate psoriasis follows an infection. Scaly The face often remains affected from undue
rashes should always raise the possibility of reluctance to use topical steroids on the face of a
psoriasis. child of this age.

149
26

Diabetes and endocrinology

has remained unchanged for the last 7 months.


Questions: Single Best Answer Her HbA1C has increased from 58 mmol/mol
(7.5%) to 90 mmol/mol (10.3%) (desired level
26.1 <58 mmol/mol). The most recent blood glucose
Ellie, a 7-year-old girl, is newly diagnosed with levels as recorded in her book are shown in
diabetes mellitus. She has been drinking lots of Fig. 26.1 (overleaf):
fluid and has had to pass urine frequently. She
has a markedly raised blood glucose (20 mmol/L) What is the most likely explanation for these
and heavy glycosuria. findings?

Which of the following is most likely to be true Select one answer only.
about her diabetes?
A. During the summer holidays she took less
Select one answer only. exercise
B. She has reduced her insulin dosage to try to
A. Her diagnosis should be confirmed with an lose weight
oral glucose tolerance test C. She is taking more insulin than she needs
B. She can be managed with oral D. She is regularly eating snacks and indulging
hypoglycaemic agents and dietary in high carbohydrate food
modification E. Some of the blood glucose measurements
C. She will have gained weight in the last few are fictitious
weeks
D. The incidence in the UK is falling
E. There is autoimmune pancreatic β-cell 26.4
damage Mohammed, a 12-year-old boy with type 1
diabetes mellitus, is reviewed in the outpatient
26.2 clinic. In spite of maintaining good control of his
James, aged 11 years, has type 1 diabetes mellitus. diabetes, his height has remained static for 9
While playing football during the mid-morning months. He says his appetite is alright, but he
break at a holiday camp, he suddenly feels faint. has lost interest in football, which is his passion,
His classmates call the supervisor who finds him as he says he can’t keep up with the other boys
lying unresponsive in the playground. any more. He just stays at home and watches
TV, but wants to be out playing football and
What should be his immediate management?
getting back his energy. A full blood count and
Select one answer only. C-reactive protein are normal and his HbA1C is
satisfactory.
A. Call an ambulance
B. Check blood glucose What is the most likely diagnosis?
C. Give a glucose drink
Select one answer only.
D. Give buccal glucose gel
E. Give insulin A. Anorexia nervosa
B. Depression
26.3 C. Growth hormone deficiency
Catherine, aged 15 years, has had type 1 D. Hypothyroidism
diabetes mellitus for 7 years. Her insulin regimen E. Inflammatory bowel disease
Insulin injection Blood
Please refer to your Doctor or Diabetes
Time

2hr after mid-day meal

2hr after evening meal


Nurse specialist as to when you should

Before mid-day meal

Before evening meal


2hr after breakfast
perform your tests.

Before breakfast
Dosage

During night
Before bed
Date Comments

Figure 26.1
Diabetes and endocrinology

26.5 vomited all her feeds today. She has become


Zeinab, aged 15 years, has had 3 months of lethargic and will no longer feed. On
diarrhoea, weight loss and palpitations. You examination she has tachycardia, with a heart
suspect hyperthyroidism. rate of 160 beats/min and a capillary refill
time of 3 seconds. A bedside blood test shows
Which combination of thyroid function test that she is hypoglycaemic.
results would confirm the diagnosis?
What is the best immediate management?
Select one answer only.
Select one answer only.
A. High TSH and high T4 levels
B. High TSH and low T4 levels A. Intravenous glucose alone for 24 hours
C. Low TSH and high T4 levels B. Intravenous saline, glucose and
D. Low TSH and low T4 levels hydrocortisone
E. Normal TSH and high T4 levels C. Oral rehydration
D. Oral rehydration and oral glucocorticoid
26.6 E. Oral rehydration, sodium supplements and
Lara, an 18-day-old female infant, has congenital oral glucocorticoid 151
adrenal hypoplasia. She presents having
26.7 26.9
Amy, aged 8 years, is rapidly putting on weight. George, a 2-week-old male infant, presents to
A photograph of her is shown in Fig. 26.2. She the Emergency Department with vomiting,
has truncal obesity and striae on her abdomen. diarrhoea and poor feeding.
26 Investigations show:
• sodium 112 mmol/L (normal range 133–145
mmol/L)
Diabetes and endocrinology

• potassium 6.8 mmol/L (normal range 3.5–6.0


mmol/L)
• urea 7.8 mmol/L (normal range 2.5–8.0
mmol/L)
• creatinine 30 mmol/L (normal range 20–65
mmol/L)
• blood glucose 1.7 mmol/L (normal range >2.6
mmol/L)
• infection screen—negative
What is the most likely cause?
Select one answer only.
A. Acute kidney injury
B. Congenital adrenal hyperplasia
C. Congenital adrenal hypoplasia
D. Cushing syndrome
E. Gastroenteritis
Figure 26.2 (Courtesy of Dr Jerry Wales)
26.10
What is the most likely cause for this? A newborn baby has recently been delivered.
Select one answer only. The midwife requests an urgent paediatric
review of the baby because she cannot tell if the
A. Corticosteroid cream for severe eczema baby is male or female (Fig. 26.3). The parents
B. Daily oral prednisolone therapy for the last 6 are asking what sex their baby is.
months
C. Ectopic ACTH (adrenocorticotropic
hormone)-producing tumour
D. High-dose corticosteroid therapy for lung
disease when she was a premature baby
E. Long-term use of inhaled corticosteroids

26.8
Luke, a 4-month-old male infant, presents with
a history of vomiting and diarrhoea. On
examination he has a heart rate of 160 beats/
min; his capillary refill time is 2–3 seconds and he
is lethargic. A bedside blood test showed that he
is hypoglycaemic. A diagnosis of Addison disease
is considered.
Figure 26.3
Which of the following combination of
biochemical results would be most likely with
Addison’s disease?
What should you tell them?
Select one answer only.
Select one answer only.
A. Hypernatraemia and hyperkalaemia with low
cortisol A. You are unable to tell if the baby is male or
B. Hypernatraemia and hypokalaemia with female, and tell the parents it is likely to be a
high cortisol mixture of both sexes, i.e. ovotesticular
C. Hyponatraemia and hyperkalaemia with disorder of sex development (DSD – or
high cortisol hermaphroditism)
D. Hyponatraemia and hypokalaemia with low B. You are unable to tell right now and a
152 cortisol detailed assessment of the baby including
E. Hyponatraemia and hyperkalaemia with low scans and blood tests will be needed before
cortisol a specialist can tell them
C. You are unable to tell right now but will be 26.12.1
able to assign a sex as soon as you get the Julie, aged 7 years, has diabetes mellitus. She is
baby’s chromosomes back admitted to hospital as she has vomited on three
D. You think it is likely to be a girl so tell them occasions. She has a 2-day history of being
the baby should be named as a female on unwell with a mild fever, sore throat and
the birth certificate pending the results decreased appetite. Her blood glucose
E. You think it is likely to be a boy so tell them measurement reads ‘high’. Although she was not
the child should be named as a male on the eating, her parents maintained her usual insulin

Diabetes and endocrinology


birth certificate pending the results dose. On examination her temperature is 37.5° C.
She is drowsy and confused. Her pulse is 150
26.11 beats/min, blood pressure 80/45 mmHg (low for
A baby is born with a disorder of sexual age) and capillary refill time 3 seconds.
differentiation. Congenital adrenal hyperplasia is Examination of her throat shows tonsillitis. Her
suspected. blood glucose is 22 mmol/L.
What blood result would confirm the diagnosis? 26.12.2
Jon is 12 years old. His brother has diabetes. He
Select one answer only. has started to drink a lot of fluids and pass a lot
A. A low testosterone of urine. He checked his blood glucose on his
B. A markedly lowered plasma 17 brother’s glucometer. It was 19 mmol/L. When he
α–hydroxyprogesterone arrives in the Emergency Department he is well
C. A markedly raised cortisol level with no signs of dehydration. Diabetes mellitus
D. A markedly raised plasma type 1 is diagnosed as his blood glucose is 21
17α-hydroxyprogesterone mmol/L. A venous blood sample shows a normal
E. A raised blood glucose pH with 2 mmol/L of ketones (within the normal
range) and a HbA1c blood test result is awaited.
26.12.3
Questions: Extended Matching Harriet, a 5-year-old girl, is known to have
diabetes mellitus type 1. She was only diagnosed
26.12 2 months ago and is on a basal bolus regime of
For each of the following scenarios chose the insulin. She is running around the garden at
best course of action (A–K) to take immediately. home when her mother notices she suddenly
becomes aggressive towards her brother and
A. Buccal glucose gel looks pale and not her usual self. Her mother
B. Diet therapy alone checks her blood glucose, which is 3 mmol/L.
C. Fluid resuscitation with normal saline (0.9%
sodium chloride) 26.12.4
D. Insulin infusion intravenously Sophie, a 9-year-old girl with type 1 diabetes
E. Intramuscular (IM) glucagon mellitus, develops a fever along with vomiting
F. Intravenous antibiotics and diarrhoea. After 2 days her mother takes her
G. Intravenous infusion of 5% glucose to the local paediatric assessment unit as she
H. Intravenous infusion of normal saline (0.9% continues to vomit. On examination her
sodium chloride) temperature is 37.5° C. She is able to talk to her
I. Intravenous sodium bicarbonate mother. She has clinical dehydration. Her pulses,
J. Oral glucose drink capillary refill time and blood pressure are
K. Subcutaneous insulin normal. Her blood glucose is 16 mmol/L.

153
B. She has reduced her insulin dosage to try to
Answers: Single Best Answer lose weight
Reducing the insulin dose would result in high
26.1 blood glucose levels and not the normal values
26 A. Her diagnosis should be confirmed with an
oral glucose tolerance test
shown in the diary.
C. She is taking more insulin than she needs
An oral glucose tolerance test is not required as
her clinical presentation and investigations have More insulin than she needs would result in low
Diabetes and endocrinology

established the diagnosis. The test is seldom blood glucose levels and the HbA1C would not
required to diagnose diabetes in children, have increased.
though it may be used to screen for cystic D. She is regularly eating snacks and indulging
fibrosis related diabetes mellitus. in high carbohydrate food
B. She can be managed with oral Regularly eating snacks and indulging in high
hypoglycaemic agents and dietary modification carbohydrate food would result in high blood
She requires insulin. glucose levels and not the normal values shown
in the diary.
C. She will have gained weight in the last few
weeks E. Some of the blood glucose measurements
Weight loss is a feature at presentation. are fictitious
Correct. Some of the blood glucose
D. The incidence in the UK is falling measurements are fictitious. Most of the values
She has type 1 diabetes mellitus. In the UK its recorded are normal but her diabetic control has
incidence is increasing and now affects 2 per deteriorated and is no longer in the desired
1000 children under 16 years of age. range.
E. There is autoimmune pancreatic β-cell 26.4
damage Anorexia nervosa
Correct. Almost all (>95%) children in the UK Eating disorders can co-exist with diabetes
have type 1 diabetes mellitus which occurs as a mellitus but he does not have the clinical
result of autoimmune destruction of pancreatic features of anorexia nervosa.
β-cells by T cells.
B. Depression
26.2 Depression is also important to consider, but in
A. Call an ambulance this case, it is not the most likely cause as he is
This will be required if he does not respond to continuing to maintain good control of his
initial treatment. The holiday camp should be diabetes and is not expressing negative feelings.
prepared for treatment of hypoglycaemia in a C. Growth hormone deficiency
known diabetic. Not an autoimmune problem. Would not explain
B. Check blood glucose his lethargy.
He has the clinical features of hypoglycaemia D. Hypothyroidism
and immediate treatment is indicated. Correct. Children with type 1 diabetes mellitus
are at increased risk of thyroid disease and
C. Give a glucose drink
should be screened annually. His static height
This is appropriate if he can take fluids, but he is
and his lethargy would support this diagnosis.
unresponsive.
He is also at increased risk of coeliac disease
D. Give buccal glucose gel and this should also be considered but is less
Correct. He is hypoglycaemic, brought on by likely as his appetite is good and his weight is
exercise. Glucose is absorbed quickly from satsfactory.
buccal glucose gel, and he should rapidly
E. Inflammatory bowel disease
wake up. He will need to have a carbohydrate-
Blood and mucus in the stool would be expected
containing drink and snack to maintain his
with ulcerative colitis and anorexia and malaise
blood glucose.
and abnormal full blood count and C-reactive
E. Give insulin protein would be expected with Crohn’s disease.
His blood glucose needs to be raised rather than
lowered. 26.5
A. High TSH and high T4 levels
Central cause for hyperthyroidism would be very
26.3
rare.
A. During the summer holidays she took less
exercise B. High TSH and low T4 levels
154 She would be expected to have had more Suggests that thyroid gland is not working.
exercise over the summer holidays. Would result in hypothyroidism.
C. Low TSH and high T4 levels E. Long-term use of inhaled corticosteroids
Correct. Hyperthyroidism usually results from She has Cushing syndrome. Inhaled steroids
autoimmune thyroiditis (Graves disease) secondary are unlikely to cause this because the doses
to the production of thyroid-stimulating absorbed systemically are low.
immunoglobulins (TSIs). The levels of thyroxine (T4)
26.8
and/or tri-iodothyronine (T3) are elevated and TSH
A. Hypernatraemia and hyperkalaemia with low
levels are suppressed to very low levels.
cortisol

Diabetes and endocrinology


D. Low TSH and Low T4 levels The laboratory findings in Addison disease are
Central hypothyroidism. low sodium.

E. Normal TSH and high T4 levels B. Hypernatraemia and hypokalaemia with high
Very early change. Would soon be followed by a cortisol
fall in TSH. Wrong way around.

26.6 C. Hyponatraemia and hyperkalaemia with high


A. Intravenous glucose alone for 24 hours cortisol
No. The child is having a salt-losing crisis. The cortisol level would be low.
D. Hyponatraemia and hypokalaemia with low
B. Intravenous saline, glucose and
cortisol
hydrocortisone
The potassium is typically high.
Correct. Lara is having an adrenal crisis. This
requires urgent treatment with intravenous E. Hyponatraemia and hyperkalaemia with
saline, glucose and hydrocortisone. Long-term low cortisol
treatment is with glucocorticoid and Correct. In Addison disease, there is
mineralocorticoid replacement and oral sodium hyponatraemia, hyperkalaemia and
chloride may be needed during infancy. The hypoglycaemia. The plasma cortisol is low and
dose of glucocorticoid needs to be increased at the plasma ACTH (adrenocorticotropic hormone)
times of illness. concentration is high. See Box 26.10 in Illustrated
Textbook of Paediatrcs for more details.
C. Oral rehydration
Always helpful but in this case not sufficient 26.9
treatment. A. Acute kidney injury
Hyponatraemia may be a feature of acute kidney
D. Oral rehydration and oral glucocorticoid
injury, but this child has a normal plasma urea
This child is seriously ill. There isn’t time to lose.
and creatinine.
E. Oral rehydration, sodium supplements and B. Congenital adrenal hyperplasia
oral glucocorticoid Correct. The combination of marked
All the correct things are being replaced here hyponatraemia, hyperkalaemia and hypoglycaemia
but this is an emergency and it requires more are suggestive of glucocorticoid deficiency, as seen
rapid correction. in congenital adrenal hyperplasia.
26.7 C. Congenital adrenal hypoplasia
A. Corticosteroid cream for severe eczema Much rarer than congenital adrenal hyperplasia
Topical steroids do not cause this because the but also can result in hyponatremia,
doses absorbed systemically are low. hyperkalaemia and hypoglycaemia. There are
X-linked (early onset) and autosomal recessive
B. Daily oral prednisolone therapy for the
(variable onset) forms.
last 6 months
Correct. Children requiring high dose oral D. Cushing syndrome
corticosteroid therapy, e.g. for Crohn’s disease or An excess of glucocorticoids, as seen in Cushing
nephrotic syndrome, may develop the features syndrome, causes an intolerance to
of Cushing syndrome. Minimized by decreasing carbohydrates (hyperglycaemia), rather than the
the dose as soon as possible. hypoglycaemia seen in this child.
C. Ectopic ACTH (adrenocorticotropic hormone)- E. Gastroenteritis
producing tumour Gastroenteritis can lead to hyponatraemic
Extremely rare in children. However, must be dehydration, but the urea would be raised and
considered if there is no history of steroid use. the potassium normal.
D. High-dose corticosteroid therapy for lung 26.10
disease when she was a premature baby A. You are unable to tell if the baby is male or
A short course of steroids in the premature female, and tell the parents it is likely to be a
period, even if high-dose, would not cause mixture of both sexes, i.e. ovotesticular disorder 155
features of Cushing syndrome at this age. of sex development (DSD – or hermaphroditism)
This is extremely rare. It is usually possible to D. A markedly raised plasma
assign the infant a sex after assessment of 17α-hydroxyprogesterone
phenotype, sex hormones and ultrasound scans Most children with congenital adrenal
of internal structures and gonads are performed. hyperplasia have 21-hydroxylase deficiency (see
26 B. You are unable to tell right now and a
detailed assessment of the baby including
Fig. 26.11 in Illustrated Textbook of Paediatrics).
The diagnosis is made by finding markedly raised
levels of the metabolic precursor 17
scans and blood tests will be needed before a α-hydroxy-progesterone.
Diabetes and endocrinology

specialist can tell them


Correct. A detailed assessment by medical, E. A raised blood glucose
surgical and psychological specialists needs to It would typically be low.
be performed followed by a full discussion with
the parents before the infant is assigned a sex.
Answers: Extended Matching
C. You are unable to tell right now but will be
able to assign a sex as soon as you get the Answer 26.12.1
baby’s chromosomes back C. Fluid resuscitation with normal saline
Sexuality is complex, and depends on more than (0.9% sodium chloride)
the baby’s phenotype, chromosomes and Julie requires immediate fluid resuscitation
hormone levels. as she has diabetic ketoacidosis and is in
D. You think it is likely to be a girl so tell them shock. See Fig. 26.7a, Illustrated Textbook of
the baby should be named as a female on the Paediatrics.
birth certificate pending the results Answer 26.12.2
If there is abnormal sexual differentiation at K. Subcutaneous insulin
birth, do not guess the infant’s gender. A Start insulin subcutaneously. Although this is
detailed assessment needs to be performed. a new diagnosis of diabetes of mellitus, Jon is
E. You think it is likely to be a boy so tell them well so it is safe to start subcutaneous insulin.
the child must be named as a male on the birth He also requires education about diabetes,
certificate pending the results changes to diet and lifestyle, training on how
If there is abnormal sexual differentiation at to administer the insulin, and what to do in an
birth, do not guess the infant’s gender. A emergency.
detailed assessment needs to be performed. Answer 26.12.3
26.11 J. Oral glucose drink
A. A low testosterone Harriet is hypoglycaemic and requires some
In congenital adrenal hyperplasia the sugar to increase her blood glucose. The
testosterone would be raised. easiest way to achieve this is with a non-diet
sugary drink. She will require complex
B. A markedly lowered plasma 17 carbohydrate snacks to maintain her blood
α–hydroxyprogesterone glucose.
In most cases, the 17 α–hydroxyprogesterone is
markedly raised. You might see this in the much Answer 26.12.4
less common congenital adrenal hypoplasia H. Intravenous infusion of normal saline
though. (0.9% sodium chloride)
Intravenous 0.9% sodium chloride solution is
C. A markedly raised cortisol level required, as she continues to vomit and has
In congenital adrenal hyperplasia the cortisol clinical dehydration. She does not require fluid
level would be low. resuscitation as she is not shocked.

156
27

Inborn errors of metabolism

What is the most likely diagnosis?


Questions: Single Best Answer
Select one answer only.
27.1
A. Congenital adrenal hyperplasia
A newborn baby is diagnosed with a
B. Familial hypercholesterolaemia
metabolic condition at 10 days of age, after
C. Glycogen storage disorder
she presented with vomiting, jaundice,
D. Phenylketonuria
hepatomegaly and liver failure. She was
E. Urea cycle defect
subsequently put on a lactose-free and
galactose-free diet.
27.3
Which enzyme is most likely to be deficient? Terry was born at term and was well at birth.
On day 2 of life, he became severely unwell
Select one answer only. with poor feeding, vomiting, acidosis and
A. Galactokinase encephalopathy. Sepsis and hypoglycaemia were
B. Galactose-1-phosphate uridyl excluded, and an inborn error of metabolism is
transferase thought to be the cause as the blood ammonia
C. Glucose-6-phosphatase level is extremely high. Which of the following is
D. Glucose-6- phosphate dehydrogenase the most likely cause?
E. Phosphoglucomutase Select one answer only.
A. Fatty acid oxidation defect
27.2 B. Glycogen storage disease
Samina is 12-years-old and has hepatomegaly. C. MCAD (Medium-chain acyl-CoA
She is short for her age, suffers from dehydrogenase deficiency)
hypoglycaemia and needs an overnight feed via D. Mucopolysaccharidosis
a nasogastric tube. E. Urea cycle defect
hypoglycaemia as a prominent feature and the
Answers: Single Best Answer blood ammonia level may be raised but not
to extremely high levels. May also present as
27.1 an acute life-threatening episode (ALTE) or
27 B. Galactose-1-phosphate uridyl
transferase
near-miss sudden infant death syndrome (SIDS).
B. Glycogen storage disease
Correct. The baby has been put on a diet that
excludes galactose. The enzyme defect is Enzyme defects prevent glycogen synthesis or
Inborn errors of metabolism

therefore likely to be one involving galactose. breakdown in liver and/or muscle. Liver forms
This infant has galactosaemia, which is a rare present with severe hypoglycaemia and
disorder resulting from deficiency of the enzyme hepatomegaly; muscle forms with exercise
galactose-1-phosphate uridyl transferase, which intolerance.
is essential for galactose metabolism. When
C. MCAD (Medium-chain acyl-CoA
lactose-containing milk feeds such as breast
dehydrogenase deficiency)
milk or infant formula are introduced, affected
Detected on neonatal biochemical screening, or
infants feed poorly, vomit, develop jaundice,
presents with acute encephalopathy and
hepatomegaly and hepatic failure. Management
hypoglycaemia on fasting, or as an acute
is with a lactose-free and galactose-free diet
life-threatening episode (ALTE) or near-miss
for life.
sudden infant death syndrome (SIDS). Acute
27.2 illness may sometimes develop before screening
C. Glycogen storage disorder results are known.
Correct. She has clinical features of the hepatic D. Mucopolysaccharidosis
form of glycogen storage disorder, where After a period of normal development
enzyme defects prevent the mobilization of development regresses, facies become coarse
glucose from glycogen and gluconeogenesis, and organomegaly develops. Skeletal
resulting in abnormal storage of glycogen abnormalities are relatively common, particularly
in liver. of the ribs and vertebrae.
27.3 E. Urea cycle defect
A. Fatty acid oxidation defect Correct. Urea cycle or organic acid disorders
These disorders, e.g. carnitine transporter may present with these clinical features. The
deficiency, may present in an infant or older marked hyperammonaemia is characteristic of
child with an illness similar to this but with urea cycle defects.

158
28

Musculoskeletal disorders

Questions: Single Best Answer


28.1
Amir is a 5-year-old boy who was born and lives
in the UK. His parents are from Pakistan. He
presents to the Emergency Department with a
high temperature and complains of pain in his
right upper arm. This has been present for 2 days
and seems to be getting worse. The pain is
described as being sharp and gets worse when
he moves his arm. He had an abscess on his right
finger 2 weeks ago for which he was prescribed Figure 28.1
oral antibiotics. On examination he has a
temperature of 39° C. There is an area of redness
and swelling over the right upper arm which is C. Perthes disease
painful when touched or moved. The white cell D. Septic arthritis
count and C-reactive protein level are raised. E. Slipped capital femoral epiphysis
What is the most likely causative organism of his
infection? 28.3
Zain is a 14-year-old boy who presents to the
Select one answer only. Emergency Department. He is a good footballer,
A. Haemophilus influenzae and after training today he complained of pain
B. Mycobacterium tuberculosis in his left knee. He has had the pain for a while
C. Salmonella typhi and it gets worse when he exercises. On
D. Staphylococcus aureus examination there is swelling over the left tibial
E. Streptococcus viridans tuberosity. He is afebrile. There is no night pain
or pain on waking.
28.2 What is the most likely diagnosis?
Aisha, a 14-year-old girl, has developed a
right-sided limp that has been present for 2 Select one answer only.
weeks. She is now complaining of pain in her A. Osteomyelitis
right hip and knee. On examination, she has a B. Osgood–Schlatter disease
temperature of 37.7° C. She is overweight (95th C. Perthes disease
centile), with a body mass index of 26 kg/m2. She D. Septic arthritis
has a decreased range of movement of the right E. Slipped capital femoral epiphysis
hip. Blood tests, including a full-blood count and
a C-reactive protein are taken and results are 28.4
found to be normal. Edward, aged 3 years, woke up complaining of
Her hip X-ray is shown in Fig. 28.1 opposite. pain in his right leg. Yesterday he refused to play
in the garden with his older brother, preferring
What is the most likely diagnosis? to watch television. Since then he has become
Select one answer only. more unwell and his mother reports he is
lethargic. On examination he has a temperature
A. Osgood–Schlatter disease of 39° C, a heart rate of 180 beats/min, and a
B. Osteomyelitis respiratory rate of 25 breaths/min. His right knee
is red and swollen and he cries if it is extended capillary refill time. There is some generalized
or flexed. On further examination you note a swelling around his knees and ankles. He has pain
small healing insect bite on his right ankle. He on passive movement of both knees and ankles.
has no other medical problems and is not
28 currently on any medication.
What is the most likely diagnosis?
Select one answer only.
Musculoskeletal disorders

A. Henoch–Schönlein purpura arthritis


B. Juvenile idiopathic arthritis
C. Osteomyelitis
D. Septic arthritis
E. Trauma

28.5
Karen, aged 5 years, presents to her general
practitioner. She has been unwell for 2 weeks
with lethargy, fever and painful wrists and knees.
On examination she has a temperature of 38.5° C
and a subtle erythematous rash on her trunk. Figure 28.2
You identify that several joints are swollen,
warm, and painful to move, including the wrists, What is the most likely diagnosis?
her right elbow, the knees, her left ankle and left
hip. She has some cervical lymphadenopathy Select one answer only.
and her spleen is palpable. She has no previous A. Henoch–Schönlein purpura
medical problems and her mother has been B. Immune thrombocytopaenic purpura
giving her paracetamol. You perform some C. Meningococcal septicaemia
blood tests and get the following results: D. Reactive arthritis (transient synovitis)
• Hb (haemoglobin), 85 g/L; WBC (white blood E. Systemic-onset juvenile idiopathic arthritis
count), 17 × 109/L; neutrophils, 10.4 × 109/L;
platelets, 366 × 109/L 28.7
• blood film: normal, no atypical lymphocytes Dominika is a 3-year-old girl. She presents with
• ESR (erythrocyte sedimentation rate): an acute onset limp which was not present on
70 mm/hour the previous day. Her mother reports that she
• ANA (antinuclear antibody): negative was unwell 2 weeks ago with a coryzal illness.
• double-stranded DNA: negative The pain is in her right leg and is present only on
• antistreptolysin O titre: normal. walking. On examination she has a temperature
of 37° C. The hip and leg look normal but on
What is the most likely diagnosis?
passive movement of her right hip, there is
Select one answer only. decreased external rotation.

A. Acute lymphoblastic leukaemia What is the most likely diagnosis from the list
B. Epstein–Barr virus infection below?
C. Post-streptococcal arthritis
Select one answer only.
D. Systemic lupus erythematosus (SLE)
E. Systemic-onset juvenile idiopathic arthritis A. Bone tumour
B. Perthes disease
28.6 C. Reactive arthritis (transient synovitis)
William is an 8-year-old boy who presents with D. Septic arthritis
joint pain. His mother reports that since E. Slipped capital femoral epiphysis
yesterday he has been refusing to walk, as his
legs are so painful. On examination his 28.8
temperature is 37° C and his heart rate is 100 You are asked to review Huw, a baby who is
beats/min. He is settled at rest and playing a 24 hours old. He was born by vaginal delivery
computer game on his iPad. On further following a normal pregnancy. The midwife has
examination you notice a rash over his lower noted that the baby’s feet look abnormal. On
limbs, buttocks, and forearms (Fig. 28.2). The rash examination the feet are turned inwards but
comprises some large (5–15 mm), red, raised appear to be of a normal size. On passive
lesions, and multiple pin-point lesions, which do movement of the foot you are able to manipulate
160 not blanch on pressure. His peripheries are warm it so that it can be fully dorsiflexed to touch the
and he has a good pulse volume with a normal front of the lower leg (neutral position).
What is the most likely diagnosis from the list
given below?
Select one answer only.
A. Positional talipes equinovarus
B. Talipes calcaneovalgus
C. Talipes equinovarus
D. Tarsal coalition

Musculoskeletal disorders
E. Vertical talus

28.9
Harry is an 18-month-old boy. He is brought to
the Emergency Department as he has stopped
walking and his mother is worried his left leg is
painful. On examination he will not weight bear,
Figure 28.3
and cries when his left leg is moved passively. He
is afebrile and the rest of his examination is
normal, except that you think his sclera has a
slight blue tinge. An X-ray is performed, which
shows a fracture of his femur, but there is also a
28.11
comment that the bones look osteoporotic. On
John is 14 years old. He presents to his general
further questioning from his mother, she cannot
practitioner as he is conscious of his physical
give any history of trauma or any incident that
appearance. He is the tallest boy in his class and
could have caused the fracture. On looking back
he does not like getting changed for sport at
through the notes you note that he broke his left
school because the other boys mock him. On
arm 4 months ago and again his mother could
examination his height is above the 98th centile,
not give any explanation as to why this had
he has long fingers, and a wide arm span. You
happened. Harry’s older sister is with them
inspect his chest (Fig. 28.4).
and has her arm in a splint having recently
fractured it.
What is the most likely diagnosis for Harry?
Select one answer only.
A. Accidental injury (unwitnessed)
B. Non-accidental injury
C. Osteogenesis imperfecta
D. Osteomyelitis
E. Osteopetrosis

28.10
John is an 8-year-old boy. He presents to the
Emergency Department with a 5-day history of
pain in his right leg. It has slowly been getting
worse. He has not had a fever. There is no history
of trauma. On examination he has a painful limp.
He has a reduced range of movement of the
right hip. He has no medical history of note, and
has been taking ibuprofen for the pain, which
has helped a little.
An X-ray of his pelvis is shown in Fig. 28.3.
Select the most likely diagnosis. Select one
answer only.
A. Fractured femur
B. Osteosarcoma
C. Perthes disease
D. Reactive arthritis (transient synovitis) 161
E. Slipped capital femoral epiphysis Figure 28.4
Select the most likely diagnosis. treatment. On examination she is afebrile. Her
right knee is swollen with only mild tenderness
A. Harrison’s sulci on passive movements of the joint. There is no
B. Kyphosis other medical history of note and she is not on
28 C.
D.
E.
Pectus excavatum
Pectus carinatum
Scoliosis
any medication.

28.13
Musculoskeletal disorders

For each of the following patients with leg pain


Questions: Extended Matching (A–L), select the most likely diagnosis. Each
option may be used once, more than once, or
28.12 not at all.
For each of the following patients select the
most likely diagnosis (A–K). Each option may be A. Acute lymphoblastic leukaemia
used once, more than once, or not at all. B. Chondromalacia patellae
C. Complex regional pain syndrome
A. Cellulitis D. Growing pains
B. Fractured femur E. Hypermobility
C. Juvenile idiopathic arthritis F. Osgood–Schlatter disease
D. Osteomyelitis G. Osteochondritis dissecans
E. Osteosarcoma H. Osteomyelitis
F. Perthes disease I. Scheuermann disease
G. Reactive arthritis (transient synovitis) J. Spondylolysis
H. Septic arthritis K. Rickets
I. Slipped capital femoral epiphysis L. Tumour – osteoid osteoma
J. Rheumatic fever
K. Vitamin D deficiency 28.13.1
Milly, a 6-year-old girl, is taken to her general
28.12.1 practitioner by her mother as she has started
Emma, aged 3½ years, presents to the waking up at night and complaining that her
Emergency Department. She has a 24-hour legs hurt. This started about 2 months ago. She
history of a limp and pain in her left thigh. There goes off to sleep okay but wakes up at around
is no history of trauma. Her temperature is midnight and cries out. Her mother feels she is in
38.9° C and her pulse is 150 beats/min. On pain and has to massage her legs before she will
examination the left thigh is swollen and held go back to sleep. On examination she looks well.
flexed and abducted. She holds it as still as Her legs look normal and she has a full range of
possible and cries if it is moved. The upper leg is movement around all the joints. The rest of her
slightly red and is warmer to touch than the examination is normal.
right leg. She has no other medical history and
her mother has given some paracetamol, which 28.13.2
had no effect on the pain. Flora is a 12-year-old girl. She presents to the
Emergency Department for the third time in 10
28.12.2 days. Her right ankle is extremely painful and she
The parents of Suliman, aged 18 months, bring is unable to weight bear, and is brought into the
him to their general practitioner. They are department in a wheelchair. She was playing
concerned because he has suddenly stopped netball 10 days ago when the ball landed on her
walking. On examination he is miserable but foot. Ever since then it has been painful and has
apyrexial. His left thigh is swollen. There is no been getting worse. She had an X-ray which
discoloration of the skin and it is not warm to showed no fracture. On examination she has
touch. On inspection it appears that he is not extreme tenderness over the medial aspect of
moving his left foot, however, he withdraws it on her foot, even when touched very lightly with a
tickling his foot, which leads him to cry. He is bit of cotton wool. The right foot feels slightly
unable to weight bear. He has no other medical colder than the left foot, but her foot pulses are
problem and is not on any medications. normal. She refuses to move the foot and will
not let you try and move it for her as it is so
28.12.3 painful. The rest of her examination is entirely
Claire is 5 years old. She presents with a swollen normal and she is systemically well with no fever.
joint. Her knee joint has been swollen for several
days. There is some mild tenderness but she 28.13.3
continues to walk, with a slight limp. Her mother Suji, a 14-year-old girl, complains of pains in her
reports that 3 weeks ago she had a bout of knees when walking upstairs. She is otherwise
gastroenteritis associated with a fever. There well. She is free of pain whilst sitting in her chair,
162 were two episodes of bloody diarrhoea and she but complains of pain in both knees when asked
vomited once. Her symptoms resolved without to stand up. On examination you notice that she
has loss of the medial arch of her feet. She has 28.14.1
no other medical problems and is not on any Jack is a 14-month-old boy. He has only recently
medication. started walking. His mother brings him to his
general practitioner as she is concerned that he
28.13.4 walks with a limp. You examine his gait and he
Joseph is a 14-year-old boy who presents to has a painless limp of his right leg. He has
his general practitioner. He complains of pain asymmetry of the skin folds around his right
and swelling in his right knee for several thigh and his right hip cannot be fully abducted.

Musculoskeletal disorders
weeks. This is worse after playing football. On He has no other medical history. He was born by
examination you note swelling over the tibial elective caesarean section because of breech
tuberosity. He has no other medical problems presentation.
and has been taking paracetamol when the pain
is present. 28.14.2
Zahra is a 4-year-old Asian girl. Her mother is
28.13.5 concerned about her leg shape. On examination
Craig is 5 years old and is complaining that his you notice that on standing, the knees are very
back hurts. This started about 4 weeks ago and widely spaced with the feet held together. On
his mother tried to ignore it as there was nothing completing the musculoskeletal examination,
to see and ibuprofen seemed to help. However, you notice that she has swollen wrist joints.
he has now started waking up at night crying There is no other medical history of note and she
out that his back is sore. He has not had a fever. is not on any medications.
His father suffers from back pain and is off work
with it at the moment. On examination he is 28.14.3
reluctant to move his back for you but there is Rachel, a 2-year-old girl, is seen for a
no obvious tenderness over his spine and there developmental check by the health visitor. She is
is mild scoliosis. His neurological examination walking on her own but the health visitor notes
is normal. that she has flat feet while she is walking. She
tries to get Rachel to stand on her tiptoes but
28.14 she cannot. When she passively extends her big
For each of the following children, select the toe an arch is demonstrated.
most likely diagnosis (A–J). Each option may be
used once, more than once, or not at all. 28.14.4
Alliah is a 2½-year-old girl. She is seen in the
A. Bow legs – normal variant paediatric outpatient department because her
B. Bow legs – rickets mother is worried about her walking, as she
C. Developmental dysplasia of the hip – late usually walks on tiptoes. She was born by normal
diagnosis vaginal delivery at 37 weeks. She started to walk
D. Kyphosis at 15 months. She never crawled but did bottom
E. Normal variant of childhood shuffle. On examination of her gait she mostly
F. Pectus excavatum walks and runs on tiptoes, and her mother
G. Pectus carinatum confirms she has done this ever since she started
H. Perthes disease walking. However, she can walk on her heels if
I. Scoliosis asked to do so. General examination of her legs
J. Slipped capital femoral epiphysis demonstrates normal tone and reflexes.

163
Answers: Single Best Answer
28.1

28 A. Haemophilus influenza
Haemophilus influenzae may cause osteomyelitis,
particularly in unimmunized children but is an
uncommon causative organism.
Musculoskeletal disorders

B. Mycobacterium tuberculosis
Tuberculosis may cause osteomyelitis but is an
uncommon cause in the UK and presents more
insidiously. It might be difficult to exclude. Consider
Figure 28.5
a chest X-ray and Mantoux test or IGRA (interferon-
gamma release assay) if his condition does not
resolve.
C. Salmonella typhi adolescent males who are physically active
In sickle cell anaemia, there is an increased risk of (particularly football or basketball). It usually
Salmonella osteomyelitis. presents with knee pain after exercise, localized
tenderness and sometimes swelling over the
D. Staphylococcus aureus
tibial tuberosity. There is often hamstring
Correct. Amir is most likely to have osteomyelitis,
tightness. It is bilateral in 25–50% of cases.
usually caused by Staphylococcus aureus, but
other pathogens include Streptococcus. In sickle C. Perthes disease
cell anaemia, there is an increased risk of Whilst pain might be referred to the knees from
Staphylococcus and Salmonella osteomyelitis. the hips, it is unlikely in this boy.
E. Streptococcus viridans D. Septic arthritis
Most infections are caused by Staphylococcus Fever absent – and he can still weight bear.
aureus but other pathogens include group A E. Slipped capital femoral epiphysis
Streptococcus species. Affects the hip. There would not be swelling over
28.2 the tibial tuberosity.
A. Osgood–Schlatter disease
28.4
Osgood–Schlatter disease affects the knees.
A. Henoch–Schönlein purpura arthritis
B. Osteomyelitis The arthritis of Henoch–Schönlein purpura
Osteomyelitis is less likely as she is not would usually be accompanied by a rash and a
systemically unwell, for example there is no fever. high fever would not be present.

C. Perthes disease B. Juvenile idiopathic arthritis


Perthes disease usually affects children aged Does not usually present so acutely with a single
5–10 years. There is degeneration of the femoral red swollen joint.
head – which is not seen here. C. Osteomyelitis
D. Septic arthritis The main differential diagnosis.
Septic arthritis is less likely as she is not D. Septic arthritis
systemically unwell. Correct. Septic arthritis. Needs to be
differentiated from osteomyelitis. Edward’s knee
E. Slipped capital femoral epiphysis
joint itself is red and swollen, with pain on
Correct. She has a slipped capital femoral
movement, making septic arthritis more likely.
epiphysis (also known as slipped upper femoral
An ultrasound and aspiration of the joint should
epiphysis). This results in displacement of
be performed to confirm this.
the epiphysis of the femoral head postero-
inferiorly (Fig. 28.5, arrow). It is most common at E. Trauma
10–15 years of age during the adolescent growth There is no history of trauma.
spurt, particularly in obese individuals.
28.5
28.3 A. Acute lymphoblastic leukaemia
A. Osteomyelitis Acute lymphoblastic leukaemia needs to be
Would be a very unusual cause, particularly considered, but is less likely as Karen has numerous
without fever. joints affected and the blood film is normal.
B. Osgood–Schlatter disease B. Epstein–Barr virus infection
164 Correct. This is osteochondritis of the patellar Epstein–Barr virus is unlikely as it rarely causes
tendon insertion at the knee, often affecting widespread arthritis.
C. Post-streptococcal arthritis 2–12 years of age. It often follows or is
Post-streptococcal arthritis is unlikely as the accompanied by a viral infection.
antistreptolysin O titre is normal, although a Presentation is with sudden onset of pain in the
repeat blood test is required to confirm that the hip or a limp. There is no pain at rest, but there is
titre remains low. decreased range of movement, particularly
external rotation.
D. Systemic lupus erythematosus (SLE)
SLE may have similar clinical features but is D. Septic arthritis
unlikely as the ANA (antinuclear antibody) and Septic arthritis of the hip joint can be difficult to

Musculoskeletal disorders
double-stranded DNA are negative. SLE is also differentiate from reactive arthritis (transient
very uncommon at this age. synovitis) but is far less common. Children with
septic arthritis are usually systemically unwell
E. Systemic-onset juvenile idiopathic
and have a fever. There may be erythema and
arthritis
swelling around the joint, but this may not be
Correct. Systemic-onset juvenile idiopathic
evident at the hip. There is severe pain on any
arthritis is the most likely diagnosis, as she has a
movement of the leg, and refusal to weight bear.
systemic illness, polyarthritis (more than four
If necessary, a normal ultrasound scan and
joints) and a salmon-pink rash. The raised
C-reactive protein evaluation will help exclude it.
neutrophil count and markedly raised ESR are
consistent with the diagnosis. E. Slipped capital femoral epiphysis
28.6 Slipped capital femoral epiphysis usually occurs
A. Henoch–Schönlein purpura in older children.
Correct. Henoch–Schönlein purpura is the most 28.8
common vasculitis of childhood, and presents A. Positional talipes equinovarus
with a purpuric rash over the lower legs and Correct. This is a common problem and is caused
buttocks, but which can also involve the by intrauterine compression. The foot is of normal
extensor surface of the elbows. It is often size, the deformity is mild and can be corrected
associated with arthritis of the ankles or knees. In to the neutral position with passive manipulation.
this case the child presented with the arthritis
rather than the rash. Other features are B. Talipes calcaneovalgus
abdominal pain, haematuria and proteinuria. Talipes calcaneovalgus is a fixed bony deformity,
in which the foot is everted and dorsiflexed.
B. Immune thrombocytopaenic purpura
The raised rash suggests a vasculitis rather than C. Talipes equinovarus
just low platelets. Also the distribution in Talipes equinovarus is a fixed bony deformity, in
Henoch–Schönlein purpura and immune which the foot is inverted and supinated. Passive
thrombocytopaenic purpura is different. movement to a normal, neutral position is not
possible. See Figs. 28.5 and 28.6 in Illustrated
C. Meningococcal septicaemia
Textbook of Paediatrics for more details.
Meningococcal sepsis is unlikely as he is afebrile,
and the purpura is restricted to his lower limbs D. Tarsal coalition
and elbows. Tarsal coalition is a diagnosis made in adolescent
years.
D. Reactive arthritis (transient synovitis)
Reactive arthritis is unlikely as there is usually a E. Vertical talus
preceding or accompanying infection, and the Vertical talus is in children with rocker bottom feet
rash would not have this distribution and who often have other congenital abnormalities.
certainly would not be purpuric.
28.9
E. Systemic-onset juvenile idiopathic arthritis A. Accidental injury (unwitnessed)
Systemic-onset juvenile idiopathic arthritis is Possible, but why the recurrent pattern?
accompanied by a high fever in a child who is
systemically unwell; the salmon-pink coloured B. Non-accidental injury
rash is widespread and blanches. Although non-accidental injury needs to be
considered, the abnormality of the bones on
28.7 X-ray makes osteogenesis imperfecta more likely.
A. Bone tumour In clinical settings child protection concerns
A bone tumour is rare at this age and usually must be addressed in parallel with investigation
presents insidiously. for osteogenesis imperfecta and other medical
B. Perthes disease conditions.
Perthes disease usually occurs in older children.
C. Osteogenesis imperfecta
C. Reactive arthritis (transient synovitis) Correct. Harry is likely to have osteogenesis
Correct. This is the most common cause of acute imperfecta (type 1). It presents with fractures in 165
hip pain in children. It occurs in children aged early childhood and the bones look osteoporotic
on X-ray. The children often have blue sclera but B. Kyphosis
this can be difficult to identify with confidence. The spine is ‘bent over’.
The condition is autosomal dominant and it is
likely his older sister Amy has it as well. C. Pectus excavatum
Correct. Pectus excavatum is also known as
28 D. Osteomyelitis
Osteomyelitis does present with a painful leg,
‘hollow chest’ or ‘funnel chest’. This boy probably
has Marfan syndrome, but it also occurs in
but would usually be associated with fever and otherwise normal children.
Musculoskeletal disorders

redness over the area. The X-ray is usually normal


at presentation. D. Pectus carinatum
This is also known as a ‘pigeon chest’ and results
E. Osteopetrosis in a protrusion of the sternum.
Osteopetrosis is also an autosomal dominant
disease that can present in its milder form with E. Scoliosis
fractures, but the bones look dense on X-ray. There is lateral curvature of the spine and rib
humping when bending over.
28.10
A. Fractured femur
Always a possibility, but the X-ray is showing a Answers: Extended Matching
specific change in the femoral head.
B. Osteosarcoma Answer 28.12.1
Worth considering but the destruction here is of H. Septic arthritis
the femoral head. It can be difficult to clinically differentiate
between osteomyelitis and septic arthritis. In this
C. Perthes disease child, the very acute history and flexed and
Correct. John has Perthes disease. This affects abducted position of the thigh at rest suggest
boys five times more frequently than girls. It septic arthritis. Ultrasound (and diagnostic
usually presents between the age of 5 and 10 aspiration), blood tests for culture and tests for
years. The X-ray Fig. 28.6 confirms the diagnosis inflammatory markers should be performed and
with flattening of the right femoral capital intravenous antibiotics started. It is important to
epiphysis. When in doubt about the X-ray, not delay treatment as this can lead to
compare both hips. permanent damage to the joint and urgent
orthopaedic review should be sought.
Answer 28.12.2
B. Fractured femur
A fractured femur may be accidental but
non-accidental injury needs to be considered in
all unexplained fractures. In osteomyelitis and
septic arthritis he would be unwell with a fever,
the left thigh would be warm and discoloured,
and he would be unwilling to move the limb
at all.
Answer 28.12.3
G. Reactive arthritis (transient synovitis)
She has a reactive arthritis. As this followed an
episode of gastroenteritis with bloody diarrhoea,
it is likely to be due to enteric bacteria (e.g.
Campylobacter).

Figure 28.6 Answer 28.13.1


D. Growing pains
This condition is poorly understood, but the
child has episodes of generalized pain in the
D. Reactive arthritis (transient synovitis) lower limbs, which often wakes them from sleep.
The X-ray would be normal. It occurs in preschool-age and school-age
children. The pain is symmetrical in the lower
E. Slipped capital femoral epiphysis
limbs. The pain is not present at the start of
More common in overweight teenage girls.
the day, and physical activity is not limited.
28.11 The physical examination is entirely normal.
A. Harrison’s sulci Hypermobility can cause pain in the limbs but
166 This would lead to indentation at points of this child did not demonstrate any hypermobility
insertion of the diaphragm. of the joints.
Answer 28.13.2 Answer 28.14.1
C. Complex regional pain syndrome C. Developmental dysplasia of the hip – late
Flora is likely to have a localized complex diagnosis
regional pain syndrome, which often presents in Jack has developmental dysplasia of the hip. This
a single foot or ankle after minor trauma. In is usually identified on one of the two hip
addition to severe pain, there may be screening checks (birth and 6–8 weeks) babies
hyperaesthesia (increased sensitivity to stimuli), receive in the UK. He was at increased risk as he
allodynia (pain from a stimulus that does not was a breech presentation. Occasionally, it is not

Musculoskeletal disorders
normally produce pain) and the affected part identified, as examination is not 100% sensitive.
(often a foot or hand) may be cool to touch with Late presentation is with a painless limp.
swelling and mottling.
Answer 28.14.2
Answer 28.13.3 B. Bow legs – rickets
B. Chondromalacia patellae This girl has bow legs (genu varum), which
There is softening of the articular cartilage of the would be a common variant of normal up until 3
patella. It most often affects adolescent females, years of age. The finding of swollen wrists leads
causing pain when the patella is tightly apposed to the diagnosis of rickets. This girl should have
to the femoral condyles, as in standing up from blood tests to confirm the diagnosis and be
sitting or on walking up stairs. It is often started on vitamin D supplementation along
associated with hypermobility and flat feet, with dietary advice and advice regarding
suggesting a biomechanical component to the increasing sunlight exposure.
aetiology.
Answer 28.14.3
E. Normal variant of childhood
Answer 28.13.4
This is a normal variant of childhood. Toddlers
F. Osgood–Schlatter disease
learning to walk usually have flat feet due to
This is an osteochondritis of the patellar
flatness of the medial longitudinal arch and the
tendon insertion at the knee, often affecting
presence of a fat pad, which disappears as the
adolescent males who are physically active
child gets older. An arch can usually be
(particularly football or basketball). It usually
demonstrated on standing on tiptoe, or by
presents with knee pain after exercise, localized
passively extending the big toe. Marked flat feet
tenderness, and sometimes swelling over the
are common in hypermobility. A fixed flat foot,
tibial tuberosity. It is bilateral in 25–50%
often painful, may indicate a congenital tarsal
of cases.
coalition and requires an orthopaedic opinion.
Rachel is unable to stand on her tiptoes, not
Answer 28.13.5 because of a pathological condition but because
L. Tumour – osteoid osteoma children are unable to do so at this age.
Craig may have a benign tumour, an osteoid
osteoma. It presents with back pain, which is Answer 28.14.4
worse at night. Back pain in young children E. Normal variant of childhood
must be taken seriously, as it is uncommon. Red Toe walking is common in young children and
flag features include a young age, waking at may become persistent, usually from habit. The
night, fever, weight loss and focal neurological child can walk normally on request. Children
signs. Osteomyelitis of the back can occur and with mild cerebral palsy can present with toe
would present with pain but is usually associated walking but an increase in tone would be found
with a fever, systemic upset and tenderness over on examination. In older boys, Duchenne
the area. muscular dystrophy should be excluded.

167
29

Neurological disorders
4-day history of fever, lethargy, vomiting and
Questions: Single Best Answer abdominal pain. She normally opens her bowels
twice a week and does intermittent urinary
29.1 catheterization three times a day with the help of
Annette is a 15-year-old girl who complains of her mother but despite this she is dribbling urine.
worsening daily occipital headaches. They occur
mainly in the mornings and sometimes wake her What is the most likely cause of her new
from sleep. Her mother says she is doing less symptoms.
well at school than previously and has become a Select one answer only.
difficult and grumpy teenager. She sometimes
vomits in the mornings. She has no other A. Constipation
medical problems though she is on the oral B. Hydrocephalus
contraceptive pill. C. Hypertension and renal failure
D. Tethering of the spinal cord
What is the most likely diagnosis?
E. Urinary tract infection
Select one answer only.
29.4
A. Idiopathic intracranial hypertension
Angelo, a 15-month-old boy, had been unwell
B. Migraine
with a runny nose and cough for a day when his
C. Medication side-effect
father brings him to the Emergency Department.
D. Tension headache
At lunch he suddenly became stiff, his eyes
E. Raised intracranial pressure due to a
rolled upwards and both his arms and legs
space-occupying lesion
started jerking for 2 minutes. He felt very hot at
the time. When examined 2 hours later, he has
29.2 recovered fully. This is the first time this has
Gerald is a 10-year-old boy. He is seen in the happened. He has a normal neurological
special school clinic with his mother, who is just examination and is acquiring his developmental
recovering from cataract surgery. He has milestones normally. He has no other medical
moderate learning difficulties and is teased problems. The triage nurse performed a blood
because of his marked facial weakness. He is glucose test, which indicated a glucose level of
unable to walk long distances. His mother says 4.2 mmol/L (within normal range).
that he struggles to release things once he grabs
them. He has no other medical problems. He has What would be the most appropriate
not had any investigations performed. investigation?
Select the most useful diagnostic test from the list. A. CT scan of the brain
B. ECG
Select one answer only. C. EEG (electroencephalography)
A. DNA testing for trinucleotide repeat D. No investigation required
expansion E. Oral glucose tolerance test
B. Electromyography (EMG)
C. Muscle biopsy 29.5
D. Nerve conduction studies Pamela is an 8-year-old girl with recurrent
E. Serum creatine kinase seizures. She has three or four seizures a month,
where she lets out a cry, her arms and legs
29.3 become stiff, her eyes roll upwards and then she
Olive is a 5-year-old girl who had a jerks her arms and legs. This lasts about 3
myelomeningocele repaired shortly after birth. minutes. Afterwards she sleeps for 2 hours and is
She uses a wheelchair for mobility. She has a then back to normal. She is doing well at school
but is sometimes missing school because of her move her arm, although she has some residual
seizures. She is currently not on any medication weakness of her mouth. She has no significant
and has no other medical problems. medical history except some episodes of
abdominal pain as a younger child. Her mother
What would be the best intervention for this child? tells you that she also suffers from headaches.
Select one answer only.
What is the most likely diagnosis?
A. Anti-epileptic drug therapy
Select one answer only.

Neurological disorders
B. Home schooling
C. Ketogenic diet A. Idiopathic intracranial hypertension
D. No intervention required B. Migraine
E. Vagal nerve stimulation C. Raised intracranial pressure due to a
space-occupying lesion
29.6 D. Subarachnoid haemorrhage
Alan is a 7-month-old male infant who was E. Tension headache
preterm, born at 28 weeks’ gestation,
birthweight 970 g, and whose family recently 29.8
arrived in this country. He is seen in a paediatric Aparna is a 2-year-old girl of Indian ethnicity
clinic because of vomiting. He had been seen who lives in the UK. She presents to her general
regularly by a doctor who was treating him for practitioner as she has been unsteady on her
gastro-oesophageal reflux. He has always feet for a day, having had diarrhoea during the
vomited but this has been getting worse and his previous week. On examination she is afebrile,
mother has noticed his eye movements are not has reduced muscle power and tone and no
normal. His examination findings can be seen in tendon reflexes can be elicited in her lower
Fig. 29.1. limbs. She is referred urgently to the paediatric
hospital and 6 hours later she is unable to stand
and the tendon reflexes in her upper limbs are
now absent. She has no other medical problems
and has been fully immunized.

What is the most likely diagnosis?

Select one answer only.


A. Guillain–Barré syndrome
B. Myasthenia gravis
C. Myotonic dystrophy
D. Poliomyelitis
E. Spinal muscular atrophy
Figure 29.1 (Courtesy of Dr Tony Hulse).
29.9
What is the most likely underlying cause for Jordain is a 7-year-old child born in Yemen who
these findings? presents to the children’s outpatient department.
Select one answer only. His mother is concerned regarding a number of
skin marks, which she has noticed over the last
A. Aqueduct stenosis year. She was not initially concerned about them,
B. Intraventricular haemorrhage but they are becoming more numerous and
C. Meningitis larger in size. She reports that Jordain’s father
D. Posterior fossa neoplasm also had similar skin marks before he passed
E. Subarachnoid haemorrhage away from ‘cancer’. On examining Jordain you
see the skin lesions shown in Fig. 29.2. He also
29.7 has a large number of freckles in his axilla region.
Sharon is an 11-year-old girl who has had He has no other medical problems and is not on
occasional headaches for the last 3 months. any medications.
Today was her first day at secondary school.
During maths class she developed her usual What is the most likely diagnosis?
throbbing left-sided headache, associated with Select one answer only.
nausea. Over the next hour, she lost her temporal
vision in her right eye and found she only had A. Ataxia telangiectasia
minimal movement in her right arm. The left side B. Friedreich ataxia
of her mouth was drooping. Her parents were C. Neurofibromatosis
called, who rushed her to hospital. She is now D. Sturge–Weber syndrome 169
feeling much better, has normal vision and can E. Tuberous sclerosis
29.11
Antonia, a 5-year-old girl, is seen by her
general practitioner. Her mother and school
teacher have noticed she has episodes where
29 she stops her activity for a few seconds, stares
blankly ahead and then resumes the activity
as if she had never stopped. These episodes
happen many times a day. She has no other
Neurological disorders

medical problems and there is no family history


of note. The EEG during an episode is shown
in Fig. 29.4.

1
2
3
4
5
6
7
8
9
10
Figure 29.2 (Courtesy of Dr Graham Clayden). 11
12
13
29.10 200 µV
14
Gregor is born at term. It was a normal 1s 15
pregnancy though the mother did not have any
16
antenatal ultrasound examinations as she was
against medical procedures. He was born by
vaginal delivery. Immediately, the midwife Figure 29.4 (Courtesy of Dr Richard Newton).
noticed the lesion shown in Fig. 29.3.

Choose the most likely diagnosis.


Select one answer only.
A. Childhood Rolandic epilepsy (benign
epilepsy with centro-temporal spikes)
B. Childhood absence epilepsy
C. Juvenile myoclonic epilepsy
D. Lennox-Gastaut syndrome
E. Infantile spasms (West syndrome)

29.12
Figure 29.3
Vijay is a 5-month-old infant who has been seen
repeatedly by his general practitioner because of
Which of the supplements listed would have colic. His mother brings him to the Accident and
reduced the risk of this problem if taken by the Emergency department as he is having episodes
mother periconceptually? of suddenly throwing his head and arms forward.
Select one answer only. These episodes occur in repetitive bursts. His
mother thinks they may be something more than
A. Folic acid just colic, as he is now not smiling or supporting
B. Iron his head as well as he did previously. He was born
C. Vitamin A at term by normal vaginal delivery and has no
170 D. Vitamin B12 other medical problems. His EEG is shown in
E. Vitamin D Fig. 29.5.
Afterwards she was groggy for a few minutes,
1 but is now back to her usual self. This has not
2 happened before. Her mother is very distressed
3 and now reports feeling faint herself. The triage
4 nurse has performed a blood glucose which
5 indicates a glucose level of 4.1 mmol/L (within
6 normal range).
7

Neurological disorders
8 29.13.2
9 Jennifer, an 11-year-old girl, is brought to the
10 Emergency Department after collapsing at
11 school. Her teacher described her standing in
12 assembly, becoming pale and collapsing to the
13 floor. She had a couple of jerking movements of
100 µV
14 her limbs lasting a few seconds. She returned to
1s 15 normal promptly. Jennifer says that she had not
16 eaten breakfast that morning, and experienced a
sensation of feeling hot, a black curtain coming
in front of her eyes, sounds becoming distant
Figure 29.5 (Courtesy of Dr Richard Newton). and feeling dizzy. The school nurse did a blood
glucose, which indicated a glucose level of
3.5 mmol/L (within the normal range). She has
Choose the most likely diagnosis. had two episodes similar to this in the past but
Select one answer only. has not presented to hospital before. She has no
other medical problems.
A. Juvenile myoclonic epilepsy
B. Lennox-Gastaut syndrome 29.13.3
C. Childhood Rolandic epilepsy (benign Bosco is an active 7-year-old black Caribbean
epilepsy with centro-temporal spikes) boy, who was referred to the outpatient
D. Childhood absence epilepsy department with a history of collapse. He
E. Infantile spasms (West syndrome) collapses during football practice or when he is
playing outside with his siblings. This has
Questions: Extended Matching happened at least six times in the last 3 months.
His mother says that he suddenly becomes pale,
29.13 loses consciousness and then slowly comes
For each of the scenarios (A–N) of children who around. He has no medical history of note and
have had a funny turn, pick the most likely has never had any investigations. His father died
diagnosis from the list. Each answer can be used suddenly at the age of 34 years.
once, more than once, or not at all.
29.13.4
A. Benign paroxysmal vertigo Rene is a lively 2-year-old boy who is seen in the
B. Blue breath-holding spells (expiratory acute paediatric assessment unit, having had an
apnoea syncope) episode of turning blue and collapsing. This is
C. Cardiac arrhythmia not the first time that this has happened. He
D. Childhood absence epilepsy recovers quickly after these events. During the
E. Hypoglycaemia consultation his mother repeatedly gives him
F. Intracranial haemorrhage sweets to keep him calm. These episodes only
G. Migraine occur when he is crying and this has led to his
H. Narcolepsy mother trying to avoid anything that will upset
I. Night terrors him. He has no other medical problems. His
J. Non-epileptic attack disorder (pseudoseizure) blood glucose today is 4.9 mmol/L.
K. Reflex asystolic syncope (reflex anoxic seizure)
L. Syncope 29.13.5
M. Tic disorder Dora is an 8-year-old girl who attends the
N. Tonic–clonic seizure outpatient department. Her mother is worried
because she used to be top of the class, but is
29.13.1 not doing as well at school this year. Whilst in
Emily, a normally fit and healthy 3-year-old girl, is clinic, you notice that Dora has an episode where
rushed to the Emergency Department. She had she suddenly stops what she is doing, stares
been playing at nursery and had banged her ahead whilst flickering her eyelids for a couple of
head against a door. Almost immediately she seconds and then resumes her previous activity
went very pale and stiff and had jerking as if nothing had happened. You ask her to blow 171
movements of her arms and legs for 20 seconds. out an imaginary candle, and the same thing
happens again. She is growing normally and is G. Gower’s sign
otherwise fit and well. H. Kernig’s sign
I. Romberg’s sign
29.14
29.15.1
29 For each scenario below (A–H), choose the most
likely diagnosis. Each diagnosis can be used
Ahmed is a 5-year-old boy who presents to the
outpatient clinic with weakness. His parents
once, more than once, or not at all.
report that he finds it difficult to climb the
stairs at home. This has been becoming
Neurological disorders

A. Childhood Rolandic epilepsy (benign


epilepsy with centro-temporal spikes) progressively worse over the preceding 6–12
B. Childhood absence epilepsy months. He is playing on the carpet with his
C. Juvenile myoclonic epilepsy brother when his mother calls him over. He has
D. Lennox–Gastaut syndrome to roll from his back onto his front in order to
E. Infantile spasms (West syndrome) stand up.

29.14.1 29.15.2
Damasco is a 6-year-old black boy who presents Christopher attends outpatient clinic for his
in the paediatric clinic. He has been referred routine follow-up. He is a 3-year-old boy who did
because his brother, with whom he shares a not breathe at birth. Following resuscitation, he
room, has woken his parents early in the required intensive care for the first 5 days of life.
morning on several occasions complaining that He now has increased tone and reflexes in his
Damasco is making unusual sounds and drooling right upper and lower limbs. On rubbing a blunt
from his mouth. By the time his parents come to instrument up the lateral side of his right foot
the room Damasco is jerking his upper and lower there is extension of the greater toe and fanning
limbs rhythmically. This lasts for 1–2 minutes. of his toes.
Afterwards Damasco complains of a funny 29.15.3
sensation on the left side of his mouth, and then Sayeed is an 8-year-old Pakistani boy who has
goes back to sleep. He is doing well at school developed muscle weakness. His mother reports
and clinical examination is normal. He has no that he has poor coordination at home and at
other medical problems. school. His speech has also recently become
29.14.2 slurred. His problems have come on slowly over
Jude is a 14-year-old boy who is on treatment for the last 12–18 months. He has no other medical
generalised tonic–clonic and absence seizures. history and is not taking any medications. On
He attends a routine clinic appointment examination you notice that he has wasting of
complaining of regularly spilling his tea in the his calves.
morning because his arm jerks involuntarily. When standing upright with his legs together
These jerks are worse if he has stayed up late the and closing his eyes he becomes unsteady and
night before. He is doing relatively well at school. starts to sway.
29.14.3 29.15.4
Andrew is a 5-year-old boy with severe learning Jane is an 11-year-old girl who presents to the
difficulties who attends a special school. He is outpatient department. She complains that she
seen in his review clinic. His epilepsy is difficult to finds it difficult to play sports such as basketball.
control and he is on two different antiepileptic In particular, she finds it difficult to run and
drugs. He has several different types of seizures: coordinate her arm movements. She was born at
sudden stiffening of his limbs causing him to fall, 32 weeks’ gestation and discharged from the
episodes of staring blankly ahead for up to 30 neonatal service at 2 years of age as her
seconds before resuming previous activities, and development was normal. Her neurological
episodes of his head dropping with a brief loss of examination appears to be normal except when
consciousness. He is fed via a gastrostomy and you are examining her gait. On heel walking her
has chronic drooling. left arm moves into a flexed position.

29.15 29.15.5
In the following clinical histories (A–I) choose the Paolo is a 7-year-old boy who presents to the
sign that is being described. Each finding can be Accident and Emergency department with fever.
used once, more than once, or not at all. His mother reports that this has been present for
the last 8 hours and that he is sleepy and wants
A. Babinski’s sign to sleep in a dark room. He has no other medical
B. Brudzinski’s sign history. On examination you note that he is
C. Chorea photophobic. Whilst he is lying on the couch you
D. Clonus flex his right knee and hip to 90°. On fully
172 E. Dyskinesia extending the knee, he complains of pain and
F. Fogs’ sign arches on his back.
D. Nerve conduction studies
Answers: Single Best Answer Difficult, costly and painful.

29.1 E. Serum creatine kinase


A. Idiopathic intracranial hypertension Serum creatine kinase is helpful in diagnosing
Idiopathic intracranial hypertension also presents muscular dystrophies, such as Duchenne.
with features of raised intracranial pressure but is
less likely to affect her behaviour and school 29.3

Neurological disorders
performance, though they can be difficult to A. Constipation
differentiate on clinical history alone. The constipation is not a new symptom but
does predispose Olive to urinary tract
B. Migraine
infection.
Migraine leads to asymmetrical features and
typically the headaches are relieved by sleep. B. Hydrocephalus
Hydrocephalus would cause lethargy and
C. Medication side-effect
vomiting, but not the other symptoms.
Analgesia overuse can cause daily headache and
is a common cause of chronic daily headache C. Hypertension and renal failure
but this is not reported. This must always be This is insidious and often presents as an
included in the history. Sometimes parents are incidental finding or with faltering growth.
unaware that paracetamol is being taken
frequently. D. Tethering of the spinal cord
Would not cause fever.
D. Tension headache
Although these headaches are very common, E. Urinary tract infection
they are often at the end of the day. Rarely do Correct. Olive had a myelomeningocele that
they wake one from sleep. has caused a neurogenic bladder (she has
to use catheterization to maintain continence)
E. Raised intracranial pressure due to a and neurogenic bowel dysfunction (she is
space-occupying lesion usually constipated). A neurogenic bladder
Correct. This girl has raised intracranial pressure predisposes to developing a urinary tract
due to a space-occupying lesion. The headaches infection, due to the stagnant urine lying in
are worsening, and occur when lying down and the bladder. Her symptoms of fever, abdominal
wake her from sleep. Raised intracranial pressure pain, vomiting, and urinary incontinence are
is associated with morning vomiting. There is most likely to be due to a urinary tract
also behaviour change and worsening infection.
educational performance. The VI (abducens)
cranial nerve has a long intracranial course and is
29.4
often affected when there is raised pressure,
A. CT scan of the brain
resulting in a squint with diplopia and inability
Persistent signs, particularly asymmetric ones,
to abduct the eye beyond the midline. It is a
should prompt imaging.
false localising sign. Other nerves are affected
depending on the site of lesion. B. ECG
Why the fever? Cardiac causes usually occur ‘out
29.2 of the blue’ or following a specific trigger:
A. DNA testing for trinucleotide repeat exercise (hypertrophic obstructive
expansion cardiomyopathy) or shock (ventricular
Correct. The most useful diagnostic test here tachycardia).
would be DNA testing for trinucleotide repeat
expansion of myotonic dystrophy. The clinical C. EEG (electroencephalography)
history suggests myotonic dystrophy because Will be ‘abnormal’ in a significant proportion of
of his learning difficulties, myotonia, facial normal children. This will lead to significant
weakness and limb weakness. It was dominantly ‘overdiagnosis’ of epilepsy. Not indicated here.
inherited from his mother, who has cataracts as
a result. D. No investigation required
Correct. Angelo had a simple febrile seizure,
B. Electromyography (EMG) secondary to a respiratory tract infection. A
You can test for the myotonia clinically rather febrile seizure is a clinical diagnosis and does not
than using electromyography, so DNA testing is require investigation. Indeed, there is no
the test most likely to give you the diagnosis. confirmatory test.
C. Muscle biopsy E. Oral glucose tolerance test
Muscle biopsies are performed when one needs This is not required as the blood glucose is
to differentiate between muscle and neuronal normal. It is rarely required in children to 173
pathology. diagnose diabetes.
29.5 this necessitates neuroimaging to exclude a
A. Anti-epileptic drug therapy vascular abnormality or structural problem.
Correct. This child should be started on an
anti-epileptic drug. Pamela has recurrent C. Raised intracranial pressure due to a

29 generalised tonic–clonic seizures which are


affecting her quality of life as she is missing
space-occupying lesion
No red flags.
school. D. Subarachnoid haemorrhage
A subarachnoid haemorrhage is rare in a child of
Neurological disorders

B. Home schooling
The solution to her missing school is control of this age. Besides, the pain is often occipital and
her seizures. the neurological symptoms would not improve
over this period.
C. Ketogenic diet
Ketogenic diets are sometimes used in children E. Tension headache
with intractable seizures. A tension headache would be symmetrical, and
the pain is not typically throbbing. In tension
D. No intervention required headaches there would not be weakness or
The seizures are impacting upon schooling and visual disturbance.
quality of life.
29.8
E. Vagal nerve stimulation A. Guillain–Barré syndrome
Vagal nerve stimulation is used in certain Correct. Guillain–Barré syndrome (post-
children with intractable seizures. infectious polyneuropathy). The classical
presentation is with ascending weakness. It may
29.6 follow campylobacter gastroenteritis. She must
A. Aqueduct stenosis be closely monitored, as a serious complication
Possible but in view of prematurity another is respiratory failure. To monitor her respiratory
diagnosis is more likely. function, you can ask her to cough (or sing) to
B. Intraventricular haemorrhage assess diaphragmatic function and involvement
Correct. This child has hydrocephalus, and the of the nerves supplying the chest muscles.
most likely underlying cause is intraventricular B. Myasthenia gravis
haemorrhage from his prematurity. This results in Myasthenia gravis presents as abnormal muscle
impairment of drainage and reabsorption of CSF fatigability, which improves with rest or
leading to post-haemorrhagic hydrocephalus. anticholinesterase drugs. It is incredibly rare!
The figure shows his large head and sun-setting
eyes, which, together with the vomiting, are all C. Myotonic dystrophy
clinical features of hydrocephalus. He will also Aparna’s illness is an acute neuropathy.
have an increasing head circumference, which
will cross centile lines. D. Poliomyelitis
Poliomyelitis is a very important differential
C. Meningitis diagnosis here, but as the child is immunized
There is no fever or clinical features of sepsis or and as polio has been eradicated in the UK it
meningitis, which makes an infection unlikely. becomes far less likely. It is on the verge of
eradication globally.
D. Posterior fossa neoplasm
The eyes show features of sun-setting, not a E. Spinal muscular atrophy
cranial nerve palsy or squint characteristic of Spinal muscular atrophy is an autosomal
posterior fossa neoplasms. The history is recessive group of disorders causing
suggestive of post-haemorrhagic hydrocephalus. degeneration of the anterior horn cells, leading
to progressive weakness and wasting of skeletal
E. Subarachnoid haemorrhage muscles. They are chronic disorders and would
Not typical in preterm infants. not present acutely as in this child.
29.7 29.9
A. Idiopathic intracranial hypertension A. Ataxia telangiectasia
Idiopathic intracranial hypertension would The skin lesions in ataxia telangiectasia are small
typically be worse on lying and/or straining. dilated blood vessels near the surface of the skin
Often there is papilloedema. or the mucous membranes.
B. Migraine B. Friedreich ataxia
Correct. The headache is typical of a migraine, Friedreich ataxia does not present with skin lesions.
being unilateral and throbbing, and associated
with nausea. However, this is a complicated C. Neurofibromatosis
174 migraine because it is associated with neurological Correct. The lesions seen in the image are
phenomena (monoplegia and hemianopia) and café-au-lait spots. This is an autosomal dominant
disorder. There are criteria for making the C. Juvenile myoclonic epilepsy
diagnosis. This boy meets the diagnosis as he Typically affects 10 to 20 year olds. Antonia’s
has six or more café-au-lait spots, has axilla history does not correspond with this diagnosis.
freckles and it is likely he has a family history, See Table 29.1 in Illustrated Textbook of
related to his father. Paediatrics for more details of epilepsy
syndromes.
D. Sturge–Weber syndrome
In Sturge–Weber syndrome there is a port-wine D. Lennox-Gastaut syndrome

Neurological disorders
stain present on the face. This condition affects children 1–3 years old,
who have multiple seizure types, but mostly
E. Tuberous sclerosis atonic, atypical (subtle) absences and tonic
In tuberous sclerosis there are a number of seizures in sleep with neurodevelopmental
different skin lesions: depigmented, ‘ash regression and behaviour disorder. Antonia’s
leaf’-shaped patches, which fluoresce under history does not correspond with this
ultraviolet light (Wood’s light), roughened diagnosis.
patches of skin (shagreen patches) usually over
the lumbar spine and/or adenoma sebaceum E. Infantile spasms (West syndrome)
(angiofibromata) in a butterfly distribution over Antonia’s history does not correspond with this
the bridge of the nose and cheeks. diagnosis.

29.10 29.12
A. Folic acid A. Juvenile myoclonic epilepsy
Correct. Folic acid prior to conception and early Vijay’s history does not correspond with
in pregnancy significantly reduces the chance of this diagnosis. See Table 29.1 in Illustrated
spina bifida (as shown in this image). It is the Textbook of Paediatrics for details of epilepsy
only vitamin supplement that reduces the risk of syndromes.
neural tube defects. B. Lennox-Gastaut syndrome
B. Iron Vijay’s history does not correspond with this
Iron supplementation is prescribed to treat diagnosis.
anaemia during pregnancy. C. Childhood Rolandic epilepsy (benign epilepsy
C. Vitamin A with centro-temporal spikes)
Vitamin A should not be supplemented in Vijay’s history does not correspond with this
pregnancy as it can harm the developing diagnosis.
fetus. D. Childhood absence epilepsy
D. Vitamin B12 Wrong age, wrong description and EEG would
Vitamin B12 is usually only considered if the show three per second spike and wave
mother is vegetarian or vegan. pattern.

E. Vitamin D E. Infantile spasms (West syndrome)


Vitamin D is recommended to be supplemented Correct. Vijay has violent flexor spasms
for healthy bone development. of the head, trunk and limbs followed by
extension. These are known as infantile
29.11 spasms. He also has developmental regression.
A. Childhood Rolandic epilepsy (benign epilepsy The EEG shown is characteristic, called
with centro-temporal spikes) hypsarrhythmia.
This condition comprises 15% of all childhood
epilepsies. The EEG shows focal sharp waves
from the Rolandic area. Important to recognize Answers: Extended Matching
as it is relatively benign and may not require
antiepileptic drugs. Occurs in 4–10-year-olds Answer 29.13.1
and remits in adolescence. Antonia’s history K. Reflex asystolic syncope (reflex anoxic
does not correspond with this diagnosis. seizure)
See Table 29.1 in Illustrated Textbook of These are triggered by pain, often after a trivial
Paediatrics for more details of epilepsy fall or knock. The child becomes pale and
syndromes. subsequently may have a generalised tonic–
clonic seizure, provoked by hypoxia. This is
B. Childhood absence epilepsy followed by a rapid recovery and there is no
Correct. The history suggests this diagnosis. In lasting neurological insult. Children with reflex
addition, the EEG corresponds with it, showing asystolic syncope often have a first-degree
3/s spike and wave discharge, which is bilaterally relative with a history of faints. A history of
synchronous during, and sometimes between, breath holding should be sought as it is easy to 175
attacks. confuse this with breath-holding attacks.
Answer 29.13.2 Answer 29.14.3
L. Syncope D. Lennox–Gastaut syndrome
Syncope is often caused by prolonged standing Andrew has different seizure types: tonic,
or a prolonged fast. There are often pre-syncopal atypical absence and atonic seizures. These
29 symptoms, including feeling hot, dizzy and/or
visual disturbance. Syncope can lead to jerking
children often have complex neurological
problems and their prognosis is, sadly, very poor.
movement (clonic movements), which can be It affects 4% of children with epilepsy and is
mistaken for a seizure. more common in boys than girls.
Neurological disorders

Answer 29.13.3 Answer 29.15.1


C. Cardiac arrhythmia G. Gower’s sign
Warning signs for a cardiac arrhythmia include The key to a positive Gower’s sign is that the
collapse during exercise, early pallor and a child has to move from a supine to a prone
family history of sudden premature death in position in order to get up off the floor (this is
otherwise fit and healthy relatives. Some types of normal until 3 years of age). It is only later that
cardiac arrhythmia (e.g. ventricular tachycardia) the classical ‘climb up the legs with the hands’
have abnormalities present on the ECG e.g long feature becomes evident, when muscle
QT, and are provoked by a shock (e.g. diving into weakness is profound. See Fig. 29.6 in Illustrated
a swimming pool). Sometimes there is a Textbook of Paediatrics.
structural heart problem (e.g. hypertrophic
obstructive cardiomyopathy) that results in ECG Answer 29.15.2
changes. A. Babinski’s sign
This comprises upward initial movement of the
Answer 29.13.4 big toe with or without fanning of the toes. A
B. Blue breath-holding spells (expiratory positive Babinski is normal until 18 months of
apnoea syncope) age. This boy has an upper-motor neurone lesion
These occur in some upset toddlers. The child secondary to perinatal brain injury.
will cry, which is immediately followed by an
episode of apnoea (breath-holding) and Answer 29.15.3
cyanosis, sometimes with loss of consciousness. I. Romberg’s sign
There is a rapid recovery after such attacks. The Romberg test is a test of balance. It is
performed by asking the child to stand with legs
Answer 29.13.5 together and arms by the side. The child should
D. Childhood absence epilepsy (typical then close his/her eyes and be observed for a full
absence seizure) minute. Romberg’s sign is positive if the patient
Often affected children do not do well at school sways or falls when his/her eyes are closed.
because of transient loss of consciousness, which Sayeed most likely has Friedreich ataxia.
impedes learning. They are often described as However, this is not a test of cerebellar function,
day-dreamers by their teachers. Absence seizures as children with cerebellar ataxia will generally
can be reproduced in the clinical setting by be unable to balance even with their eyes open!
asking the child to hyperventilate for 2 minutes.
Asking the child to blow on a toy windmill and Answer 29.15.4
keep it spinning quickly or blow out an F. Fogs’ sign
imaginary candle is helpful (as well as fun). Subtle asymmetries in gait may be revealed by
Fogs’ test – children are asked to walk on their
Answer 29.14.1 heels, the outside of their feet and then the
A. Childhood Rolandic epilepsy (benign inside of their feet. Watch for the pattern of
epilepsy with centro-temporal spikes) abnormal movement in the upper limbs.
This classically presents between the age of 4–10 Observe them running. This girl probably has
years, with tonic–clonic seizures in sleep or subtle cerebral palsy, which may be related to
simple partial seizures, with abnormal feelings in her premature birth.
the tongue and distortion of the face. It is also
known as benign childhood epilepsy with Answer 29.15.5
centro-temporal spikes due to the EEG, which H. Kernig’s sign
shows focal sharp waves from the Rolandic i.e. Kernig’s sign is positive if there is pain on
centro-temporal area. extending the knee. This suggests meningism. To
undertake the test, the hip and knee are flexed
Answer 29.14.2 to 90°. The knee is the extended to 180°.
C. Juvenile myoclonic epilepsy Brudzinski’s sign also indicates meningism; the
Myoclonic jerks are worse in the morning, patient should be lying in a supine position and
especially if the child is sleep deprived. These the sign is positive if there is involuntary lifting
children may also have generalised tonic–clonic of the legs when their head is lifted off the
176 seizures and absences. couch.
30

Adolescent medicine

Select one answer only.


Questions: Single Best Answer
A. Malignant disease
30.1 B. Heart disease
Which of these Western European countries has C. Infection
the highest incidence of teenage motherhood? D. Injury and poisoning
E. Neurological disease
Select one answer only.
A. Denmark 30.4
B. France Olivia is a 16-year-old girl who presents to the
C. Portugal local pharmacy for ‘emergency contraception’.
D. Spain She has had unprotected sex with her boyfriend,
E. United Kingdom who is also 16 years old. She has no other
medical problems and is not currently on any
medications.
30.2
Claire is 15 years old. She visits her general For how long after unprotected intercourse is
practitioner as she wants to go on the oral emergency contraception effective?
contraceptive pill. She is sexually active and has
a boyfriend with whom she has been in a Select one answer only.
relationship for 6 months. Her mother is not A. 12 hours
aware she has come to see the doctor today, and B. 24 hours
is also not aware she is sexually active. Claire C. 48 hours
does not want her mother to know she is getting D. 72 hours
the oral contraceptive pill. She has no other E. 1 week
medical problems. She has regular periods and
her blood pressure is normal.
Questions: Extended Matching
What advice should you give her?
30.5
Select one answer only. From the list of medical disorders encountered
A. You will only give her the pill if her parents in adolescence, select the most likely diagnosis
are present to consent in writing (A–K) for each scenario. Each diagnosis can be
B. You will prescribe the pill and promise not used once, more than once, or not at all.
to tell her parents A. Acne vulgaris
C. You will prescribe the pill and encourage her B. Asthma
to tell her mother that she is going to start C. Depression
taking the pill D. Epilepsy
D. You will prescribe the pill but will inform her E. Chronic fatigue syndrome
parents you are doing so F. Anorexia nervosa
E. She cannot have the pill as she is not legally G. Somatic symptoms
allowed to have sex H. Risk-taking behaviour (including smoking,
drug abuse, alcohol)
30.3 I. Pregnancy
Which of the following is the most common J. Malignancy
cause of death in 15–19-year-olds in the UK? K. Psychosis
30.5.1 out of character; he was previously a kind and
Anika, a 15-year-old girl, presents with her hard-working boy. His school performance has
mother to her general practitioner. She complains also deteriorated. He also complains of an
of vomiting. This is present most often in the occipital headache that is worse in the morning
30 morning and is associated with abdominal pain,
fatigue, and breast tenderness. You ask her if she
or when he bends down. His mother sometimes
finds vomit in the sink. Examination is normal
has had unprotected sex, and she denies this. She except for a squint. On checking his eye
has a suprapubic mass. She has no other medical movements, you find that he is unable to deviate
Adolescent medicine

problems and is not on any medications. his left eye laterally.


30.5.2 30.5.4
Sam, 16-years-old, is brought to his general Bennu, a 14-year-old girl whose parents are from
practitioner because his parents are worried Egypt, is sent to the school nurse by her maths
about him. They complain that he stays locked in teacher because she is complaining of a
his room and is not doing well at school. He headache. She complains of a headache at least
often refuses to get out of bed because he has a once a week and often feels tired. There is no
headache. You speak to Sam alone. He pattern to her headaches. She has no other
complains of being bored all the time and says symptoms and is doing well at school. She has
that there is nothing he enjoys. He feels lots of friends and is a sociable teenager. There
hopeless. He no longer goes out with his friends. are no abnormalities on examination. She has no
He complains of a headache, which is of gradual other medical problems and is on no
onset and like a tight band around his head. He medications.
is a fan of ‘ER’, a medical drama, is particularly
concerned about one of the characters who 30.5.5
suffered from a brain tumour. He is convinced Gareth is a 15-year-old boy who has become
that he also has a brain tumour. increasingly withdrawn. He frequently argues
with his mother, particularly when she believes
30.5.3 he should stay at home to do his homework
Keith, a 15-year-old boy, attends the paediatric rather than leaving the house to meet his new
outpatient department with his mother. Over the group of friends. His mother believes that this
last 6 weeks he has become increasingly new group of friends are using drugs. She is very
aggressive, and last week was suspended from upset that he has even been cautioned by the
school for throwing his book at a teacher. His police for urinating in public whilst intoxicated.
mother says that this behaviour is completely He has no other medical problems.

178
groups. Risk taking behaviour is an important
Answers: Single Best Answer problem, and alcohol is implicated in many cases.
Injury and poisoning account for almost half
30.1 (43%) of deaths in 15–19-year-olds.
A. Denmark
In 2014 Denmark had the lowest rate of 30.4
motherhood in the European Union area with D. 72 hours
rates of 1.1 per 1000 women aged 15–17 years. Correct. Emergency contraception is available

Adolescent medicine
from a pharmacist without prescription for those
B. France aged 16 years and over, and on prescription for
France has a relatively low rate of motherhood those under 16 years. If taken within 72 hours, it
with approximately 4.4 births per 1000 young has a 2% failure rate.
women aged 15–17 in 2014.
C. Portugal
Portugal has reduced rates of motherhood in Answers: Extended Matching
those under the age of 18 consistently over the
last decade and still has rates lower than in the Answer 30.5.1
UK. In 2014 these were 5.3 per 1000. I. Pregnancy
The symptoms described are of pregnancy.
D. Spain Teenagers often present with symptoms rather
Spain has been consistently one of the better than missed periods. She may well deny
performers in the European Union. Rates remain unprotected sex whilst her mother is present.
low but not as low as the Nordic countries and
the Netherlands. Answer 30.5.2
C. Depression
E. United Kingdom Sam has features of depression including
Correct. In 2014 there were 6.8 live births per 1000 apathy, inability to enjoy himself, decline in
young women aged 15–17 in the UK. The highest school performance and hypochondriacal
rates were seen in Bulgaria and Romania with rates ideas. His headache is most likely a tension
of 35.5 per 1000 and 28.2 per 1000 respectively. headache.
Data can be found at http://www.ons.gov.uk/ons/
dcp171778_353922.pdf for data on conceptions Answer 30.5.3
and http://www.ons.gov.uk/ons/rel/vsob1/ J. Malignancy
births-by-area-of-usual-residence-of-mother Keith has symptoms of a brain tumour. His
–england-and-wales/2012/sty-international behaviour has changed, and he has a headache
-comparisons-of-teenage-pregnancy.html for most suggestive of raised intracranial pressure. The
up to date statistics (Published February 2014). squint is a VI nerve palsy, a ‘false-localizing sign’
of raised intracranial pressure. Fundoscopy
30.2 and blood pressure measurement should be
C. You will prescribe the pill and encourage undertaken and he should have an urgent
her to tell her mother that she is going to brain scan.
start taking the pill
Correct. It is usually desirable for the parents to Answer 30.5.4
be informed and involved in contraception G. Somatic symptoms
management. She should be encouraged to tell Bennu has no symptoms of depression, and
them or allow the doctor to, but if the young there are no associated danger signs (‘red-flag’
person is competent to make these decisions for signs) with the headache. Somatic symptoms are
herself, in the UK the courts have supported common in teenagers, with 25% complaining of
medical management of these situations without headache more than once a week.
parental knowledge. This is referred to as Fraser
Guidelines (See Chapter 5. Care of the Sick Child Answer 30.5.5
and Young Person in Illustrated Textbook of H. Risk-taking behaviour
Paediatrics). Gareth has started to explore ‘adult’ behaviours
including smoking, drinking and drug use. These
30.3 ‘risk-taking’ behaviours reflect Gareth’s search for
D. Injury and poisoning new and enjoyable experiences, as well as
Correct. In the UK, the mortality rate has exerting independence from his mother and
declined more slowly in adolescents than in other rebelling against her wishes.

179
31

Global child health

31.3
Questions: Single Best Answer Which of the following causes of death in
childhood is likely to increase most in the next
31.1 15 years?
Worldwide, which of the following conditions is
most likely to result in the death of an individual Select one answer only.
child before their fifth birthday?
A. HIV infection
Select one answer only. B. Malaria
C. Malnutrition
A. Gastroenteritis D. Pneumonia
B. HIV (Human immunodeficiency virus) E. Trauma
infection
C. Injuries
D. Malaria
E. Prematurity

31.2
Worldwide, which of the following factors is
most important in predicting neonatal mortality?
Select one answer only.
A. Birth order
B. Household wealth
C. Maternal educational achievement
D. Paternal educational achievement
E. Urban versus rural residency
differences from maternal educational
Answers: Single Best Answer achievement.

31.1 C. Maternal educational achievement


A. Gastroenteritis Correct. A well-educated mother is much less
This is a very important cause of morbidity but likely to have a neonatal death than a poorly
since the 1970s and the widespread use of oral educated one. See Figure 31.3 in Illustrated
rehydration solution, deaths from gastroenteritis Textbook of Paediatrics for comparisons

Global child health


have fallen. It is now responsible for 9% of between different ‘modifiable’ factors.
deaths of children under 5 years of age.
D. Paternal educational achievement
B. HIV infection This has an effect but is not as marked as the
A worldwide catastrophe that shows signs of effect of maternal education.
improvement. Prevention of mother to child E. Urban versus rural residency
transmission has helped considerably. It is now
Urban living is associated with lower mortality
responsible for 1% of deaths of children under
but the effect is less than that seen in maternal
5 years of age.
education.
C. Injuries 31.3
This is the leading cause of death in older
A. HIV infection
children. Globally, it is responsible for 6% of
Death in children from HIV infection should not
deaths of children under 5 years of age.
increase because of prevention of mother to
D. Malaria child transmission and increasing availability and
An important treatable cause of death. It is efficacy of treatment.
now responsible for 5% of deaths of children
B. Malaria
under 5 years of age.
Mortality from malaria is decreasing because of
E. Prematurity insecticide-treated bed nets, mosquito control,
Correct. Prematurity accounts for 16% of deaths and early treatment, often by community health
in children under 5 years of age. Overall, 44% of all workers.
deaths occur in the neonatal period and of these
C. Malnutrition
prematurity is the leading cause. Pneumonia is the
Food poverty is an important problem but it is
next most common cause of death, responsible
not predicted to increase.
for 13% of all deaths in children under 5.
D. Pneumonia
31.2 Pneumonia is a major cause of death during
A. Birth order childhood. It is particularly important in younger
Birth order does affect childhood mortality but children but is not increasing.
the effect is small.
E. Trauma
B. Household wealth Correct. Mortality from unintentional injury,
There is still a difference in neonatal mortality mainly through road traffic incidents, is
according to wealth but it is less than the predicted to increase.

181
Index
Page numbers followed by “f” indicate figures, “t” Autosomal dominant disorder
indicate tables, and “b” indicate boxes. achondroplasia, 48-49, 54
Marfan syndrome, 49, 54, 54t
A Autosomal dominant inheritance, 47, 47f, 52
Abdomen Autosomal recessive disorder, sickle cell disease, 49, 54
ultrasound of, 34, 38, 126, 130 Autosomal recessive inheritance, 46, 46f, 51
X-ray of, 35, 38
ABO incompatibility, 65, 68 B
Accidental injury, 40, 40f, 43 Babinski sign, 172, 176
Accidents Baby, assessment of, 152-153, 152f, 156
burns, 34, 35f, 38 Bacterial infection, in eczema, 143, 148
poisoning and, 33-39 Bacterial meningitis, 85, 90
Achondroplasia, autosomal dominant disorder, 48-49, Balanoposthitis, broad-spectrum antibiotic and warm
54 baths for, 117, 119
Acute lymphoblastic leukaemia, 7 Barrel chest, 7-8
Adolescent medicine, 177 Basal ganglia lesion, 7
Adrenal tumour, 70, 72 Beneficence, sick child and, 23, 25
Advice, for emotional and behavioural problems, 139, Benign epilepsy, with centro-temporal spikes. see
141 Childhood Rolandic epilepsy
Airway opening manoeuvres, paediatric emergencies Benzoyl peroxide, topical, for acne, 143, 143f, 148
and, 28, 32 Biliary atresia, 65, 68, 121, 123
Alcohol, 36, 39 Bilirubin, level of, 63, 67
Allergy, 93-94 Biopsy, excision, 127, 130
Cow’s milk protein, 11 Birth order, neonatal mortality and, 180-181
Analgesia, 88-89, 92 Birthweight, cardiovascular disease and, 74, 76
intravenous, 34, 38 Bite marks, non-accidental injury and, 41, 41f, 44
Anaphylaxis, 93-94 Blood culture, for leukaemia, 124, 128
Anorexia nervosa, 138, 140 Blood glucose
Anti-epileptic drug therapy, 168-169, 174 hepatomegaly and, 120, 122
Antibiotics, for leukaemia, 124, 128 for hypoglycaemia, 29, 32
Antipyretic, 88-89, 92 level, paediatric emergencies and, 28, 31
Aorta, coarctation of, 106, 108 measurements of, 150, 151f, 154
Aortic stenosis, 105-106, 105f, 108-109 Blood tests, for assessment of baby, 152-153, 152f, 156
Apgar score, 57, 61 Blood transfusion, 133, 136
Appendicitis, 80, 84 Blue breath-holding spells (expiratory apnoea syncope),
Arthritis 171, 176
reactive, 160, 162, 165-166 Bone fracture, parietal, 33, 33f, 37, 37f
septic, 159-160, 162, 164, 166 Bone marrow aspirate, 126, 130
systemic-onset juvenile idiopathic, 160, 165 Bordetella pertussis, 95, 100
ASD. see Atrial septal defect (ASD) Bow legs-rickets, 163, 167
Asthma, 1-2, 96, 101 BPD. see Bronchopulmonary dysplasia (BPD)
history and examination of, 6, 6f, 8 Brain, CT or MRI scan of, 15, 19
Ataxic (hypotonic) cerebral palsy, developmental Brain tumour, 126, 130
problems and, 16, 20 Breast milk
Atopic eczema, 146, 146f, 149 disadvantages of, 74-75, 77
Atresia, biliary, 121, 123 jaundice, 65, 68
Atrial septal defect (ASD), 105-106, 105f, 108 Broad-spectrum antibiotic, for balanoposthitis, 117, 119
Atrioventricular septal defect, 46, 50 Bronchiolitis, 98, 102
Auditory brainstem response audiometry, 11 history and examination of, 5, 5f, 8
Autism spectrum disorder Bronchopulmonary dysplasia (BPD), 65, 65f, 68, 98, 98f,
age of, 13, 18 102
developmental problems and, 13, 18 Buccal glucose gel, 150, 154
Autoimmune pancreatic β-cell damage, 150, 154 Burns, 34, 35f, 38
Autonomy, for child, 23, 25 ‘glove and stocking’ distribution of, 42, 42f, 44
C Depression
Candida napkin rash, 142, 143f, 147 adolescents and, 178-179
Cardiac arrhythmia, 171, 176 maternal postnatal, 10, 12
Cardiac compressions Dermatological disorders, 142-149
positions on chest, 26, 26f, 30 Development, 9-12
using hands encircling method, 28, 32 age, 9, 11
Cardiac disorders, 104-109 child, poverty on, 1-2
Cataract, developmental problems and, 15, 15f, 19 health surveillance, 10, 12
Catecholamines, urinary, 124, 126, 128, 130 hearing, 9-12

Index
Cavernous haemangioma, 58, 58f, 62 milestones, 10, 12
Cerebellar lesion, 7 normal, 9, 11
Chest X-ray, 96, 100 routine check, 9, 11
Chickenpox (varicella zoster virus), 88, 88f, 92, 125, 125f, vision, 9-12
129 Developmental delay, 15
Child abuse Developmental dysplasia of the hip (DDH), 2, 163,
medical, 41, 41f, 43 167
physical abuse, 40, 43 Developmental milestone, by median age, 10, 12
sexual abuse, 40-41, 43 Developmental problems, and child with special needs,
Child development. see Development 13-20
Child protection, 40-44. see also Child abuse Dextrocardia with situs inversus, 106, 108
Child psychiatrist, for emotional and behavioural Diabetes, 150-156
problems, 139, 141 type 1, 75, 77
Childhood, normal variant of, 163, 167 Diabetic ketoacidosis, 79, 83
Childhood absence epilepsy, 170-172, 170f, 175-176 Diaphragmatic hernia, 65, 65f, 68
Childhood Rolandic epilepsy, 172, 176 Dietician, care for disabled child, 17, 20
Children Digoxin, 36, 39
age for, admitted to hospital, 21, 24 Distraction, for reducing pain, 22, 25
in society, 1-2 DNA FISH analysis, developmental problems and,
Children’s Coma Scale, 31t 16, 19
Chondromalacia patellae, 162-163, 167 Down syndrome, 48, 48f, 53
Choreoathetoid cerebral palsy, 7 characteristic clinical manifestations, 53b
Chronic fatigue syndrome, 139, 141 nondisjunction, meiotic, 45, 50
Chronic non-specific diarrhoea, 78, 81 Duchenne muscular dystrophy, 57, 62
Ciliary dyskinesia, primary, 106, 108 Duodenal atresia, 64, 68
Clean catch urine, 110, 114 Duty, moral obligation and, 23, 25
Coarctation of the aorta, 66, 69 Dyskinetic cerebral palsy, developmental problems and,
Coeliac disease, 78, 81 16, 20
Coeliac screen, 71, 73
Cognitive behavioural therapy, for emotional and E
behavioural problems, 139, 141 Electroencephalography (EEG), developmental problems
Colds, 96, 101-102 and, 16, 19
Complex regional pain syndrome, 162, 167 Emergencies, paediatric, 26-32
Congenital abnormalities, 56, 60 Emergency contraception, 177, 179
Congenital adrenal hyperplasia, 152, 155 Endocrinology, 150-156
Congenital infection screen, 16, 19 Epiglottitis, 95, 95f, 100
Consanguinity, autosomal recessive inheritance, 46, 46f, Epilepsy
51 childhood absence, 170-172, 170f, 175-176
Constipation, 78, 80-81, 84, 138-140 childhood Rolandic, 172, 176
Contraception, emergency, 177, 179 Epstein-Barr virus, 87, 88f, 92
Convergent squint, right, history and examination of, 4, Erythema toxicum, 58, 58f, 62
4f, 7 Excision biopsy, 127, 130
Cortisol, low, Addison disease and, 152, 155 Expiratory apnoea syncope. see Blue breath-holding
Coryza, 96, 102 spells
Cough, 96, 101 Explanation, for emotional and behavioural problems,
Cow’s milk protein allergy, 11, 75, 77 139, 141
Coxsackie A, 86, 86f, 90-91 Eye, opening of, Glasgow Coma Score and, 31t
Creams, for atopic eczema, 143, 148
Creatine kinase, developmental problems and, 16, 19 F
Cricopharyngeus muscle, 36, 36f, 39 Familial short stature, 71-72
Cushing syndrome, clinical features, 53, 53b Femur
Cystic fibrosis, 2, 96-97, 97f, 99, 101-103 fractured, 162, 166
genetics and, 47, 47f, 52 midshaft fracture of, 40, 40f, 43
Cystitis, 110, 114 Fifth disease (Parvovirus infection), 88, 92
Cytomegalovirus, 57, 61 Fluid resuscitation, 153, 156
intravenous access and, 34, 37
D Fogs’ sign, 172, 176
DDH. see Developmental dysplasia of the hip (DDH) Folic acid, 170, 170f, 175
Death, in children, 1-2 Foreign body inhalation, 98, 98f, 102 183
Denmark, teenage motherhood, 177, 179 Fossa tumour, posterior, 125-126, 125f, 129
Fractured femur, 162, 166 Hepatitis B, 121, 123
Fragile X syndrome, 48, 53-54, 54b, 54f vaccination, 120-122
France, teenage motherhood, 177, 179 Hepatomegaly
Functional abdominal pain, 79, 82 blood glucose and, 120, 122
history and examination of, 3, 7
G serum conjugated and unconjugated bilirubin and,
Galactosaemia, 121, 123 120, 122
Galactose-1-phosphate uridyl transferase, 157-158 Hereditary spherocytosis, 131, 135
Gastro-oesophageal reflux, 78-79, 82 High-flow oxygen therapy, paediatric emergencies and,
Index

Gastroenteritis, children and, 180-181 28, 32


Gastroenterology, 78-84 Hip, developmental dysplasia of, 2
abdominal pain in, 79-80, 82 Hirschsprung disease, 64, 68, 79, 83
constipation in, 78, 80-81, 84 Hookworm infestation, 7
Hirschsprung disease in, 79, 83 Household wealth, neonatal mortality and, 180-181
inflammatory bowel disease in, 84 Human immunodeficiency virus (HIV), children and,
GCS. see Glasgow Coma Score (GCS) 180-181
Genes, 70, 72 Hydrocortisone, intravenous, 151, 155
Genetics, 45-55 17α-hydroxyprogesterone, 153, 156
cystic fibrosis, 47, 47f, 52 Hyperkalaemia, Addison disease and, 152, 155
uniparental disomy, 49, 55, 55f Hypernatraemic dehydration, rehydration for, 79,
Genital disorders, 117-119 82, 83f
surgical repair for, 117, 117f, 119 Hypersensitivity, 93-94
Genu varum. see Bow legs-rickets Hypoglycaemia, 57, 61
Glanular hypospadias, management of, 117-119, 118f Hyponatraemia, Addison disease and, 152, 155
Glasgow Coma Score (GCS), 27, 30, 31t Hypothyroidism, 120, 120f, 122
Global child health, 180-181 diabetes and, 150, 154
Glomerulonephritis, 112, 116 Hypovolaemic shock, 34-35, 38
Glucose
intravenous, 151, 155 I
oral, 153, 156 Idiopathic precocious puberty, 70, 72
Glucose-6-phosphate dehydrogenase deficiency, 131, Immune thrombocytopenia, 131, 135
135 Immune thrombocytopenic purpura (ITP), 42, 42f, 44
Glycogen storage disorder, 157-158 Immunization, 87, 91
Gonococcus on swab, 40-41, 43 Inborn errors of metabolism, 157-158
Gower sign, 172, 176 Infant, development of, 75, 77
G6PD deficiency, 65, 68-69 Infantile seborrhoeic dermatitis, 143, 147
Gradual rehabilitation programme, 139, 141 Infantile spasms (West syndrome), 170-171, 171f, 175
Group B streptococcus, 64, 67 Infection
Growing pains, 162, 166 in axilla, 144, 144f, 148
Growth, 70-73 and immunity, 85-92
hormone, 70, 72 Inflammatory bowel disease, 80, 84
restricted, 56, 60 Inhaled foreign body (left side), history and examination
Guillain-Barré syndrome, 169, 174 of, 5-6, 6f, 8
Guttate psoriasis, 144-145, 145f, 149 Initial fluid resuscitation, in shock, 27, 31
Injuries
H adolescents and, 177, 179
Haematological disorders, 131-137 children and, 180-181
Haemoglobin Insulin, subcutaneous, 153, 156
in infants, 134, 137 Intracranial pressure, raised, 168, 173
and platelets, low, 124-125, 125f, 128 Intramuscular adrenaline, as medication for wheezing,
Haemolytic-uraemic syndrome, 111, 115 28, 32
Haemophilia A, 133, 136 Intramuscular (IM) administration, vitamin K, 22, 25
Haemophilus influenzae, 95, 95f, 100 Intranasal diamorphine, 22, 25
Haemorrhage, neurosurgical referral and, 33-34, Intravenous administration, of given drugs, 22, 24
34f, 37 Intravenous antibiotics, 63, 67, 89, 92
Haemorrhagic disease of the newborn, vitamin K for kidney and urinary tract disorders, 110,
deficiency and, 75, 77 114-115
Harrison sulcus, history and examination of, 4, 4f, 8 Intravenous desferrioxamine, 36, 38
Health surveillance, in United Kingdom, 10, 12 Intravenous immunoglobulin, 89, 92
Hearing, child development, 9-12 Intravenous morphine via nurse-controlled pump, 22,
Hearing loss 25
mild conductive, in right ear, 14, 14f, 18-19 Intraventricular haemorrhage, 169, 169f, 174
severe sensorineural, in both ears, 14, 15f, 19 Intussusception, 79, 83
Hearing test, for developmental problems, 13, 18 as cause of pain and shock, 27, 31
Heart murmur, 56, 60 Investigation, febrile seizure and, 168, 173
Hemiplegia, left, history and examination of, 4, 7 Iron, 36, 39
Henoch-Schönlein purpura, 144, 145f, 149, 160, 160f, Iron-deficiency anaemia, 132, 132f, 136
184 165 Iron supplements, 131-132, 135
Hepatitis A, 121, 123 ITP. see Immune thrombocytopenic purpura (ITP)
J N
Juvenile myoclonic epilepsy, 172, 176 Naevus flammeus, 58, 58f, 62
Nebulized treatment, for sick child, 22, 24-25
K Neonatal infection, risk of, 57, 62
Karyotype, 71, 73 Neonatal medicine, 63-69
Kawasaki disease, 86, 91 Neonatal sepsis, 64, 68
Kerion, 143, 144f, 148 Neonates, mortality, 180-181
Kernig sign, 172, 176 Nephroblastoma, 126, 130
Kidney and urinary tract disorders, 110-116 Nephrotic syndrome, 111, 111f, 115

Index
diagnosis of, 112 Neuroblastoma, 126, 130
renal investigations of, 112-113 Neurofibromatosis, 169, 170f, 174-175
Kidneys, poorly functioning fetal, 56, 60 Neurological disorders, 168-176
Klinefelter syndrome (47, XXY), 47-48, 52, 54 Neuromuscular junction, 7
clinical features, 52b Night terrors, 139, 141
Kwashiorkor, 75, 77 Non-allergic food hypersensitivity, 93-94
Non-maleficence, principle, sick child and, 23, 25
L Nondisjunction, meiotic (Down syndrome), 45, 50
Laryngotracheobronchitis (croup), 97-98, 97f, 102 Noonan syndrome, 47, 47f, 53
Left-hand preference, developmental problems and, 13, Normal variant of childhood, 163, 167
18 NSAID, for sick child, 23, 25
Left-to-right shunt, 104, 107 Nutrition, 70, 72, 74-77
Lennox-Gastaut syndrome, 172, 176
Leukaemia, 46, 51, 124, 128 O
acute lymphoblastic, 126, 130, 133, 136 Occupational therapist, care for disabled child, 17, 20
Leukocoria, 125, 129 Oestrogen, 70, 72
Lipid profile, nutrition and, 74, 76 Ointments, for atopic eczema, 143, 148
Liquid form, of antibiotics, 22, 24 Oligohydramnios, 56, 60
Liver disorders, 120-123 OME. see Otitis media with effusion (OME)
Lower motor neurone lesion, 7 Oncology ward, paediatric assessment, 21, 24
Lymphoblastic leukaemia, acute, 126, 130, 133, 136 Opiate poisoning, paediatric emergencies, 27, 27f, 30
Opiate use, maternal, 57, 62
M Oral antibiotic, 88, 92
Magnetic resonance imaging scan, 126-127, 130 Oral antihistamine, as medication for urticarial rash, 28,
Malaria 32
children and, 180-181 Oral contraceptive pill, adolescents and, 177, 179
history and examination of, 3, 7 Oral paracetamol, regular, 22, 25
Malaria parasites (Plasmodium sp.), 87, 88f, 92 Oral rehydration solution, 89, 92
Malignancy, adolescents and, 178-179 Oral steroid, 99, 103
Malignant disease, 33, 37, 124-130 Osgood-Schlatter disease, 159, 163-164, 167
Malnutrition, childhood death and, 180-181 Osteogenesis imperfecta, 161, 165-166
Malrotation, 80, 84 Osteoid osteoma, 163, 167
Marasmus, 75, 77 Otitis media, 99, 103
Marfan syndrome, autosomal dominant disorder, 49, 54, Otitis media with effusion (OME), 96-97, 97f, 102
54t Oxygen, 93-94
Maternal educational achievement, neonatal mortality
and, 180-181 P
Maternal postnatal depression/stress, 10, 12 Paediatric emergencies, 26-32
Measles virus, 87, 87f, 92 Paediatrician, for disabled child, 16, 20
Meconium ileus, 64, 68 Pain management, for children, 22-23
Medication, administration of, 21-22 Paracetamol, 36, 39
Medulloblastoma, 125, 129, 129f Parenting classes, 138, 140
Meningococcal sepsis, 85, 85f, 90 Parvovirus infection (fifth disease), 88, 92
Meningococcal septicaemia, 42, 42f, 44 Patau syndrome (trisomy 13), 48, 53
Mental health, child and adolescent, 138-141 Paternal educational achievement, neonatal mortality
Mesenteric adenitis, 80, 84 and, 180-181
Metabolism, inborn errors of, 157-158 PDA. see Persistent ductus arteriosus (PDA)
Metered dose inhaler, 96, 101 Pectus carinatum, 8
Migraine, 80, 84, 169, 174 Pectus excavatum, 8, 161-162, 161f, 166
Milia, 58, 58f, 62 Perinatal medicine, 56-62
Molluscum contagiosum, 145, 145f, 149 Perineal contamination, 112, 116
Mongolian blue spots, 59, 59f, 62 Pernasal swab, 95, 100
Morphine, regular oral long-acting, with rapid action Persistent ductus arteriosus (PDA), 105-106, 105f,
oramorph for breakthrough pain, 22, 25 108
Mortality, child, 180-181 Perthes disease, 161, 161f, 166, 166f
Motherhood, teenage, 177, 179 Pertussis, 99, 103
Motor response, Glasgow Coma Score and, 31t PEX1gene, 46, 51
Murmur, 104, 106-108 Physiotherapist, care for disabled child, 17, 20
Musculoskeletal disorders, 159-167 Plasma creatinine, and electrolytes, 113, 116 185
Mycoplasma pneumoniae, 144, 144f, 148 Platelets, and haemoglobin, low, 124-125, 125f, 128
Pleural effusion (right sided), history and examination Shoulder dystocia, 56, 60
of, 6, 6f, 8 Sick children, care of, 21-25
Pneumocystis jiroveci (carinii) pneumonia, 86, 86f, 91 Sickle cell disease, 7, 132, 132f, 136
Pneumonia, 80, 84, 98-99, 98f, 102-103 Sleeping, supine, as factor for sudden infant death
childhood death and, 180-181 syndrome, 28, 32
right sided, history and examination of, 5, 5f, 8 Slipped capital femoral epiphysis, 159, 159f, 164, 164f
Pneumothorax, 64, 64f, 68 Smoking, pregnancy and, 56, 60
Poisoning Society, child in, 1-2
accidents and, 33-39 Somatic symptoms, adolescents and, 178-179
Index

adolescents and, 177, 179 Sore throat, 95, 95f, 100


Polygenic disorder, 49, 55 Spain, teenage motherhood, 177, 179
Polymerase chain reaction, 85, 90 Spastic bilateral cerebral palsy, age of, 13, 18
Port wine stain, 58, 58f, 62 Spastic diplegia cerebral palsy, developmental problems
Portugal, teenage motherhood, 177, 179 and, 16, 20
Positional talipes equinovarus, 160-161, 165 Spastic hemiplegia cerebral palsy, developmental
Poverty, on child development, 1-2 problems and, 16, 20
Prednisolone, oral Spastic quadriplegia cerebral palsy, 7, 16, 19
for nephrotic syndrome, 111, 115 Specialist health visitor, care for disabled child, 17, 20
therapy, 152, 152f, 155 Speech and language therapist, care for disabled child,
Pregnancy, adolescents and, 178-179 17, 20
Premature thelarche, 70, 72 Spherocytosis, hereditary, 131, 134-135, 137
Prematurity, children and, 180-181 Squint, child development and, 9, 11
Primary ciliary dyskinesia, history and examination of, Staphylococcal scalded skin syndrome, 42, 42f, 44
4-5, 5f, 8 Staphylococcus aureus, 88, 92, 159, 164
Proteus sp., 110, 114 Sternal recession, 8
Psychologist, care for disabled child, 17, 20 Stevens-Johnson syndrome, 142, 142f, 147
Puberty, 70-73 Strawberry naevus, 58, 58f, 62
Pulmonary stenosis, 105-106, 105f, 108 Streptococcal infection, 142, 142f, 147
Pupils, checking of, paediatric emergencies and, 29, 32 Streptococcal tonsillitis, group A, 95, 95f, 100
Pyloric stenosis, 64, 68, 79, 83 Stress, maternal postnatal, 10, 12
Supraventricular tachycardia, 104, 107
R Surfactant deficiency, 63, 63f, 67
Rash, 93-94 Syncope, 171, 176
Reactive arthritis (transient synovitis), 160, 162, 165-166 Systemic-onset juvenile idiopathic arthritis, 160, 165
Reassurance, 138, 140
for emotional and behavioural problems, 139, 141 T
Recombinant factor VIII, 133, 136-137 T4 level, high, 151, 155
Reflex anoxic seizure. see Reflex asystolic syncope Talipes equinovarus, positional, 160-161, 165
Reflex asystolic syncope (reflex anoxic seizure), 171, Testicular swelling, 117, 119
175 Testis, torsion of, 117, 119
Rehydration, for hypernatremic dehydration, 79, 82, 83f Testosterone, 70, 72
Renal osteodystrophy, vitamin D supplements for, Tetracycline, oral, for acne, 143, 143f, 148
111-112, 116 Tetralogy of Fallot, 8
Renal stone, 110, 114 β−thalassaemia trait, 133, 136
Renal ultrasound, 112-113, 116 Thelarche, premature, 70, 72
Respiratory disorders, 95-103 Thrill, 104, 107
Respiratory distress syndrome, 56, 61 Thrombocytopenia, immune, 131, 135
Resuscitation algorithms, stimulation of baby, 27, 30 Thyroid function test, 46, 51
Retinoblastoma, 125, 129 Thyroid stimulating hormone (TSH), 71, 73
Reverse end-diastolic flow, in umbilical artery, 56, 60 level, low, 151, 155
Rhinovirus, 87, 92 Toe walking, 167
Risk-taking behaviour, adolescents and, 178-179 Topical administration, for significant eczema, 22, 22f,
Robertsonian translocation, unbalanced, 45-46, 50 25
Romberg sign, 172, 176 Topical anaesthetic, for pain, 22, 25
Rotavirus, gastroenterology and, 78, 81 Total fluid requirement, for vomiting, 27-28, 31
Runny nose, 33, 37 Toxoplasmosis, 57, 61
Rural residency, neonatal mortality and, 180-181 Transient synovitis, 160, 162, 165-166
Transient tachypnoea of the newborn, 66, 69
S Transposition of the great arteries, 105, 108
Salbutamol, inhaled, via metered dose inhaler and Trauma, child and, 180-181
spacer, 99, 103 Trinucleotide repeat expansion, 168, 173
Salicylates (aspirin), 36, 39 Truth-telling, 23, 25
Saline, intravenous, 151, 153, 155-156 TSH. see Thyroid stimulating hormone (TSH)
Scabies, 145, 145f, 149 Tuberculosis, 86-87, 86f, 91
Scans, for assessment of baby, 152-153, 152f, 156 Tuberculosis meningitis, 85, 90
Senior doctor, person to inform parents, 21, 24 Tumour
Septic arthritis, 159-160, 162, 164, 166 brain, 126, 130
186 Septicaemia, paediatric emergencies and, 26, 26f, 30 posterior fossa, 125-126, 125f, 129
Shock, hypovolaemic, 34-35, 38 Wilms, 126, 130
Turner syndrome, 48, 53 Ventricular septal defect (VSD), 104-108, 105f
clinical features, 52b Verbal response, Glasgow Coma Score and, 31t
Twin pregnancy, risk in, 56 Vision, child development, 9-12
Type 1 diabetes, 75, 77 Visual reinforcement audiometry, 9, 11
Typical absence seizure. see Childhood absence Vitamin D deficiency, 56, 60, 74-77, 74f
epilepsy Vitamin D supplements, for renal osteodystrophy,
111-112, 116
U Vitamin K, 133, 136
Ultrasound, renal, 112-113, 116 concentration of, in breast milk, 74-75, 77

Index
Umbilical granuloma, 64, 64f, 68 deficiency of, and haemorrhagic disease of the
Uniparental disomy, 49, 55, 55f newborn and, 75, 77
United Kingdom, teenage motherhood, 177, 179 von Willebrand disease, 133, 136
Upper motor neurone lesion, history and examination VSD. see Ventricular septal defect (VSD)
of, 3-4, 3f, 7
Upper respiratory tract infection, 96, 102 W
Urban residency, neonatal mortality and, 180-181 Warm baths, for balanoposthitis, 117, 119
Urea cycle defect, 157-158 West syndrome. see Infantile spasms
Urinary catecholamines, 124, 126, 128, 130 Whooping cough, 99, 103
Urinary dipstick, 111, 115 Wilms tumour, 7, 126, 130
Urinary tract infection, 112, 116, 168, 173 Wilson disease, 121, 121f, 123
due to Proteus sp., 110, 114
Urine microscopy, and culture, 112-113, 116 X
Urticarial rash, 93-94, 93f X-linked recessive disorder, 45, 45f, 50
X-linked recessive inheritance, 45, 45f, 50
V
Vagal stimulation manoeuvre, 104, 107 Y
Varicella zoster, 57, 61, 88, 88f, 92, 125, 125f, 129 Young person, care of, 21-25

187
This page intentionally left blank

You might also like